Einführung in die Wissenschaftstheorie [4 ed.] 9783534264483, 3534264487

Der renommierte Philosoph Gerhard Schurz erläutert die logischen Probleme wissenschaftlichen Denkens ebenso wie die Wiss

120 4 2MB

German Pages [268] Year 2014

Report DMCA / Copyright

DOWNLOAD PDF FILE

Table of contents :
Cover
Titel
Impressum
Inhalt
Vorwort
1. Einleitung
1.1 Aufgaben und Zielsetzungen der Wissenschaftstheorie
1.2 Philosophische Positionen in der Wissenschaftstheorie
1.2.1 Empirismus und Rationalismus
1.2.2 Logischer Empirismus
1.2.3 Kritischer Rationalismus
1.2.4 Historische Wissenschaftstheorie und Relativismus
1.2.5 Weitere Positionen in Kürze
1.3 Zusammenfassung, einführende Literatur und Übungen
2. Gemeinsame Grundlagen der Wissenschaften
2.1 Methode und Voraussetzungen der Wissenschaftstheorie
2.2 Gemeinsame erkenntnistheoretische Annahmen
2.3 Gemeinsame methodologische Merkmale
2.4 Wissenschaftliche Disziplinen und ihre Klassifikation
2.5 Die Frage der Wertneutralität und das Abgrenzungsproblem
2.5.1 Das Webersche Wertfreiheitspostulat
2.5.2 Wertneutralität und Zweck-Mittel-Schlüsse
2.5.3 Das Abgrenzungsproblem
2.5.4 Präzisierung der Wertneutralitätsforderung
2.6 Wissenschaftliches Schließen und Argumentieren
2.6.1 Deduktion und Induktion
2.6.2 Popper und die Bedeutung induktiven Schließens in den Wissenschaften
2.6.3 Abduktion und Schluss auf die beste Erklärung
2.6.4 Monotone und nichtmonotone Schlüsse
2.7 Weiterführende Themen
2.7.1 Konstruktiver Realismus versus radikaler Konstruktivismus
2.7.2 Zur Theorie(un)abhängigkeit von Beobachtungen
2.7.3 Zum Unterschied zwischen Erfahrungssätzen und Werturteilen
2.8 Zusammenfassung, einführende Literatur und Übungen
3. Das analytische Instrumentarium: Sprache, Logik und Wahrscheinlichkeit
3.1 Begriffe und Begriffsarten
3.1.1 Klassifikation von Begriffen nach ihrem logischen Typ
3.1.2 Syntax, Formalisierung und Semantik
3.1.3 Klassifikation von Begriffen nach ihrem Inhaltstyp
3.1.4 Klassifikation von Begriffen nach ihrem Abstufungstyp (Skalentyp)
3.1.4.1 Qualitativ-klassifikatorische Begriffe und Nominalskalen
3.1.4.2 Komparative Begriffe und Ordinalskalen
3.1.4.3 Quantitative Begriffe
3.2 Klassifikation von Sätzen nach dem Inhaltstyp
3.3 Logische Wahrheit und deduktive Logik
3.4 Bedeutungskonventionen und definitorische Wahrheit
3.5 Klassifikation von Sätzen nach ihrem Allgemeinheitsgrad
3.6 Generelle Sätze, Gesetzesartigkeit, Determinismus und Indeterminismus
3.7 Gehalt von Sätzen und Gehaltsarten
3.8 Verifikation, Falsifikation, Bestätigung und Schwächung
3.9 Objektive (statistische) und subjektive (epistemische) Wahrscheinlichkeit
3.10 Weiterführende Themen
3.10.1 Dispositionsbegriffe
3.10.2 Herausforderungen an die logisch-definitorisch-synthetisch-Unterscheidung
3.10.3 Relevanz und Irrelevanz in logischen Schlüssen
3.10.4 Weiterführendes zur Wahrscheinlichkeit
3.10.4.1 Mathematische Gesetze der Wahrscheinlichkeit
3.10.4.2 Probleme des statistischen und des subjektiven Wahrscheinlichkeitsbegriffs
3.10.4.3 Principal principle und engste Referenzklasse
3.11 Zusammenfassung, einführende Literatur und Übungen
4. Gesetzeshypothesen und ihre empirische Überprüfung
4.1 Die Relevanzbedingung
4.1.1 Relevanz von strikten Gesetzen
4.1.2 Relevanz von statistischen Gesetzen
4.2 Die empirische Überprüfung von Gesetzeshypothesen auf Wahrheit und Relevanz
4.2.1 Der strikte Fall: die Methode der Übereinstimmung und des Unterschieds
4.2.2 Methodische Induktion: die Findung von strikten Gesetzeshypothesen
4.2.3 Der statistische Fall: die Methode der Akzeptanz- u.Konfidenzintervalle und der Signifikanztests
4.2.3.1 Überprüfung auf vermutliche Wahrheit – die Methode der Akzeptanzintervalle
4.2.3.2 Auffindung statistischer Hypothesen und Konfindenzintervalle
4.2.3.3 Überprüfung auf vermutliche Relevanz – die Methode der signifikanten Unterschiede
4.2.3.4 Statistische Repräsentativität
4.2.3.5 Teststatistik, Inferenzstatistik und die Likelhood-Intuition
4.2.4 Fehlerquellen in der statistischen Methode
4.3 Korrelation und Kausalität
4.3.1 Versteckte Variablen
4.3.2 Kausalrichtung
4.4 Die Anwendung statistischer Hypothesen auf den Einzelfall
4.5 Weiterführende Themen
4.5.1 Statistische Methoden für quantitative Variablen
4.5.2 Die Rechtfertigung der Likelihood-Intuition innerhalb der subjektiven Wahrscheinlichkeitstheorie
4.6 Zusammenfassung, einführende Literatur und Übungen
5. Theorien und ihre empirische Bewertung
5.1 Theoretische Begriffe und multiple Zuordnungsgesetze
5.2 Das Beispiel der Newtonischen Physik
5.3 Grundlegendes zur Theorienstatik: Struktur wissenschaftlicher Theorien
5.4 Methodische Merkmale (guter) wissenschaftlicher Theorien
5.4.1 Systemcharakter, empirische Kreativität, Globalität und Vereinheitlichung
5.4.2 Holismus der Bedeutung, des empirischen Gehalts, und der Theorienüberprüfung (Duhem-Neurath-Quine-These)
5.4.3 Theoriennetze
5.5 Das Beispiel der Piagetschen kognitionspsychologischen Entwicklungstheorie
5.6 Grundlegendes zur Theoriendynamik
5.6.1 Das Lakatos-Modell der Theorienrevision
5.6.2 Theorienbewertung, Theorienvergleich und Theorienfortschritt
5.7 Das Beispiel der Adorno-Milgramschen Theorie des autoritären Charakters
5.8 Weiterführende Themen
5.8.1 Instrumentalismus und Realismus
5.8.2 (Nicht-)Eliminierbarkeit und (Un-)Definierbarkeit theoretischer Begriffe
5.8.3 Allgemeine Theorien der Bestätigung
5.8.3.1 Hypothetisch-deduktive Bestätigung
5.8.3.2 Subjektiv-probabilistische Bestätigung
5.8.3.3 Die Goodman-Paradoxie
5.9 Zusammenfassung, einführende Literatur und Übungen
6. Erklärung – und was alles dazu gehört
6.1 Das deduktiv-nomologische Modell der Erklärung
6.2 Erklärung versus Voraussage und Begründung
6.2.1 Nicht jeder Glaubensgrund ist ein Realgrund
6.2.2 Kausalität und Gesetzesartigkeit im Erklärungsrahmen
6.2.3 Die Bedingung der prognostischen Funktion
6.2.4 Irrelevanz und Redundanz – die logischen Probleme der DN-Erklärung
6.3 Probabilistische Erklärungsmodelle
6.3.1 Induktiv-Statistische Erklärungen nach Hempel
6.3.2 Die Bedingung der maximalen Bestimmtheit
6.3.3 Bedingungen an den Wahrscheinlichkeitswert: konfligierende Intuitionen
6.4 Normische Erklärungen und die Erklärung menschlicher Handlungen
6.5 Weiterführende Themen
6.5.1 Gesetzesartigkeit
6.5.1.1 Naturgesetze versus Systemgesetze
6.5.1.2 Gesetzesartigkeit i. w. S. und kontrafaktische Konditionalsätze
6.5.1.3 Gesetzesartigkeit i. e. S. und physikalische Notwendigkeit
6.5.2 Kausalität
6.5.2.1 Singuläre und generelle Kausalbeziehung
6.5.2.2 Kausale Präemption und Überdetermination
6.5.2.3 Kausale Prozesse
6.5.2.4 Kausale Graphen und probabilistische Kausalanalyse
6.5.2.5 Interventionistischer Ansatz
6.6 Zusammenfassung, einführende Literatur und Übungen
Literaturverzeichnis
Definitionen, Merksätze, und Abbildungen
Autorenregister
Sachregister
Lösungen zu ausgewählten Aufgaben
Recommend Papers

Einführung in die Wissenschaftstheorie [4 ed.]
 9783534264483, 3534264487

  • 0 0 0
  • Like this paper and download? You can publish your own PDF file online for free in a few minutes! Sign Up
File loading please wait...
Citation preview

Gerhard Schurz

Einführung in die Wissenschaftstheorie 4. Auflage

Einbandgestaltung: Peter Lohse, Büttelborn Abbildung: Symbolische Darstellung der Durchbrechung des mittelalterlichen Weltbildes, 1888. Aus: Camille Flammarion: L’atmosphère, et la météorologie populaire, Paris 1888. i akg-images.

Die Deutsche Nationalbibliothek verzeichnet diese Publikation in der Deutschen Nationalbibliografie; detaillierte bibliografische Daten sind im Internet über http://www.dnb.d-nb.de abrufbar.

Das Werk ist in allen seinen Teilen urheberrechtlich geschützt. Jede Verwertung ist ohne Zustimmung des Verlags unzulässig. Das gilt insbesondere für Vervielfältigungen, Übersetzungen, Mikroverfilmungen und die Einspeicherung in und Verarbeitung durch elektronische Systeme. 4., überarbeitete Auflage 2014 i 2014 by WBG (Wissenschaftliche Buchgesellschaft), Darmstadt 1. Auflage 2006 Die Herausgabe dieses Werkes wurde durch die Vereinsmitglieder der WBG ermöglicht. Satz: Lichtsatz Michael Glaese GmbH, Hemsbach Einbandgestaltung: schreiberVIS, Bickenbach Gedruckt auf säurefreiem und alterungsbeständigem Papier Printed in Germany Besuchen Sie uns im Internet: www.wbg-darmstadt.de

ISBN 978-3-534-26448-3 Elektronisch sind folgende Ausgaben erhältlich: eBook (PDF): 978-3-534-73861-8 eBook (epub): 978-3-534-73862-5

Inhalt Vorwort

. . . . . . . . . . . . . . . . . . . . . . . . . . . . . . . .

1. Einleitung . . . . . . . . . . . . . . . . . . . . . . . . . . . . . 1.1 Aufgaben und Zielsetzungen der Wissenschaftstheorie . . 1.2 Philosophische Positionen in der Wissenschaftstheorie . . 1.2.1 Empirismus und Rationalismus . . . . . . . . . . 1.2.2 Logischer Empirismus . . . . . . . . . . . . . . . 1.2.3 Kritischer Rationalismus . . . . . . . . . . . . . . 1.2.4 Historische Wissenschaftstheorie und Relativismus 1.2.5 Weitere Positionen in Kürze. . . . . . . . . . . . . 1.3 Zusammenfassung, einführende Literatur und Übungen .

10

. . . . . . . . .

11 11 12 12 13 15 16 17 20

2. Gemeinsame Grundlagen der Wissenschaften . . . . . . . . . . 2.1 Methode und Voraussetzungen der Wissenschaftstheorie . 2.2 Gemeinsame erkenntnistheoretische Annahmen . . . . . . 2.3 Gemeinsame methodologische Merkmale . . . . . . . . . 2.4 Wissenschaftliche Disziplinen und ihre Klassifikation . . . 2.5 Die Frage der Wertneutralität und das Abgrenzungsproblem 2.5.1 Das Webersche Wertfreiheitspostulat . . . . . . . . 2.5.2 Wertneutralität und Zweck-Mittel-Schlüsse . . . . . 2.5.3 Das Abgrenzungsproblem . . . . . . . . . . . . . . 2.5.4 Präzisierung der Wertneutralitätsforderung . . . . . 2.6 Wissenschaftliches Schließen und Argumentieren . . . . . 2.6.1 Deduktion und Induktion . . . . . . . . . . . . . . 2.6.2 Popper und die Bedeutung induktiven Schließens in den Wissenschaften . . . . . . . . . . . . . . . . 2.6.3 Abduktion und Schluss auf die beste Erklärung . . . 2.6.4 Monotone und nichtmonotone Schlüsse . . . . . . 2.7 Weiterführende Themen . . . . . . . . . . . . . . . . . . . 2.7.1 Konstruktiver Realismus versus radikaler Konstruktivismus . . . . . . . . . . . . . . 2.7.2 Zur Theorie(un)abhängigkeit von Beobachtungen . 2.7.3 Zum Unterschied zwischen Erfahrungssätzen und Werturteilen . . . . . . . . . . . . . . . . . . . . . 2.8 Zusammenfassung, einführende Literatur und Übungen . .

21 21 26 29 32 39 39 41 43 44 47 47

62 64

3. Das analytische Instrumentarium: Sprache, Logik und Wahrscheinlichkeit . . . . . . . . . . . . . . . . . . . . . . . . . 3.1 Begriffe und Begriffsarten . . . . . . . . . . . . . . . . . .

66 66

49 52 54 56 56 57

6

Inhalt

3.1.1 3.1.2 3.1.3 3.1.4

Klassifikation von Begriffen nach ihrem logischen Typ Syntax, Formalisierung und Semantik . . . . . . . . Klassifikation von Begriffen nach ihrem Inhaltstyp . Klassifikation von Begriffen nach ihrem Abstufungstyp (Skalentyp) . . . . . . . . . . . . . . 3.1.4.1 Qualitativ-klassifikatorische Begriffe und Nominalskalen . . . . . . . . . . . . . . . 3.1.4.2 Komparative Begriffe und Ordinalskalen . 3.1.4.3 Quantitative Begriffe . . . . . . . . . . . . 3.2 Klassifikation von Sätzen nach dem Inhaltstyp . . . . . . . 3.3 Logische Wahrheit und deduktive Logik . . . . . . . . . . . 3.4 Bedeutungskonventionen und definitorische Wahrheit . . . 3.5 Klassifikation von Sätzen nach ihrem Allgemeinheitsgrad . 3.6 Generelle Sätze, Gesetzesartigkeit, Determinismus und Indeterminismus . . . . . . . . . . . . . . . . . . . . . . . 3.7 Gehalt von Sätzen und Gehaltsarten . . . . . . . . . . . . 3.8 Verifikation, Falsifikation, Bestätigung und Schwächung . . 3.9 Objektive (statistische) und subjektive (epistemische) Wahrscheinlichkeit . . . . . . . . . . . . . . . . . . . . . 3.10 Weiterführende Themen . . . . . . . . . . . . . . . . . . . 3.10.1 Dispositionsbegriffe . . . . . . . . . . . . . . . . . 3.10.2 Herausforderungen an die logisch-definitorischsynthetisch-Unterscheidung . . . . . . . . . . . . . 3.10.3 Relevanz und Irrelevanz in logischen Schlüssen . . 3.10.4 Weiterführendes zur Wahrscheinlichkeit . . . . . . 3.10.4.1 Mathematische Gesetze der Wahrscheinlichkeit . . . . . . . . . . . . 3.10.4.2 Probleme des statistischen und des subjektiven Wahrscheinlichkeitsbegriffs . 3.10.4.3 Principal principle und engste Referenzklasse . . . . . . . . . . . . . . . 3.11 Zusammenfassung, einführende Literatur und Übungen . .

4. Gesetzeshypothesen und ihre empirische Überprüfung . . . . . 4.1 Die Relevanzbedingung . . . . . . . . . . . . . . . . . . 4.1.1 Relevanz von strikten Gesetzen . . . . . . . . . . 4.1.2 Relevanz von statistischen Gesetzen . . . . . . . 4.2 Die empirische Überprüfung von Gesetzeshypothesen auf Wahrheit und Relevanz . . . . . . . . . . . . . . . . . . 4.2.1 Der strikte Fall: die Methode der Übereinstimmung und des Unterschieds . . . . . . . . . . . . . . . 4.2.2 Methodische Induktion: die Findung von strikten Gesetzeshypothesen . . . . . . . . . . . . . . . .

66 70 71 73 74 75 76 79 83 85 89 93 97 98 99 102 102 104 106 109 109 112 115 117

. . . .

120 120 120 123

.

127

.

128

.

131

Inhalt

4.3

4.4 4.5

4.6

4.2.3 Der statistische Fall: die Methode der Akzeptanz- u. Konfidenzintervalle und der Signifikanztests . . . . 4.2.3.1 Überprüfung auf vermutliche Wahrheit – die Methode der Akzeptanzintervalle . . . 4.2.3.2 Auffindung statistischer Hypothesen und Konfindenzintervalle . . . . . . . . . . . . 4.2.3.3 Überprüfung auf vermutliche Relevanz – die Methode der signifikanten Unterschiede 4.2.3.4 Statistische Repräsentativität . . . . . . . . 4.2.3.5 Teststatistik, Inferenzstatistik und die Likelhood-Intuition . . . . . . . . . . . . 4.2.4 Fehlerquellen in der statistischen Methode . . . . . Korrelation und Kausalität . . . . . . . . . . . . . . . . . . 4.3.1 Versteckte Variablen . . . . . . . . . . . . . . . . . 4.3.2 Kausalrichtung . . . . . . . . . . . . . . . . . . . . Die Anwendung statistischer Hypothesen auf den Einzelfall Weiterführende Themen . . . . . . . . . . . . . . . . . . . 4.5.1 Statistische Methoden für quantitative Variablen . . 4.5.2 Die Rechtfertigung der Likelihood-Intuition innerhalb der subjektiven Wahrscheinlichkeitstheorie . . . . . . . . . . . . . . . . . . . . . . . . Zusammenfassung, einführende Literatur und Übungen . .

5. Theorien und ihre empirische Bewertung . . . . . . . . . . . . . 5.1 Theoretische Begriffe und multiple Zuordnungsgesetze . . 5.2 Das Beispiel der Newtonischen Physik . . . . . . . . . . . 5.3 Grundlegendes zur Theorienstatik: Struktur wissenschaftlicher Theorien . . . . . . . . . . . . . . . . . 5.4 Methodische Merkmale (guter) wissenschaftlicher Theorien 5.4.1 Systemcharakter, empirische Kreativität, Globalität und Vereinheitlichung . . . . . . . . . . . . . . . . 5.4.2 Holismus der Bedeutung, des empirischen Gehalts, und der Theorienüberprüfung (Duhem-NeurathQuine-These) . . . . . . . . . . . . . . . . . . . . . 5.4.3 Theoriennetze . . . . . . . . . . . . . . . . . . . . 5.5 Das Beispiel der Piagetschen kognitionspsychologischen Entwicklungstheorie . . . . . . . . . . . . . . . . . . . . . 5.6 Grundlegendes zur Theoriendynamik . . . . . . . . . . . . 5.6.1 Das Lakatos-Modell der Theorienrevision . . . . . . 5.6.2 Theorienbewertung, Theorienvergleich und Theorienfortschritt . . . . . . . . . . . . . . . . . . 5.7 Das Beispiel der Adorno-Milgramschen Theorie des autoritären Charakters . . . . . . . . . . . . . . . . . . . . 5.8 Weiterführende Themen . . . . . . . . . . . . . . . . . . .

133 134 136 137 141 142 143 146 146 152 154 156 156

160 163 166 166 176 184 188 188

189 190 192 196 196 199 208 211

7

8

Inhalt

. .

211

. . . . . .

. . . . . .

213 216 216 217 218 220

6. Erklärung – und was alles dazu gehört . . . . . . . . . . . . . . . 6.1 Das deduktiv-nomologische Modell der Erklärung . . . . . 6.2 Erklärung versus Voraussage und Begründung . . . . . . . 6.2.1 Nicht jeder Glaubensgrund ist ein Realgrund . . . . 6.2.2 Kausalität und Gesetzesartigkeit im Erklärungsrahmen . . . . . . . . . . . . . . . . . . 6.2.3 Die Bedingung der prognostischen Funktion . . . . 6.2.4 Irrelevanz und Redundanz – die logischen Probleme der DN-Erklärung . . . . . . . . . . . . . 6.3 Probabilistische Erklärungsmodelle . . . . . . . . . . . . . 6.3.1 Induktiv-Statistische Erklärungen nach Hempel . . . 6.3.2 Die Bedingung der maximalen Bestimmtheit . . . . 6.3.3 Bedingungen an den Wahrscheinlichkeitswert: konfligierende Intuitionen . . . . . . . . . . . . . . 6.4 Normische Erklärungen und die Erklärung menschlicher Handlungen . . . . . . . . . . . . . . . . . . . . . . . . . 6.5. Weiterführende Themen . . . . . . . . . . . . . . . . . . . 6.5.1 Gesetzesartigkeit . . . . . . . . . . . . . . . . . . . 6.5.1.1 Naturgesetze versus Systemgesetze . . . . 6.5.1.2 Gesetzesartigkeit i. w. S. und kontrafaktische Konditionalsätze . . . . . . . . . . . . . . 6.5.1.3 Gesetzesartigkeit i. e. S. und physikalische Notwendigkeit . . . . . . . . . . . . . . . 6.5.2 Kausalität . . . . . . . . . . . . . . . . . . . . . . . 6.5.2.1 Singuläre und generelle Kausalbeziehung . 6.5.2.2 Kausale Präemption und Überdetermination . . . . . . . . . . . . . 6.5.2.3 Kausale Prozesse . . . . . . . . . . . . . . 6.5.2.4 Kausale Graphen und probabilistische Kausalanalyse . . . . . . . . . . . . . . . 6.5.2.5 Interventionistischer Ansatz . . . . . . . . 6.6 Zusammenfassung, einführende Literatur und Übungen . .

223 223 225 227

5.9

5.8.1 Instrumentalismus und Realismus . . . . . . . . 5.8.2 (Nicht-)Eliminierbarkeit und (Un-)Definierbarkeit theoretischer Begriffe . . . . . . . . . . . . . . 5.8.3 Allgemeine Theorien der Bestätigung . . . . . . 5.8.3.1 Hypothetisch-deduktive Bestätigung . 5.8.3.2 Subjektiv-probabilistische Bestätigung 5.8.3.3 Die Goodman-Paradoxie . . . . . . . Zusammenfassung, einführende Literatur und Übungen

Literaturverzeichnis

. . . . . . . . . . . . . . . . . . . . . . . . . .

227 229 229 230 230 232 233 235 237 237 237 237 238 239 239 240 241 242 243 244 247

Inhalt

Definitionen, Merksätze, und Abbildungen . . . . . . . . . . . . . .

257

Autorenregister

. . . . . . . . . . . . . . . . . . . . . . . . . . . .

259

. . . . . . . . . . . . . . . . . . . . . . . . . . . . . .

263

Lösungen zu ausgewählten Aufgaben . . . . . . . . . . . . . . . . .

267

Sachregister

9

Vorwort Dieses Buch hat sich aus meinen Einführungsvorlesungen und dem zugehörigen Einführungsskriptum in die Wissenschaftstheorie entwickelt. Alle Großkapitel enthalten am Ende kompakt dargestellte weiterführende Themen. Auf diese Weise bezweckt dieses Buch nicht nur eine Einführung zu bieten, sondern zugleich einen Überblick über die verschiedenen Teilgebiete der Wissenschaftstheorie in ihrem Zusammenhang zu liefern. Das Buch wendet sich gleichmaßen an Studierende und Fachleute, gleichermaßen an Philosoph(inn)en und an Einzelwissenschaftler(innen) aller Fachrichtungen. Durch den Brückenschlag zwischen Standardwissenschaftstheorie und Nachbargebieten wie Wertproblematik oder Statistik, und durch ausgesuchte Theorienbeispielen aus Natur- und Sozialwissenschaften versucht dieser Band ein tiefgreifendes Verständnis des komplexen Phänomens ,Wissenschaft‘ zu ermöglichen. Das Buch ist self-contained und somit voraussetzungsfrei lesbar. Einsatz in der Lehre: Lässt man die weiterführenden Themen sowie diverse Details weg, dann entspricht der so beschränkte Stoff einer einsemestrigen zweistündigen Einführungsvorlesung. Nimmt man Details und weiterführende Themen hinzu, so füllt der Stoff zwei aufeinander aufbauende zweistündige Einführungsvorlesungen. Der Kernstoff dieser Einführung wird durch Definitionen und Merksätze besonders hervorgehoben. Am Ende jedes Großkapitels finden sich einer Kurzzusammenfassung, Hinweise zu einführender Literatur sowie Fragen und Aufgaben. Das Gesamtliteraturverzeichnis folgt am Schluss, nebst Verzeichnissen und Registern. Nummerierungskonventionen: Die Nummerierung von Abbildungen erfolgt nach folgendem Muster: „Abb. KapitelNr-Nr“. Beispiel: „Abb. 2.1-2“ ist die 2. Abbildung von Kap. 2.1. Analog werden Definitionen, Merksätze und Hervorhebungen (eingerückte Textstellen) mit „KapNr-Nr“ nummeriert. Beispiel: „(Def. 4.2-2)“ ist die 2. Definition von Kapitel 4.2. „(3.1-4)“ ist die 4. Hervorhebung von Kap. 3.1. Für wertvolle Hilfe bedanke ich mich bei Georg Dorn, Axel Bühler, Veronika Linke, Marc Breuer, Eckhart Arnold, Hannes Leitgeb, Wolfgang Spohn und Reinhard Kleinknecht. Bleibt mir nur noch, den geschätzten Leserinnen und Lesern viel Freude und Gewinn bei der Lektüre dieses Buches zu wünschen. Anmerkung zur 4. Auflage: Wie in der 3. Auflage wurden in der 4. Auflage Literaturaktualisierungen vorgenommen und kleine Fehler berichtigt. Merksatz (Ms. 6.5-1) zu kausalen Graphen wurde wesentlich verbessert. Düsseldorf 10.10.2013

Gerhard Schurz

1. Einleitung 1.1 Aufgaben und Zielsetzungen der Wissenschaftstheorie Die Wissenschaft bestimmt heutzutage in hohem Maße unser gesellschaftliches Leben. Inwiefern und inwieweit können wir den Resultaten der Wissenschaft vertrauen? Es gibt eine wissenschaftliche Disziplin, welche Fragen wie diese systematisch untersucht: die Wissenschaftstheorie. Sie ist jene Wissenschaftsdisziplin, welche die Funktionsweise wissenschaftlicher Erkenntnis untersucht, ihre Zielsetzungen und ihre Methoden, ihre Leistungen und ihre Grenzen. So wie die Wissenschaften selbst hat sich auch die Wissenschaftstheorie aus der Philosophie heraus entwickelt und wird heute arbeitsteilig sowohl von Wissenschaftsphilosophen wie von Einzelwissenschaftlern betrieben. Man unterscheidet zwischen allgemeiner und spezieller Wissenschaftstheorie. Spezielle Wissenschaftstheorien sind auf einzelne Disziplingattungen bezogen wie z. B. Physik, Biologie, Psychologie oder Human- und Sozialwissenschaften. Die allgemeine Wissenschaftstheorie fragt nach jenen Erkenntnisbestandteilen, die allen Wissenschaftsdisziplinen mehr oder weniger gemeinsam sind. Ihre Hauptfragen sind die folgenden: (i) (ii) (iii) (iv) (v) (vi)

wie ist eine wissenschaftliche Sprache aufgebaut? was sind die Regeln für die Gültigkeit eines Argumentes? was zeichnet eine wissenschaftliche Beobachtung aus? worin besteht eine Gesetzeshypothese, und worin eine Theorie? wie werden Gesetzeshypothesen und Theorien empirisch überprüft? was leistet eine wissenschaftliche Voraussage, was eine Kausalerklärung?

Die vorliegende Einführung widmet sich diesen Fragen der allgemeine Wissenschaftstheorie. Zugleich werden aber auch Anwendungen auf Themen spezieller Wissenschaftstheorien behandelt und charakteristische Unterschiede zwischen einzelnen Wissenschaftssparten untersucht. Zu den allgemeinsten Fragen der Wissenschaftstheorie gehören die folgenden: (vii) gibt es eine objektive Wahrheit bzw. eine objektiv erkennbare Realität? (viii) welcher Zusammenhang besteht zwischen Wissenschaft und Werturteilen? In Frage (vii) geht Wissenschaftstheorie in Erkenntnistheorie über (s. Kap. 2.2), und in Frage (viii) geht Wissenschaftstheorie in Metaethik über (s. Kap. 2.5). Neben ihrer grundsätzlichen Bedeutung, welche in der Beantwortung der erläuterten Fragen liegt, besitzt die Wissenschaftstheorie eine Reihe von bedeutenden Anwendungen, innerhalb sowie außerhalb der Wissenschaften.

Hauptfragen der allgemeinen Wissenschaftstheorie

12

1. Einleitung Anwendungen der Wissenschaftstheorie

Die wissenschaftsinternen Anwendungen der Wissenschaftstheorie liegen unter anderem in der Lieferung von Grundlagen- und Methodenwissen, welches Entscheidungshilfen für kontroverse oder neue einzelwissenschaftliche Fragen zur Hand gibt, weiters in der Herausarbeitung interdisziplinärer Gemeinsamkeiten, ferner in der Vermittlung argumentativer Kompetenz und Kritikfähigkeit, und nicht zuletzt in der Rolle der Wissenschaftstheorie als Wegbereiterin für neue Wissenschaftsdisziplinen. Unter den wissenschaftsexternen Anwendungen der Wissenschaftstheorie auf gesellschaftliche Problemzusammenhänge seien zwei besonders hervorgehoben: (a) Das wissenschaftstheoretische Abgrenzungsproblem ist von hoher gesellschaftlicher Bedeutung. Es besteht in der Frage, welche Teile unseres Ideengutes den Status objektiv-wissenschaftlicher Erkenntnis beanspruchen dürfen, im Gegensatz zu subjektiven Werthaltungen, parteilichen Ideologien oder religiösen Überzeugungen. Nur allgemeinverbindliche Erkenntnisse sollen gemäß dem Grundkonsens demokratisch-säkularer Informationsgesellschaften in staatlichen Bildungseinrichtungen unterrichtet werden. Brisant wurde diese Frage z. B. in der Auseinandersetzung mit der Bewegung des Kreationismus in den USA: so berief sich die berühmte Entscheidung des Richters W. R. Overton von 1981 auf Abgrenzungskriterien von Wissenschaft gegenüber religiösem Glauben (s. Bird 1998, 2 – 9). (b) Ebenso bedeutend ist die Funktion von wissenschaftstheoretischer Aufklärung, um der Gefahr des ideologischen Missbrauchs von Wissenschaft und ihren Resultaten entgegenzuwirken. Politiker, Medien und Wirtschaftsvertreter berufen sich gerne auf Expertenwissen, welches dabei leider nicht selten für vorgefasste Zwecke einseitig oder verfälscht dargestellt wird (s. dazu Kap. 4.2.4 – 4.4).

1.2 Philosophische Positionen in der Wissenschaftstheorie 1.2.1 Empirismus und Rationalismus. Obgleich die Bezeichnung „Wissenschaftstheorie“ erst im 20. Jahrhundert eingeführt wurde, ist die Disziplin der Wissenschaftstheorie so alt wie die Wissenschaften selbst und hat sich mit deren Entwicklung kontinuierlich mitentwickelt (s. Losee 1977). Die Geschichte der Wissenschaftstheorie beginnt mit Aristoteles (384 – 322 v. Chr.), dem großen Wissenssystematisierer der Antike. Aristoteles war wesentlich erfahrungsorientierter als sein Lehrer Platon. Dennoch war auch Aristoteles, so wie die meisten Philosophen nach ihm, ein Anhänger des sogenannten fundamentalistischen Erkenntnisprogramms. In diesem Erkenntnisprogramm geht man davon aus, dass echtes Wissen nur möglich ist, wenn es auf einem Fundament von sicheren und notwendigen Prinzipien ruht, welche nicht durch unsichere Erfahrung, sondern durch rationale Intuition gewonnen werden (vgl. Albert 1980, 11 – 18). Aristoteles sprach in diesem Zusammenhang von „intuitiver Induktion“ (s. Losee 1977, 16 f.). In der gegenwärtigen Erkenntnis- und Wissenschaftstheorie hat sich dagegen

1.2 Philosophische Positionen in der Wissenschaftstheorie

das fallibilistische Erkenntnisprogramm durchgesetzt, welches davon ausgeht, dass unsere Erkenntnis der Realität grundsätzlich fehlbar ist und wissenschaftliches Wissen zwar mehr oder weniger gut bestätigt, aber niemals irrtumssicher sein kann. Gestützt auf die großartigen Erfolge der naturwissenschaftlichen Methode in der Neuzeit – mit Pionieren wie z. B. Galileo Galilei (1564 – 1642), Isaac Newton (1642 – 1727) oder Charles Darwin (1809 – 1882) – hat sich in der Philosophie die einflussreiche Strömung des Empirismus etabliert, von Francis Bacon (1561 – 1626) und John Locke (1632 – 1704) bis zu David Hume (1711 – 1776) und John Stuart Mill (1806 – 1873). Zum anderen hat sich das fundamentalistische Erkenntnisprogramm in der Strömung des Rationalismus und verwandten Richtungen weiterentwickelt – von René Descartes (1596 – 1650) über Gottfried W. Leibniz (1646 – 1716) bis zu Immanuel Kant (1724 – 1804). Den grundlegenden Unterschied zwischen den empiristischen und den rationalistischen Strömungen kann man so charakterisieren. Für Empiristen sind jene Sätze, welche sich apriori – also allein durch den Verstand und mit rationaler Gewissheit – begründen lassen, eingeschränkt auf die sogenannten analytischen Sätze, deren Wahrheit auf Logik und begrifflichen Konventionen beruht. Solche Sätze besitzen keinen Realgehalt – sie sagen nichts über die wirkliche Welt aus; dies tun nur synthetische Sätze. Für Rationalisten gibt es dagegen auch apriorisch begründbare Sätze mit Realgehalt, sogenannte synthetische Sätze apriori. Doch sowohl Descartes‘ wie Kants Versuche, apriorische Prinzipien der Erfahrungswissenschaft zu begründen, wurden von der weiteren Entwicklung der Naturwissenschaft widerlegt. Die Entwicklung der modernen Wissenschaftsphilosophie ist auf das engste mit der Einsicht in die Uneinlösbarkeit des fundamentalistischen Erkenntnisprogramms verbunden. Auch die klassischen Systeme des philosophischen Empirismus trugen lange Zeit reduktionistische und fundamentalistische Züge. Die skeptischen und zur epistemischen Bescheidenheit aufrufenden Konsequenzen des Empirismus wurden erst von Hume konsequent ausformuliert (s. dazu Kap. 2.6.1, 6.5.1.3). Hume zeigte, dass die zwei Kernstücke der wissenschaftlichen Methode, das Kausalitätsprinzip und das Induktionsprinzip, weder logisch noch empirisch begründbar sind, und dieses Problem sollte die Philosophie bis in die heutigen Tage beschäftigen. Die anscheinende Begründungsinsuffizienz des Empirismus gab rationalistischen Nachfolgeströmungen wieder Auftrieb, in welchen das fundamentalistische Erkenntnisprogramm jedoch nach und nach aufgegeben bzw. durch pragmatisierte oder historisierte ,Rationalismen‘ ersetzt wurde (s. Kap. 1.2.5). Im 20. Jahrhundert haben sich post-empiristische und post-rationalistische Ansätze einander beträchtlich genähert, und in diesem Spannungsfeld hat sich auch die gegenwärtige Wissenschaftstheorie entwickelt. 1.2.2 Logischer Empirismus. Zu den wichtigsten Entstehungsursachen der modernen Wissenschaftstheorie und Analytischen Philosophie gehört der logische Empirismus, der insbesondere vom Wiener Kreis (aber auch z. B. vom nahestehenden Berliner Kreis um Hans Reichenbach, u. a. m.) entwickelt wurde. Beim Wiener Kreis handelte es sich um eine Gruppe von Ein-

Das fallibilistische Erkenntnisprogramm

Empirismus versus Rationalismus

Wiener Kreis

13

14

1. Einleitung

Empirischer Reduktionismus

zelwissenschaftlern und Philosophen in Wien, deren Kern Moritz Schlick (1882 – 1936), Otto Neurath (1882 – 1945) und Rudolf Carnap (1891 – 1970) bildeten. Anknüpfend an Ernst Mach (1838 – 1916) bemühte sich diese Gruppe um eine Neubegründung des Empirismus und der wissenschaftlichen Philosophie insgesamt (s. Stadler 1997, Schurz 2003). Das Neuartige ihrer Situation war die Entwicklung der modernen Logik, die erst Ende des 19. Jahrhunderts einsetzte. Durch die moderne Logik wurden beliebige sprachliche Erkenntnissysteme mit mathematischer Präzision darstellbar, und so war es die Hoffnung des Wiener Kreises, nun endlich das methodische Rüstzeug für eine wissenschaftlich fortschreitende Philosophie gefunden zu haben (vgl. Schlick 1930/31, 5 f.). Mitte der 1930er Jahre musste sich der in politischer Hinsicht linksliberal bis sozialistisch orientierte Wiener Kreis unter dem Druck der Nationalsozialisten auflösen. Die Mehrheit seiner Mitglieder emigrierte teils über Umwege in die USA, wo sich die logisch-empiristische Bewegung in Vereinigung mit verwandten angloamerikanischen Richtungen zur Analytischen Philosophie weiterentwickelte und in der Nachkriegszeit auch in Europa Fuß fasste. Was die heutige Wissenschaftstheorie vom logischen Empirismus lernen kann, sind weniger bestimmte Einzelthesen als die hohen Standards begrifflicher und argumentativer Genauigkeit. In ihrer Phase bis 1935 war die Wissenschaftsphilosophie des logischen Empirismus positivistisch und reduktionistisch verengt. In der späteren Phase haben die logischen Empiristen ihre verengten Positionen nach und nach verworfen und durch Thesen ersetzt, auf die die Bezeichnung „Empirismus“ oder „Positivismus“ im üblichen Sinn nicht mehr zutrifft. In einem Punkt hatte sich der logische Empirismus sehr früh von seinen klassischen Vorgängern gelöst: in der Zurückweisung der Infallibilität von Beobachtungssätzen. In der sogenannten Protokollsatzdebatte setzte sich – nicht bei Schlick, wohl aber bei Neurath und Carnap – die Ansicht durch, dass auch Beobachtungssätze wie „dort ist ein Tisch“ prinzipiell fehlbar sind. Der Empirismus besteht in dieser Sicht nur mehr darin, dass Beobachtungssätzen im Gesamtsystem der Erkenntnis eine epistemisch bevorzugte Rolle zukommt (vgl. Carnap 1932/33, Neurath 1934, 113). Am empirischen Reduktionismus wurde dagegen noch lange Zeit festgehalten. Der klassische Empirismus vertrat folgende Reduktionsthese: alle ,seriösen‘ wissenschaftlichen Begriffe müssen durch Definitionsketten auf Beobachtungsbegriffe zurückführbar sein. Dieses Reduktionsprogramm wurde vom logischen Empirismus in seiner Frühphase übernommen (Carnap 1928). Erst später setzte sich die Ansicht durch, dass theoretische Begriffe wie „Kraft“, „elektrisches Feld“ oder „menschlicher Charakter“ nicht durch Beobachtungsbegriffe definierbar sind, sondern weit über das unmittelbar Beobachtbare hinausgehen (Carnap 1956; Hempel 1951). In der sogenannten Standardwissenschaftstheorie der 1960er Jahre war das klassische empiristische Erkenntnismodell bereits aufgegeben. Übrig blieb ein minimaler Empirismus, der in der Forderung bestand, dass wissenschaftliche Theorien empirische Konsequenzen haben müssen, an denen sie überprüft werden können. Damit wurden zwei Abgrenzungslinien durchlässig, an denen der frühe logische Empirismus festzuhalten versuchte. Erstens wurde die Abgrenzung

1.2 Philosophische Positionen in der Wissenschaftstheorie

zwischen Wissenschaft und Metaphysik durchlässig: denn es stellt sich heraus, dass Prinzipien, die isoliert betrachtet empirisch konsequenzenlos sind, im Verein mit anderen theoretischen Sätzen neue empirische Konsequenzen erzeugen können (vgl. Hempel 1951, § 4; Stegmüller 1970, 293 – 295; s. Kap. 5.4). Auf diese Weise gewann die postpositivistische Wissenschaftstheorie neuen Zugang zur Diskussion metaphysischer Prinzipien wie Realismus und Kausalität. Zweitens wurde die Abgrenzung zwischen analytischen und synthetischen Sätzen durchlässig, denn die Bedeutung theoretischer Begriffe wird durch die gesamte jeweilige Hintergrundtheorie bestimmt (Carnap 1956; Quine 1951). Verbunden damit brach auch die Einengung der Analytischen Philosophie auf sprachinterne Fragen zusammen, die Carnap (1950a) vertreten hatte. Quine (1960) entwickelte dagegen eine naturalistische Auffassung, derzufolge es einen kontinuierlichen Übergang gibt zwischen Erfahrungswissenschaft und Wissenschaftsphilosophie. 1.2.3 Kritischer Rationalismus. Assoziiert mit dem Wiener Kreis war auch Karl Popper (1902 – 1994), der Begründer des sogenannten kritischen Rationalismus. Poppers Wissenschaftstheorie trug von Anbeginn an jene anti-reduktionistischen Züge, zu denen sich der logische Empirismus erst nach Jahren der Wandlung durchgerungen hatte. Wissenschaftliche Theorien können nach Popper beliebig weit über die Erfahrung hinausgehen, wenn sie nur an ihr überprüfbar sind. Überprüfung sollte sich nach Popper in Form von möglichst strengen Falsifikationsversuchen vollziehen. Dabei berief sich Popper auf die logische Asymmetrie von Verifikation und Falsifikation bei strikten (ausnahmslosen) Allsätzen wie z. B. „alle Metalle leiten Strom“. Solche Gesetzeshypothesen können durch keine endliche Menge von Beobachtungen verifiziert werden, aber bereits durch ein einziges Gegenbeispiel falsifiziert werden (Popper 1935/76, Kap. I – IV). Für Popper war Falsifizierbarkeit das entscheidende Abgrenzungskriterium zwischen Wissenschaft und Spekulation: Wissenschaftliche Theorien sind nicht verifizierbar, aber sie müssen falsifizierbar sein. Wenn sie nur einmal falsifiziert wurden, dann werden sie als falsch ausgeschieden; haben sie aber viele Falsifikationsversuche erfolgreich bestanden, dann gelten sie als bewährt. Auch Poppers Falsifikationismus war in der Folgediskussion mannigfacher Kritik ausgesetzt. Lakatos (1974) zeigte auf, dass wissenschaftliche Theoriensysteme so gut wie nie aufgrund eines einzigen Gegenbeispiels verworfen werden, sondern zunächst durch ad hoc Modifikationen gegenüber widerspenstigen Erfahrungsdaten immunisiert werden (s. Kap. 5.6.1). Ebenfalls harter Kritik ausgesetzt war Poppers Antiinduktivismus – d. h. seine These, Wissenschaft könne gänzlich ohne Induktion auskommen (s. Kap. 2.6.2). Poppers kritischer Rationalismus ist von einem klassischen Rationalismus weit entfernt. Nirgendwo wird in Poppers Philosophie behauptet, man könne Realerkenntnis durch erfahrungsunabhängige apriori Intuition begründen; im Gegenteil hatte dies Popper immer abgelehnt. Somit ist Poppers Philosophie ebenso postrationalistisch wie der späte logische Empirismus postpositivistisch ist. Stärker als der späte logische Empirismus hat Popper allerdings betont, dass Beobachtungssätze nicht bloß fehlbar sind, sondern

Abgrenzung Wissenschaft – Metaphysik

Falsifizierbarkeit als Abgrenzungskriterium

15

16

1. Einleitung

auch theoriebeladen sind, sodass die Grenze zwischen Beobachtungs- und theoretischen Begriffen nicht scharf gezogen werden könne (1935/76, 73 – 76; Neuer Anhang X 374 ff.). Mit dieser Argumentation kam der kritische Rationalismus dem Angriff des Relativismus unwillentlich ein Stück weit entgegen.

Wissenschaftliche Paradigmen

Wissenschaftliche Revolutionen

1.2.4 Historische Wissenschafstheorie und Relativismus. Ende der 1950er Jahre wurde vom Wissenschaftshistoriker Thomas Kuhn (1967) ein fundamentaler Angriff auf die Standardwissenschaftstheorie vorgetragen, der diese noch weiter verunsichern sollte. Faktische Wissenschaft, so Kuhn, verhält sich anders als es ihr die Wissenschaftstheoretiker ,vorschreiben‘ wollen. Das von Kuhn entwickelte alternative Wissenschaftsmodell ist eher historisch-soziologisch als logisch-kognitiv angelegt. Kuhn zufolge vollzieht sich Wissenschaftsentwicklung auf der Grundlage sogenannter Paradigmen, wie etwa das Paradigma der klassischen (Newtonschen) Physik oder das der (Darwinschen) Evolutionstheorie. Ein Kuhnsches Paradigma enthält zumindest folgende drei Komponenten: (i) sehr allgemeine theoretische Prinzipien oder Modellvorstellungen, (ii) Musterbeispiele erfolgreicher Anwendungen, und (iiii) methodologisch-normative Annahmen (s. Kuhn 1977, Hoyningen-Huene 1989, Schurz 1998a). Das wissenschaftssoziologische Korrelat des Paradigmas ist die Scientific Community, eine Gemeinschaft von Fachexperten, welche an einem Paradigma festhält und an seiner Weiterentwicklung arbeitet. Kuhn zufolge bestimmt ein Paradigma nicht nur die grundlegenden Prinzipien und Problemstellungen, nicht nur die Interpretation der Beobachtungsdaten – nein, es bestimmt sogar die Beobachtungsdaten selbst, denn alle Beobachtung ist theoriegeladen: es gibt nach Kuhn keine theorie- bzw. paradigmenneutrale Beobachtung. Diese starke These übernimmt Kuhn von Hanson (1958). Wissenschaftsentwicklung vollzieht sich Kuhn zufolge in zwei sich ablösenden Phasen, einer normalwissenschaftlichen und einer revolutionären Phase. Die gemeinsame Akzeptanz eines Paradigmas ermöglicht in der normalwissenschaftlichen Phase kontinuierlichen Wissensfortschritt. Wenn sich widerspenstige Daten, sogenannte Anomalien, einer kohärenten Erklärung durch das Paradigma widersetzen, werden diese Konflikte durch mehr oder minder ad hoc vorgenommene Modifikationen der aktuellen Theorieversionen innerhalb des akzeptierten Paradigmas bereinigt. Häufen sich jedoch solche Anomalien, so beginnen jüngere Gelehrte nach einem neuen Paradigma zu suchen. Sobald ein solches gefunden ist, tritt die Wissenschaftsentwicklung für eine gewisse Zeit in eine revolutionäre Phase ein, in der zwei Paradigmen um die Vorherrschaft kämpfen. Als Beispiele führt Kuhn (1967) den Übergang von der ptolemäischen zur kopernikanischen Astronomie oder den von der Newtonschen zur Einsteinschen Physik an. Da während eines Wechsels des Paradigmas jedoch alle gemeinsamen Rationalitätsstandards weggefallen sind und alle bisherigen Erfahrungsdaten neu ,gesehen‘ werden, sind die zwei konkurrierenden Paradigmen, gemäß Kuhns ,berüchtigter‘ Inkommensurabilitätsthese, rational unvergleichbar, und der Kampf um die Vorherrschaft findet anstatt in Form eines kognitiven Leistungsvergleichs in der Form eines wissenschaftspolitischen Machtkampfes statt, in dem die Anhänger des alten Paradigmas schließlich aussterben,

1.2 Philosophische Positionen in der Wissenschaftstheorie

wodurch sich das neue Paradigma durchsetzt und eine neue normalwissenschaftliche Phase einläutet. Kuhns alternatives Wissenschaftsmodell war in den folgenden Jahrzehnten einer kontroversen Diskussion ausgesetzt. Neben einer Gegnerschaft bildete sich eine eher gemäßigte und eine eher radikale Richtung von Kuhnianern heraus (vgl. Hoyningen-Huene 1989, 207 f.). Die radikalere Richtung gab dem Relativismus starken Auftrieb – am deutlichsten ausgeprägt in Feyerabends ,Anarchistischer Erkenntnistheorie‘ (1976). Dem Argument der Theorieabhängigkeit wissenschaftlicher Beobachtungen und seinen relativistischen Konsequenzen konnte die Standardwissenschaftstheorie nichts Wirksames entgegensetzen. Denn sie hatte ja alle inhaltlichen Fragen über die Natur dessen, was Beobachtung sei, aus der verengten Perspektive der Wissenschaftslogik verbannt. Für Popper war die Frage, was als Beobachtungssatz zugelassen wird, wissenschaftslogisch eine Sache reiner Konvention (1935/76, 71 – 74), und nicht viel anders sahen es die logischen Empiristen. In Kap. 2.7.2 wird sich zeigen, dass sich ein anderes Bild ergibt, sobald man eine empirisch-kognitive Perspektive einnimmt. Letztendlich: würde es keinen Unterschied geben zwischen dem, was die Erfahrung zeigt, und dem, was man theoriegelenkt vermutet, so wäre Erfahrungswissenschaft eine permanente Selbsttäuschung. Einer Theorie der Erfahrungswissenschaften, welche diesen zentralen Unterschied nicht erklären kann, haftet etwas äußerst Unbefriedigendes an.

1.2.5 Weitere Positionen in Kürze 1.2.5.1 Pragmatische Wissenschaftstheorie. In den 1980er Jahren sprachen mehrere Autoren von einer pragmatischen Wende der Wissenschaftstheorie (Stegmüller 1983, Kap. XI). Leider aber zerfällt die pragmatische Philosophie und die Begriffsverwendung von „Pragmatik“ insgesamt in zwei entgegengesetzte Lager. Während das eine Lager (z. B. Ch. S. Peirce, Rescher 1998, van Fraassen 1980, Schurz 1983) Pragmatik im erkenntnisinternen Sinne versteht, fasst das andere Lager (z. B. W. James, Rorty 1982, Stich 1990, evtl. Putnam 1995) Pragmatik in einem erkenntnisexternen Sinn auf. Für das erkenntnisexterne Lager haben die pragmatischen Komponenten von Erkenntnis nichts mit ihrer Wahrheit zu tun. Für das erkenntnisinterne Lager sind es dagegen gerade diese pragmatischen Komponenten, welche den Begriff der Wahrheit in einem nicht-zirkulären Sinn etablieren sollen. Während das erkenntnisexterne Lager Erkenntnis in Abhängigkeit von beliebigen Interessen setzt und darin ein zwingendes Argument für die subjektive Relativität von Erkenntnis erblickt, liefert für das interne Lager der Bezug auf erkenntnisinterne Zwecke erst die Möglichkeit eines zirkelfreien Erkenntnisaufbaus. Die beiden Lager sind derart verschieden in ihrem Pragmatikverständnis, dass man sie nicht unter einen Begriff subsumieren sollte. Aus diesem Grund vermeide ich heutzutage, im Gegensatz zu früheren Arbeiten, die Bezeichnung „pragmatisch“. 1.2.5.2 Metaphysischer Realismus. Als Reaktion auf die Probleme des logisch-empiristischen Wissenschaftsmodells haben etliche Wissenschaftsphilosophen vorgeschlagen, wieder zu jenen metaphysischen Ansätzen des

Interne versus externe Pragmatik

17

18

1. Einleitung

Theorieabhängigkeit der Bedeutung und Referenz

Realismus und der Notwendigkeit zurückzukehren, von denen sich die Wissenschaftstheorie des frühen 20. Jahrhunderts eigentlich befreien wollte. Beispielsweise ist es eine Konsequenz der postpositivistischen Wissenschaftstheorie, dass der Begriff „Masse“ in der Newtonschen und in der Einsteinschen Physik etwas verschiedenes bedeutet. Diese Theorieabhängigkeit war auch eine Hauptstütze für Kuhns Inkommensurabilitätsthese. Gegen diese Konsequenz hat der frühere Putnam (z. B. 1979, 27 ff., 55 f.), zusammen mit Kripke (1972, 55 – 59), folgende Argumentation entwickelt: Wissenschaftsfortschritt im Sinn einer realistischen Wahrheitsannäherung sei überhaupt nur möglich, wenn die Referenz wissenschaftlicher Begriffe, d. h. ihr Bezug zur Realität, starr fixiert sei, so dass diese Referenz in allen möglichen Theorien bzw. möglichen Welten mit metaphysischer Notwendigkeit immer dieselbe bleibt. Wie aber eine Reihe von Argumenten zeigt, reicht das erfahrungsbezogene Wissen oft nicht aus, um eine Fixierung der Referenz theoretischer Begriffe zu garantieren (vgl. Bird 1998, 108 – 120; Ladyman et al. 2007). Klassifikationssysteme für wissenschaftliche (z. B. chemische oder biologische) Arten sind vom wissenschaftlichen Hintergrundwissen abhängig. Die Vorstellung, eine notwendige Beziehung zwischen Sprache und Realität könne erfahrungsunabhängig etabliert werden, ist letztlich ein Rückfall in ein fundamentalistisches Erkenntnismodell. Der spätere Putnam (1977, 1990) hat übrigens seine frühere metaphysischrealistische Position wieder aufgegeben. 1.2.5.3 Strukturalistische Wissenschaftstheorie. In der Hoffnung, der Kuhnschen Herausforderung besser begegnen zu können, hat Stegmüller (1973a) vorgeschlagen, den bisherigen „statement view“ der Standardwissenschaftstheorie durch einen sogenannten „non statement view“ zu ersetzen. Dahinter verbirgt sich die auf Suppes (1957, 246 ff.) und Sneed (1971) zurückgehende Idee, wissenschaftliche Theorien besser als mengentheoretische Modellsysteme anstatt als Aussagensysteme einer formalen Sprache zu rekonstruieren. Solche Modellsysteme werden auch Strukturen genannt: daher die Bezeichnung „strukturalistische“ Wissenschaftstheorie. Während Stegmüller in (1973a, z. B. 119, 134) darin eine radikale Umwälzung erblickt, sieht er später im Übergang von logischen zu mengentheoretischen Formalisierungsmethoden nur mehr einen „pragmatisch-psychologischen Grund“ (1986, 25). Denn auch mengentheoretische Modelle werden durch Aussagen einer mengentheoretischen Sprache dargestellt, weshalb zwischen dem ,statement view‘ und dem ,non statement view‘ einfache Übersetzungsmöglichkeiten bestehen (s. French 2008; Schurz 2013b). In der Detailrekonstruktion einzelwissenschaftlicher Theorien hat die strukturalistische Wissenschaftstheorie eindrucksvolle Leistungen erbracht. 1.2.5.4 Naturalismus und kognitive Wende. Für Quine wird naturalisierte Erkenntnistheorie zu einer empirischen Disziplin, die „wissenschaftlich untersucht, wie der Mensch zur Wissenschaft kommt“ (Quine 1976, 17). Von einer Reihe jüngerer Erkenntnis- und Wissenschaftstheoretiker wurde das naturalistische Programm emphatisch aufgegriffen (vgl. Papineau 1993; Kornblith 1994). Eine erste Konkretisierung des Naturalismus ist die evolutionäre Erkenntnistheorie (z. B. Campbell 1984): eine Reihe von Aspekten unserer Erkenntnis konnten durch evolutionäre Betrachtungen aufgeklärt

1.2 Philosophische Positionen in der Wissenschaftstheorie

werden. Kritisch ist anzumerken, dass naturalistische Ansätze selbst gewisse begründungsbedürftige erkenntnistheoretische Voraussetzungen machen, z. B. bzgl. Realismus und Induktion, und daher die klassische Erkenntnistheorie nicht zur Gänze ersetzen können. Eine zweite Konkretisierung des Naturalismus ist die sogenannte kognitive Wende. Sie war Ausdruck davon, dass sich mittlerweile die Kognitionswissenschaften (Cognitive Sciences) als eigenes interdisziplinäres Fachgebiet herausgebildet hatten, in welchem die Grundlagen des menschlichen Erkenntnisvermögens nicht nur logisch-philosophisch, sondern auch empirisch-psychologisch im Rahmen der Kognitionspsychologie (z. B. Anderson 2001), und algorithmisch-computermodelliert im Rahmen der Künstlichen Intelligenzforschung untersucht wurden (z. B. Thagard 1999). In der Folge begannen eine Reihe von Philosophen vermehrt, kognitionswissenschaftliche Fragestellungen und Ansätze in die Erkenntnis- und Wissenschaftstheorie einzubringen. 1.2.5.5 Radikaler Konstruktivismus. Der ,radikale‘ Konstruktivismus wurde von Entwicklungspsychologen wie Glasersfeld (1985) und Biologen wie Maturana und Varela (1984) entwickelt. Seine zentrale Argumentation besteht darin, vom Konstruktcharakter unserer Wahrnehmungen und Vorstellungen von der Wirklichkeit darauf zu schließen, dass es keine erkennbare Wirklichkeit gibt, die an sich gegeben wäre. In Kap. 2.7.1 wird zu zeigen versucht, dass diese Argumentation auf einem Fehlschluss beruht. 1.2.5.6 Hermeneutik und Kritische Theorie: Die im 19. Jahrhundert entstandene Philosophie der Geisteswissenschaften geht auf Friedrich Schleiermacher (1768 – 1834), Wilhelm Dilthey (1833 – 1911) und Wilhelm Windelband (1848 – 1915) zurück. Sie entstand als methodische Abgrenzungsbewegung der Geisteswissenschaften gegenüber den expandierenden Naturwissenschaften. Im Zentrum dieser Philosophie steht der Versuch, die Hermeneutik als Lehre des zwischenmenschlichen Verstehens für den Zweck einer solchen methodischen Abgrenzung heranzuziehen. Der Methodendualismus der Philosophen der Geisteswissenschaften wurde zu folgender These zugespitzt: in den Naturwissenschaften erklären wir, in den Geisteswissenschaften verstehen wir. Während naturwissenschaftliche Erklärungen auf allgemeinen Gesetzeshypothesen beruhen, würde geisteswissenschaftliches Verstehen ohne Rekurs auf Gesetzeshypothesen vor sich gehen, denn Geistig-Seelisches unterliegt keinen strengen Gesetzesmechanismen, und Verstehen hat direkten Zugang zum menschlichen Geist. Die Hermeneutik hat eine weit vor Schleiermacher zurückreichende Tradition, in der von einer rigiden Abgrenzung zu Logik und Naturwissenschaft nicht die Rede ist (s. Scholz 2001, Teil I.B). Der Methodendualismus des 19. Jahrhunderts hat auch die Methodendebatte im 20. Jahrhundert stark beeinflusst (vgl. Apel 1979 versus Haussmann 1991). Die human- und sozialwissenschaftliche Kontroverse über quantitative versus qualitative Methoden ist teilweise eine Wiederauflage dieses alten Methodenstreites (vgl. Lamnek 1988). Seitens der analytischen Wissenschaftstheorie wurden demgegenüber eine Reihe von Ansätzen entwickelt, in denen die Einheit von Hermeneutik und erfahrungswissenschaftlicher Methode herausgearbeitet wird (Bühler 2003; sowie Kap. 6.4). Die von Horkheimer, Adorno, Marcuse, Habermas entwickelte kritische Theorie kombiniert die methodendualistische

Kognitive Wende

Philosophie der Geisteswissenschaften

Methodendualismus

19

20

1. Einleitung

Variante der Hermeneutik mit ihrer Lehre von der unvermeidlichen Interessensgebundenheit aller Erkenntnis (z. B. Habermas 1968, Apel 1979). Diese Lehre befindet sich im Widerspruch zur Forderung der Wertneutralität von Wissenschaft, die in Kap. 2.5 behandelt wird.

1.3 Zusammenfassung, einführende Literatur und Übungen 1.3.1 Zusammenfassung (Auswahl). Die Wissenschaftstheorie untersucht die Funktionsweise, die Ziele und Methoden, sowie die Leistungen und Grenzen der Wissenschaften. Neben ihrer wissenschaftsinternen Bedeutung besitzt sie auch gesellschaftliche Anwendungen, wie z. B. das Abgrenzungsproblem von Wissenschaft gegenüber bloßem Glaube, und das Aufklärungsproblem gegenüber ideologischem Missbrauch von Wissenschaft. 1.3.2 Einführende Literatur. Eine ausgezeichnete Einführung in die Geschichte der Wissenschaftstheorie bietet Losee (1977). Umfassend über die Geschichte des logischen Empirismus informiert Stadler (1997). Zur Wissenschaftstheorie Poppers siehe Keuth (1998, Hg.). Zur Philosophie Kuhns s. Hoyningen-Huene (1989). Weiterführendes zum Paradigmenbegriff in Schurz/Weingartner (1998, Hg.). Zum wissenschaftstheoretischen Realismus s. Leplin (1984). 1.3.3 Fragen und Aufgaben Zu Kap. 1.1: Was sind die sechs Hauptfragen der allgemeinen Wissenschaftstheorie? Zu Kap. 1.2: 1) Worin besteht der Hauptunterschied zwischen empiristischen und rationalistischen Denkrichtungen? 2) Was versteht man unter empirischem Reduktionismus? 3) Was besagt das Abgrenzungskriterium des kritischen Rationalismus? 4) Diskutieren Sie Kuhns Inkommensurabilitätsthese!

2. Gemeinsame Grundlagen der Wissenschaften 2.1 Methode und Voraussetzungen der Wissenschaftstheorie Prima facie lassen sich zwei gegensätzliche Auffassungen zur Aufgabenstellung und Methode der Wissenschaftstheorie unterscheiden: 1.) Der normativen Auffassung zufolge hat Wissenschaftstheorie die Aufgabe, zu sagen, was Wissenschaft sein sollte, und wie sie betrieben werden sollte. Zu diesem Zweck muss Wissenschaftstheorie angeben, worin wissenschaftliche Rationalität besteht, und aufgrund welcher Kriterien sich eine wissenschaftliche Hypothese rational rechtfertigen lässt. 2.) Der deskriptiven Auffassung zufolge hat Wissenschaftstheorie dagegen die Aufgabe, zu sagen, was Wissenschaft de fakto ist und wie sie betrieben wird. Zu diesem Zweck muss Wissenschaftstheorie die faktischen Wissenschaften in ihrer historischen Entwicklung und gegenwärtigen Struktur so gut wie möglich beschreiben und erklären. Die normative Auffassung ist historisch älter und gegenwärtig immer noch weit verbreitet. Sowohl die logischen Empiristen wie die kritischen Rationalisten vertraten diese Auffassung. Die deskriptive Gegenposition wurde dagegen erst mit der durch Kuhn (1967) ausgelösten historischen Wende der Wissenschaftstheorie aktuell; wir finden sie z. B. bei Strukturalisten (z. B. Stegmüller 1973 a, 297 ff.; Balzer 1982, 1 – 5) oder bei Naturalisten (z. B. Bird 1998, 266 f.; Giere 1999, 157 – 163). Welche Position ist nun die richtige? Die Position der Normativisten gründete sich auf die bekannte, auf Hans Reichenbach (1938, 6 f.) zurückgehende Unterscheidung zwischen dem Entdeckungs- bzw. Entstehungszusammenhang von wissenschaftlicher Erkenntnis (context of discovery), und ihrem Begründungs- bzw. Rechtfertigungszusammenhang (context of justification). Die prima facie plausible Argumentation der Normativisten lässt sich wie folgt aufschlüsseln (s. z. B. Popper 1935/76, Kap. I.2): (2.1-1) Argumentation der Normativisten: Prämisse 1: Für die Beurteilung des Erkenntniswertes einer Hypothese ist nur die Frage der rationalen Rechtfertigbarkeit ihres Wahrheitsanspruches maßgeblich. Prämisse 2: Ihre faktische Entstehungsgeschichte, die Gründe ihrer Entdeckung oder Erfindung usw., sind hierfür dagegen völlig irrelevant. Konklusion: Daher sollte sich die Wissenschaftstheorie nur mit der rationalen Rechtfertigung von Wissenschaft beschäftigen und sich nicht um ihren Entstehungszusammenhang kümmern – letzteres ist die Sache von Wissen-

Normative vs. deskriptive Wissenschaftstheorie

22

2. Gemeinsame Grundlagen der Wissenschaften

schaftsgeschichte, -soziologie oder -psychologie, aber nicht Sache von Wissenschaftstheorie.

Normativistischer Fehlschluss

Da seit Kuhn (1967) die normativistische Argumentation sehr pauschal kritisiert wurde, wollen wir uns klarmachen, worin die Berechtigung und worin der Fehler dieser Argumentation liegt. Zunächst ist die Argumentation logisch korrekt – wenn man als analytisch wahr unterstellt, dass es der Wissenschaftstheorie um die Beurteilung des Erkenntniswertes von Hypothesen geht. D. h., wären die Prämissen des Argumentes wirklich wahr, dann wäre auch die Konklusion wahr. Der Fehler liegt auch nicht in Prämisse (1) – auch diese These scheint analytisch wahr zu sein, insofern es Erkenntnis um Wahrheit geht und um nichts sonst. Dagegen ist Prämisse (2) vom ,Bazillus‘ des normativistischen Fehlschlusses infiziert. Denn die historische Genese einer wissenschaftlichen Hypothese umfasst ja nicht nur erkenntnisexterne Momente, die erkenntnisirrelevant sind, sondern auch erkenntnisintern relevante Momente, z. B. die experimentellen Methoden, die Wissenschaftler de fakto verwenden, und die rationalen Hypothesenbewertungen, die sie de fakto vornehmen. Beispielsweise beschreibt der organische Chemiker Kekulé, dass er seine bahnbrechende Hypothese des ringförmigen Benzolringes zum ersten Mal im Traum gewonnen habe (s. Hempel 1974, 28). Für die empirische Rechtfertigung dieser Hypothese hat dies natürlich keine Rolle gespielt. Andererseits wurde das Boyle-Mariottesche Gasgesetz durch systematische induktive Datenextrapolation gewonnen (Langley et al. 1987, 81) – und dies spielt für die empirische Rechtfertigung dieses Gesetzes eine entscheidende Rolle. Von den erfolgreichen de-fakto Methoden und Argumentationen der Wissenschaften sollten die Wissenschaftsphilosophen lernen; aber das können sie nur, wenn sie sich diese ansehen. Was die Normativisten statt Prämisse 2 zumeist nur sagen wollen, oder zumindest nur sagen sollten, ist folgende nahezu triviale Abschwächung. (2.1-2) Prämisse 2*: Der erkenntnisexterne Anteil der Entwicklungsgeschichte einer Hypothese ist für die Beurteilung ihres Erkenntniswertes irrelevant.

Bedeutung der Wissenschaftsgeschichte

Die Konklusion des Argumentes (2.1-1) folgt natürlich nicht mehr, wenn man Prämisse 2 durch die schwächere Prämisse 2* ersetzt. Und in dieser Konklusion wird der Fehler der normativistischen Argumentation gravierend. Um dies zu erkennen, muss man sich nur folgende Frage vorlegen: wie gelangen denn die Wissenschaftstheoretiker zu den von ihnen aufgestellten Regeln und Kriterien der wissenschaftlichen Methode? Es wäre naiv, anzunehmen, dass sich diese Regeln und Kriterien durch reine Logik oder bloße Intuition gewinnen lassen könnten; dazu ist Wissenschaft viel zu komplex. Das wäre ebenso, als wenn jemand ein Lehrbuch über Methoden des Brückenbauens schreiben möchte, ohne sich die de-fakto Geschichte des Brückenbauens oder zumindest Brücken-Musterbeispiele genau anzusehen. Die Wissenschaftstheorie kann und soll also aus den Musterbeispielen der Wissenschaftsgeschichte lernen. Wir gelangen damit zu folgendem Resultat: obwohl die Aufgabe der Wissenschaftstheorie primär darin be-

2.1 Methode und Voraussetzungen der Wissenschaftstheorie

steht, die Methoden und Kriterien rationaler Wissenschaft herauszufinden, so ist es hierzu dennoch unumgänglich, auch die faktischen Wissenschaften in ihrer historischen Entwicklung zu studieren. Aufgrund der Kuhnschen Kritik kristallisierte sich die Einsicht heraus, dass die Wissenschaftsmodelle des logischen Empirismus und kritischen Rationalismus in vielen Hinsichten zu simpel waren, um reale Wissenschaft zu erfassen (vgl. Stegmüller 1986, 20). Als Reaktion darauf proklamierten jüngere Wissenschaftstheoretiker, Wissenschaftstheorie solle sich überhaupt auf die deskriptive Analyse der Wissenschaften beschränken. Aber auch diese Position ist weit übertrieben. Die Frage nach der Definition und den Kriterien wissenschaftlicher Rationalität muss natürlich im Zentrum der Wissenschaftstheorie bleiben, denn diese Frage ist es ja, was die Wissenschaftstheorie als Disziplin zusammenhält und von Wissenschaftsgeschichte und Wissenschaftssoziologie unterscheidet. Zusammengefasst ist Wissenschaftstheorie somit eine Disziplin, die sowohl deskriptive wie normative Bestandteile enthält (s. auch Stegmüller 1973b, 9 ff.). Die Methode der Wissenschaftstheorie lässt sich am besten als rationale Rekonstruktion bezeichnen. Diese Methode bewegt sich zwischen zwei Polen: einem (sogenannten) deskriptivem Korrektiv, welches adäquat rekonstruiert werden soll, und einem normativen Korrektiv, welches Rationalitätsnormen beinhaltet. Das deskriptive Korrektiv enthält Musterbeispiele erfolgreicher wissenschaftlicher Erkenntnisse sowie aber auch Mustergegenbeispiele von widerlegten wissenschaftlichen Hypothesen. Über diese Muster(gegen)beispiele besteht rationaler Konsens, auf den sich Wissenschaftstheorie zumindest vorläufig stützen darf. Das normative Korrektiv beinhaltet zunächst eine oberste erkenntnisinterne Zielvorgabe (oder Norm), welche folgendes besagt (s. auch Weingartner 1978, § 3.2):

Deskriptives und normatives Korrektiv

(2.1-3) Das oberste Erkenntnisziel (Z) der Wissenschaft besteht in der Findung von möglichst wahren und gehaltvollen Aussagen, Gesetzen oder Theorien, über einen bestimmten Gegenstandsbereich. Dabei ist ein Satz umso gehaltvoller, je mehr Konsequenzen er besitzt (s. Kap. 3.7). Die Einschränkung auf gehaltvolle wahre Aussagen ist bedeutsam, denn Wahrscheinlichkeit und Gehalt von Hypothesen sind oft gegenläufig. Man kann die Wahrheitschancen von Hypothesen maximieren, indem man nur triviale Tautologien äußert, wie etwa „die Sonne dreht sich um die Erde oder auch nicht“. Andererseits ist es leicht, sehr gehaltvolle und beeindruckende Hypothesen vorzutragen, wenn man auf ihre Wahrheit keine Rücksicht nimmt, wie z. B. „ich habe ein Perpetuum Mobile erfunden“. Die eigentliche Kunst des Wissenschafters besteht darin, Hypothesen zu formulieren, die sich sowohl empirisch bewahrheiten als auch als gehaltvoll und konsequenzenreich erweisen. Das Ziel der wahren gehaltvollen Aussage besagt allein jedoch noch überhaupt nichts, solange nicht zumindest in groben Zügen umrissen ist, was unter ,Wahrheit‘ verstanden werden soll. Jeder Mensch, welchem ,Denkstil‘ er auch zugehören mag, beruft sich gerne auf das Wahrheitsziel, nur dass dabei

Gehaltvolle Wahrheit als Zielvorgabe der Wissenschaft

23

24

2. Gemeinsame Grundlagen der Wissenschaften

Minimale erkenntnistheoretische Annahmen

jeder unter Wahrheit etwas anderes verstehen mag. Das wissenschaftliche Erkenntnisziel gewinnt daher erst greifbaren Inhalt durch die zweite, vom Wissenschaftstheoretiker vorausgesetzte Komponente, ein minimales gemeinsames erkenntnistheoretisches Modell der Wissenschaften, das Begriffen wie Wahrheit etc. erst Sinn verleiht. Wir werden in Kap. 2.2 dieses erkenntnistheoretische Modell der Wissenschaften genau explizieren. Hier sei es nur in Stichpunkten umrissen: es enthält (1) einen minimalen Realismus, (2) die Einstellung der Fallibilität (Fehlbarkeit), (3) das Streben nach Objektivität, (4) einen minimalen Empirismus, sowie (5) das Streben nach logischer Klarheit. Obwohl dieses minimale erkenntnistheoretische Modell nicht aus normativen, sondern aus deskriptiven Behauptungen besteht, rechnen wir es dennoch zum normativen Korrektiv der Wissenschaftstheorie, da es dem obersten Wissenschaftsziel erst klaren Sinn verleiht und von der Wissenschaftstheorie normalerweise vorausgesetzt wird. Gegeben das so beschriebene normative und das deskriptive Korrektiv, so kann die Methode der rationalen Rekonstruktion wie folgt präzisiert werden: (Def. 2.1-1) Methode der rationalen Rekonstruktion: Entwickle verallgemeinerte und logisch möglichst präzise Modelle von wissenschaftlicher Erkenntnis, die einerseits auf das deskriptive Korrektiv zutreffen, d. h. den Erfolg seiner Musterbeispiele und den Misserfolg seiner Mustergegenbeispiele optimal erklären können, und die andererseits sich unter Voraussetzung des normativen Korrektivs rechtfertigen lassen, und zwar als optimale Mittel, um unter Voraussetzung des minimalen erkenntnistheoretischen Modells das oberste wissenschaftliche Erkenntnisziel zu erreichen.

Anwendung der Wissenschaftstheorie auf kontroverse Bereiche

Die Tätigkeit der Wissenschaftstheorie besteht also darin, aus dem deskriptiven Korrektiv verallgemeinerte Modelle zu abstrahieren, die zugleich dem normativen Korrektiv entsprechen. In gewisser Weise besagt dies schon der Begriff der rationalen Rekonstruktion: man zeichnet etwas nach und erweist es dadurch zugleich als rational. Zusammengefasst zeigt dies das Schema in Abb. 2.1-1. Solange beide Teilaufgaben der rationalen Rekonstruktion – deskriptive Adäquatheit und normative Rechtfertigbarkeit – in Harmonie miteinander erfolgreich durchführbar sind, ist die wissenschaftstheoretische Rekonstruktionsarbeit geglückt und ihre Resultate können als bewährt gelten. Natürlich ist nicht zu erwarten, dass alles, was man in der faktischen Wissenschaft antrifft, auch rational gemäß dem normativen Korrektiv ist. Daher enthält das deskriptive Korrektiv der Wissenschaftstheorie auch nur anerkannte Musterbeispiele und -gegenbeispiele. Haben sich wissenschaftstheoretische Modelle bewährt, so kann man sie in einem weiteren Schritt dann auch auf kontroverse Bereiche einzelner Disziplinen anwenden, in denen keine Einigkeit vorliegt – mag es sich dabei um das Objektivitätsproblem in der Quantenmechanik, das Teleologieproblem in der Biologie oder das Erklären-Verstehen-Problem in den Geisteswissenschaften handeln. Genau hier

2.1 Methode und Voraussetzungen der Wissenschaftstheorie

NORMATIVES KORREKTIV Allgemeines Erkenntnisziel (Revision?)

(Minimales) Erkenntnistheoretisches Modell (Rechtfertigung)

(Revision?)

WISSENSCHAFTSTHEORETISCHE REKONSTRUKTION Modelle von Beobachtung, Experiment, Gesetz, Theorie, Erklärung, Bestätigung, Falsifikation und Schwächung, Theorienfortschritt, usw. (empirische Stützung) Faktische Wissenschaft: Musterbeispiele und -gegenbeispiele

(Anwendung) Faktische Wissenschaft: Kontroverse Beispiele

DESKRIPTIVES KORREKTIV

Abb. 2.1-1: Die Methode der rationalen Rekonstruktion

vermag dann die Wissenschaftstheorie den Einzelwissenschaften wertvolle Entscheidungshilfen in die Hand zu geben. Einerseits also lernt die Wissenschaftstheorie aus den faktischen Wissenschaften und wird durch sie korrigiert; andererseits kann sie nach erfolgreicher Rekonstruktionsarbeit die Wissenschaften bei der Lösung ihrer Probleme anleiten. Allerdings muss die Zielsetzung der rationalen Rekonstruktion, deskriptive Wissenschaftsanalyse und normative Wissenschaftsrechtfertigung zusammenzubringen, nicht immer aufgehen. Dies ist beispielsweise dann der Fall, wenn Wissenschaftstheoretiker von unrealistisch hohen oder einseitigen normativen Standards ausgehen, wodurch es ihnen in der Folge hartnäckig nicht gelingen wird, ihr normatives Korrektiv mit der deskriptiven Wissenschaftsanalyse zusammenzubringen. In solchen Fällen muss die Wissenschaftstheorie dazu übergehen, ihr eigenes normatives Korrektiv abzuändern. Wenn sie das tut – und derartiges ist schon mehrmals in der Geschichte der Wissenschaftstheorie geschehen – so befindet sich die Wissenschaftstheorie, wie man in Anlehnung an die Terminologie Kuhns sagen kann, nicht mehr in einer normalwissenschaftlichen, sondern in einer revolutionären Phase. Man kann also auch die Geschichte der Wissenschaftstheorie in Analogie zur Phasenlehre, die Kuhn für die Geschichte der Naturwissenschaften entworfen hat, in zwei Phasen einteilen: in normalwissenschaftliche Phasen, in welchen das normative Korrektiv unbehelligt vorausgesetzt wird, und in revolutionäre Phasen, in welchen dieses normative Korrektiv hinterfragt und modifiziert wird. Abschließend sei dieses Modell rationaler Rekonstruktion mit zwei ähnlichen Charakterisierungen der wissenschaftstheoretischen Methode verglichen. Carnap (1950b, § 2) hatte das Verfahren der Wissenschaftstheorie als Explikation beschrieben. Dabei wird ein vages Konzept der Alltags- oder Wissenschaftssprache wie z. B. der Begriff der Wahrscheinlichkeit (das Explikandum) durch einen exakt definierten Begriff (das Explikatum) ersetzt,

Selbstanwendung der Wissenschaftstheorie

25

26

2. Gemeinsame Grundlagen der Wissenschaften Carnaps Verfahren der Explikation

Goodmans Überlegungsgleichgewicht

wobei das Explikatum vom Explikandum in Hinsicht auf Vagheits- und Ambiguitätselimination abweichen muss, aber dennoch mit ihm möglich gut übereinstimmen soll, und zugleich möglichst fruchtbar und einfach sein soll. Es bleibt bei Carnap aber ungeklärt, wie die genaue Kombination dieser sich teilweise widersprechenden Forderungen aussehen soll. Verglichen dazu enthält unser Modell der rationalen Rekonstruktion deutlichere normative und deskriptive Vorgaben, wobei der Möglichkeit von Inkohärenzen zwischen den beiden durch die Unterscheidung zwischen normalwissenschaftlichen und revolutionären Phasen der Wissenschaftstheorie Rechnung getragen wird. Eine Ähnlichkeit besteht auch zwischen rationaler Rekonstruktion und der Methode des reflexiven Überlegungsgleichgewichtes nach Goodman (1955/75, 85 – 89) und Rawls (1979, 38, 68 – 71). Während letztere Methode jedoch rein kohärenztheoretisch funktioniert, insofern es um die wechselseitige Anpassung von methodologischen Regeln und Intuitionen geht, ist die Methode der rationalen Rekonstruktion nach der Seite ihres deskriptiven Korrektivs hin empirisch fundiert; und auch das normative Korrektiv besteht nicht aus bloßen Intuitionen, sondern aus erkenntnistheoretischen Hypothesen im Zusammenhang mit einer normativen Zielvorgabe.

2.2 Gemeinsame erkenntnistheoretische Annahmen

Strukturelle Korrespondenztheorie der Wahrheit

Das minimale erkenntnistheoretische Modell, das allen empirischen Wissenschaftsdisziplinen mehr oder weniger gemeinsam ist, lässt sich durch fünf erkenntnistheoretische Annahmen (E1 – E5) beschreiben: E1 – Minimaler Realismus: Dieser Annahme zufolge gibt es eine Wirklichkeit bzw. Realität, die unabhängig vom (gegebenen) Erkenntnissubjekt existiert. Es wird nicht unterstellt, dass alle Eigenschaften dieser Realität erkennbar sind. Die Möglichkeit grundsätzlicher Erkenntnisgrenzen wird offengelassen und kann nicht apriori, sondern nur angesichts des faktischen Erkenntniserfolges der Wissenschaften beantwortet werden. Wissenschaftliche Disziplinen bezwecken, möglichst wahre und gehaltvolle Aussagen über abgegrenzte Bereiche dieser Realität aufzustellen. Der Begriff der Wahrheit wird dabei im Sinn der strukturellen Korrespondenztheorie verstanden, derzufolge die Wahrheit eines Satzes in einer strukturellen Übereinstimmung zwischen dem Satz und dem von ihm beschriebenen Teil der Realität besteht. Dieser von Alfred Tarski (1936a) präzisierte strukturelle Wahrheitsbegriff unterstellt somit keine direkte Widerspiegelungsbeziehung zwischen Sprache und Wirklichkeit. Er ist auch damit verträglich, dass streng genommen falsche Theorien in einem approximativen Sinne wahr bzw. wahrheitsnahe sein können (dazu Kap. 5.6.2). E2 – Fallibilismus und kritische Einstellung: Es gibt keinen unfehlbaren ,Königsweg‘ zu korrespondenztheoretischer Wahrheit. Der Annahme des Fallibilismus zufolge ist jede wissenschaftliche Behauptung mehr oder minder fehlbar; wir können uns ihrer Wahrheit daher nie absolut sicher sein, aber wir können ihre Wahrheit als für mehr oder weniger wahrscheinlich befinden. Daher kommt alles darauf an, im Sinne von Annahme E4 unten,

2.2 Gemeinsame erkenntnistheoretische Annahmen

durch empirische Überprüfung herauszufinden, wie es um die Wahrscheinlichkeit der Wahrheit einer wissenschaftlichen Hypothese bestellt ist. Mit dem Fallibilismus ist somit eine kritische Einstellung verbunden, derzufolge keine Aussage von der Kritik ein- für allemal ausgeschlossen werden darf. E3 – Objektivität und Intersubjektivität: Die Wahrheit einer Aussage muss dieser Annahme zufolge objektiv gelten, d. h., sie muss unabhängig von den Einstellungen und Wertungen des Erkenntnissubjekts bestehen, da ja gemäß Annahme E1 auch die Realität unabhängig davon besteht, und Wahrheit die Übereinstimmung von Aussage und Realität ist. Die Objektivitätsannahme folgt daher bereits aus Annahme E1. Die Charakterisierung von Objektivität als Subjektunabhängigkeit hilft uns in der Wissenschaftspraxis allerdings nicht weiter, da es immer Subjekte sind, die Aussagen aufstellen und Hypothesen formulieren. Wir haben keinen direkten Zugang zur objektiven Wahrheit, sondern können uns ihr nur indirekt über – nicht sichere, aber wahrscheinliche – Kriterien nähern. Die zweite Teilaussage von Annahme E3 besagt, dass ein zentrales wissenschaftliches Kriterium für Objektivität und indirekt auch für Wahrheit in der Intersubjektivität von Aussagen liegt: wenn sich die Wahrheit einer Aussage überhaupt überzeugend begründen lässt, so muss sich jede kognitiv hinreichend kompetente Person von der Wahrheit dieser Aussage nach hinreichender Kenntnisnahme der Datenlage zumindest ,im Prinzip‘ überzeugen lassen. E4 – Minimaler Empirismus: Mit der Ausnahme von Formalwissenschaften wie der Mathematik, auf die wir noch zu sprechen kommen, muss der Gegenstandsbereich einer Wissenschaft im Prinzip der Erfahrung bzw. der Beobachtung zugänglich sein. Denn letztlich kann nur durch wahrnehmende Beobachtung verlässliche Information über die Realität erlangt werden – nur durch Wahrnehmung stehen wir mit der Realität in informationellem Kontakt. Empirische Beobachtungen sind somit ein zentraler Schiedsrichter für die wissenschaftliche Wahrheitssuche: an ihnen müssen wissenschaftliche Gesetzeshypothesen und Theorien überprüft werden. Dies ist die zentrale Aussage von Annahme E4. Beobachtungen sind zwar nicht infallibel (das wäre ein Widerspruch zur Annahme E2), aber bei ihnen ist Intersubjektivität und praktische Sicherheit am leichtesten und schnellsten erzielbar. Minimal ist diese Art von Empirismus, weil nicht behauptet wird, dass sich alle wissenschaftlichen Begriffe bzw. Sätze durch Definitionsketten auf Beobachtungen zurückführen lassen müssen oder gar durch sie beweisbar sind. Wissenschaftliche Theorien dürfen und sollen auch über das sprechen, was der Beobachtung nicht unmittelbar zugänglich ist; entscheidend ist nur, dass sie empirische Konsequenzen besitzen, an denen sie sich überprüfen lassen. E5 – Logik im weiten Sinn: Durch Anwendung präziser logischer Methoden zur Einführung von Begriffen, zur Formulierung von Sätzen sowie zur Bildung korrekter Argumente kann man dem Ziel der Wahrheitssuche (gemäß Annahmen E1 – E4) am effektivsten näher kommen. Denn die Bedeutung eines Satzes steht nur dann genau fest, wenn die in ihm vorkommenden Begriffe genau präzisiert wurden. Und nur für Sätze mit präzise formulierter Bedeutung sind deren logische Konsequenzen präzise ermittelbar. Schließlich ist nur dann, wenn die Konsequenzen einer Hypothese genau

Empirische Überprüfbarkeit

27

28

2. Gemeinsame Grundlagen der Wissenschaften

Empirische Bewährbarkeit des Realismus

Probleme der Intersubjektivität

bekannt sind, diese Hypothese gemäß Annahme E4 präzise empirisch überprüfbar. Das Verfahren der empirischen Überprüfung erfordert also an allen Stellen die Anwendung logischer Methoden – im weiten, nicht auf deduktive Logik eingeschränkten Sinn von Logik. Unser minimales Erkenntnismodell macht so wenig Annahmen wie möglich, jedoch soviel wie nötig, um sinnvoll gegenstandsbezogene Wissenschaft betreiben zu können. Einige seiner Annahmen verdienen nähere Erläuterung. Wir nennen den Realismus in Annahme E1 „minimal“, weil er lediglich die objektive Existenz der Wirklichkeit behauptet. Ob und bis zu welchem Grad die Realität auch objektiv erkennbar ist, kann gemäß Annahme E2 nicht apriori beantwortet werden, sondern nur durch den Erfolg der wissenschaftlichen Erkenntnismethode. Dadurch unterscheidet sich der minimale Realismus von allen Spielarten eines metaphysischen Realismus. Wir behaupten in Annahmen E4 und E5 lediglich zwei minimale Ebenen, in denen Erkenntnis einen Ausgangspunkt und Prüfstein hat: einerseits die Beobachtung, die uns einen zwar lückenhaften, aber reliablen und intersubjektiven Zugang zur Wirklichkeit ermöglicht, und andererseits die Logik im weiten Sinn, die ein universelles Denkwerkzeug bereit stellt, aber von sich aus noch nichts über die Wirklichkeit besagt. In unserer Charakterisierung von Intersubjektivität in Annahme E3 haben wir uns auf ,hinreichend kompetente‘ Personen bei ,hinreichender Kenntnis der Datenlage‘ beschränkt. Denn Personen, die den Gehalt einer Hypothese nicht verstehen oder die Datenlage nicht kennen, besitzen nicht die Fähigkeit, diese Hypothese auf ihre Wahrheit hin zu überprüfen. Die Schwierigkeit dieser Einschränkung zeigt uns, dass Intersubjektivität keinesfalls als Definition von Objektivität oder Wahrheit verwendet werden darf (so wie z. B. bei Kamlah und Lorenzen 1973, 119), da Kompetenz ein unsicheres und graduelles Kriterium ist, weshalb sich auch noch so viele als kompetent befundene Personen kollektiv irren können. Charles S. Peirce hat versucht, die Definition von Wahrheit als intersubjektiver Konsens vor solchen Einwänden zu retten, indem er sich auf das fiktive Endresultat einer idealen Forschergemeinschaft bezog (s. Apel 1976, Hg., 1. Teil, Kap. II.7). Aber selbst in dieser kontrafaktischen Version muss intersubjektiver Konsens nicht mit korrespondenztheoretischer Wahrheit übereinstimmen. Es gibt unzählige Propositionen, über die selbst eine ideale Forschergemeinschaft nie Konsens erzielen kann, weil sie keinen evidentiellen Zugang dazu besitzt, obwohl es keinen Grund gibt, anzunehmen, diese Propositionen seien in ihrem Wahrheitswert ontologisch unbestimmt – z. B. „wie viele (bzw. soundsoviele) Viren befanden sich in Cäsars Nase, als er den Rubikon überschritt?“. Zusammengefasst kann Intersubjektivität nur als unsicheres Kriterium für Objektivität dienen. Beim minimalen Realismus von Annahme E1 handelt es sich um einen hypothetisch-konstruktiven Realismus (s. Kap. 2.7.1). Zugleich trägt dieser Realismus naturalistische Züge, da die philosophische Hypothese der Erkennbarkeit und sogar die der Existenz einer subjektabhängigen Realität sich letztlich, wie alle anderen Bestandteile des wissenschaftlichen Gesamtsystems, über den Erfolg der Erfahrungserkenntnis bewähren muss, und zwar als beste Erklärung dieses Erfolges (im Sinne des abduktiven Schlusses, s. Kap. 2.6.3). Es sei

2.3 Gemeinsame methodologische Merkmale

betont, dass der minimale Realismus auch kompatibel ist mit den berechtigten Zweifeln am ,vollen‘ Realismus in den philosophischen Interpretationen der Quantenmechanik (hierzu s. Bartels 1986, Kap. 4). Zu Vermeidung von Missverständnissen ist hinzuzufügen, dass die Korrespondenztheorie der Wahrheit sich zwar vorzüglich als Definition von Wahrheit eignet, jedoch von sich aus keinerlei Kriterien hergibt, um die Wahrheit eines Satzes herauszufinden (vgl. Rescher 1977; Brendel 1999, 123). Den Weg zu solchen Kriterien weisen unsere Annahmen E5 und E4: ermittle die empirischen Konsequenzen deiner Hypothese und überprüfe sie anhand empirischer Beobachtungen oder Experimente. Um beispielsweise die Wahrheit des Satzes „diese Blume ist rot“ herauszufinden, bringt mich die korrespondenztheoretische Einsicht, dass dieser Satz wahr ist, wenn diese Blume rot ist, keinen Schritt weiter; ich muss vielmehr in der Lage sein, das durch den ostensiven Individuenterm „diese Blume“ bezeichnete Objekt sowie die mit „rot“ ausgedrückte Wahrnehmungsqualität visuell zu erfassen, um die Wahrheit dieses Satzes durch einen einfachen Wahrnehmungsakt überprüfen zu können.

Definition versus Kriterien der Wahrheit

2.3 Gemeinsame methodologische Merkmale Aus dem obersten Erkenntnisziel (Z) zusammen mit den fünf erkenntnistheoretischen Annahmen (E1 – 5) folgen sehr plausibel vier methodologische Merkmale (M1 – 4), die allen im weiten Sinn empirischen Wissenschaften gemeinsam sind. Zunächst eine begriffliche Vorklärung. Unter einem Beobachtungssatz verstehen wir einen Satz, der einen beobachtbaren Sachverhalt ausdrückt. Wir unterscheiden zwischen aktualen und potentiellen Beobachtungssätzen. Aktual nennen wir einen Beobachtungssatz, wenn der von ihm ausgedrückte Sachverhalt tatsächlich beobachtet wurde. Bloß potentiell ist dagegen ein Beobachtungssatz, wenn der von ihm ausgedrückte Sachverhalt noch nicht beobachtet wurde, aber beobachtet werden könnte, z. B. zu einem späteren Zeitpunkt oder an einem anderen Ort. Bloß potentielle Beobachtungssätze drücken empirische Voraussagen aus wie z. B. „morgen wird eine Sonnenfinsternis eintreten“. Damit können wir die vier gemeinsamen methodologischen Merkmale wie folgt formulieren (ähnlich bei Weingartner 1978, §§ 3.1 – 3.7): M1: Wissenschaft sucht nach möglichst allgemeinen und gehaltvollen hypothetischen Sätzen, die in einer wissenschaftlichen Sprache abgefasst sind. Dabei handelt es sich in allen Disziplinen um Gesetze und Theorien, und in einigen Disziplinen (z. B. den historischen Wissenschaften) auch um hypothetische Singulärsätze. M2: Wissenschaft sucht nach möglichst vielen (und möglichst relevanten) aktualen Beobachtungssätzen, welche die Resultate von Beobachtungen, Experimenten oder Messungen wiedergeben. M3: Wissenschaft versucht, mithilfe der allgemeinen und hypothetischen Sätze die zum gegenwärtigen Zeitpunkt bekannten aktualen Beobachtungssätze zu erklären, sowie neue und noch unbekannte potentielle Beobachtungssätze vorauszusagen.

Aktuale vs. potentielle Beobachtungssätze

29

30

2. Gemeinsame Grundlagen der Wissenschaften

Erklärung und Voraussage

Bedeutung für das planende Handeln

Drei Ebenen der wissenschaftlichen Methode

M4: Wissenschaft versucht, ihre allgemeinen und hypothetischen Sätze empirisch zu überprüfen, und zwar indem die vorausgesagten (potentiellen) Beobachtungssätze mit den gegenwärtig bekannten aktualen Beobachtungssätzen verglichen werden. Stimmen die letzteren mit den ersten überein, so war die Voraussage erfolgreich (sie wird dann zu einer erfolgreichen Erklärung) und das Gesetz bzw. die Theorie ist bestätigt bzw. bewährt. Stehen die letzteren mit den ersteren in logischem Widerspruch, so war die Voraussage gescheitert und das Gesetz bzw. die Theorie ist falsifiziert oder im Fall einer bloß statistischen Voraussage zumindest geschwächt. Von einer Erklärung spricht man nur dann, wenn der erklärte Sachverhalt bereits eingetreten und bekannt ist, während bei einer Voraussage der vorausgesagte Sachverhalt neu und noch unbekannt ist. Logisch gesehen haben sowohl Erklärungen wie Voraussagen die Form deduktiver oder probabilistischer (d. h. wahrscheinlichkeitsmäßiger) Argumente (s. Kap. 6). Die vier methodologischen Merkmale hängen mit den erkenntnistheoretischen Annahmen der Wissenschaften sehr eng zusammen. Merkmal M1 ergibt sich aus dem Erkenntnisziel Z, denn Gesetze und Theorien sind Musterbeispiele allgemeiner gehaltvoller Sätze. M2 ergibt sich aus Annahme E4 (verschärft durch Annahmen E2 und E3): um etwas über die vermutliche Wahrheit ihrer Hypothesen herauszufinden, muss die Wissenschaft empirische Daten sammeln. Um ihre allgemeinen Hypothesen anhand dieser Daten zu überprüfen, müssen die logischen oder probabilistischen Konsequenzen der Hypothesen in Form von Voraussagen bzw. potentiellen Erklärungen ermittelt werden und diese Konsequenzen mit den Daten verglichen werden – in die Methodenbausteine M3 und M4 fließen daher Annahmen E4 und E5 ein. Erfolgreiche Voraussagen und Erklärungen sind nicht nur für das wissenschaftliche Oberziel der Findung wahrer gehaltvoller Sätze wesentlich. Voraussagen wie Erklärungen sind zugleich für das planende Handeln des Menschen überlebenswichtig. Voraussagen ermöglichen es dem Menschen, seine Zukunft dadurch zu planen, dass er sich selbst dem vorausgesagten Ereignisverlauf anpasst. Erklärungen haben mit Ursachenfindung zu tun (s. Kap. 6.2); sie liefern dem Menschen Ursachenwissen und versetzen ihn dadurch in die Lage, in die Welt kausal einzugreifen, um so den Ereignisverlauf seinen eigenen Wünschen anzupassen. Die wissenschaftlichen Unterziele der Voraussage und der kausalen Kontrolle wurden von der Menschheit seit dem Steinzeitalter verfolgt. Wer voraussagen kann, dass in einem bestimmten Gebiet nahrhafte Pflanzen wachsen, kann gezielt dorthin wandern und damit der eigenen Stammesgruppe eine hohe Fortpflanzungsrate bescheren. Wer aber die Ursachen des Wachstums nahrhafter Pflanzen kennt, wer sie anzupflanzen und zu bewässern weiß, der braucht nicht länger Sammler bleiben, sondern kann Ackerbauer werden. In fortgeschrittenen Wissenschaften spielt sich das wissenschaftliche Erklärungs- und Voraussageverfahren und das damit zusammenhängende Überprüfungsverfahren nicht auf zwei, sondern auf drei Ebenen ab: aktuale Beobachtungssätze auf der untersten Ebene, empirische Gesetzeshypothesen auf der mittleren Ebene und wissenschaftliche Theorien auf der obersten Ebene. Zwischen der ersten und zweiten, und zwischen der zweiten und

2.3 Gemeinsame methodologische Merkmale

dritten Ebene, finden jeweils die Verfahrensmethoden M3 und M4 statt, wie in Abb. 2.3-1 dargestellt. Die dreistufige Arbeitsteilung ist aus vielerlei Gründen vorteilhaft; in allen fortgeschrittenen Wissenschaftsdisziplinen hat sich die Arbeitsteilung zwischen den experimentellen Praktikern (Ebene 1 und 2) und den Theoretikern (Ebene 2 und 3) entwickelt. Wissenschaftliche Theorien Voraussage Erklärung

Bestätigung Schwächung

Empirische Gesetze Voraussage Erklärung

Bestätigung Schwächung

Aktuale Beobachtungssätze

Abb. 2.3-1: Die drei Ebenen der wissenschaftlichen Methode

Das Beispiel in Abb. 2.3-2 soll die drei Ebenen in ihrem Zusammenwirken verdeutlichen. In diesem weitgehend selbsterklärenden Beispiel stellt man sich, betreffend die unterste und mittlere Ebene, am besten das 16. Jahrhundert vor, als Rauchen aus Übersee gerade eingeführt und seine Wirkung weitgehend unbekannt war. Raucher-Neulinge berichten über Schwindelgefühl; dies wird durch die empirische Gesetzeshypothese G korrekt erklärt bzw. vorausgesagt, wodurch G bestätigt wird. Gewohnheitsraucher, bei denen kein Schwindelgefühl mehr auftritt, schwächen dann jeTHEORIE T ( plus Hintergrundwissen) T1: Kohlenmonoxid im Rauch reduziert den Sauerstoffgehalt im Blut T2: Sauerstoffmangel im Blut führt zu Schwindelgefühl Bestätigung

Erklärung

Bekanntes empirisches Gesetz G

Bestätigung

Erklärung

Bekannte Fälle

Voraussagen

Neue empirische Gesetze:

EMPIRISCHES GESETZ G: Rauchen führt zu Schwindelgefühl Bestätigung

THEORIEMODIFIKATION T*: } Kohlendioxidmangel führt auch bei Sauerstoffüberschuss zu Schwindelgefühl

Bestätigung

Schwächung

Schwindel bei starkem Smog

Bestätigung

Schwindel bei Hyperventilation

GESETZESMODIFIKATION G*: } außer bei Gewohnheitsrauchern Voraussagen

Schwächung

Neue Fälle: ein Nichtraucher AKTUALE BEOBACHTUNGSSÄTZE

Bestätigung

ein Gewohnheitsraucher

Abb. 2.3-2: Beispiel für die drei Ebenen der wissenschaftlichen Methode

Konkretes Beispiel

31

32

2. Gemeinsame Grundlagen der Wissenschaften

doch G. Dies führt dazu, die Hypothese G gemäß G* zu modifizieren (im Sinne von Lakatos 1974). Auf der obersten Ebene – dazu stelle man sich besser das 20. Jahrhundert vor, als sich die Biochemie entwickelte – geht es nun darum, das empirische Gesetz G theoretisch zu erklären, was durch die beiden theoretischen Gesetze T1 und T2 zusammen mit weiterem Hintergrundwissen gelingt. Das vorausgesetzte theoretische Hintergrundwissen besagt, dass Sauerstoff im Blutkreislauf durch Andockung an die roten Blutkörperchen transportiert wird; dass Kohlenmonoxid durch seine chemische Ähnlichkeit mit Sauerstoff die Andockstellen besetzen und für Sauerstoffaufnahme blockieren kann; dass weiters bei Sauerstoffmangel im Blut das Großhirn mit Schwindelgefühl als Vorzeichen einer Ohnmacht reagiert, wobei sich der kritische Reaktionsschwellenwert durch Gewohnheitsbildung (G*) nach oben verschiebt. Aus diesem theoretischen Erklärungswissen folgen nun – und das ist typisch für Theorien – qualitativ neue empirische Gesetzesprognosen: z. B. müssten ähnliche Symptome überall, wo der Kohlenmonoxidgehalt der Atmosphäre hoch ist, auftreten (z. B. bei Smog), was zutrifft und damit die Theorie bestätigt. Andere Gesetzesphänomene, wie z. B. dass auch bei Hyperventilation Schwindelgefühl auftritt, scheinen sich der Theorie zu widersetzen, und zwingen dazu, diese zu modifizieren: in diesem Fall wird zu viel Sauerstoff eingeatmet und zu viel Kohlendioxid ausgeatmet, was über eine Erhöhung der freien Calciumionenkonzentration u. a. zu einer Verengung der Gehirnblutgefäße führt, was die Sauerstoffzufuhr zum Gehirn reduziert und zu Schwindelgefühl führt. Obwohl als Einführung gedacht, vermag dieses Beispiel einige Komplexitäten wissenschaftlicher Erkenntnisprozesse zu verdeutlichen.

2.4 Wissenschaftliche Disziplinen und ihre Klassifikation In diesem Kapitel wollen wir untersuchen, inwieweit unsere erkenntnistheoretischen Annahmen E1 – 5 und die methodologischen Merkmale M1 – 4 auf die existierenden Wissenschaftsdisziplinen zutreffen. Zu diesem Zweck gehen wir von folgender gegenstandsbezogenen Einteilung von wissenschaftlichen Disziplinen aus, der wir keine ,entgültige‘, sondern nur heuristische Bedeutung zumessen. Manche Disziplinen sind Mischdisziplinen und unter „auch:“ angeführt. Nicht alle Disziplinen können angeführt werden – weitere Spezialdisziplinen wären nachzutragen: Einteilung von Wissenschaftsdisziplinen

(2.4-1) Wissenschaftliche Disziplinen, klassifiziert nach ihrem Gegenstandsgebiet: Wissenschaften … 1) von der Natur: Physik, Chemie, Biologie, Geologie, Medizin (Astronomie, Kosmologie, Geographie, Paläontologie, auch: biologische Evolutionsgeschichte) 2) von der Technik: Maschinenbau, Elektrotechnik, …, auch: Computerwissenschaft

2.4 Wissenschaftliche Disziplinen und ihre Klassifikation

3) vom Menschen: Psychologie (auch: Pädagogik, Medizin, Kognitionswissenschaft) 4) von der Gesellschaft: Soziologie, Ökonomie, Politikwissenschaft (auch: Anthropologie, Ethnologie, Geographie) 5) von der Geschichte (der Menschheit): Geschichtswissenschaft (auch: Anthropologie, Ethnologie, Philosophie als Geistesgeschichte) 6) von den kulturellen (geistigen, sozialen) Schaffensprodukten des Menschen: Rechtswissenschaften, Sprachwissenschaften, Literaturwissenschaft, Kunst- und Musikwissenschaft, Medienwissenschaften (auch: Pädagogik, Religionswissenschaft) 7) von formalen Strukturen (Formalwissenschaften): Mathematik (Logik, Statistik, theoretische Informatik, Systemtheorie …), formale Methodologie und Wissenschaftstheorie 8) von den allgemeinen Grundlagen der geistigen Welterfassung: Philosophie (Erkenntnistheorie, Wissenschaftstheorie und theoretische Philosophie; Ethik, Ästhetik und praktische Philosophie) 9) von Gott: Theologie (auch: Religionswissenschaft) Unsere Klassifikation hat den Vorteil, von den werthaften Färbungen, mit denen Schlagwortkategorien wie Naturwissenschaften, Geisteswissenschaften oder Realwissenschaften (usw.) behaftet sind, unabhängig zu sein. Versucht man, diese Schlagwortkategorien auf unsere gegenstandsorientierte Klassifikation aufzustülpen, so zeigt sich, dass über die Zuordnungen nicht immer Konsens besteht, und dass die verschiedenen Zuordnungsmöglichkeiten teilweise mehr wissenschaftspolitisch als wissenschaftstheoretisch motiviert sind. Beispielsweise bildet die Disziplinengruppe 1 den Kern der Naturwissenschaften. Aber auch die Mathematik rechnet sich zu den Naturwissenschaften, obwohl sie wissenschaftstheoretisch zum Typ der Formalwissenschaften gehört. In jüngerer Zeit rechnet sich auch die Psychologie überwiegend den Naturwissenschaften zu, obwohl sie ebenso den Human- und Sozialwissenschaften zugerechnet werden könnte. Die Disziplinengruppen 5 und 6 bilden den Kernbestand der Geisteswissenschaften. Die Philosophie (8) steht dagegen den Naturwissenschaften ebenso nahe wie den Geisteswissenschaften. Von der Intention fungiert der Begriff der Human- und Sozialwissenschaften als ein im Vergleich zum Begriff der Geisteswissenschaften modernisierter Abgrenzungsbegriff zu den Naturwissenschaften. Ob die sogenannten Human- und Sozialwissenschaften nur die Disziplinengruppen 3 und 4, oder auch 5 und 6 mitumfassen, ist terminologisch nicht festgelegt. Im Begriff der Realwissenschaften (bzw. Substanzwissenschaften) werden alle Disziplinen zusammengefasst, die einen realen Gegenstandsbereich haben; also alle Disziplingruppen außer den Formalwissenschaften. Die Theologie kann nur dann zu den Realwissenschaften gerechnet werden, wenn man die Realität Gottes annimmt; da diese Realität wissenschaftlich unbegründbar ist, ist diese Zurechnung fragwürdig. Zunächst wollen wir den besonderen Status von Disziplinengruppe 7, also von Mathematik und Formalwissenschaften erläutern. In ihrer Gesamtheit machen Annahmen E1 – 5 und Merkmale M1 – 4 nur für die empiri-

Probleme der Klassifikation wissenschaftlicher Disziplinen

33

34

2. Gemeinsame Grundlagen der Wissenschaften

Der Sonderstatus der Formalwissenschaften

Philosophie der Mathematik

schen Realwissenschaften Sinn. Mathematik und Formalwissenschaften besitzen keinen konkreten, empirisch zugänglichen Gegenstandsbereich, sondern beschäftigen sich mit formalen und abstrakten Strukturen. Die Mathematik besitzt also keine empirische Basis, und deshalb treffen auf sie die Annahme E4 und die methodologischen Merkmale M2 und M4, welche eine empirischen Basis voraussetzen, nicht zu. Das heißt nicht, dass die Mathematik eine spekulative Disziplin wäre, die ohne eine empirische Basis Aussagen über die Realität macht: die reine Mathematik macht gar keine Aussagen über die (empirisch zugängliche) Realität. Sie entwickelt lediglich formale Strukturmodelle, charakterisiert sie durch abstrakte und möglichst allgemeine Axiome und Definitionen, und leitet daraus logische Konsequenzen ab. Es wäre beispielsweise sinnlos zu sagen, das Axiomensystem der reellen Zahlen oder das der euklidischen Geometrie sei an sich ,wahr‘ oder ,falsch‘. Solche Behauptungen lassen sich nur machen, wenn mathematische Begriffe empirisch interpretiert werden, wenn z. B. den reellen Zahlen reale Orte oder Zeitpunkte, und den mathematischen Vektoren reale Längen zugeordnet werden. Dann ist aber die Geometrie nicht länger eine mathematische Theorie, sondern sie wird zu einer physikalischen Theorie. Einstein hat dies einmal so formuliert (vgl. Carnap 1973, 77): „Insofern sich die Lehrsätze der Mathematik auf die Wirklichkeit beziehen, sind sie nicht sicher; und insofern sie sicher sind, beziehen sie sich nicht auf die Wirklichkeit.“ Man kann also sagen: gerade weil sie die Frage der empirischen Interpretation ihrer Modelle ausklammert, ist die Mathematik die strengste aller Wissenschaften. Freilich will die Mathematik sich in erster Linie mit solchen Strukturmodellen beschäftigen, von denen man weiß oder glaubt, dass sie sich auf gewisse Bereiche der Realität anwenden lassen. In dieser Hinsicht können mathematische Disziplinen mehr oder weniger fruchtbar sein für die Realwissenschaften, doch sie sind niemals im realwissenschaftlichen Sinn wahr oder falsch. Die Annahmen E2, E3, E5 und das Methodenmerkmal M1 treffen auch auf die Mathematik uneingeschränkt zu (die Fallibilitätsannahme E2 insofern, als auch mathematische Beweise nicht irrtumssicher sind). Annahme E1 des minimalen Realismus trifft nur indirekt zu, insofern mathematische Strukturen den Zweck haben, in Realwissenschaften angewandt zu werden. Der Methodenbaustein M3 insofern trifft zu, als Mathematiker an der Herleitung verschiedenster Theoreme interessiert sind, welche wichtige Eigenschaften ihrer Modelle ausdrücken. Es sei nicht unerwähnt, dass es in der Philosophie der Mathematik über die Natur mathematischer Gegenstände gegensätzliche Auffassungen gibt (s. z. B. Hart 1996, Hg.; Shapiro 2000). Gemäß der am meisten verbreiteten Auffassung sind mathematische Objekte konstruierte, konzeptuelle und abstrakte Objekte. Realobjekte wie z. B. fünf rote Rosen können mathematische Objekte wie z. B. die Zahl 5 exemplifizieren, aber sie können niemals mit ihnen identisch sein. Diese Auffassung des konzeptuellen Strukturalismus wurde von der Mathematikergruppe Bourbaki (1961) und Philosophen der Mathematik wie Field (1980) detailliert ausgearbeitet. Der gegensätzliche Standpunkt in der Philosophie der Mathematik ist der mathematische Rea-

2.4 Wissenschaftliche Disziplinen und ihre Klassifikation

lismus, der annimmt, dass es mathematische Objekte per se gibt. Diese Position vertraten z. B. Frege (1918, 43) und Gödel (1947, 271 f.). Dabei handelt es sich aber ebenfalls um abstrakte und platonische, d. h. nicht in der raumzeitlichen Wirklichkeit lokalisierte Objekte, die nicht empirisch, sondern durch mathematische Intuition zugänglich sind. Zusammengefasst sind Mathematik und die weiteren Formalwissenschaften eine Spezialisierung aus dem Gesamtsystem der Wissenschaft, welche sich mit formalen Modellen, Algorithmen und Methoden beschäftigt, und diese Spezialisierung hat sich in der Geschichte der Wissenschaft als ausgesprochen nützlich erwiesen. Wir charakterisieren die Formalwissenschaften daher als Voraussetzungs- und Hilfswissenschaften, die den realwissenschaftlichen Disziplinen formale Grundlagen und strukturelle Modelle mit bekannten Eigenschaften zur Verfügung stellen. Stellen wir uns nach dieser Vorabklärung die Frage: In welchem Grade treffen die Annahmen E1 – 5 und Methodenbausteine M1 – 4 auf die aufgeführten Disziplingruppen mit Ausnahme der Formalwissenschaften zu? Offensichtlich passen E1 – 5 und M1 – 4 auf alle Disziplinen der Gruppen 1 – 3 (Naturwissenschaften, technische Disziplinen, Psychologie). Es gibt auch nichts, was gegen die Anwendung auf die sozialwissenschaftlichen und historischen Disziplinen der Gruppen 4 und 5 spricht. Minimaler Realismus, Fallibilismus, Objektivität und Intersubjektivität (E1 – 3) und logische Klarheit (E5) sind in allen diesen Disziplinen akzeptierte Standards; dasselbe gibt für die Suche nach allgemeinen Hypothesen, nach Erklärungen bzw. Voraussagen (M1, M3). In allen diesen Disziplinen liegen empirische Daten vor, z. B. in Form von Resultaten psychologischer Experimente, Interviewdaten, ökonomischen und demographischen Daten, und in den Geschichtswissenschaften in Form von historischem Text- und Quellenmaterial, usw. Damit sind auch Annahme E4 und Bausteine M2 und M4 erfüllt. Freilich ist es in Bereichen der Sozialwissenschaft und Geschichte aufgrund der Komplexität des Gegenstandes wesentlich schwieriger, allgemeine Gesetzeshypothesen und Theorien zu finden – doch das Zutreffen der methodischen Bausteine verlangt ja nur, dass in den betreffenden Wissenschaften nach den darin genannten Zielen sinnvoll gesucht werden kann, was nicht schon heißt, dass dieses Ziel überall gleich gut erreicht wird. Dieselben Überlegungen gelten für weite Bereiche der Disziplinen aus Gruppe 6. Auch hier gibt es empirische Daten – sprachliche Texte, Kunstoder Musikwerke aus verschiedenen Kulturen oder Stilepochen (usw.) bilden beispielsweise das Datenmaterial der Sprach- und Literatur-, Kunst- und Musikwissenschaften. Vertreter der hermeneutischen Richtung wenden an dieser Stelle ein, dass die ,Daten‘ der Sozial- und Geisteswissenschaften, wie z. B. sprachliche Äußerungen oder Texte, immer schon interpretativer Natur seien, weshalb geisteswissenschaftliches Verstehen einem unauflöslichen hermeneutischen Zirkel ausgesetzt sei (vgl. Gadamer 1975, 250 ff.). Doch eine Reihe von Wissenschaftstheoretikern haben herausgearbeitet, dass die Überprüfung von Interpretationshypothesen über das, was ein Autor in einem Text sagen wollte, ähnlich ,holistisch‘ funktioniert wie die Überprüfung naturwissenschaftlicher Theorien (vgl. Stegmüller 1979a, Follesdal 2003). Die in Kap. 1.2.5.6 erwähnte methodendualistische Schule

Überprüfung des gemeinsamen Wissenschaftsmodells

Hermeneutischer Zirkel

35

36

2. Gemeinsame Grundlagen der Wissenschaften

Wert- und Normsätze in der Jurisprudenz

wendet wiederum ein, dass es in den Geisteswissenschaften keine allgemeine Gesetzeshypothesen gäbe, sodass sich die Tätigkeit dieser Disziplinen auf den Methodenbaustein M2, also auf die Zusammenstellung singulärer historischer Fakten gemäß der ,idiographischen Methode‘ beschränken müsse. In der Tat gibt es in sozial- und geisteswissenschaftlichen Disziplinen so gut wie keine strikten (ausnahmslosen) Gesetze. Es gibt aber statistische und insbesondere normische Gesetzeshypothesen („wenn A, dann normalerweise B“), die für Erklärungen und Hypothesenüberprüfungen in diesen Disziplingruppen unerlässlich sind (näheres Kap. 3.5, 4.2.3, 6.4). Alle Methodenbausteine der empirischer Wissenschaften können daher auf die Geisteswissenschaften übertragen werden, auch wenn nicht alle Angehörige dieser Disziplinengruppe davon Gebrauch machen. Dennoch gibt es ein vorwiegend in den Geisteswissenschaften anzutreffendes Phänomen, das in einem entscheidenen Gegensatz zur wissenschaftlichen Methode gemäß E1 – 5 und M1 – 4 zu stehen scheint: nämlich das Phänomen von Wert- und Normsätzen. Wir erläutern dieses Problem zunächst anhand einer Teildisziplin der Rechtswissenschaften, nämlich der Jurisprudenz oder Rechtsdogmatik, d. h. der Wissenschaft der Rechtsprechung. Diese Disziplin untersucht konkrete Handlungen und Tatbestände daraufhin, wie sie sich in das gegebene Rechtssystem einordnen lassen, und speziell, ob sie unter eine strafbare Handlungsweise fallen. Der auf Kelsen (1960) zurückgehende Rechtspositivismus hatte die Auffassung vertreten, dass die Jurisprudenz ein wertfreies systematisch-deduzierendes Verfahren sei, das sich lediglich auf das positive, d. h. vom Gesetzgeber festgesetzte, Recht stützt. Doch Kelsens These ist rechtsphilosophisch umstritten. War z. B. die Entwendung von Privateigentum im Einkaufladen Mundraub oder Diebstahl? (ein Beispiel in Savigny et al. 1976, 101 f.). Oder: handelte es sich bei der Unterbringung eines Verbrechers im eigenen Heim bereits um eine Mittäterschaft? (ein Beispiel von Kuhlen 2000, 37). In solchen und anderen Fällen folgt aus den Rechtsbestimmungen des Gesetzbuchs und der empirischen Tatschilderung noch nicht, worunter die gegebene Tat fällt; und der Gesetzesausleger bzw. Richter hat hier die Aufgabe, die inhaltlich unterbestimmten Gesetzesbestimmungen auszulegen. In Savigny et al. (1976, 106, 144 ff.) wird gezeigt, dass sich diese Auslegungen häufig auf Wertannahmen stützen, die nicht im Gesetzestext stehen, sondern dem ,intuitiven Rechtsbewusstsein‘ entnommen sind. Nun kann der Rechtswissenschaftler sicher zeigen, dass unter Voraussetzung gewisser Wertannahmen (oder ,Billigkeitsannahmen‘) eine gewisse Gesetzesauslegung adäquat ist. Doch wie soll er wissenschaftlich begründen, dass ein bestimmter Notstand ausreicht, um von ,Mundraub‘ anstatt von ,Diebstahl‘ zu sprechen? Oder dass ein Unterwäschewerbeplakat noch den ,guten Sitten‘ entspricht oder schon gegen die ,Würde der Frau‘ verstößt? (Hilgendorf 2000, 15, 24). Die Frage, die sich also stellt, nicht nur in Teilgebieten der Rechtswissenschaft, sondern auch in anderen Disziplinen mit human- oder sozialwissenschaftlichem Bezug, ist diese: lassen sich fundamentale Wertannahmen wissenschaftlich begründen? In medias res: die von uns vorgeschlagene Antwort lautet: nein. Das heißt nicht, dass Wert- und Normsachverhalte in den Wissenschaften gar nicht

2.4 Wissenschaftliche Disziplinen und ihre Klassifikation

auftreten. Einerseits lassen sich die de-fakto Werteinstellungen von Personen empirisch erheben, und andererseits lassen sich spezielle Wertsätze aus vorausgesetzten fundamentaleren Wertsätzen (und weiteren Prämissen) logisch deduzieren. Doch eine fundamentale wissenschaftliche Wertbegründung ist unmöglich. Unsere Auffassung befindet sich im Einklang mit dem Standardverständnis von (empirischer) Wissenschaft und beansprucht keine Originalität; doch unsere Begründung dieser Antwort sollte eine gewisse Originalität aufweisen. Denn die Wertfreiheitsforderung ist von Positivismuskritikern einer so scharfen Kritik unterzogen worden, dass sie heutzutage nicht mehr als selbstverständlich vorausgesetzt werden kann, sondern einer sorgsamen Ausarbeitung bedarf, der wir uns in Kap. 2.5 zuwenden. Oben haben wir wissenschaftliche Disziplinen nach ihrem Gegenstandsbereich untergliedert. Zum Abschluss dieses Kapitels erläutern wir zwei weitere Klassifikationsmöglichkeiten. Eine methodologisch bedeutsame Klassifikation der Realwissenschaften ist die folgende, nach der ,Eindringlichkeit‘ ihres empirischen Charakters: 1) Empirische Wissenschaften (im Sinn des minimalen Empirismus): Darunter sind alle Disziplinen zu verstehen, die sich auf empirische Daten irgendwelcher Art beziehen, ob diese Daten in freier Feldbeobachtung, durch Studium von Quellenmaterial oder im Laborexperiment gewonnen wurden. Zu dieser umfangreichsten Gruppe zählen also alle durch die Annahmen E1 – 5 und Methodenbausteine M1 – 4 beschriebenen Disziplinen, inklusive der Geisteswissenschaften (im hier verstandenen Sinn). Nur Disziplinen, deren Theorien auf reiner Spekulation beruhen, liegen außerhalb dieses Bereichs – wie z. B. Mystik, Esoterik, spekulative Geistes-,Wissenschaft‘, oder dogmatisch-bekenntnishafte Anteile der Theologie. Gemäß unseren erkenntnistheoretischen Annahmen ist es zweifelhaft, ob es sich dabei überhaupt um Realwissenschaften handelt. 2) Experimentelle Wissenschaften: Diese Untergruppe empirischer Disziplinen zeichnet sich dadurch aus, dass sie ihre Daten in Form von kontrollierten Experimenten gewinnt. Die Bedeutung des kontrollierten Experiments gegenüber freier Feldbeobachtung liegt darin, dass in einem Experiment gezielt gewisse Konstellationen von Bedingungen hergestellt werden, welche Aufschluss über die kausale Relevanz bzw. Irrelevanz von Faktoren geben können (s. Kap. 4.2 – 4.3), während man in freier Feldbeobachtung eventuell ,ewig‘ darauf warten muss, dass der ,Zufall‘ genau die erwünschte Konstellation von Bedingungen beschert. Aus diesem Grund können experimentelle Wissenschaften wesentlich schneller und effektiver in einen empirischen Gegenstandsbereich eindringen als Wissenschaften, die auf bloße Beobachtung bzw. auf passive Rezeption des historischen Datenmaterials angewiesen sind. Das experimentelle Kriterium trennt die Disziplinen der Gruppen 1 – 3 zumindest tendenziell von jenen der Gruppen 4 – 6. Denn es ist sowohl aus technischen wie aus ethischen Gründen kaum möglich, mit Gesellschaftsgruppen, Institutionen oder Gesetzesentwürfen Experimente zu machen. Ethische Bedenklichkeitsfragen treten freilich schon bei biologischen oder psychologischen Experimenten auf. 3) Sezierende Wissenschaften: Eine signifikante Untergruppe der experimentellen Wissenschaften sind das, was ich – in Ermangelung eines besse-

Wertfreiheit

Kontrollierte Experimente

37

38

2. Gemeinsame Grundlagen der Wissenschaften

Realanalyse und Realsynthese in der Naturwissenschaft

ren Wortes – die ,sezierenden‘ Wissenschaften nenne. Darunter verstehe ich Wissenschaften, die ihre Gegenstände nicht bloß diversen experimentellen Bedingungen aussetzen, um sie theoretisch-begrifflich zu analysieren, sondern die ihre Gegenstände materiell analysieren, d. h. auseinandernehmen bzw. sezieren – eventuell sogar, um sie danach, sofern dies noch möglich, wieder ,zusammenzubauen‘ bzw. synthetisieren zu können. Ich meine, dass es dieser weit über das Experiment hinausgehende sezierende Charakter ist, der das wesentliches Charakteristikum der klassischen Naturwissenschaften wie Physik, Chemie, Biologie und Medizin ausmacht, im Gegensatz etwa zur experimentellen Verhaltensforschung oder Psychologie. Die Chemie und Biochemie hat es gelernt, die Gegenstände unserer Umgebung in ihre molekularen und schließlich atomaren Bestandteile zu zerlegen (Analytische Chemie) und neu zusammenzusetzen (Synthetische Chemie), und die Atomphysik hat dasselbe auf der subatomaren Ebene getan, wobei es hier um wesentlich gewaltigere Energiemengen geht. Die Analyse und Synthese von Lebewesen, an der die heutigen Naturwissenschaften arbeiten, von Stammzellenforschung, Gentechnik, Neurophysiologie bis zu künstlicher Intelligenz und Robotik, könnte darauf hinauslaufen, in ferner Zukunft einmal den ganzen Menschen künstlich zu verändern und letztlich erzeugen zu können. Wie immer gilt auch hier: wissenschaftliche Erkenntnisse können zu verschiedenen, ,guten‘ wie ,bösen‘ Zwecken verwendet werden. Sicher aber werden die damit möglich gewordenen Gefahren immer größer. Daher wird auch die Bedeutung der Wissenschaftsethik, welche unter anderem darin besteht, der technischen Umsetzung von wissenschaftlichen Möglichkeiten Grenzen zu setzen, in Zukunft noch viel größer werden als heute. Die Klassifikation ist in Abb. 2.4-1 zusammengefasst.

Spekulation Empirische Wissenschaften Experimentelle Wissenschaften Sezierende Wissenschaften

Abb. 2.4-1: Klassifikation der Realwissenschaften

Unabhängig davon lassen sich wissenschaftliche Disziplinen gemäß unseren bisherigen Ausführungen auch nach dem Grad der Quantifizierung und Mathematisierung ihrer Methodologien wie folgt einteilen:

2.5 Die Frage der Wertneutralität und das Abgrenzungsproblem

Graduelle Einteilung wissenschaftlicher Methodologien nach zunehmendem (L) logisch-mathematischen Präzisionsgrad: 4 logisch-mathematisch „quantitativ“ Logik Statistik und Messtheorie Technologie

Quantitative vs. qualitative Methoden

natursprachlich „qualitativ“ Hermeneutik Inhaltsanalyse Feldforschung

Quantitative Methoden liegen auf der linken und qualitative Methoden auf der rechten Seite dieses Spektrums. In beiden Fällen handelt es sich um wissenschaftliche Methoden: die ideologische Polarisierung zwischen einem quantitativen und einem qualitativen Methodenparadigma (wie z. B. in Lamnek 1988) erscheint aus allgemein-wissenschaftstheoretischer Sicht unnötig und übertrieben.

2.5 Die Frage der Wertneutralität und das Abgrenzungsproblem 2.5.1 Das Webersche Wertfreiheitspostulat. Die erste detaillierte Ausarbeitung der Wertfreiheitsforderung geht auf einen Hauptbegründer der Soziologie zurück, nämlich auf Max Weber (1864 – 1920). Weber war in eine Kontroverse mit Schmoller verwickelt, der die Auffassung propagierte, die Sozialwissenschaft solle sittliche Werturteile über die anzustrebende Gesellschaftsordnung erarbeiten. Gegen die Scheinobjektivität von solchen ,Kathederwertungen‘, d. h. mit der Autorität des Wissenschaftlers verkündeten Werturteilen, trat Weber energisch auf: Objektivität könne in der Wissenschaft nur erreicht werden, wenn der Wissenschaftler sich auf deskriptive Tatsachenaussagen beschränke und diese von seinen Werteinstellungen klar trenne (Weber 1968, Kap. II und X; vgl. Stegmüller 1979b, 177 – 181). Die Hauptgründe für diese Selbstbeschränkung der Wissenschaft auf deskriptive Urteile, die auch Max Weber anführt, lassen sich so zusammenfassen: Werte sind keine Eigenschaften, die den Gegenständen selbst innewohnen, sondern beruhen auf subjektiven Interpretationen durch uns Menschen. Ebenso sind Normen keine objektiven Tatsachen, sondern menschengemachte Forderungen, die gewissen Werten zur Realisierung verhelfen sollen. Es gibt bei der Frage der Überprüfung von Wert- und Normsätzen nichts, was einer empirischen Basis gemäß Annahme E4 und Methodenbausteinen M2 und M4 entspricht. Andererseits sind Wert- und Normsätze keineswegs bloß formale Behauptungen, die rein logisch begründbar wären. Daher gibt es im Bereich der Normen und Werte keine Objektivität im erfahrungswissenschaftlichen Sinne. Es liegt letztlich in der Freiheit des Menschen, sich zu gewissen Normen und Werten zu bekennen. Diese Begründung des Wertfreiheitspostulates ist prima facie nicht nur theoretisch, sondern auch praktisch plausibel. Jedermann weiß aus Erfahrung, dass die Herstellung eines Konsenses in Wertfragen zwischen Ange-

Webers Wertfreiheitsforderung

39

40

2. Gemeinsame Grundlagen der Wissenschaften

Positivismuskritik

Werte in den Wissenschaften

hörigen verschiedener Kulturen viel schwieriger ist als die Herstellung eines Konsenses in Sachfragen. Kein strenger Islamist würde an der westlichen Naturwissenschaft Anstoß finden; das westliche Wertesystem erscheint ihm dagegen abwegig. Dennoch wurde die Wertfreiheitsforderung speziell ab den 1960er Jahren seitens neomarxistischer Richtungen wie die der kritischen Theorie massiv kritisiert, und unter dem Namen „Positivismusstreit“ (Adorno et al. 1969) oder „Werturteilsstreit“ (Albert/ Topitsch 1971) flammte die Weber-Schmollersche Diskussion erneut auf. Die Vertreter der kritischen Theorie warfen der empirischen Sozialwissenschaft voralledem vor, eine rein deskriptiv bzw. ,positivistisch‘ vorgehende Sozialwissenschaft würde den Status Quo der unterdrückerischen Gesellschaftsverhältnisse nur theoretisch reproduzieren und dadurch implizit sanktionieren, während es einer kritischen Wissenschaft um die Veränderung der gesellschaftlichen Verhältnisse gehen müsse (vgl. Dahms 1994, 17). Dieser Vorwurf beruht auf einem fatalen Missverständnis: zur Erforschung des Deskriptiven gehört natürlich auch die Erforschung der Gesetzmäßigkeiten, und damit die Erforschung des naturgesetzlich, technisch oder praktisch Möglichen und Unmöglichen. Daher ist es gerade auch die empirische Sozialwissenschaft, die aufzeigen kann, wie gesellschaftliche Verhältnisse verändert werden können, und eben dadurch zu der von den Vertretern der kritischen Theorie geforderten Emanzipation beitragen kann. De fakto waren fast alle ,Neopositivisten‘ des Wiener Kreises politisch fortschrittlich orientiert und sahen viele ihrer Arbeiten in engem Zusammenhang mit gesellschaftlichen Veränderungen (Dahms 1994, 125). Der Unterschied ist nur, dass die kritisch-empirische Sozialwissenschaft, so wie sie im ,Positivismusstreit‘ von Popper und Albert gegenüber Horkheimer und Adorno verteidigt wurde, es mit dem Aufzeigen gesellschaftlicher Veränderungsmöglichkeiten belässt, und den werthaften Entschluss der demokratischen Meinungsbildung überlässt, anstatt auch noch diesen quasi-wissenschaftlich begründen zu wollen. Wenngleich also der Hauptvorwurf der kritischen Theorie ins Leere trifft, wurden in der Folgezeit auch berechtigte Einwände vorgetragen, die einer feineren Diskussion bedürfen. So wurde nicht nur von kritischen Theoretikern (z. B. Habermas 1968, 149 f.; s. Dahms 1994, 375), sondern auch von jüngeren analytischen Philosophen eingemahnt, dass Wert- und Normfragen keinesfalls einem Irrationalismus überantwortet werden dürfen. In der Tat ist eine rationale Behandlung von Wertfragen sowohl möglich als auch gesellschaftlich überaus notwendig, und seit den 1970er Jahren hat sich die Disziplin der Analytischen Ethik rasch etabliert (z. B. Frankena 1994). Auch in dieser Hinsicht ist es ratsam, sich Max Weber zuzuwenden, der die Wertfreiheitsforderung angesichts solcher Einwände bereits subtil differenziert hatte. Erstens, so Weber, ist es dem Sozialwissenschaftler durchaus möglich, das faktische Vorliegen von Wert- und Normensystemen deskriptiv zu erforschen, ohne dabei selbst zu werten (Weber 1968, 500 – 502). Wer die Vielehe im Islam erforscht, muss weder ein Befürworter noch ein Gegner derselben sein. Die Behauptung, dass eine Person oder Gesellschaft an gewisse Werte glaubt, ist selbst keine wertende, sondern eine deskriptive Behauptung, zu deren Bestätigung lediglich empirische Daten benötigt werden.

2.5 Die Frage der Wertneutralität und das Abgrenzungsproblem

Zweitens, so ebenfalls Weber (1968, 510 ff.), kann der Wissenschaftler qua Formalwissenschaftler logische Beziehungen zwischen Wert- und Normsätzen feststellen. Mittlerweile sind die hierfür relevanten Formaldisziplinen der deontischen Logik und der Entscheidungstheorie weit entwickelt (z. B. Aqvist 1984; Raiffa 1973). Drittens kann der Wissenschaftler mithilfe von Zweck-Mittel-Schlüssen aus vorgegebenen Normen und deskriptivem Wissen eine Reihe abgeleiteter Normen gewinnen und diese als Mittelempfehlungen an die Gesellschaft oder Politik weitergeben. 2.5.2 Wertneutralität und Zweck-Mittel-Schlüsse. Die Auffindung geeigneter Mittel für gegebene Zwecke ist ein wichtiger Aufgabenbereich deskriptiver Wissenschaft und gehorcht folgender logischen Form:

Zweck-MittelSchluss

(Def. 2.5-1) Das allgemeine Schema des Zweck-Mittel-Schlusses: Deskriptive Zweck-Mittel-Hypothese: M ist unter den gegebenen Umständen U ein notwendiges – oder alternativ: ein optimales – Mittel für die Realisierung von Z. Daher: Gegeben (fundamentale Norm:) Zweck Z soll realisiert werden, dann (abgeleitete Norm:) soll auch Mittel M realisiert werden. Dieser Zweck-Mittel-Schluss wird in den meisten ethischen Theorien als analytisch gültig akzeptiert (s. Schurz 1997a, Kap. 11.4). Dabei wird im Zweck-Mittel-Schluss vorausgesetzt, dass es sich bei M entweder um ein notwendiges oder ein optimales Mittel für Z handelt. Ist M dagegen ein bloß hinreichendes Mittel M für einen Zweck Z, dann ist der Zweck-MittelSchluss nicht gültig, denn ein und derselbe Zweck Z besitzt im allgemeinen verschiedene hinreichende Realisierungsmittel, und bei vielen dieser hinreichenden Mittel überwiegt der Schaden ihrer Nebenfolgen den Nutzen der Zweckerreichung. Für den Zweck, frische Luft ins Zimmer zu lassen, ist beispielsweise auch das Aufbrechen der Wand ein hinreichendes Mittel; ein optimales Mittel hierfür ist dagegen das Öffnen eines Fensters, und ein notwendiges Mittel hierfür ist irgendeine Öffnung ins Freie. Die fundamentale Norm übernimmt der Wissenschaftler z. B. vom Politiker oder von der Gesellschaft, und die mithilfe seines deskriptiven Wissens daraus abgeleitete Norm gibt er als Mittelempfehlung an die selbigen zurück. Dabei ist wesentlich, dass der Wissenschaftler seine Empfehlung explizit relativiert auf die jeweils vorausgesetzte fundamentale Norm. Illustrieren wir dies an einem Beispiel von Unterrichtstheorien aus der Erziehungswissenschaft (für das folgende vgl. Dubs 1982; Patry 1991). Angenommen, als anzustrebender Fundamentalwert sei kognitiver Lernerfolg (Beherrschung des Lernstoffs) vorgegeben. Aufgrund seiner Lerntheorie gelangt ein Unterrichtstheoretiker zu der deskriptiven Hypothese, dass kognitiver Lernerfolg genau dann maximiert wird, wenn zwischen 70 und 80 % der Unterrichtszeit lehrergesteuert (,gelenkt‘) abläuft. Der Unterrichtstheoretiker gibt diese Konklusion als abgeleitete Wert- bzw. Normaussage an die Lehrer weiter. Er befindet sich allerdings nur dann im Einklang mit der

Notwendige, hinreichende und optimale Mittel

41

42

2. Gemeinsame Grundlagen der Wissenschaften

Emanzipatorische Funktion der Wertneutralität

Erkenntnisparadigmen nach Habermas

Kritik der Positivismuskritik

Wertneutralitätsthese, wenn er seine Mittelempfehlung hypothetisch formuliert: wenn ihr maximalen kognitiven Lernerfolg wollt, dann solltet ihr zwischen 70 und 80 % der Unterrichtszeit lehrergesteuert unterrichten. Wird die Explizitmachung vorausgesetzter Werte unterlassen, dann kann dies ideologisch bedenkliche Folgen haben. Angenommen, in unserem Beispiel hätten die Lehrer andere Fundamentalzwecke als der Wissenschaftler; sie sehen soziales Lernen als gleichrangiges Unterrichtsziel neben kognitivem Lernerfolg an. Es sei nun deskriptiv erwiesen, eventuell sogar vom selben Wissenschaftler, dass für eine solche Zwecksetzung lediglich 60 % lehrergesteuerter Unterricht neben 40 % Eigen- und Gruppenaktivität die optimale Unterrichtsform sei. Angenommen aber, in unserem Beispiel gäbe unser Wissenschaftler seine Mittelkonklusion nicht hypothetisch sondern kategorisch weiter: 80 % des Schulunterrichts soll lehrergesteuert sein, und die Lehrer glauben ihm. Dann manipuliert der Wissenschaftler die Lehrer, denn er verschweigt, dass seine Mittelaussage nur unter seiner Wertannahme gilt, welche die Lehrer nicht teilen, und daher gemessen an ihren Zwecken vom Wissenschaftler schlecht beraten werden. Analoges gilt für viele andere Anwendungsfälle, von der Medizin (ein Arzt berät einen Patienten) über die geographische Raumplanung (Raumplaner beraten Politiker) bis hin zur Rechtswissenschaft (der Rechtsanwalt berät das vor der Scheidung stehende Ehepaar). In allen solchen Fällen ist es höchst bedeutsam, dass der wissenschaftliche ,Experte‘ die in seinen Mittelempfehlungen vorausgesetzten Wertannahmen explizit macht, denn nur so kann der beratene ,Kunde‘ herausfinden, ob dies auch wirklich seine eigenen Werte sind. Wir weisen damit nachdrücklich darauf hin, dass die Wertneutralitätsforderung keine dogmatisch-szientistische, sondern eine kritisch-emanzipatorische Funktion besitzt. Dies steht in Gegensatz zu der einflussreichen Dreiteilung von wissenschaftsphilosophischen Paradigmen nach ihrem jeweiligen Erkenntnisinteresse, die auf Habermas zurückgeht. Dieser unterscheidet nämlich wie folgt (vgl. Habermas 1968, 155 ff.; Konegen und Sondergeld 1985, 139, u. a. m.): 1.) empirisch-analytisches Paradigma – technisch-instrumentelles Erkenntnisinteresse 2.) hermeneutisches Paradigma – praktisches Erkenntnisinteresse 3.) kritisch-dialektisches Paradigma – emanzipatorisches Erkenntnisinteresse. Doch diese Zuordnung ist unrichtig. Wissenschaft in dem hier vertretenen, von Habermas vermutlich als empirisch-analytisch klassifiziertem Sinn, ist gar keinem bestimmten wissenschaftsexternen Erkenntnisinteresse zugeordnet, sondern dient unmittelbar nur dem wissenschaftsinternem Erkenntnisziel, der Suche nach möglichst allgemeinen und gehaltvollen Wahrheiten. Diese lassen sich ebenso gut für technische Zwecke wie für praktische Interpretationszwecke oder politische Reformzwecke auswerten. Dem empirischanalytischen Programm das Fehlen praktisch-emanzipatorischer Interessen zu unterstellen, ist schlichte Demagogie. Sich zur Wertneutralität zu bekennen, heißt für den Wissenschaftler eben nicht, auf werthaftes Engagement zu verzichten (vgl. Prim/Tilman 1979, 139), oder sich seiner gesellschaftlichen

2.5 Die Frage der Wertneutralität und das Abgrenzungsproblem

Verantwortung nicht bewusst zu sein. Wesentlich dafür ist nur, dass der Wissenschaftler seine fundamentalen Wertannahmen explizit macht und seine Zweck-Mittel-Schlüsse hypothetisch relativiert. Es sind nicht unterschiedliche Erkenntnisinteressen, sondern es ist der unterschiedliche Umgang mit der Wertneutralitätsfrage, worin sich die drei Habermasschen Methodenparadigmen eigentlich unterscheiden (s. Schurz 1998a). Um eine klare Trennung von Deskription und Wertung sind (leider!) nur die Vertreter des empirisch-analytischen Methodenprogramms bemüht, wogegen in hermeneutischen Ansätzen beschreibende Aussagen und Wertungen oft bewusst nicht unterschieden werden (vgl. Hilgendorf 2000, 39 ff.), und in kritisch-dialektischen Ansätzen gewisse emanzipatorische Normen in quasi-wissenschaftlicher Tarnung vorausgesetzt werden (vgl. Kriz et al. 1990, 151 f.). Die Schlussfolgerungen aus unseren bisherigen Überlegungen lassen sich wie folgt zusammenfassen. Die Forderung, Wissenschaft müsse gänzlich frei von Wertaussagen sein, ist sicherlich zu stark. Wert- und Normaussagen dürfen in dreierlei Form in den Wissenschaften auftreten, ohne den Geist des Weberschen Wertfreiheitspostulates zu verletzen: (1.) Wissenschaften können das empirische Vorliegen von Wertsystemen untersuchen; dann handelt es sich bei den diesbezüglichen Hypothesen nicht mehr um Wertoder Normsätze, sondern um deskriptive Sätze; (2.) Wissenschaften können logische Beziehungen zwischen Wert- und Normsätzen untersuchen, und (3.) sie können mithilfe von Zweck-Mittel-Schlüssen abgeleitete Werte relativ zu vorausgesetzten Fundamentalwerten empfehlen. Lediglich die Begründung von Fundamentalwerten liegt außerhalb des Bereichs deskriptiver Wissenschaft. Wir nennen diese These von nun an die Wertneutralitätsthese und kürzen sie mit (W) ab. 2.5.3 Das Abgrenzungsproblem. In der skizzierten Linie können wir nun alle Disziplinen analysieren und damit auch die Grenzen des Wissenschaftlichen bestimmen. Solange beispielsweise die Rechtswissenschaft empirisch vorliegende Rechtssysteme analysiert, oder logische und Zweck-Mittel-Beziehungen zwischen Normen feststellt, bleibt sie deskriptive Wissenschaft. Die logische Analyse von ethischen Werten und Normen betreibt nicht nur die juristische Dogmatik, sondern auch die analytische Ethik, insbesondere ihre Teildisziplin der sogenannten Metaethik, die sich ebenfalls zwanglos in das logisch-deskriptive Wissenschaftsprogramm einfügt. Literatur- und Kunstwissenschaften, die sich mit der ästhetischen Wirkung verschiedener Literatur- und Kunstgattungen befassen, fügen sich in den Rahmen empirisch-deskriptiver Wissenschaft ein, solange unter ästhetischer Wirkung eine faktische Wirkung auf den Menschen gemeint ist. Lediglich wenn absolute, nicht auf einem faktisch gegebenen Kunststil relativierte Aussagen über ästhetische Werte gemacht werden, wird der Rahmen deskriptiver Wissenschaften gesprengt. Dasselbe trifft auch auf die Richtungen der humanistischen Psychologie und Pädagogik zu, welche Werte wie Selbstentfaltung und Würde an die Stelle von objektiver Analyse setzen (Bühler und Allen 1982; Schiefele et al. 1979). Religionswissenschaften, sofern sie faktisch vorliegende Religionssysteme historisch oder logisch rekonstruieren, gehören zum deskriptiven Wissenschaftsprogramm, während

43

44

2. Gemeinsame Grundlagen der Wissenschaften

Abgrenzung von Wissenschaft

Einheitswissenschaft

Theologien, die gewisse religiöse Annahmen, die das „Credo“ der Religion ausmachen, dogmatisch voraussetzen, in doppelter Weise die Grenzen der Wissenschaft verlassen: erstens dort, wo sie spekulative Annahmen über Wesen und Existenz Gottes voraussetzen, und zweitens dort, wo sie fundamentale Wertannahmen voraussetzen. Zusammenfassend grenzen wir den Bereich der Wissenschaften wie folgt ein: Wissenschaften im hier verwendeten Sinn sind alle im weiten Sinne empirischen Disziplinen (gemäß E1 – 5, M1 – 4 und W) inklusive ihrer formalen Hilfswissenschaften und Zweck-Mittel-basierten Anwendungsdisziplinen. Diese Charakterisierung von „Wissenschaft“ liefert uns eine Abgrenzung der Wissenschaften von nichtwissenschaftlichen Disziplinen bzw. geistigen Betätigungsfeldern. Sie fungiert damit als Vorschlag zur Lösung des Abgrenzungsproblems. Dabei verstehen wir dieses Abgrenzungsproblem in einem anderen Sinne, als es der logische Empirismus oder der kritische Rationalismus verstand. Erstens identifizieren wir das Nichtwissenschaftliche nicht mit dem kognitiv Sinnlosen (wie z. B. Carnap 1961). Diese Identifikation ist inakzeptabel, weil empirisch gehaltlose Sätze wie „Gott existiert“ oder fundamentale Normsätze wie „Helfe deinem Nächsten“ dennoch verstanden werden können, also eine kommunizierbare Bedeutung haben. Zweitens stehen wir solchen Sätzen keineswegs abwertend gegenüber; wir bezeichnen sie lediglich deshalb als nichtwissenschaftliche Sätze, weil ihnen jegliche Möglichkeit einer empirischen Überprüfung fehlt. Drittens sehen wir die praktische Bedeutung des Abgrenzungsproblems anders als Carnap. Mit der Lösung des Abgrenzungsproblems verfolgen wir kein Eliminationsanliegen, sondern ein Erklärungsanliegen. Die faktischen Unterschiede in der evolutionären Erfolgs- bzw. Wirkungsbilanz der empirische Wissenschaften im Vergleich zu Ethik, Religion oder Kunst sind derartig markant, dass sie geradezu nach einer Erklärung rufen. Wer die machtvolle Eigendynamik von Wissenschaft und Technik auch nur im Ansatz verstehen will, muss herausfinden, was es ist, das Wissenschaft von nichtwissenschaftlichen Ideensystemen derart stark unterscheidet. Kurzum: Wissenschaftstheorie, die praktische Bedeutung erlangen will, muss sich dem Abgrenzungsproblem stellen. Aus unserer Abgrenzung des Wissenschaftsbegriffs ergibt sich auch ein Beitrag zur Frage der Einheitswissenschaften. Die Versuche, das Programm der Einheitswissenschaft auf dem Weg einer empirischen oder ontologischen Reduktion von höheren Wissenschaften auf die Physik zu leisten (Oppenheim/Putnam 1958; Causey 1977), können als kaum aussichtsreich gelten. Die Einheitlichkeit der Wissenschaften, wenn es sie gibt, ist vielmehr im Aufweis von epistemisch-methodologischen Gemeinsamkeiten zu suchen. Unsere Ausführungen zeigen, dass die Idee der Einheitlichkeit von Wissenschaften in bescheidenerer, aber umso nachhaltigerer Version weiterlebt: mit den erkenntnistheoretischen Annahmen E1 – 5, den Methodenbausteinen M1 – 4 und der Wertneutralitätsthese W haben wir jene Eigenschaften zu formulieren versucht, die allen empirischen Disziplinen trotz ihrer Unterschiedlichkeiten gemeinsam sind. 2.5.4 Präzisierung der Wertneutralitätsforderung. Um zur endgültigen Formulierung zu gelangen, bedarf es zwei weiterer Präzisierungsschritte.

2.5 Die Frage der Wertneutralität und das Abgrenzungsproblem

(1.) Eine Reihe von Autoren hat argumentiert, dass Wissenschaft schon deshalb nicht in einem absoluten Sinn wertfrei bzw. wertneutral sein kann, weil das Unternehmen „Wissenschaft“ ja selbst auf gewissen Werten bzw. Zielsetzungen beruht: den wissenschaftsinternen Werten (z. B. Schmidt 1971, 359 ff.; Sober 2007, 110). Wir haben in Kap. 2.1 die Suche nach möglichst allgemeinen und gehaltvollen Wahrheiten als das übergeordnete Erkenntnisziel aller Wissenschaften angeführt. Dieser fundamentale wissenschaftsinterne Wert wird in wissenschaftlichen Begründungen implizit vorausgesetzt und kommt insofern in allen Wissenschaften vor. Die Wertneutralitätsthese muss daher auf fundamentale wissenschaftsexterne Wertannahmen eingeschränkt werden. Man beachte, dass diese Einschränkung nicht impliziert, dass auch unsere These von der erfahrungswissenschaftlichen Unbegründbarkeit fundamentaler Wertsätze auf wissenschaftsexterne Werte einzuschränken wäre. Auch der wissenschaftsinterne Grundwert ist nicht erfahrungswissenschaftlich begründbar; er ist vielmehr per definitionem das oberste Ziel von Wissenschaft. (2.) Genügt die Einschränkung auf fundamentale wissenschaftsinterne Wert- bzw. Normsätze für eine haltbare Fassung der Wertneutralitätsforderung? Nein – sofern man Wissenschaft nicht lediglich als Begründungsprozess, sondern als gesellschaftlichen Prozess im Auge hat. Denn obwohl Werte nicht wissenschaftlich begründbar sind, so fließen doch an unzähligen Stellen des wissenschaftlichen Erkenntnisprozesses Wertungen notgedrungen mit ein, so dass wir zeigen müssen, in welcher Phase dieses Prozesses die Wertneutralitätsforderung relevant wird. Gehen wir aus von der üblichen Einteilung des wissenschaftlichen Forschungsprozesses in drei Phasen, nämlich dem Entstehungszusammenhang (EZ), in dem die wissenschaftlich relevanten Probleme zuallererst konzipiert werden, dem Begründungszusammenhang (BZ), in dem Daten erhoben und Hypothesensysteme generiert und überprüft werden, sowie dem Verwertungszusammenhang (VZ), in dem gut gesicherte Erkenntnisse zu verschiedensten gesellschaftlichen Zwecken verwertet werden. In (EZ) wird unter anderem der Untersuchungsgegenstand festgelegt und nach Relevanzgesichtspunkten selektiert. Dieser Prozess ist auch von wissenschaftsexternen Werten abhängig: welches Problem wichtig ist, entscheiden unter anderem wirtschaftliche oder politische Auftragsinteressen, oder die subjektiven Interessen der Einzelforscher. In (VZ) kommen wissenschaftsexterne Werte massiv ins Spiel: ob physikalische Hypothesen zu friedlichen oder zu Kriegszwecken, ob psychologische Erkenntnisse zur gesteigerter Arbeitseffizienz oder zur Förderung der Familienbeziehung eingesetzt werden, hängt davon ab, wer sie verwendet bzw. verwenden darf, weil er sie finanziert hat. Was die Wertneutralitätsthese fordert, bezieht sich ausschließlich auf den Begründungszusammenhang: in (BZ) dürfen wissenschaftsexterne Werte keine Rolle spielen. Damit können wir die Wertneutralitätsthese abschließend wie folgt formulieren und schematisch darstellen: (Ms. 2.5-1) Wertneutralitätsthese (bzw. -forderung): Ein bestimmter Bereich der Wissenschaften, nämlich ihr Begründungszusammenhang, soll frei sein von fundamentalen wissenschaftsexternen Wertannahmen

Wissenschaftsinterne vs. -externe Werte

Drei Phasen des Forschungsprozesses

45

46

2. Gemeinsame Grundlagen der Wissenschaften EW & IW

nur IW

EW & IW

EZ

BZ

VZ

Abb. 2.5-1: Schematische Darstellung der Wertneutralitätsforderung Legende: EZ – Entstehungszusammenhang, BZ – Begründungszusammenhang, VZ – Verwertungszusammenhang; IW – wissenschaftsinterne Werte, EW – wissenschaftsexterne Werte

Korrektiver Rückbezug des BZ auf den EZ

Eine wichtige Konsequenz dieses Schemas der Wertneutralität ist die korrektive Rückbezüglichkeit des Begründungszusammenhanges auf die Problemselektion im Entstehungszusammenhang. Die in (EZ) vorgenommene Selektion der untersuchten Parameter bzw. Merkmale ist nämlich keinesfalls erkenntnisneutral, sondern wirkt sich auf den Begründungszusammenhang aus, insofern sie das, was sich als Resultat der wissenschaftlichen Analyse ergeben kann, einschränkt. Es kann sich im Verlauf der wissenschaftlichen Forschung herausstellen, dass zur Erfassung der relevanten Ursachen die Berücksichtigung von Parametern nötig ist, von denen in (EZ) bereits abstrahiert wurde. Daher darf die in (EZ) vorgenommene Selektion nur vorläufig sein und muss einer späteren Kritik und Korrektur durch Erkenntnisse in (BZ) zugänglich sein, und zwar auch dann, wenn dies den externen Ausgangswerten, also den Ursprungszielen des Forschungsprojekts, zuwiderläuft. Andernfalls könnte man nicht von einer Unabhängigkeit des (BZ) von externen Wertentscheidungen sprechen. Wir illustrieren dies an einem Beispiel: die klassische Schulmedizin war lange Zeit rein biologisch-physiologisch orientiert; zu einem gewissen Teil ist sie dies auch noch heute. Eine solche Schulmedizin abstrahiert also bereits im Entstehungszusammenhang medizinischer Forschung von psychischen Parametern. Mittlerweile kennt man viele organische Krankheiten mit psychischen Ursachen, und die Bedeutung der psychosomatischen Interaktion ist nicht von der Hand zu weisen. Ideologische Kämpfe von Befürwortern und Gegnern psychosomatischer Medizin mögen entbrennen. Doch wissenschaftstheoretisch gesehen ist die Frage des psychosomatischen Zusammenhangs keine ideologische, sondern eine theoretische Frage: um sie überhaupt sachlich untersuchen zu können, muss die Einschränkung auf biologisch-physiologische Faktoren in medizinischen Modellen aufgegeben und der Bereich psychischer Phänomene miteingeschlossen werden. Mit anderen Worten, die ursprüngliche und extern motivierte Einschränkung im (EZ) muss aufgrund von im (BZ) gewonnenen Erkenntnissen aufgegeben werden.

2.6 Wissenschaftliches Schließen und Argumentieren

2.6 Wissenschaftliches Schließen und Argumentieren In den Methodenbausteinen M3 und M4 finden wissenschaftliche Prozesse des Argumentierens und Schließens statt. Dabei hat man es mit sehr unterschiedlichen Schlussprozessen zu tun, die in diesem Kapitel näher erläutert werden. 2.6.1 Deduktion und Induktion. Wenn wir gemäß M3 von einer strikt-allgemeinen Gesetzeshypothese zusammen mit Beobachtungssätzen auf weitere Beobachtungssätze schließen, so handelt es sich um einen logisch-deduktiven Schluss. Schließen wir dagegen gemäß M4 aus der Tatsache, dass viele aktuale Beobachtungssätze bisher eine Gesetzeshypothese bestätigt haben, auf die vermutliche Wahrheit der Hypothese, so handelt es sich um einen induktiven Schluss, genauer gesagt, um einen induktiven Generalisierungsschluss. Beispiele: (2.6-1) Deduktiver Schluss (Logik i. e. S.): Alle Fische sind Kiemenatmer. Dieses Tier ist ein Fisch.

Induktiver Generalisierungsschluss: Alle bisher beobachteten Fische (Nr. 1, 2, …, n) waren Kiemenatmer.

Also ist dieses Tier ein Kiemenatmer. Also sind (wahrscheinlich) alle Fische Kiemenatmer. Sicher: Wahrheitsübertragung in Unsicher: Wahrheitsübertragung nur allen möglichen Welten. in genügend ,uniformen‘ möglichen Welten. Der entscheidende Unterschied liegt darin, dass nur deduktive Schlüsse die Wahrheit mit Sicherheit, also in allen möglichen ,Welten‘, von den Prämissen auf die Konklusion übertragen: wenn die Prämissen (die Sätze oberhalb des Trennungsstriches) wahr sind, dann ist auch die Konklusion (der Satz unterhalb des Trennstriches) mit Sicherheit wahr. Deduktive Schlüsse bilden den Gegenstand der Logik im engeren Sinne. (Manche Autoren stellen der deduktiven Logik eine induktive Logik gegenüber, z. B. Carnap 1950b, § 43 ff.; diese Terminologie ist jedoch umstritten.) Induktive Generalisierungsschlüsse sind hingegen grundsätzlich unsicher – der Doppelstrich in (2.6-1) zeigt diese Unsicherheit an. Denn die Prämissen eines induktiven Generalisierungsschlusses sprechen nur über die bisher beobachteten Anwendungsfälle, während die Konklusion eine unbeschränkte Generalisierung auf alle und insbesondere alle zukünftigen Anwendungsfälle vornimmt. Man sagt daher auch, induktive Schlüsse sind gehaltserweiternd. Nur in solchen Umständen bzw. Welten, die hinreichend regelhaft sind, deren Zukunft der Vergangenheit hinreichend ähnlich ist, kann man verlässlich aus der Wahrheit der Prämissen auf die Wahrheit einer induktiven Konklusion schließen. Nichts kann logisch garantieren, dass der zukünftige Weltverlauf dem bisherigen Weltverlauf gleichen wird. Schon morgen könnten mittlerweile ausgestorbene kiemenlose Fische entdeckt werden, oder durch eine Mutation kiemenlose Fische entstehen.

Deduktives vs. induktives Schließen

47

48

2. Gemeinsame Grundlagen der Wissenschaften

Humes Induktionsproblem

Dies war (in groben Zügen) das Hauptargument von David Hume, der als erster Philosoph gezeigt hat, dass eine Begründung des induktiven Schließens mit den Mitteln der deduktiven Logik unmöglich ist. Hume hat aber, noch weitergehend, die rationale Begründbarkeit von Induktion insgesamt bezweifelt (s. Hume 1748, 4. Abschn./2. Teil, 6. Abschn.). Induktion lässt sich, zweitens, auch nicht durch Beobachtung rechtfertigen, denn in induktiven Schlüssen schließt man ja auf das, was man noch nicht beobachtet hat. Und drittens lässt sich induktives Schließen auch nicht durch Induktion rechtfertigen, denn dies würde geradewegs in einen Zirkelschluss führen: wenn man induktives Schließen für verlässlich hält, weil es sich bisher bewährt hat, so setzt man dabei voraus, was sich bisher bewährt hat, wird sich – per Induktionsschluss – auch in Zukunft bewähren. In einem solchen Zirkelschluss setzt man genau das voraus, was man zeigen will, und zeigt dadurch gar nichts. Alle drei traditionellen wissenschaftlichen Begründungsmethoden – Logik, Erfahrung und Induktion – scheinen also für die Induktion zu versagen. Hume hat auch als erster gezeigt, dass eine wahrscheinlichkeitstheoretische Umformulierung des Induktionsproblems nicht weiterhilft. Wenn man argumentiert, dass induktive Schlüsse nicht immer, aber zumindest meistens bzw. mit hoher Wahrscheinlichkeit von wahren Prämissen zu wahren Konklusionen führen, so setzt man ein probabilistisches Induktionsprinzip voraus, demzufolge sich die bisher beobachteten Ereignishäufigkeiten auf die Zukunft bzw. auf die nichtbeobachteten Fälle übertragen lassen, was nur dann der Fall ist, wenn unsere Wirklichkeit hinreichend uniform ist. Das waren die Gründe, die Hume zu der zutiefst skeptischen These führten, induktives Schließen sei keiner rationalen Begründung fähig, sondern beruhe lediglich auf psychologischer Gewohnheit. Eine befriedigende Lösung des Humeschen Induktionsproblems zu finden wäre von größter Wichtigkeit. Und dennoch blieb dieses Problem bis heute weitgehend ungelöst. Eine Reihe von Lösungsversuchen, auf die hier nicht näher eingegangen werden kann, wurden in der analytischen Philosophie des 20. Jahrhunderts zwar diskutiert, doch sie fanden keine allgemeine Akzeptanz (s. Stegmüller 1971; Swinburne 1974; Rescher 1987; Howson 2000; Ladyman 2002, Kap. 1). Der überwiegenden Meinung gegenwärtiger Philosophen, das Induktionsproblem sei theoretisch unlösbar, schließe ich mich allerdings nicht an. Eine aussichtsreiche Rechtfertigung der Induktion, die von den bisher genannten Argumenten nicht getroffen wird, liegt m.E. in der auf Reichenbach zurückgehenden entscheidungstheoretischen Rechtfertigungsidee von Induktion als bester Erkenntnisalternative (Reichenbach 1949, § 91). Dieser Idee zufolge sind Induktionen das Beste, was wir tun können, wenn wir unser Erkenntnisziel, wahre Allaussagen und speziell Voraussagen über die Zukunft zu gewinnen, überhaupt erreichen wollen. Wir können zwar nicht beweisen, dass wir damit immer oder auch nur häufig Erfolg haben; aber wenn wir überhaupt verlässliche Aussagen über die Zukunft gewinnen wollen, so ist Induktion der einzige oder zumindest der beste Weg dazu. Auch gegen Reichenbachs Idee gibt es schwerwiegende Einwände (s. Skyrms 1989, 82 – 84); es gibt aber auch begründete Hoffnungen, die mit diesem Ansatz verbundenen Probleme spieltheoretisch lösen zu können (s. Schurz 2008b).

2.6 Wissenschaftliches Schließen und Argumentieren Das Allgemeine (Gesetze und Theorien)

induktiver Aufstieg

Abb. 2.6-1: Das induktivdeduktive Schema

deduktiver Abstieg

Das Besondere (Beobachtungen)

Bisher haben wir Induktion und Deduktion im Sinne des sogenannten induktiv-deduktiven Schemas erläutert. Diesem Schema zufolge, welches sich bereits bei Aristoteles findet, ist Induktion eine Methode, um vom Besonderen auf das Allgemeine zu schließen, und Deduktion eine Methode, um von Allgemeinen zurück auf das Besondere zu schließen (s. Abb. 2.6-1). Obwohl dieses Schema einige wichtige methodische Merkmale der Wissenschaften einfängt, so ist es doch in mehreren Hinsichten übervereinfacht bzw. inkorrekt. Die binäre Einteilung von Schlussarten, auf der es beruht, ist nämlich zu eng – es gibt weitaus mehr Schlussarten, die wir hier kurz auflisten. Was das Allgemeine betrifft, so müssen wir dabei zwischen strikten Allsätzen und statistischen Generalisierungen unterscheiden.

Das induktivdeduktive Schema

1. Deduktive Schlusstypen: 1.1 Vom Allgemeinen auf das Besondere – z. B.: Alle F sind G, dies ist ein F, daher: dies ist ein G. 1.2 Vom Allgemeinen auf das Allgemeine – z. B. Alle F sind G, Alle G sind H, daher: Alle F sind H. 1.3 Vom Besonderen auf das Besondere – z. B. dies ist F oder G, dies ist nicht F, daher: dies ist G. Dagegen gibt es keine deduktiven Schlüsse vom Besonderen auf das (essentiell) Allgemeine.

Deduktive und induktive Schlussarten

2. Induktive Schlusstypen (s. auch Carnap 1950b, 207 f.): 2.1 Vom Besonderen auf das Allgemeine: 2.1.1 Der strikt-induktive Generalisierungsschluss: Alle bisher beobachteten Fs waren Gs, also sind (wahrscheinlich) alle Fs Gs. 2.1.2 Der statistisch-induktive Generalisierungsschluss: r % aller bisher beobachteten Fs waren Gs, also sind (wahrscheinlich) zirka r % aller Fs Gs. 2.2 Vom Besonderen auf das Besondere – der induktive Voraussageschluss: Alle bisher beobachteten Fs waren Gs, also ist wahrscheinlich auch das nächste F ein G. 2.3 Vom Allgemeinen auf das Besondere – der induktiv-statistische Spezialisierungsschluss: r % aller Fs sind Gs, dies ist ein F, also wird dies mit r % Glaubenswahrscheinlichkeit ein G sein. 2.6.2 Popper und die Bedeutung induktiven Schließens in den Wissenschaften. Der Begründer des kritischen Rationalismus, Karl Popper, hat die These vertreten, dass das Induktionsproblem einerseits unlösbar sei, dass aber andererseits die Methode der Erfahrungswissenschaft gänzlich ohne induktive Schlüsse auskommt (1935/76, Kap. I). Im folgenden wollen wir aufgrund einer Unterscheidung von drei Arten von Induktion aufzeigen, dass

49

50

2. Gemeinsame Grundlagen der Wissenschaften

Drei Arten der Induktion

Poppers erste Induktionskritik

Carnaps induktive Wahrscheinlichkeit

Poppers Argumente wichtige Einsichten zutage fördern, dass aber seine radikale Behauptung, Erfahrungswissenschaft käme ohne Induktion aus, unhaltbar ist (s. auch Schurz 1998b, 2002a). (1.) Die methodische Induktion. Diese Auffassung von Induktion knüpft direkt an das in Kap. 2.6.1 erläuterte induktiv-deduktive Schema an. Ihr zufolge ist Induktion eine Methode, um von einzelnen Beobachtungen durch deren induktive Generalisierung auf allgemeine Gesetze und Theorien zu schließen. Popper kritisiert an der methodischen Induktion die darin involvierte Ansicht, dass Induktion die zentrale Methode der Gewinnung von Gesetzeshypothesen und Theorien sei. Der Glaube, dass eine solche Entdeckungsmethode in der Wissenschaft überhaupt benötigt werde, beruht, so Popper, auf einer Verwechslung von Entdeckungs- und Begründungszusammenhang (1935/76, Kap. I.1 – I.2). Wie wissenschaftliche Hypothesen gewonnen werden, ob durch Induktion, Intuition oder durch trial & error, ist für den Begründungszusammenhang von Wissenschaft gänzlich belanglos. Methodische Induktion sei somit für den Begründungszusammenhang von Wissenschaft entbehrlich. Bringen wir uns dagegen Kap. 2.1 in Erinnerung: es muss nicht sein, aber es kann sein, dass bestimmte Methoden der Hypothesengewinnung zugleich ein gewisses Maß an Begründung mitliefern. Dies trifft z. B. auf den in Kap. 2.6.1 erläuterten strikt-induktiven und statistisch-induktiven Generalisierungsschluss zu: dieser Schluss ist eine Methode, eine empirische Gesetzeshypothese zu entdecken, welche diese zugleich induktiv begründet, d. h. vorläufig bestätigt. Raffiniertere induktive Entdeckungsalgorithmen finden sich in Langley et al. (1987) (s. Kap. 4.2.2). Hypothesengenerierungsverfahren, welche zugleich eine Begründung mitliefern, sind kognitiv natürlich wesentlich effizienter als blinde trial & error-Verfahren. In zwei wichtigen Punkten ist Poppers Kritik an der methodischen Induktion jedoch im Recht. (1.) Hypothesen müssen nicht durch methodische Induktion gewonnen werden – alle Hypothesengewinnungsmethoden sind erlaubt und verdienen kritische Überprüfung. (2.) Zwar gibt es induktive Generierungsmethoden für empirische Gesetzeshypothesen. Für die Konstruktion von wissenschaftlichen Theorien gibt es solche induktiven Generierungsmethoden im allgemeinen nicht mehr. Schon sehr einfache Theorien – wie z. B. das Galileische Fallgesetz, welches die Idealisierung des reibungsfreien Falls unterstellt – sind nicht mehr allein durch induktive Verallgemeinerungsprozeduren aus der Beobachtung gewinnbar, denn sie enthalten Begriffe, die über das Beobachtbare hinausgehen – sogenannte theoretische Begriffe, wie z. B. „reibungslose Bewegung“, „Masse“ oder „Kraft“. Poppers Kritik an der Gewinnbarkeit wissenschaftlicher Theorien durch methodische Induktion (s. 1983, 118) ist daher weitgehend berechtigt. (2.) Die logische Induktion. Die logische Induktionsauffassung, die auf Carnap (1950b) zurückgeht, versteht Induktion nicht als eine Methode der Entdeckung, sondern der Begründung von wissenschaftlichen Hypothesen, nämlich als Methode der Feststellung des Bestätigungsgrades von Theorien aufgrund gegebener Beobachtungsdaten. Dies soll durch Errichtung eines Systems der induktiven Wahrscheinlichkeit geschehen, worin die induktive

2.6 Wissenschaftliches Schließen und Argumentieren

Wahrscheinlichkeit einer Theorie T bei gegebenen Beobachtungsdaten B auf rationale Weise bestimmt werden kann (zu Weiterführungen dieses Ansatzes s. z. B. Carnap/Jeffrey 1971; Hintikka/Suppes 1966, Hg.). Statt dem induktiven Bestätigungsgrad kann man auch alternativ den Grad der Wahrheitsnähe einer Theorie T bei gegebenen Beobachtungsdaten B als zentrales Bewertungsmaß ansehen (dazu Kap. 5.6.2). Poppers Kritik an der logischen Induktionsauffassung besagt nun folgendes: wir können zwar feststellen, dass relativ zu gegebenen Beobachtungen eine Theorie T1 besser bestätigt bzw. bewährt ist als eine andere Theorie T2, oder dass T1 wahrheitsnäher ist als T2, doch wir können keinen Absolutbetrag für die Wahrscheinlichkeit bzw. Wahrheitsnähe von T1 bzw. T2 angeben, denn schon morgen kann eine neue Theorie T3 entwickelt werden, die sowohl T1 wie T2 weit in den Schatten stellt (z. B. Popper 1983, 19 f., 23). Poppers Argument stellt eine wesentliche Einsicht in die Grenzen der logischen Induktionsauffassung dar. Denn um die induktive Wahrscheinlichkeit oder die Wahrheitsnähe einer Theorie T bei gegebenen Beobachtungsdaten B als numerischen Wert zu bestimmen, muss man unter anderem über eine vollständige Liste aller betrachteten Alternativtheorien {T1, …, Tn} verfügen. Eine nicht-willkürliche Aufstellung einer solchen Liste ist aber zumindest im Fall von Theorien unmöglich. Zu einer gegebenen Theorie T gibt es immer unnennbar viele mögliche Alternativtheorien, da diese beliebige neue theoretische Begriffe enthalten können. Es gibt keinen begrenzten sprachlichen Möglichkeitsraum, der alle möglichen Alternativtheorien enthält (s. auch Howson 2000, 46). Erneut liegt die Sachlage im Falle empirischer Hypothesensysteme anders. Hier ist es möglich, eine endliche Liste aller möglichen Alternativhypothesen in einem begrenzten Beobachtungsvokabular aufzustellen. Betrachten wir z. B. die Menge aller Intervallhypothesen der Form Hn = „zwischen n und n+1 % aller Menschen sind braunäugig“, mit 0  n  99, und nehmen wir eine Gleichverteilung der Ausgangswahrscheinlichkeiten an, so lässt sich die induktive Wahrscheinlichkeit von Hn, gegeben eine Stichprobe von m Menschen, darunter k ( m) braunäugigen, aufgrund der Kolmogorovschen Wahrscheinlichkeitsaxiome und dem principal principle berechnen (s. Kap. 4.5.2). Zusammengefasst stellt Poppers Kritik der logischen Induktion für empirische Gesetzeshypothesen eine Übertreibung dar; sie trifft jedoch auf den Fall wissenschaftlicher Theorien unumwunden zu. Aufgrund Poppers Kritik an der logischen Induktion können wir konkurrierende Theorien in Hinblick auf ihren Bewährungs- bzw. Schwächungsgrad bei gegebener Datenlage lediglich komparativ (d. h. vergleichend) bewerten. Haben wir einen solchen empirischen Erfolgsvergleich der derzeit vorhandenen Alternativtheorien durchgeführt, so stützen wir unsere zukünftigen Prognosen und Handlungen auf jene Theorie(n), die bisher am erfolgreichsten waren, m. a. W. auf die, welche sich bisher am besten bewährt haben. Dies wird selbstverständlich auch von Popper so gesehen (1983, 65; 1974, 34). Daraus folgt, dass auch im Kern von Poppers ,deduktivistischem‘ Bewährungsprogramm ein fundamentaler Induktionsschritt enthalten ist – eine dritte Art von Induktion, die ich mit Musgrave (2002) als die epistemische Induktion bezeichne:

Poppers zweite Induktionskritik

51

52

2. Gemeinsame Grundlagen der Wissenschaften

Unverzichtbarkeit der epistemischen Induktion

Komparative Theorienbewertung

(3.) Die epistemische Induktion besagt folgendes: Wenn eine Theorie T1 bisher (explanativ und prognostisch) erfolgreicher war als T2, so ist es relativ zum gegebenen Beobachtungsstand wahrscheinlich, dass T1 auch in Zukunft erfolgreicher sein wird als T2. Die bisher ermittelten Erfolgspräferenzen zwischen gegebenen Theorien werden also induktiv in die Zukunft projiziert. Das Induktionsprinzip heißt „epistemisch“, weil dabei nicht Objekthypothesen, sondern epistemische Metahypothesen über den Bewährungsgrad unserer Hypothesen induktiv projiziert werden (vgl. Kuipers‘ rule of success, 2000, Kap. 6.1.3). Das epistemische Induktionsprinzip ist in der Tat für alle empirischen Wissenschaftsdisziplinen unverzichtbar. Würde dieses Prinzip nicht angenommen werden, so wäre die Poppersche Methode der Bewährungsproben ohne jegliche Pointe. Bisheriger Erfolg wäre dann einfach irrelevant für unser zukünftiges Handeln. Obwohl beispielsweise die Auffassung, dass schwere Körper auf der Erde zu Boden fallen und nicht frei schweben, sich bisher besser bewährt hat als die gegenteilige Auffassung, so wäre dies kein Grund, diese Auffassung auch zur Grundlage unseres zukünftigen Handelns zu machen, da logisch gesehen schon ab morgen alle Körper frei zu schweben beginnen könnten. Leider hat Popper zeitlebens, in überbetonter Abgrenzung zum logischen Empirismus, jegliche Art der Induktion verurteilt. Doch seine diesbezüglichen Argumente in (1983, z. B. 66 f.) scheinen mir eher auf sprachliche Wortwendungen als auf sachliche Gründe hinauszulaufen (s. Schurz 1998b). Auch kritische Rationalisten wie Watkins (1984, 340 f.) und Musgrave (2002) haben das epistemische Induktionsprinzip anerkannt. Epistemische Induktion, so halten wir fest, ist immer nur komparativ. Eine wichtige Konsequenz davon ist folgende: gemäß der epistemischen Induktion ist der Bewährungsgrad einer Theorie grundsätzlich doppelt relativ, und zwar – relativ zum gegebenen Stand des Beobachtungswissens, sowie – relativ zum derzeitigen Stand der Alternativtheorien. 2.6.3 Abduktion und Schluss auf die beste Erklärung. Mit induktiven Generalisierungsschlüssen lässt sich zwar von Einzelbeobachtungen auf empirische Gesetzeshypothesen schließen, nicht aber von empirischen Gesetzeshypothesen auf wissenschaftliche Theorien. Denn mit induktiven Schlüssen ist es nicht möglich, neue theoretische Begriffe in die Konklusion einzuführen, welche in den Prämissen nicht enthalten sind. Es gibt jedoch eine weitere – allerdings kontroverse – Schlussart, mit der dies möglich ist, und zwar die Abduktion bzw. der Schluss auf die beste Erklärung. Grob geschlossen schließt man hier von einer beobachteten Wirkung auf eine vermutete Ursache, z. B. von einer sich dahinschlängelnden Spur im Sand auf eine Sandviper, die hier vorbeikroch. An dieser Stelle muss vor einem terminologischen Missverständnis gewarnt werden. Einige Autoren fassen den Begriff der Induktion in einem weiten Sinn auf – sie identifizieren induktive Schlüsse mit jeglicher Art gehaltserweiternder bzw. nicht-deduktiver Schlüsse, rechnen also auch die abduktiven Schlüsse zu den induktiven hinzu (z. B. Earman 1992; Bird 1998, 13). Ich verstehe hier Induktion immer im oben angeführten Hume-

2.6 Wissenschaftliches Schließen und Argumentieren

schen Sinne, und sehe Abduktion als eine zweite nicht-deduktive bzw. gehaltserweiternde Schlussart an. Die Schlussart der Abduktion geht auf C. S. Peirce zurück. In (1878) explizierte Peirce den abduktiven Schluss formal als Retrodiktionsschluss von „Alle Fs sind Gs“ und „dies ist ein G“ auf „dies ist ein F“. Diese Charakterisierung von Abduktion dominiert zwar die computerwissenschaftliche Literatur zur Abduktion (s. Flach/Kakas 2000); sie ist aber viel zu eng, um das zu erfassen, was Peirce als Wissenschaftstheoretiker mit Abduktion bezweckte. Für Peirce ist es wesentlich, dass durch abduktives Schließen in den Wissenschaften neue theoretische Modelle eingeführt werden können (1903, § 170). Z. B. schloss Newton aus der Bewegung der Planeten um die Sonne abduktiv auf die Existenz einer Gravitationskraft. Auch der erkenntnistheoretische Schluss von innersubjektiven Wahrnehmungserlebnissen auf Realbeobachtungen ist ein abduktiver Schluss: die Annahme, dass meine Seherlebnisse durch von mir unabhängig existierende Objekte bewirkt werden, ist die beste Erklärung für die immer wiederkehrenden Regelmäßigkeiten in meinen visuellen Erlebnissen (s. z. B. Moser 1989, 91 f.). Wie in Schurz (2008a) gezeigt wird, umfasst Abduktion eine ganze Familie von Schlussarten, die folgendes Schema gemeinsam haben.

Abduktion nach Peirce

(2.6-2) Allgemeines Schema der Abduktion (s. auch Peirce 1903, § 189; Niiniluoto 1999): Prämisse 1: Ein erklärungsbedürftiges (singuläres oder generelles) Faktum E. ,Prämisse‘ 2: Ein Hintergrundwissen W, das für eine gewisse Hypothese(nmenge) H impliziert: H ist eine potentielle Erklärung für E (d. h., ist H wahr, so erklärt H E). Abduktive Vermutung: H ist wahr. Wie Peirce selbst betont hat, ist der Geltungsstatus einer abduktiv erschlossenen Hypothese ein ganz vorläufiger: die abduzierte Hypothese muss im weiteren Verlauf durch Deduktion und Induktion empirisch getestet werden, um den Charakter einer wahrscheinlichen Hypothese anzunehmen (1903, § 171). Zudem gibt es im gegebenem Hintergrundwissen immer mehrere mögliche Hypothesen, H1, … Hn, welche die gegebene Evidenz E potentiell erklären; und die abduktive Vermutung wählt jene Hypothese aus, die am plausibelsten erscheint. In diesem Sinne hat Harman (1965) das Peircesche Abduktionskonzept in den folgenden Schluss auf die beste bzw. best-verfügbare Erklärung (SBE) transformiert: (2.6-3) (Schluss auf die beste Erklärung – SBE): Schließe abduktiv auf die plausibelste Erklärungshypothese unter allen möglichen bzw. im Hintergrundwissen verfügbaren Erklärungshypothesen für E. Gemäß dem Verständnis von Harman (1965) und anderen SBE-Vertretern (z. B. Lipton 1991, Bartelborth 1996) wird in einem SBE-Schluss vom empirischen Erfolg einer Theorie auf ihre vermutliche Wahrheit im realistischen

Harmans SBE

53

54

2. Gemeinsame Grundlagen der Wissenschaften

Kuipers’ Korrektur von Harmans SBE

Sinn zu geschlossen. Kuipers (2000, Kap. 7.5.3) bringt dagegen denselben Einwand ins Spiel, den Popper gegen die logische Induktion vorbrachte: de fakto können wir immer nur zu komparativen Urteilen gelangen, denen zufolge eine Theorie vermutlich wahrheitsnäher ist als alle uns bekannten Alternativtheorien. Kuipers schlägt daher vor, das Schema der Abduktion bzw. des SBE in folgender Weise zu modifizieren: erstens spricht er, statt vom „Schluss auf die beste Erklärung“, vom „Schluss auf die beste Theorie“, und zweitens wird in Kuipers‘ Vorschlag von der Prämisse, dass eine Theorie die empirisch adäquateste unter allen bekannten Alternativtheorien ist, nicht darauf geschlossen, dass T vermutlich wahr ist, sondern nur darauf, dass T unter allen bekannten Alternativtheorien die wahrheitsnächste Theorie ist. Das Zusammenwirken des epistemischen Induktionsschlusses und des abduktiven Schlusses auf die beste Theorie ist in Abb. 2.6-2 dargestellt. (1) Evidenz: Tk ist unter den Alternativtheorien T1, …, Tn bisher am empirisch erfolgreichsten h h epistemischer Induktionsschluss h h f (2) Instrumentalistische Konklusion: Tk ist unter T1, …, Tn empirisch am adäquatesten (daher auch in Zukunft empirisch am erfolgreichsten)

h h abduktiver Schluss h auf die beste Theorie h f (3) Realistische Konklusion: Tk ist unter T1, …, Tn am wahrheitsnächsten. Abb. 2.6-2: Zusammenwirken von epistemischer Induktion und Abduktion

Realismus und Instrumentalismus

In Abb. 2.6-2 wird auch erkenntlich, wie diese beiden Schlussarten mit den wissenschaftstheoretischen Positionen des Instrumentalismus und des Realismus zusammenhängen. Wissenschaftstheoretische Instrumentalisten wie z. B. van Fraassen akzeptieren den Schluss von (1) auf (2), lehnen aber den abduktiven Schluss von (2) auf (3) ab, denn ihnen zufolge können Theorien nur mehr oder weniger empirisch adäquat, aber nicht im realistischen Sinne wahr oder falsch sein (van Fraassen 1980, 11 f.; 1989, 142 f.). Wissenschaftstheoretische Realisten glauben dagegen an die vermutliche Wahrheit bzw. Wahrheitsnähe von gut bestätigten Theorien; für sie ist gerade der Schluss von (2) auf (3) wesentlich. 2.6.4 Monotone und nichtmonotone Schlüsse. Mit der Unsicherheit von nicht-deduktiven (induktiven oder abduktiven) Schlüssen verbindet sich ihre Eigenschaft der Nichtmonotonie. Im Gegensatz dazu sind deduktive Schlüssen monoton. Genauer gesagt ist die Monotonie eines Schlusses folgende Eigenschaft:

2.6 Wissenschaftliches Schließen und Argumentieren

(Def. 2.6-1) Monotonie: Ein gültiger Schluss von Prämissen P1, …, Pn auf die Konklusion K heißt monoton g. d. w. (genau dann, wenn) er auch nach Hinzufügung beliebiger weiterer Prämissen gültig bleibt.

(Ms. 2.6-1) Alle deduktiven Schlüsse sind monoton, d. h., sie erfüllen die Monotonieregel: P1, …, Pn /K ist gültig r für beliebige Q: Q, P1, …, Pn /K ist gültig. Unsichere Schlüsse besitzen diese Monotonieeigenschaft nicht. Die logische Untersuchung nichtmonotonen Schließens ist seit den 1980er Jahren ein bedeutendes Gebiet geworden (vgl. Gabbay et al. 1994; Schurz 2001b). Wir schränken den Begriff der Gültigkeit auf deduktive Schlüsse ein (einfacher Schlussstrich „/“) und sprechen im Fall von unsicheren Schlüssen stattdessen von ihrer (nicht-deduktiven) Korrektheit (doppelter Schlussstrich „//“). Die Korrektheit von induktiven oder abduktiven Schlüssen kann verloren gehen, wenn den Prämissen weitere Informationen hinzugefügt werden, welche implizieren, dass die als wahrscheinlich vermutete Konklusion in diesem Fall nicht eintrat. Man nennt solche Informationen auch Ausnahmeinformationen.

Unsichere Schlüsse sind nichtmonoton

(Ms. 2.6-2) Nichtmonotonie macht folgendes möglich: P1, …, Pn //K ist korrekt; jedoch: es gibt Q, so dass: Q, P1, …, Pn // K ist inkorrekt. Ein nichtmonotoner Schluss hat also immer nur vorläufige Geltung und kann durch neue Information zunichte gemacht werden. Es mag eingewendet werden, dass dies auch bei deduktiven Schlüssen möglich ist, nämlich dann, wenn neue Information uns dazu bringt, nicht mehr an die Wahrheit der bisherigen Prämissen zu glauben. Dies ist richtig, doch im Gegensatz dazu kann in nichtmonotonen Schlüssen neue Information den Schluss auch dann zunichte machen, wenn sie die Wahrheit der alten Prämissen gar nicht tangiert. Z. B. können wir aus allen bisher beobachteten weißen Schwänen induktiv-generalisierend und vorläufig schließen, dass vermutlich alle Schwäne weiß sind. Aber schon die nächste Beobachtung kann uns einen schwarzen Schwan liefern, und die Hinzufügung dieser Information zu unseren Beobachtungsprämissen macht den Induktionschluss zunichte, denn der neue schwarze Schwan falsifiziert unsere bisherige Konklusion, ohne dass dadurch unsere bisherigen Beobachtungen über weiße Schwäne falsch werden würden. Der wahrscheinlichkeitstheoretische Grund der Nichtmonotonie ist folgender: aus der Tatsache, dass die bedingte Wahrscheinlichkeit von A unter der Annahme (,Prämisse‘) B hoch ist, folgt nicht immer, dass auch die Wahrscheinlichkeit von A unter der Annahme von B und einer weiteren Annahme C hoch ist. Die Vorläufigkeit nichtmonotoner Schlüsse drückt sich darin aus, dass sie eine andere epistemische Anwendungsregel besitzen als monotone Schlüs-

Wahrscheinlichkeitstheoretischer Grund der Nichtmonotonie

55

56

2. Gemeinsame Grundlagen der Wissenschaften

se. Dabei verstehen wir unter dem epistemischen Hintergrundsystem (bzw. Hintergrundwissen) W einer Person bzw. Personengruppe die Menge aller Propositionen, die diese in begründeter Weise als wahr akzeptiert. (Ms. 2.6-3) Anwendungsregel monotoner Schlüsse: Ist der Schluss „P1, …, Pn /K“ deduktiv gültig, dann sollst du K deinem Wissenssystem W hinzufügen, solange es die Prämissen enthält.

(Ms. 2.6-4) Anwendungsregel nicht-monotoner Schlüsse: Ist der Schluss „P1, …, Pn //K“ nicht-monoton korrekt, dann sollst du K deinem Wissenssystem W hinzufügen, solange es die Prämissen enthält, und diese Prämissen alle Propositionen in W umfassen, die für die Konklusion K von Relevanz sind. Für induktive Voraussage- und Generalisierungsschlüsse entspricht (Ms. 2.6-4) Reichenbachs Regel der ,engsten Referenzklasse‘ und Carnaps ,Forderung des empirischen Gesamtdatums‘ (s. Kap. 3.9, 6.3.2).

2.7 Weiterführende Themen 2.7.1 Konstruktiver Realismus versus radikaler Konstruktivismus Fehlschluss des radikalen Konstruktivismus

Der hypothetisch-konstruktive Realismus von Kap. 2.2 unterscheidet sich markant von dem in Kap. 1.2.5.5 erwähnten radikalen Konstruktivismus. Der Fehlschluss des radikalen Konstruktivismus sei hier kritisch nachgezeichnet (typisch z. B. in Kriz et al. 1990, 19 f.): (2.7-1) Prämisse des radikalen Konstruktivismus = epistemischer Konstruktivismus: Unsere Wahrnehmung und Vorstellung von der Wirklichkeit ist nicht etwas ,Gegebenes‘, sondern das Ergebnis einer aktiven kognitiven Konstruktion. Konklusion des radikalen Konstruktivismus = ontologischer Konstruktivismus (Idealismus): Daher ist auch die Wirklichkeit selbst nicht ,an sich‘ gegeben, sondern durch uns ,konstruiert‘ bzw. ,konstituiert‘. Der konstruktive Realismus teilt zwar die Prämisse (1), aber weist die vermeintliche Konklusion (2) weit von sich. Der ungültige Übergang vom epistemischen zum ontologischen Konstruktivismus wird von Konstruktivisten häufig durch den folgenden Begründungsschritt untermauert (s. Kriz et al. 1990, 20; Glasersfeld 1985): (2.7-2) Begründungsschritt des radikalen Konstruktivismus: Es ist unmöglich, über eine Wirklichkeit an sich – unabhängig von ihrer erkenntnismäßigen Konstruktion – etwas auszusagen. Dasjenige, worüber wir immer nur etwas aussagen können, ist die von uns konstruierte Wirklichkeit.

2.7 Weiterführende Themen

Der subtile Fehler dieses Begründungsschrittes liegt darin, dass dabei der Begriff des „aussagens“ im naiv-realistischen Widerspiegelungssinn aufgefasst wird. Dann ist es in der Tat unmöglich, über einen Gegenstand „unabhängig von seiner erkenntnismäßigen Konstruktion“ – was in diesem Fall heißt: „unbeeinflusst von dieser Konstruktion“ – etwas auszusagen. Die moderne Wahrheitstheorie fasst die Beziehung zwischen wahrer Aussage und ihrem Gegenstand jedoch nicht als quasi-identische Widerspiegelung, sondern als strukturelle Korrespondenz auf, die gewisse Informationen überträgt, aber weder vollständig noch eindeutig sein muss. Dass der Satz „diese Blume ist rot“ wahr ist, heißt lediglich, dass das vom singulären Term „diese Blume“ bezeichnete Objekt die vom Prädikat „rot“ ausgedrückte Eigenschaft besitzt: wie viele andere Eigenschaften diese Blume sonst haben mag und wie viele Rotnuancen es auch geben mag, spielt für diese strukturelle Korrespondenz keine Rolle. In dieser Sicht liegt keine Inkohärenz darin, zu sagen, mit unseren erkenntnismäßigen Konstruktionen sagen wir etwas über eine Wirklichkeit aus, die unabhängig von unseren erkenntnismäßigen Konstruktionen existiert, wenngleich diese erkenntnismäßigen Konstruktionen keine quasi-identischen Widerspiegelungen der Wirklichkeit, sondern nur unvollkommene strukturelle Abbildungen derselben darstellen. Die unabhängig existierende Wirklichkeit wird eben nicht als in unseren Vorstellungen unmittelbar gegeben angenommen, wie im metaphysischen Realismus, sondern sie wird lediglich als System hypothetischer Entitäten postuliert, dessen Existenz die empirischen Erfolge unserer Erkenntnis am besten erklären kann. Giere (1999, 174 ff.) und Kuipers (2000, Kap. 13) haben diese Position den ,konstruktiven Realismus‘ genannt, und ich bezeichne diese Position als den hypothetisch-konstruktiven Realismus, um zu betonen, das auch der Realismus letztlich eine fallible Hypothese darstellt. Zusammengefasst ist es gerade die radikal-konstruktivistische Argumentation, die am naiven Realismus klebt, insofern sie die naiv-realistische Deutung der Gegenstände unserer Aussagen dazu benutzt, diese Gegenstände als subjektiv-konstruiert nachzuweisen; wogegen aus hypothetisch-realistischer Sicht der Begründungsschritt (2.7-2) und damit die ganze Argumentation haltlos wird.

2.7.2 Zur Theorie(un)abhängigkeit von Beobachtungen Ein zentraler Stützpfeiler gegenwärtiger erkenntnisrelativistischer Strömungen (s. Kap. 1.2.2.3) ist die These, dass es theoriefreie Erfahrung schlechterdings nicht gibt – jede Beobachtung setze schon in ,irgendeiner‘ Form Theorie voraus. Gegenwärtig wird diese Theorieabhängigkeitsthese von der Mehrheit von Wissenschaftstheoretikern gewissermaßen ,halbherzig‘ akzeptiert (s. z. B. Chalmers 1994, 20 – 31). Eine vollständige Theorieabhängigkeit von Beobachtung müsste aus empirischer Wissenschaft aber ein zirkuläres Unternehmen machen, dem jede Aussicht auf Objektivität fehlt (ebenso Nagel 1979, 79). Denn angenommen, zwei Vertreter der rivalisierenden Theorien T1 und T2 beobachten dasselbe Raumzeitgebiet g. Ihre Theorien implizieren für g jeweils die sich widersprechenden Prognosen P1 und P2. Wäre vollständige Theorieabhängigkeit gegeben, dann wären die

Der hypothetischkonstruktive Realismus

57

58

2. Gemeinsame Grundlagen der Wissenschaften

Theorieabhängigkeit und Rechtfertigungszirkel

Wahrnehmungstäuschungen

Beobachtungen der Theorievertreter nur durch ihre theoretischen Vorerwartungen bestimmt. Der Vertreter von T1 würde also P1 beobachten, wo der Vertreter von T2 P2 beobachtet. Somit wäre jede Prognose eine selbsterfüllende Prognose. Eine unabhängige Überprüfung von Theorien wäre unmöglich; vielmehr entstünde ein totaler Rechtfertigungszirkel: die Anhänger der Theorie T glauben an ihre Theorie, weil sie deren Prognosen beobachtet haben, und dies deshalb, weil sie an die Theorie glauben. Totale Rechtfertigungszirkel sind epistemisch wertlos. Partielle Zirkel, in denen unabhängige Daten eingespeist werden, können dagegen durchaus epistemischen Wert besitzen. Im folgenden seien einige zentralen Argumente für verschiedene Arten von Theorieabhängigkeit sowie die zugehörigen Gegenargumente in Form von ,Antworten‘ knapp skizziert. Es soll sich zeigen, wie sich eine bestimmte Art von Theorieunabhängigkeit von Beobachtungen im engen Sinne von Wahrnehmungen gegen diese Argumente verteidigen lässt. 1.) Erfahrung ist theoriegeleitet: Eine Selektion der Erfahrung hinsichtlich relevanter Aspekte und Merkmale ist in jeder Forschung unerlässlich, schon aus Komplexitätsgründen. Diese Selektion ist theoriegeleitet: welche Theorie ich vertrete, bestimmt, nach welchen Beobachtungen ich suche (s. z. B. Chalmers 1994, 37 f.; van Fraassen 1980, 56 ff.; Albert 1980, 52 f.). Antwort: Dies ist richtig. Doch die Tatsache, dass Beobachtungen theorieabhängig selektiert werden, impliziert nicht, dass die Beobachtungen selbst theorieabhängig sind, und dass eine intersubjektive Überprüfung der beobachtungsselektierenden Theorien unmöglich wäre. Auch Vertreter der jeweils anderen Theorie T2 können die von Vertretern der Theorie T1 gemachten Beobachtung selbst anstellen, und würden dann deren Richtigkeit anerkennen. 2.) Wahrnehmung ist ein (unbewusster) Konstruktions- und Interpretationsprozess: Die visuellen Gehirnzentren erzeugen durch raffinierte neuronale Konstruktionsmechanismen aus den zweidimensionalen Netzhauteindrücken des Lichtes ein dreidimensionales bewusst erlebtes Sehbild. In gewissen meist künstlich erzeugten Situationen führen diese unbewussten visuellen Konstruktionsprozesse zu visuellen Ambiguitäten oder Täuschungen, wie dies durch kognitionspsychologische Befunde eindrucksvoll belegt wird. Bekannt sind z. B. Kippbilder, wie das Jastrowsche Bild des Entenhasen, ein Bild, das je nachdem als Ente oder Hase gesehen wird, und welches von Wittgenstein (1945, Teil II.x, 520), Hanson (1958, 12) und Kuhn (1967, 152 ff.) als Beleg der ,Theorieabhängigkeit‘ der Wahrnehmung verwendet wird. Über eine Fülle weiterer solcher optischer ,Illusionen‘ informiert Rock (1984). Antwort: Die kognitionspsychologischen Befunde zum Konstruktionscharakter des Sehprozesses widerlegen nur den sogenannten direkten Realismus (Dancy 1985, 144), demzufolge wir die Dinge so sehen, wie sie an sich sind. In allen kognitionspsychologischen Befunden zeigt sich aber ein Weiteres: unsere visuellen Wahrnehmungsprozesse und ihr Resultat sind in geradezu hartnäckiger Weise unabhängig von unserem erworbenen Hintergrundwissen (s. hierzu Fodor 1984, 34 ff.; Pylyshyn 1999; Rock 1984, 228 f.). Wahrnehmungstäuschungen stellen sich für jedermann in derselben Weise ein, auch dann, wenn die betreffende Person darüber aufgeklärt wurde, dass es sich um eine Wahrnehmungstäuschung handelt. Unsere Wahr-

2.7 Weiterführende Themen

Wahrnehmung unabhängig von erworbenem Wissen

Beobachtungsdaten im ,weiten Sinn ,

nehmungsprozesse beruhen auf angeborenen Mechanismen, die sich in einer millionenjährigen Evolutionsgeschichte herausgebildet und bewährt haben. Die Wahrnehmungspsychologie vermag insbesondere zu erklären, warum dieselben Mechanismen, die unter normalen Umgebungsbedingungen die Realität visuell korrekt abbilden, unter gewissen nicht-normalen Bedingungen optische Illusionen erzeugen können. Man könnte unter der ,Theorieabhängigkeit‘ der Wahrnehmung höchstens die Tatsache verstehen, dass Wahrnehmung von ,angeborenen Theorien‘ abhängt, die in der Sehrinde unseres Gehirns fest verdrahtet sind – aber so wollen wir den Begriff nicht verstehen. Mit Theorieabhängigkeit der Wahrnehmung meinen wir immer nur: Abhängigkeit der Wahrnehmung von erworbenem Hintergrundwissen. Visuelle Wahrnehmung in diesem Sinn ist – zumindest in hohem Maße bzw. bis auf wenige Ausnahmen – theorieunabhängig: Personen mit unterschiedlichstem Hintergrundwissen machen angesichts desselben (deutlich ausgeprägten) visuellen Reizes dieselben Wahrnehmungen. Hanson (1958, 5 – 8) und Kuhn haben die erläuterten wahrnehmungspsychologischen Befunde zumindest stellenweise so interpretiert, als ob unser Sehbild von erworbenen Hintergrundtheorien abhängt. So sagt Kuhn, es fand eine Verschiebung des Sehbildes statt, als man den Uranus vor 1781 als Fixstern ohne Bewegung sah, jedoch ab 1781 als einen sich bewegenden Planeten (1967, 151). In solchen Fällen handelt es sich nach Kuhn um einen paradigmatischen Gestaltwechsel, vergleichbar mit einem Kippbild (1967, 164). Aufgrund obiger Ausführungen ist diese Interpretation zurückzuweisen. In Fällen wie diesen hat sich nicht die Sehwahrnehmung der Wissenschaftler geändert. Schon gar nicht handelte es sich um ein Kippbild im wahrnehmungspsychologischen Sinn. Was sich geändert hat, war lediglich die theoretische Interpretation ihrer Wahrnehmung. 3.) Wissenschaftliche ,Beobachtungsdaten‘ (in einem weiteren Sinn) sind theorieabhängig: In der Wissenschaft ist ein weiterer Beobachtungsbegriff üblich, demzufolge man auch Galaxien, Bakterien oder Elektronen mit entsprechenden Messinstrumenten beobachten kann. Eine Reihe von Autoren, darunter auch Kuhn in vielen Passagen, beziehen ihre Theorieabhängigkeitsthese auf diesen Beobachtungsbegriff im weiteren Sinn (Putnam 1962, 244; Kuipers 2000, § 13.3; Giere 1999, 80; Adam 2002, 12 f.; u. a. m.). Antwort: Solche Beobachtungsdaten im weiteren Sinn sind theorieabhängige Interpretationen von Wahrnehmungen. Bei den hierbei vorausgesetzten Theorien handelt es sich in erster Linie um Theorien über die Funktionsweise von Messinstrumenten, die zum als unproblematisch betrachteten Vorwissen des Wissenschaftlers gerechnet werden. Der pragmatisch erweiterte Beobachtungsbegriff im weiteren Sinne ist für das effiziente Vorwärtskommen der Wissenschaft zweifellos sehr bedeutend – denn man kann nicht immer alles gleichzeitig überprüfen. Unter Beobachtung im engen Sinn verstehen wir im folgenden aber immer nur (visuelle) Wahrnehmung, und wir behaupten die Theorieunabhängigkeit nur für diesen engen Begriff. Wenn der Chemiker von der Beobachtung von Säure aufgrund des Lackmuspapiertests spricht, dann war das, was er im engen Sinn beobachtete, der rote Farbton des Lackmuspapiers; der Schluss auf die chemische Säure war das Resultat einer theoretischen Interpretation. Selbst im Falle sprachlicher Da-

59

60

2. Gemeinsame Grundlagen der Wissenschaften

Kontinuitätsargument

Sprach- und Kulturabhängigkeit

ten der Humanwissenschaften liegt die primäre Wahrnehmung auf der Ebene sprachlicher Laut- und Textgestalten. Deren konventionell-semantische Interpretation erfolgt innerhalb der eigenen Muttersprache spontan und unbewusst – dass es sich dabei aber um einen Interpretationsprozess und nicht um ,Wahrnehmung‘ handelt, wird jedem klar, der dabei ist, eine fremde Sprache zu lernen. 4.) Das Kontinuitätsargument: Maxwell (1962, 7), Carnap (1976, 253 – 254) und Hempel (1974, 112 – 119) wiesen darauf hin, es gäbe einen kontinuierlichen Übergang von Beobachtbarkeit mit dem freien Auge über Zwischenstufen wie Beobachtbarkeit mittels einer Brille oder Lupe, bis hin zu Beobachtbarkeit mit dem Teleskop, dem Lichtmikroskop, dem Radarschirm oder dem Elektronenmikroskop. Eine Reihe von Autoren schlossen daraus, jede Grenzziehung zwischen perzeptueller Beobachtung im engen Sinn und theorieabhängiger Beobachtung im weiten Sinn sei willkürlich. Antwort: Erstens impliziert die Tatsache, dass es einen kontinuierlichen Übergang zwischen zwei Merkmalen wie z. B. Schwarz und Weiß gibt, noch keineswegs, dass es keinen Unterschied zwischen Schwarz und Weiß gibt (van Fraassen 1980, 214). Und zweitens gibt es in diesem Übergang markante Schnitte. Während Beobachtung im engen Sinn nämlich kein erworbenes Hintergrundwissen bedarf, steigt das erforderliche Hintergrundwissen zur Bedienung der oben genannten Messinstrumente sukzessive an (man stelle sich hierzu jeweils vor, was ein Eingeborener mit diesen Instrumenten ohne Instruktion anstellen würde). 5.) Beobachtung ist sprach- und kulturabhängig: Die These von der Sprachabhängigkeit von Erfahrung wurde voralledem durch den Philosophen Wilhelm von Humboldt und durch die kulturethnologischen Forschungen von Sapir und Whorf populär gemacht (s. Kutschera 1975, Kap. 4.1). Beispielsweise verfügen viele Eingeborenenstämme über andere und weniger differenzierte Farbbegriffe als Angehörige der westlichen Zivilisation, woraus die Kulturethnologen schlossen, dass diese Eingeborenen die Farben ihrer Umwelt tatsächlich anders wahrnehmen als wir. Umgekehrt besitzen die Eskimos Berichten zufolge viel mehr Begriffe für Schneesorten als wir, usw. (s. Whorf 1963, 15; Kutschera 1975, 300 ff.). Whorf leitete daraus sein linguistisches Relativitätsprinzip ab, demzufolge wir nur das wahrnehmen können, was in unserer Sprache durch Begriffe vorgezeichnet wird. Antwort: Man muss zugestehen, dass unterschiedliche Kulturen bis zu einem gewissen Grade unterschiedliche Begriffssysteme entwickeln, in denen sie die Wahrnehmungen ihrer unterschiedlichen Umwelten beschreiben. Daraus folgt aber nicht, dass ihre Sinneserfahrungen selbst sprachabhängig sind. Dies ließe sich nur dann folgern, wenn Angehörige dieser Kulturen auch nicht in der Lage wären, die jeweiligen Beobachtungsbegriffe anderer Kulturen durch ostensives Training zu erlernen. Diese Lernfähigkeit ist jedoch durchgängig vorhanden. In diesem Sinne können auch die Zuni-Indianer den Begriff „orange“, über den sie nicht verfügen, nach kurzem Training erlernen; die Tiv und die Dani konnten auf diese Weise unzählige Farben erlernen, für die sie in ihrer Muttersprache über keine Begriffe verfügen (Garnham/Oakhill 1994, 49 – 51; Berlin/Kay 1999, 24). Ebenso könnte ein Europäer die Schneebegriffe der Eskimos ostensiv erwerben. Wesentlich an einem ostensiven

2.7 Weiterführende Themen

Lernexperiment ist, dass es nonverbal funktioniert. Denn ansonsten könnte der Sprachrelativist argumentieren, der Zuni-Indianer hätte die Farbe „orange“ erst durch den Erwerb der westlichen Farbsprache zu sehen gelernt. Quine (1960, Kap. II.7 – II.8) hat ostensive Lernexperimente als Methode der Übersetzung zwischen radikal fremden Kulturen vorgeschlagen. Beobachtungsbegriffe als ostensiv erlernbare Begriffe: Ein ostensives Lernexperiment besteht aus einer Trainingsphase und einer Testphase. In der Trainingsphase wird der auf Beobachtbarkeit zu überprüfende Begriff durch ein der Versuchsperson (Vp) unbekanntes Kunstwort „X“ vorgestellt, sodass die Vp vor dem Lernexperiment mit dem Kunstwort aufgrund ihres Hintergrundwissens keine Bedeutung verbindet. Der durch das Kunstwort bezeichnete Begriff wird der Vp anhand einer Menge von positiven und negativen Beispielen illustriert, und es wird jeweils die Information „X ja“ bzw. „X nein“ gegeben. Der zu leistende ostensive Lernprozess der Vp besteht darin, anhand der vorgelegten Beispiele die Bedeutung von „X“, also das „X“ entsprechende wahrnehmbare Eigenschaftsbündel, herauszuabstrahieren. Ob dies der Vp gelungen ist, wird in der anschließenden Testphase ermittelt, in welcher der Vp eine Reihe neuer positiver und negativer Beispiele vorgeführt werden, und gefragt wird „ist dies ein X?“. Wenn nun ein Begriff von beliebigen Menschen unabhängig von ihrer Sprache und ihrem Hintergrundwissen problemlos ostensiv erlernbar ist, dann ist dies ein nahezu zwingender Grund, diesen Begriff als theorieunabhängig anzusehen. Würde ein Zuni-Indianer das Farbmerkmal „orange“ gar nicht wahrnehmen, weil er den Begriff dazu nicht besitzt, so würde er auch nicht den Unterschied zwischen den orangen, roten und gelben Bildern in der Trainingsphase wahrnehmen, und es wäre unerklärlich, wie er dann die Testphase erfolgreich bestehen könnte. Tatsächlich aber scheinen Wahrnehmungsbegriffe die Eigenschaft zu besitzen, von beliebigen Menschen unabhängig von ihrem Hintergrundwissen rein ostensiv erlernbar zu sein. Für theoretische Begriffe trifft diese ostensive Erlernbarkeit nicht zu – es wäre chancenlos, einem Eingeborenen z. B. den physikalischen Kraftbegriff rein ostensiv, ohne jahrelange Ausbildung in fremder Sprache und Naturwissenschaft, beibringen zu wollen. Herkömmlich wird ein Beobachtungsbegriff als ein Begriff charakterisiert, dessen Zutreffen auf einen beobachtbaren Gegenstand durch bloße Wahrnehmung festgestellt werden kann (z. B. Carnap 1936/37, 454 f.). Um diese Charakterisierung zu überprüfen, müssen wir uns auf die Introspektion von Personen verlassen – es ist aber bekannt, dass viele Menschen in ihren vermeintlichen Beobachtungen Wahrnehmung und theoretische Interpretation unterschiedslos vermengen. Obige Überlegungen liefern uns jedoch eine Möglichkeit, Beobachtungsbegriffe im engen Sinne in einer empirisch transparenten Weise wie folgt zu explizieren. (Def 2.7-1): Ein Begriff ist ein Beobachtungsbegriff (im engen Sinn) g.d.w. er von allen Menschen unter Normalbedingungen der Beobachtung unabhängig von ihrem Hintergrundwissen und ihrer Sprach- und Kulturzugehörigkeit in einem ostensiven Lernexperiment erlernbar ist.

Ostensives Lernen

Unverlässlichkeit der Introspektion

61

62

2. Gemeinsame Grundlagen der Wissenschaften Normalbedingungen der Beobachtung

Die Normalbedingungen der Beobachtung umfassen dabei (i) die für die Wahrnehmung erforderlichen physikalischen Normalbedingungen (geeignete Lichtverhältnisse, usw.), (ii) biologische Normalbedingungen (die Person hat keinen empirisch feststellbaren Defekt der Sinnesorgane oder des Nervensystems), und (iii) psychologische Normalbedingung (die Person ist im psychisch normalen Wachzustand, und ihre ja/nein-Äußerungen sind wahrhaftig). Sprachliche Normalbedingungen (die Person versteht die zur Formulierung benutzen Worte richtig) werden nicht benötigt, da im ostensiven Lernexperiment ja die Sprache umgangen wird.

2.7.3 Zum Unterschied zwischen Erfahrungssätzen und Werturteilen

Das Normalbedingungsargument

Die Wertneutralitätsforderung beruht auf der These, dass fundamentale Norm- und Wertsätze keiner erfahrungswissenschaftlichen Begründung fähig sind. Diese These wird nicht von allen Wissenschaftstheoretikern und Ethikern geteilt. Daher wollen wir sie im folgenden gegen die wichtigsten Einwände verteidigen. Es gibt eine Reihe von unterschiedlichen metaethischen Positionen zur Frage der Begründbarkeit von fundamentalen Normund Wertsätzen, die hier nicht erläutert werden können (vgl. Schurz 1997a, Kap. 11.3). Die entscheidende Herausforderung für unsere Begründung der Wertneutralitätsthese sind die Ansätze des sogenannten Wertempirismus, welche eine besondere Art von ,Werterfahrung‘ als Überprüfungsbasis für die Ethik annehmen, in Analogie zur gewöhnlichen Erfahrung als Überprüfungsbasis für Erfahrungswissenschaften (s. Czaniera 2001, Kap.1 – 2). In der Wissenschaftstheorie wurde diese Position von Weingartner (1978, Kap. 7.15 ff.) vertreten, der annimmt, ethische Hypothesen werden durch Basiswertsätze und Basisnormen überprüft. Ein Beispiel für einen Basiswertsatz wäre etwa: ich sehe einen Menschen, der einem Armen Geld gibt, und empfinde dies spontan als gute Tat. Im folgenden möchte ich anhand von zwei Argumenten zeigen, dass sich solche ,Werterfahrungen‘ ganz anders verhalten als echte Sinneserfahrungen: das Normalbedingungsargument und das Unabhängigkeitsargument. Das Normalbedingungsargument besagt folgendes: Intersubjektiv abweichende Wahrnehmungsurteile können nahezu immer auf Defekte in den Normalbedingungen der Beobachtung zurückgeführt werden. Auf intersubjektiv abweichende Werturteile trifft dies im allgemeinen nicht zu. Betrachten wir als Beispiel den angenommen wahren Beobachtungssatz dort brennt ein Feuer. Die These der Intersubjektivität eines solchen Beobachtungssatzes besagt, dass diesem Satz jeder oder nahezu jeder Mensch zustimmen würde, sofern die in Kap. 2.7.2 erläuterten Normalbedingungen der Beobachtung erfüllt sind. Würde eine Person tatsächlich hartnäckig und wiederholt in visueller Nähe des Feuers behaupten „ich sehe aber kein Feuer“, so hätten wir allen Grund, einen Defekt in diesen Normalbedingungen anzunehmen, und wir würden diesen Defekt durch einen unabhängigen Test nachweisen können. Eine solche Person wäre höchstwahrscheinlich blind, oder gehirngeschädigt, oder sie befände sich in einem Halluzinationsstadium. Aus diesem Grund wären wir auch dazu berechtigt, einen solchen Menschen von seinem Beobachtungsdefekt heilen zu wollen.

2.7 Weiterführende Themen

Doch nehmen wir an, unter denselben Normalbedingungen der Beobachtung würde eine erwachsene Person einem Basiswertsatz wie z. B. „dass diese Person vor dem Sterben bewahrt wird, ist gut“ systematisch nicht zustimmen, obwohl ihm angenommen 99 % aller Personen zustimmen. Dann wären wir nicht berechtigt anzunehmen, dass ein Defekt in den ,moralischen Normalbedingungen‘ dieser Person vorliegt, d. h., dass diese Person z. B. moralisch minderwertig sei und davon geheilt werden sollte, genauso wie ein Blinder von seiner Blindheit geheilt werden sollte. Vielmehr müssen wir die Haltung dieser Person als abweichende Werthaltung prima facie achten und – sofern dadurch die Interessen anderer Personen nicht geschädigt werden – auch zulassen. Bei den Eskimos ist es zum Beispiel üblich, eine Person, die so altersschwach geworden ist, dass sie sich nicht mehr selbst ernähren kann, nicht zu füttern, sondern sterben zu lassen, weil man darin das natürliche Anzeichen dafür sieht, dass ihr Geist die irdische Welt verlassen will: sollen wir deshalb die Ethik der Eskimos als ,moralisch defektiv‘ erklären und von ihren Defekten zu ,heilen‘ versuchen? Würden wir mit moralischen Basiswertsätzen genauso verfahren wie mit Beobachtungssätzen, so könnten wir in gefährliche Nähe zu mittelalterlich-religiösen Moralpraktiken geraten. Ich will mit dieser These nicht leugnen, dass es tatsächlich Fälle von psychopathologischer Gewissensblindheit gibt, die einer psychologischen Heilung bedürfen: aber diese Fälle sind nicht der Regelfall. So dürfte eine extreme Behauptung wie „ich liebe es, andere Menschen zu töten“ bei jedermann die berechtigte Reaktion auslösen „dieser Mensch hat einen Defekt“ – aber diese Situation trifft nur auf die wenigsten abweichenden Wertbasissätze zu, wogegen die analoge Situation im Falle abweichender Wahrnehmungsurteile der Regelfall ist. Ganz in diesem Sinn beschreiben Prim und Tilman (1979, 93 f.) ein Experiment zur Objektivität des Beobachtungssatzes „vor mir liegt ein Stück Kreide“, das sie an ihren Studenten durchführten: die Studenten reagierten äußerst unwillig und erklärten, ein Dissens in solchen Fragen sei ein Problem für den Augenarzt, und keines der Philosophie. Das Unabhängigkeitsargument geht auf Toulmin (1950, 127) zurück und besagt folgendes: die Art und Weise, wie wir unsere Wahrnehmungen emotiv-werthaft empfinden bzw. interpretieren, ist von unseren ethischen Hintergrundannahmen abhängig. Unsere Wahrnehmungseindrücke bleiben dagegen auch dann gleich, wenn sich unsere deskriptiven Hintergrundannahmen radikal verändern. Beispielsweise sehen wir tagtäglich die Sonne über das Himmelsgewölbe wandern. Nehmen wir an, bislang seien wir Geozentriker gewesen (die Sonne dreht sich um die ruhende Erde); doch nun werden wir, nach gründlicher Lektüre von Kepler und Newton, zu Heliozentrikern (die Erde dreht sich um die ruhende Sonne). Dann haben wir unsere Hintergrundtheorie radikal gewechselt – doch wie schon in Kap. 2.7.2 erläutert, wird unser Seherlebnis dadurch in keiner Weise beeinflusst: nach wie vor sehen wir die Sonne tagtäglich in einem Bogen über den Himmel wandern. Das Seherlebnis ist unabhängig von erworbenen Hintergrundannahmen. Ganz anders im Falle werthaft-moralischer Empfindungen. Wenn ich einen Menschen sehe, der eine Menge Geld den Armen spendet, dann

Der Umgang mit abweichenden Werthaltungen

Das Unabhängigkeitsargument

63

64

2. Gemeinsame Grundlagen der Wissenschaften

habe ich spontan die moralische Empfindung: dies war eine gute Tat. Wenn ich jedoch erfahre, dass es sich bei dem Spender um einen Politiker kurz vor seiner Wahl handelt, der diese Spende zum kalkulierten Zwecke der Gewinnung von Wählerstimmen investierte und sich ansonsten mehr um das Wohl der Reichen als um das der Armen kümmert, so stellt sich bei mir angesichts desselben Wahrnehmungserlebnisses eine ganz andere moralische Empfindung ein: nun ich empfinde den Politiker als heuchlerisch. Das Normalbedingungs- und das Unabhängigkeitsargument bekräftigen unsere These, dass Wert- und Normurteile nicht auf Sinneserfahrungen beruhen, sondern subjektive Interpretationen sind, weswegen eine fundamentale Begründung von Wert- und Normsätzen nicht möglich ist.

2.8 Zusammenfassung, einführende Literatur und Übungen 2.8.1 Zusammenfassung (Auswahl). Die Methode der Wissenschaftstheorie ist die rationale Rekonstruktion. Sie entwickelt anhand faktisch vorliegender Musterbeispiele von Wissenschaft (deskriptives Korrektiv) ein allgemeines Modell von Wissenschaft, welches dem normativen Korrektiv entspricht. Letzteres besteht aus der wissenschaftsinternen Zielvorgabe – der Suche nach gehaltvollen Wahrheiten – sowie aus den erkenntnistheoretischen Annahmen E1) minimaler Realismus, E2) Fallibilismus, E3) Objektivität, E4) minimaler Empirismus und E5) Logik (i. w. S.). In allen wissenschaftlichen Disziplinen – mit Ausnahme der Formalwissenschaften, auf die auch Annahme E4 nicht zutrifft – finden sich ferner vier gemeinsame methodologische Merkmale, nämlich M1) die Suche nach möglichst gehaltvollen Hypothesen, M2) die Rekrutierung von Beobachtungen bzw. empirischen Daten, M3) die Erklärung und Voraussage von Beobachtungen durch die Hypothesen in M1, sowie M4) die Überprüfung dieser Hypothesen durch Vergleich der gemäß M3 vorausgesagten Beobachtungen mit den gemäß M2 aktual gefundenen. Wissenschaftliche Disziplinen lassen sich nach ihrem Gegenstandsbereich klassifizieren, nach dem Grad ihrer empirischen Eindringlichkeit, sowie dem Grad ihrer Quantifizierung. Eine bedeutende Abgrenzung von Wissenschaft gegenüber nichtwissenschaftlichen Überzeugungssystemen liefert die Wertneutralitätsthese, welche besagt, dass in den wissenschaftlichen Begründungszusammenhang keine wissenschaftsexternen Werte einfließen dürfen. Die Schlussarten, welche in den Wissenschaften eine Rolle spielen, untergliedern sich in die drei Gruppen: deduktive, induktive und abduktive Schlüsse (bzw. Schlüsse auf die beste Erklärung). 2.8.2 Einführende Literatur. Zur Methode der rationalen Rekonstruktion s. Stegmüller (1979c). Der Klassiker des Werturteilsstreites ist Albert/Topitsch (1971, Hg.). Über den Positivismusstreit informiert Dahms (1994). Zum Induktionsproblem s. Rescher (1987). Zum Schluss auf die beste Erklärung s. Bartelborth (1996). Adam (2002) informiert über die Frage der Theorie(un)abhängigkeit von Erfahrung. Zur kognitiven Psychologie des Sehens s. Rock (1984).

2.8 Zusammenfassung, einführende Literatur und Übungen

2.8.3 Fragen und Aufgaben. Zu Kap. 2.1: 1) Wie lautet die Argumentation der Normativisten, und worin besteht ihr Fehler? 2) Erläutern Sie die Methode der rationalen Rekonstruktion. Zu Kap. 2.2: 1) Erläutern Sie die fünf Annahmen des gemeinsamen erkenntnistheoretischen Modells der Wissenschaften. 2) Was ist der Unterschied zwischen Falsifizierbarkeit, Falsifiziertheit und Fallibilität? Zu Kap. 2.3: 1) Worin bestehen die vier Methodenbausteine der Wissenschaften? Zu Kap. 2.4: 1) Inwiefern sind die Begriffe „Naturwissenschaften“, „Humanund Sozialwissenschaften“ und „Geisteswissenschaften“ mehrdeutig? 2) Wodurch unterscheiden sich empirische, experimentelle und sezierende Wissenschaften? Ordnen Sie Erziehungswissenschaft, Humanmedizin und Kulturwissenschaft in diese Klassifikation ein. Zu Kap. 2.5: 1) Erläutern Sie den Zweck-Mittel-Schluss anhand von Beispielen aus Ihrem Fachgebiet. 2) Erläutern Sie die Habermassche Dreiteilung von Wissenschaftsdisziplinen nach ihrem Erkenntnisinteresse. Was besagt die Kritik daran? 3) Erläutern Sie die Wertneutralitätsthese sowie alle in ihr vorkommenden Begriffe. Zu Kap. 2.6: 1) Um welche Schlussart handelt es sich bei folgenden Schlüssen? (i) Die meisten Lichtschalter funktionieren. Ich drücke den Lichtschalter. Also geht das Licht an. (ii) Alle Menschen sind sterblich. Aristoteles ist ein Mensch. Also ist auch Aristoteles sterblich. (iii) Bisher hat mein Kühlschrank gut funktioniert. Also kann ich mich auch in Zukunft (bzw. alternativ: morgen) auf ihn verlassen. (iv) Immer wenn Gregor von seinem Bruder spricht, nimmt sein Gesicht gespannte Züge an. Also hat er ein Problem mit seinem Bruder. 2) Erläutern Sie das Prinzip der methodischen, der logischen und der epistemischen Induktion. Wie lauten die Popperschen Argumente gegen die ersten beiden Induktionsarten? 3) Erläutern Sie den Schluss auf die beste Erklärung anhand eines Beispiels. 4) Welche der Schlüsse in 1) sind monoton und welche nichtmonoton? Zu Kap. 2.7: 1) Erläutern Sie die Argumentation des radikalen Konstruktivismus und ihre Kritik durch den hypothetisch-konstruktiven Realismus. 2) Eine Person könnte jemanden übersehen, (a) weil sie ihn absichtlich ignoriert, (b) weil sie blind ist, (c) weil sie schläft, oder (d) weil es stockdunkel ist. Welche Normalbedingung der Beobachtung ist dabei jeweils verletzt? 3) Erläutern Sie das Normalbedingungs- und das Unabhängigkeitsargument.

65

3. Das analytische Instrumentarium: Sprache, Logik und Wahrscheinlichkeit 3.1 Begriffe und Begriffsarten

Begriffe

Die logischen Grundbausteine jeder Sprache sind ihre Begriffe. Begriffe sind bedeutungsvolle sprachliche Ausdrücke, die grammatisch unterhalb der Satzebene liegen. „Gras“ und „grün“ sind Begriffe; „Gras ist grün“ ist ein Satz. Begriffe bezeichnen zwar etwas, sie können selbst jedoch noch nicht wahr oder falsch sein; dies können nur die aus Begriffen gebildeten Sätze. Begriffe können atomar oder auch komplex sein; z. B. ist „Gras“ ein atomarer und „grünes Gras“ ein komplexer Begriff. Sätze sind aus Begriffen zusammengesetzt, und von der Natur seiner Begriffe hängt die Natur eines Satzes ab. Daher beschäftigen wir uns zuerst mit den unterschiedlichen wissenschaftlichen Begriffsarten, bevor wir die unterschiedlichen wissenschaftlichen Satzarten behandeln. Begriffe können in dreierlei Hinsichten klassifiziert werden: erstens nach ihrem logischen Typ, zweitens nach ihrem Inhaltstyp, und drittens nach ihrem Skalentyp.

3.1.1 Klassifikation von Begriffen nach ihrem logischen Typ Logische Grammatik

Singuläre Terme

Die logische Grammatik klassifiziert Begriffe und Sätze nach ihrer logischsemantischen Funktion. Abb. 3.1-1 präsentiert die Übersicht. 1. Nichtlogische Begriffe dienen dazu, etwas in der Welt zu bezeichnen oder auszudrücken. Logische Begriffe haben dagegen nur eine strukturelle Funktion. 1.1 Singuläre Begriffe bzw. Terme bezeichnen immer ein bestimmtes Individuum bzw. Einzelding. Singuläre Terme unterteilt man weiter in: (1.1.1) Eigennamen (z. B. „Saul Kripke“, „Salzburg“), (1.1.2) ostensive (hinweisende) bzw. indexikalische Terme (z. B. „dieser Mensch dort“, „jetzt“, „hier“, „ich“, „du“, usw.), und (1.1.3) definite Deskriptionen mithilfe von kennzeichnenden Eigenschaften („dasjenige x, welches die Eigenschaft F hat“). Logische Notation: atomare singuläre Terme heißen auch Individuenkonstanten und werden meist durch Kleinbuchstaben a, b, … (bzw. a1, a2, …) dargestellt. 1.2. Generelle Begriffe 1. Stufe werden auf singuläre Terme angewandt: diese Anwendung nennt man Prädikation, und durch sie entsteht ein atomarer Satz. 1.2.1 Prädikate (1. Stufe) bezeichnen Merkmale, die auf Individuen zutreffen können oder nicht. Man unterschiedet zwischen Eigenschaftsbegriffen wie z. B. „x ist groß“, und Artbegriffen wie z. B. „x ist ein Mensch“. Logische Notation: Prädikate werden meist durch Großbuchstaben F, G, …. (bzw. F1, F2, …) dargestellt. Prädikate sind einstellig, d. h. sie besitzen eine Argumentstelle, welche man als Variable x zum Prädikatbuchstaben hinzu-

3.1 Begriffe und Begriffsarten Singuläre Begriffe bzw. Terme Prädikate Nichtlogische

Generelle Begriffe 1. Stufe

Begriffe

Relationen Funktionen

Generelle Begriffe höherer Stufe Logische

Wahrheitsfunktionale Satzoperatoren (Aussagenlogik AL)

Begriffe

Quantoren, Variablen 1. Stufe (Prädikatenlogik PL 1. Stufe) Intensionale Satzoperatoren (Modallogik ML) Quantoren, Variablen höherer Stufe (PL höherer Stufe) Elementbeziehung (Mengentheorie) (Mathematische Begriffe)

Abb. 3.1-1: Klassifikation von Begriffen nach ihrem logischen Typ

schreibt: „Fx“ für „x ist ein F“. Steht z. B. „Fx“ für „x ist groß“, und „a“ für „Peter“, dann steht „Fa“ für den Satz „Peter ist groß“. 1.2.2 Ein n-stelliges Relationszeichen (1. Stufe) bezeichnet eine Relation zwischen n gegebenen Individuen. Logische Notation durch Großbuchstaben R, Q, …. Steht z. B. „Rxy“ für die 2-stellige Relation „x ist Bruder von y“, „a“ für „Peter“, und „b“ für „Paul“, dann steht der atomare Satz „Rab“ für „Peter ist Bruder von Paul“. 1.2.3 Funktionszeichen: Ein einstelliges (bzw. n-stelliges) Funktionszeichen bezeichnet eine Funktion, die einem (bzw. n) gegebenen Individuum(en) ein anderes Individuum eindeutig zuordnet. Logische Notation: man benutzt dafür meist die Kleinbuchstaben f, g, …. Steht z. B. „f(x)“ für „die Mutter von x“, dann bezeichnet „f(a)“ die Mutter von a (z. B. von Peter), und „b = f(a)“ steht für „b ist die Mutter von a“. In der Mathematik spielen Funktionen als Operationen über Zahlen eine grundlegende Rolle; z. B. ist die Addition + eine zweistellige Funktion, die je zwei Zahlen x und y ihre Summe + (x, y) zuordnet; dabei schreibt man „+ (x, y)“ in die infix-Notation „x + y“. Auch alle quantitativen (z. B. physikalischen) Eigenschaften von Individuen sind als Funktionen darzustellen: so bezeichnet m(a) die Masse des Körpers a, und „m(a) = 5,6 kg“ steht für „die Masse des Körpers a beträgt 5,6 Kilogramm“. 1.3 Prädikate und Funktionen höherer Stufe werden auf andere Prädikate oder Sätze angewandt und bilden dadurch Sätze. Ein Beispiel für ein höherstufiges Prädikat ist der Begriff der Gesetzesartigkeit eines Prädikates oder Allsatzes (s. Kap. 3.6), und ein Beispiel für eine höherstufige Funktion ist der Wahrscheinlichkeitsbegriff. 2. Logische Begriffe: Es gibt unterschiedliche logische Begriffstypen. Sie werden in entsprechenden Typen von Logiken zum Ausdruck gebracht. 2.1 Wahrheitsfunktionale (extensionale) Satzoperatoren – Aussagenlogik AL:

Prädikate und Relationsbegriffe

Funktionsbegriffe

67

68

3. Das analytische Instrumentarium Satzoperatoren der Aussagenlogik

Quantoren der Prädikatenlogik

Logische Notation: A, B, … stehen im folgenden für beliebige Sätze. Die wichtigsten aussagenlogischen Satzoperatoren sind folgende: – die Negation : (lies: : A – nicht A), – die Konjunktion ^ (lies: A ^ B – A und B) – die Disjunktion (das einschließende Oder) _ (lies: A _ B – A oder B oder beides). – die sogenannte ,materiale‘ Implikation ! (lies: A ! B – wenn A, dann B), – die ,materiale‘ Äquivalenz $ (lies: A $ B – A genau dann wenn (g. d. w.) B). – Konvention: „: $ “ steht für „per definitionem äquivalent“; ebenso für „:=“. Weitere Satzoperatoren sind damitdefinierbar. Z. B. ist das ausschließen de Entweder-Oder _ definiert als A _ B :$ (A _ B) ^ : (A ^ B) (A oder B, aber nicht beides). Die Gesetze dieser Satzoperatoren expliziert die (nichtmodale) Aussagenlogik. Allgemein erzeugt ein Satzoperator in Anwendung auf gegebene Sätze einen neuen komplexen Satz. Ein Satzoperator heißt wahrheitsfunktional (oder extensional), wenn der Wahrheitswert des komplexen Satzes eindeutig durch die Wahrheitswerte seiner Teilsätze (Argumente) bestimmt ist. Die Wahrheitsfunktionalität der aussagenlogischen Satzoperatoren äußert sich in ihren Wahrheitstafeln: jeder Satz A ist entweder wahr oder falsch; : A wahr g. d. w. A falsch ist; A ^ B ist wahr g.d.w. sowohl A als auch B wahr ist; usw. Beachte: die materiale Implikation A ! B wird als wahr definiert g. d. w. A falsch ist oder B wahr ist, d. h. es gilt: A ! B: $ :A _ B. Die materiale Implikation ist schwächer als das natursprachliche wenn-dann, da ihre Wahrheit keinen inhaltlichen Zusammenhang zwischen A und B voraussetzt. 2.2 Quantoren 1. Stufe und Individuenvariablen – Prädikatenlogik PL 1. Stufe: Quantoren 1. Stufe quantifizieren über Individuen. Die zugehörigen Variablen 1. Stufe heißen Individuenvariablen, für die wir x, y, … (x1, x2, …) schreiben. Die beiden wichtigsten Arten von Quantoren sind: – der Allquantor 8x (lies: 8xFx – für alle x gilt: Fx), und – der Existenzquantor 9x: (lies: 9xFx – für mindestens ein x gilt: Fx). Der semantische Bereich von Individuen, auf den sich ein Quantor bezieht, heißt sein Individuenbereich (bzw. domain) D. So sagt 8xFx, dass alle Individuen in D Merkmal F haben, und 9xFx, dass es Individuen in D gibt, die Merkmal F haben. Wenn nichts hinzugesagt wird, wählt man als Individuenbereich den universalen Bereich, d. h. die Klasse aller Individuen im Gegenstandsbereich der betreffenden Wissenschaft. Im Ausdruck „8xFx“ wird die Individuenvariable „x“ durch den Quantor „ 8x“gebunden. Die Teilformel „Fx“ in „8xFx“ nennt man den syntaktischen Bereich des Quantors „8x“. Ausdrücke, deren sämtliche Individuenvariablen (wenn solche vorkommen) gebunden sind, nennt man geschlossene Formeln bzw. Sätze. Der Ausdruck „Fx“ heißt offene Formel, weil seine Individuenvariable „x“ frei ist, d. h. durch keinen Quantor gebunden wird. Durch offene Formeln wie z. B. „Fx ^ Gx“ werden komplexe generelle Begriffe ausgedrückt. Eine ausgezeichnete logische Relation ist die Identität: x = y steht für „x ist identisch mit y“, d. h. „x“ und „y“ bezeichnen dasselbe Individuum. Die

3.1 Begriffe und Begriffsarten

Logik der Sätze, die aus singulären Termen, generellen Begriffen 1. Stufe, extensionalen Satzoperatoren und Quantoren 1. Stufe gebildet werden, ist die Prädikatenlogik (PL) 1. Stufe (Einführungswerke z. B. Klenk 1989, Essler et al. 1991). Die PL 1. Stufe enthält die AL als Teilfragment und gehört zum wichtigsten logischen Rahmenwerk der Wissenschaftstheorie und der Mathematik, weil sich darin die typenfreie Mengenlehre darstellen lässt, auf welcher die gesamte klassische Mathematik aufbaut. 2.3 Intensionale Satzoperatoren sind Satzoperatoren, die nicht wahrheitsfunktional sind. Ein Beispiel ist der Notwendigkeitsoperator F („FA“ für „A ist notwendig“): der Wahrheitswert von „FA“ ist nicht schon durch den Wahrheitswert von A festgelegt. Stehe z. B. A für den wahren Satz „morgen regnet es“, und B für den wahren Satz „2 + 2 = 4“, dann sind sowohl A und B wahr, aber nur FB ist wahr, während FA falsch ist, denn dass es morgen regnet, ist keine Notwendigkeit. Andere Beispiele sind deontische Satzoperatoren („A ist geboten“), usw. Die Logik intensionaler Satzoperatoren ist die verallgemeinerte Modallogik (s. z. B. Hughes/Cresswell 1996). 2.4 Quantoren und Variablen 2. (bzw. höherer) Stufe quantifizieren analog über Merkmale oder Sachverhalte und werden in PL 2. (bzw. höherer) Stufe behandelt. Notation: wir bezeichnen Variablen höherer Stufe durch Großbuchstaben X, Y, …. Die Logik höherstufiger Begriffe und Quantoren ist die logische Typentheorie. 2.5. Mengentheoretische Symbole: Mengen sind Zusammenfassungen von Individuen zu einem komplexen Individuum namens ,Menge‘. – Logische Notation: – {x: Fx} steht für die Menge aller Individuen x, die die Eigenschaft F besitzen, – {a1, …, an} steht für die Menge bestehend aus den Individuen a1, …, an. – (a1, …, an) steht dagegen für die geordnete Folge bestehend aus a1, …, an. – “x 2 y“ steht für „x ist Element der Menge y“. Beachte: Mengen sind invariant unter Vertauschungen und Wiederholungen ihrer Elemente; z. B. gilt {a, b} = {a, a, b} = {b, a} = {b, a, a, b} (usw.). Dagegen verlangen geordnete Folgen bzw. n-Tupel eine bestimmte Anordnung, welche auch Wiederholungen gestattet: (a, b, b, c) 6¼ (a,b,c) 6¼ (b,a,c). Die Mengentheorie wird zur informellen Logik bzw. Metalogik gerechnet, denn auch „2“ ist ein logischer Begriff in einem weiteren Sinn. In der typenfreien Mengenlehre werden Prädikationen der Form wie „a ist ein F“ durch Elementbeziehungen „a ist ein Element der Klasse aller Fs“ ersetzt, wobei die Klasse aller Fs als abstraktes Individuum betrachtet wird. Da diese Ersetzung für Prädikate beliebig hoher Stufen iterierbar ist, kann auf diese Weise jeder höherstufige Ausdruck der logischen Typentheorie in einen mengentheoretischen Ausdruck 1. Stufe übersetzt werden. Die Axiome der typenfreien Mengenlehre sind in der Sprache der PL 1. Stufe formulierbar; sie gehen aber über rein logische Prinzipien hinaus (Einführung z. B. Ebbinghaus 2003). Weitere mengentheoretische Notationen werden, soweit erforderlich, im Text erklärt. 2.6. Mathematische Begriffe haben mit logischen Begriffen gemeinsam, dass sie abstrakt-konzeptuelle Objekte bzw. Funktionen bezeichnen. Sie können entweder durch Eigenaxiome mathematischer Theorien oder durch

Mengentheoretische Begriffe

69

70

3. Das analytische Instrumentarium

mengentheoretische Definitionen charakterisiert werden. Zu den wichtigsten mathematischen Begriffen gehören (i) die Begriffe für verschiedene Zahlenarten (z. B. IN für die Menge der natürlichen und IR für die Menge der reellen Zahlen), sowie (ii) die Funktionsbegriffe, welche Operationen über solchen Zahlen ausdrücken (Addition, Multiplikation, usw.).

3.1.2 Syntax, Formalisierung und Semantik Syntax Semantik Pragmatik

Man unterteilt die Dimensionen der Sprache in Syntax, Semantik und Pragmatik. Die Syntax beschäftigt sich mit der grammatischen Form von sprachlichen Ausdrücken, die Semantik mit den Bedeutungen von Ausdrücken, und die Pragmatik schließlich mit den jenen Eigenschaften von Ausdrücken, die durch ihren Kontext bestimmt sind. Es folgen einige Erläuterungen zu Syntax und Semantik. Aus atomaren Begriffen werden gemäß syntaktischen Formregeln Terme und Sätze beliebig hoher Komplexität zusammengesetzt. Die Übersetzung eines natursprachlichen Satzes in einen Satz einer logischen Sprache nennt man Formalisierung. Hier einige Beispiele:

Formalisierung

(3.1-1) Beispiele für Formalisierung: Übersetzungen: a – dieses Tier, Rx – x ist ein Rabe, Sx – x ist schwarz. (a) Dieses Tier ist ein Rabe. Formalisierung: Ra. (b) Alle Raben sind schwarz. Teilformalisierung: Für alle x gilt: wenn x ein Rabe ist, dann ist x schwarz. Formalisierung: 8x(Rx ! Sx). (c) Einige Raben sind nicht schwarz. Teilformalisierung: Es gibt ein x, sodass x ein Rabe ist und nicht schwarz ist. Formalisierung: 9x(Rx ^ :Sx).

Bedeutung und Referenz

Logische Formalisierung ist für die fortgeschrittene Wissenschaftstheorie ein wichtiges Hilfsmittel, um die logische Struktur von Sätzen aufzudecken und darauf aufbauend logische Zusammenhänge zu erkennen. Formalisierung ist jedoch ist kein Selbstzweck, sondern wird durch ihre kognitive Nützlichkeit im Vergleich zum formalen Aufwand gerechtfertigt. In diesem Buch wird von Formalisierung so wenig als möglich Gebrauch gemacht. Für die meisten Zwecke genügen Teilformalisierungen wie (3.1-1) oben. In der Semantik wird zwischen zwei Arten von semantischen Bezügen unterschieden: jeder Ausdruck hat einerseits eine Bedeutung, und andererseits einen Gegenstandsbezug bzw. eine Referenz. (Die Terminologien sind allerdings unterschiedlich; vgl. Kutschera 1975, Kap. 2; Runggaldier 1990, 153.) Die Bedeutung eines Begriffs ist das, was jedem kompetenten Sprecher bekannt sein muss, um den Begriff richtig zu verstehen. Mithilfe seines Wissens um die Bedeutung und seines empirischen Wissens über die Welt ist ein Sprecher in der Lage, die Referenz seiner Begriffe mehr oder weniger (un)vollständig zu bestimmen (s. Kap. 1.2.5.2). Worin die Bedeutung genau besteht (in der Vorstellung, im Gedanken, in der Definition, im Gebrauch?), darüber gibt es in der Semantik kontroverse Auffassungen. Die Referenz eines singulären Terms ist unkontroversiell ein Individuum. Die Referenz von Prädikaten ist ebenfalls kontrovers: einige Autoren (z. B. Carnap 1972, Kap. I) identifizieren die Referenz eines Prädikates mit seiner Extension,

3.1 Begriffe und Begriffsarten

d. h. der Menge von Individuen, die unter das Prädikat fallen; andere Autoren (z. B. Fodor 1990, 93) identifizieren die Referenz eines Prädikates mit der durch es bezeichneten Realeigenschaft bzw. natürlichen Art. Die Bedeutung von Sätzen identifiziert man mit der durch den Satz bezeichneten Proposition. Gemäß traditioneller Auffassung bezeichnen logisch oder analytisch äquivalente Sätze dieselbe Proposition. Der Gegenstandsbezug von Sätzen wird manchmal mit Wahrheitswerten identifiziert (z. B. bei Frege 1892), aber auch mit Ereignissen oder Realsachverhalten. Unkontroversiell ist folgende Definition von „Sachverhalt“ und „Tatsache“: jeder deskriptive Satz drückt einen Sachverhalt aus; ist der Satz wahr, so besteht der Sachverhalt real und ist dann eine Tatsache. In der auf Alfred Tarski (1936a) zurückgehenden logischen Semantik werden sprachlichen Ausdrücken formale Gegenstandsbezüge in Form von extensionalen (d. h. mengentheoretischen) Interpretationen zugeordnet. Wird beispielsweise der Individuenkonstante a das Individuum a und dem Prädikat F die Klasse von Individuen F (jeweils aus dem zugrundeliegenden Individuenbereich D) zugeordnet, dann ist in der so erhaltenen Interpretation der Satz „Fa“ wahr g.d.w. (genau dann wenn) a 2 F. Wir gehen darauf nicht näher ein (zur mathematischen Logik s. z. B. Rautenberg 2002). Eine extensionale Interpretation einer Sprache heißt auch ein Modell für diese Sprache. Schreibt man obiges Minimodell für die Symbole „a“ und „F“ in die Form (D; a, F), dann entspricht dies einer ,non-statement‘-Struktur der strukturalistischen Wissenschaftstheorie (s. Balzer et al. 1987).

3.1.3 Klassifikation von Begriffen nach ihrem Inhaltstyp Diese wissenschaftstheoretisch besonders bedeutsame Klassifikation betrifft nur nichtlogische Begriffe. Abb. 3.1-2 zeigt die Übersicht. (logische Begriffe)

Beobachtungsbegriffe empirische Begriffe deskriptive Begriffe

empirische Dispositionsbegriffe

(nichtlogische

theoretische Begriffe

Begriffe) präskriptive Begriffe

Normbegriffe Wertbegriffe

Abb. 3.1-2: Klassifikation von Begriffen nach ihrem Inhaltstyp

Zunächst ist zwischen deskriptiven und präskriptiven Begriffen zu unterscheiden. Dabei fassen wir Norm- und Wertbegriffe unter der Hareschen Bezeichnung der präskriptiven Begriffe zusammen (Hare 1952). Es gibt logisch gesehen nur wenige präskriptive Begriffe: die normativen Begriffe des Gebotenseins, Verbotenseins oder Erlaubtseins, und die qualitativen, kom-

Satz und Proposition

Sachverhalt und Tatsache Logische Semantik Extensionale Interpretation

Modell

71

72

3. Das analytische Instrumentarium

Beobachtungsbegriffe i. e. S.

Empirische Dispositionsbegriffe

parativen und quantitativen Wertbegriffe (X ist ethisch oder ästhetisch wertvoll, bzw. wertvoller als Y, bzw. wertvoll im Grade so-und-so). Die Wissenschaftstheorie beschäftigt sich vorwiegend mit deskriptiven Begriffen und Sätzen, welche die erfahrbare Welt beschreiben. Es gibt ungeheuer viele deskriptive Begriffe; dabei kann es sich um singuläre Begriffe oder generelle Begriffe verschiedenen Typs handeln. Die wissenschaftstheoretisch wichtigste Unterscheidung innerhalb der deskriptiven Begriffe ist die zwischen empirischen und theoretischen Begriffen, und innerhalb der empirischen Begriffe die zwischen Beobachtungsbegriffen und empirischen Dispositionsbegriffen. (1.) Ein genereller Beobachtungsbegriff im engen Sinn (i. e. S.) drückt ein sinnlich wahrnehmbares Merkmal oder einen solchen Merkmalskomplex aus. Gemäß Kap. 2.7.2 ist ein sinnlich wahrnehmbares Merkmal ein solches, das Menschen unabhängig von ihrem Hintergrundwissen ostensiv erlernen können. Beispiele: „x ist rot“, „x ist ein Rabe“, „x ist länger als y“, „Der Zeiger des Messgerätes x zeigt auf 5“, usw. Ein singulärer Beobachtungsterm bezeichnet ein wahrnehmbares Individuum. Die klassischen und die frühen logischen Empiristen forderten, alle deskriptiven Begriffe müssten Beobachtungsbegriffe sein. Tatsächlich sind nur wenige Begriffe echte Beobachtungsbegriffe in diesem engen Sinn. Man beachte folgende Subtilität: strenggenommen ist nicht ein Merkmal als solches wahrnehmbar, sondern nur eine bestimmte Merkmalsexemplifizierung an einem wahrnehmbaren Individuum. So drückt „ist rot“ ein Beobachtungsmerkmal in Anwendung auf sichtbare Gegenstände aus; ich kann es aber auch auf unbeobachtbare Individuen wie z. B. Mikroben oder Moleküle anwenden, und an diesen Individuen ist das Merkmal nicht wahrnehmbar, da sie zu klein sind, um sichtbar zu sein. Insofern hat Quine Recht, wenn er betont, die primäre Ebene von „Beobachtbarkeit“ ist nicht die von Begriffen, sondern die von „Gelegenheitssätzen“ wie „dies da ist rot“ (1960, 25; 1995, 3 ff.). Putnam (1962, 242 f.) hat in dieser Tatsache ein grundsätzliches Hindernis für eine klare Definition von Beobachtungsbegriffen gesehen. Durch unser Kriterium der ostensiven Erlernbarkeit wird dieses Problem wie folgt gelöst: ein Merkmal ist genau dann ostensiv erlernbar, wenn es zumindest einige wahrnehmbare Exemplifikationen an wahrnehmbaren Individuen besitzt. (2.) Ein empirischer Dispositionsbegriff drückt eine empirische Disposition aus. Darunter wird die gesetzesmäßige Regelmäßigkeit eines Individuums verstanden, unter gewissen empirischen Umständen bzw. Testbedingungen eine gewisse beobachtbare Reaktion bzw. ein beobachtbares Resultat zu liefern. Beispiel: Der empirische Dispositionsbegriff „x ist wasserlöslich“ ist definitorisch äquivalent mit folgender gesetzesartigen Allhypothese, generalisiert über Zeitpunkte: „immer wenn x ins Wasser gegeben wird bzw. würde, dann wird bzw. würde sich x auflösen“. Gesetzesartigkeit bzw. das kontrafaktische Konditional („würde“) gehen zwar über das Beobachtbare hinaus (näheres dazu in Kap. 3.6, 3.10.1). Dennoch handelt es sich bei Dispositionsbegriffen um empirisch operationalisierte Begriffe, deren Bedeutung nicht theorieabhängig ist (s. dazu Kap. 5.1). Weitere Beispiele: „x ist hart“, „x wiegt auf der Balkenwaage 4 kg“, „der Mensch x

3.1 Begriffe und Begriffsarten

ist deutschsprachig“, usw. Empirische Dispositionsbegriffe entsprechen dem, was die Operationalisten (Bridgeman 1936) von allen wissenschaftlichen Begriffen forderten. Tatsächlich sind aber theoretische Begriffe nicht auf empirische Dispositionsbegriffe reduzierbar (s. Kap. 5.1). (3.) Ein empirischer Begriff im engen Sinn ist ein Beobachtungsbegriff (i. e. S.) oder ein empirischer Dispositionsbegriff. (4.) Ein theoretischer Begriff im weiten Sinn (i. w. S.) ist jeder deskriptive Begriff, der kein empirischer Begriff (i. e. S.) ist, aber dennoch eine Bedeutung hat. Ein theoretischer Begriff i. w. S. muss in keiner wissenschaftlichen Theorie vorkommen, er kann auch auf Spekulation basieren (z. B. „die erste Ursache alles Seienden“). (5.) Ein theoretischer Begriff im engen Sinn ist ein theoretischer Begriff i. w. S., dessen Bedeutung durch eine wissenschaftliche Theorie eingeführt wird. Will man die Bezogenheit auf eine bestimmte Theorie T explizit machen, dann spricht man von einem T-theoretischen Begriff – dies ist der Theoretizitätsbegriff der strukturalistischen Wissenschaftstheorie (s. Stegmüller 1986, 33). Beispiele: physikalische Kraft, chemische Substanzarten, biologischer Genotyp, psychologischer Charakter, Intention, Gesellschaftsordnung, etc. Konvention: Wenn nichts hinzugesagt wird, verstehen wir Beobachtungsbegriffe, empirische Begriffe und theoretische Begriffe jeweils im engen Sinn. (6.) Wie in Kap. 2.7.2 erläutert, verwenden eine Reihe von Wissenschaftstheoretikern Beobachtungsbegriffe bzw. empirische Begriffe in folgendem weiten Sinne: sie verstehen darunter jeden Begriff, der ein messbares Merkmal bzw. ein Datum ausdrückt. Solche empirischen Begriffe im weiten Sinne sind theorieabhängig – ihre Bedeutung hängt speziell von jenen Theorien ab, die die Funktionsweise der benutzten Messinstrumente beschreiben und als unproblematisch vorausgesetzt werden. Die Grenze zwischen empirischen Begriffen i. w. S. und theoretischen Begriffen i. e. S. kann nur pragmatisch und kontextabhängig gezogen werden. Zwischen empirischen Begriffen i. e. S. und theoretischen Begriffen i. w. S. existiert dagegen eine zwar graduelle, aber objektive Grenze.

3.1.4 Klassifikation von Begriffen nach ihrem Abstufungstyp (Skalentyp) Die meisten Merkmale in unserer Welt können nicht nur zutreffen oder nicht zutreffen; sie können auch mehr oder weniger stark ausgeprägt sein. Der Abstufungstyp eines Begriffs gibt die Feinheit seiner Ausprägungsstufen wieder. Danach lassen sich wissenschaftliche Begriffe wie in der Übersicht von Abb. 3.1-3 klassifizieren. Qualitative Begriffe sind gar nicht bzw. nur im ja-nein-Sinn abgestuft, komparative sind im Sinne einer Ordnung gestuft, und quantitative Begriffe sind am stärksten, nämlich kontinuierlich abgestuft. In der Statistik gibt man die Abstufungen eines Merkmals durch Zahlen wieder und spricht vom Skalentyp eines Begriffs (z. B. Bortz 1985, 28 – 33). Je stärker abgestuft ein Begriff ist, umso schärfere Information kann man mit ihm wiedergeben, und umso strengeren Tests kann man die mit ihm formulierten Hypothesen unterziehen. Die einzelnen Abstufungstypen seien nun erläutert.

Theoretische Begriffe i. w. S. und i. e. S.

73

74

3. Das analytische Instrumentarium Klassifikatorische Begriffe Nominalskalen (Kategorialskalen) qualitative Begriffe Komparative Begriffe Ordinalskalen (Rangskalen) Intervallskalen (Differenzskalen) quantitative Begriffe Verhältnisskalen (Ratioskalen)

Skalentyp

Abb. 3.1-3: Klassifikation von Begriffen nach ihrem Skalentyp

Skalentyp

3.1.4.1 Qualitativ-klassifikatorische Begriffe und Nominalskalen (Kategorialskalen). Mithilfe von qualitativen oder klassifikatorischen Begriffen unterteilt man die Objekte eines Objektbereichs D in gewisse Untergruppen D1, …, Dn, die durch die Zugehörigkeit zu gewissen Merkmalen bzw. Kategorien M1, …, Mn definiert sind. Dabei wird gefordert: (a) die Klassifikation muss disjunkt sein, d. h. die Merkmale Mi bzw. die zugehörigen Merkmalsgruppen müssen sich gegenseitig ausschließen, und (b) die Klassifikation muss exhaustiv sein, d. h. die Merkmale bzw. zugehörigen Merkmalsgruppen müssen den ganzen Objektbereich D ausschöpfen. Die Menge F := {M1, …, Mn} der Merkmalskategorien einer Klassifikation nennt man eine klassifikatorische (monadische) Begriffsfamilie (s. Carnap/ Jeffrey 1971, 43 ff.). Der einfachste Fall eines qualitativen Begriffes ist ein analytisch unabhängiger Grundbegriff, z. B. Fx. Er bildet die zweiwertige Begriffsfamilie {Fx,:Fx}: z. B. x ist ein Mensch oder nicht. Die Bedingungen der Disjunktivität und Exhaustivität gelten in diesem Fall schon rein logisch. Es gibt auch Begriffsfamilien mit mehr als zwei Kategorien; z. B. die Familie der Farbbegriffe {rot, gelb, grün, blau, …}, oder die Familie sozialer Schichten {Unterschicht, Mittelschicht, Oberschicht}, usw. Die Bedingungen der Disjunktivität und Exhaustivität gelten in diesen Fällen zumeist aufgrund von analytischen Konventionen. Jeder Begriffsfamilie korrespondiert ein zugehöriges Attribut – z. B. korrespondiert der Begriffsfamilie {Unter-, Mittel-, Oberschicht} das Attribut „soziale Schicht von x“. Formal ist ein Attribut eine Funktion höherer Stufe, die jedem Objekt aus D seine Merkmalskategorie zuordnet. Wenn man die Kategorien Mi durch natürliche Zahlen i ersetzt, also z. B. „Unter-, Mittel-, Oberschicht“ durch „1,2,3“, dann erhält man eine Nominalskala im engeren Sinn. Beispiel: soziale Schicht von Herrn Meier = 2. Für Nominalskalen ist die Zahlenzuordnung allerdings unwesentlich und willkürlich. In der Statistik nennt man eine Funktion, die angewandt auf verschiedene Objekte verschiedene Zahlenwerte annehmen kann, auch eine Variable; im Fall einer zweiwertigen Begriffsfamilie spricht man von binärer Variable. Statistische Variablen sind Funktionen; man hüte sich vor einer Verwechslung mit Variablen im logischen Sinn.

Anforderungen an Klassifikationen

Begriffsfamilie und Attribut

3.1 Begriffe und Begriffsarten

3.1.4.2 Komparative Begriffe und Ordinalskalen (Rangskalen). Mithilfe von komparativen Begriffen werden die Objekte eines Gegenstandsbereichs D hinsichtlich ihrer Merkmalsausprägung in eine Rangordnung gebracht, d. h. in Ranggruppen D1, …, Dn eingeteilt (s. Abb. 3.1-4): D1 enthält alle D-Objekte mit dem niedrigsten Ausprägungsgrad des Merkmals, D2 alle Objekte mit dem nächstgrößeren Ausprägungsgrad, usw. Die Objekte („o“) innerhalb einer Ranggruppe haben den gleichen Ausprägungsgrad. Ordnet man den Ranggruppen Di die natürlichen Zahlen i zu, so erhält man eine Ordinalskala bzw. Rangskala. Die entsprechende Zuordnungsfunktion rM: D !{1, …, n} ist der ordinalskalierte Funktionsbegriff bzw. die ordinalskalierte ,Variable‘. Dn

o o

Rang n

Rangskalen

zunehmende Ausprägungsstärke

# D2

o

D1

o

o o

o o

Rang 2

o

Rang 1

Abb. 3.1-4: Rangskalierung von D

Die Rangplätze einer Ordinalskala liefern lediglich komparative Information; quantitative Differenzen zwischen den Ausprägungsrängen sind unbekannt bzw. undefiniert. Man kann also nicht annehmen, die numerischen Rangdifferenzen entsprechen quantitativen Merkmalsdifferenzen. Beispielsweise wird in der Mohs-Skala die Härte von Mineralien durch wechselseitige Ritzvergleiche festgelegt: ritzt Mineral A Mineral B, aber nicht umgekehrt, so hat A einen höheren Härtegrad. Mohs gelangte damit zu einer zehnstufigen Härteskala: 1 (Talk), 2 (Gips), 3 (Kalkspat), bis zu 9 (Korund) und 10 (Diamant). Man darf daraus nicht schließen, die Härtedifferenz zwischen Talk und Gips sei z. B. der neunte Teil der Härtedifferenz zwischen Talk und Diamant. Auch psychische Einstellungsmerkmale haben zumeist nur Ordinalskalenniveau: ich kann z. B. sagen, dass mir eine Speise besser schmeckt als eine andere, ohne sagen zu können, um wieviel sie mir besser schmeckt. Ein Erhebungsverfahren, das man in den empirischen Sozialwissenschaften bei Einstellungsmessungen durchgängig anwendet, besteht darin, Versuchspersonen (Vpn) aufzufordern, die zu beurteilenden Sachverhalte in eine vorgegebene Ordinalskala einzuordnen. Ein Fragebogen zur Einstellung von Vpn gegenüber dem Alter könnte z. B. folgende Frage enthalten: Bitte kreuzen Sie an: Ich halte die Sicherung der Alterspensionen für: ganz wichtig eher wichtig bin unentschieden eher unwichtig ganz unwichtig

Man gelangt damit zu einer 5-stufigen Ordinalskala, von ganz unwichtig (1) bis zu ganz wichtig (5). Die Einstellungen der befragten Personen werden so in eine Rangreihung gebracht. Der logisch grundlegendste Begriff von Ordinalskalen ist der KleinerGleich-Begriff: x M y besagt, dass x das Merkmal M in höchstes so hohem

Einstellungsmessung

75

76

3. Das analytische Instrumentarium

Axiome einer Quasi-Ordnung

Intransitivität der ungefähren Gleichheit

Metrisierung

Ausprägungsgrad wie y besitzt. Damit definiert man den Begriff der MGleichheit: x  My :$ x  My ^ y  M x, der besagt, dass x und y Merkmal M im gleichen Ausprägungsgrad besitzen, und den Kleiner-Begriff xM wie z. B. „ist länger als“ über einen Gegenstandsbereich D in Form einer Verhältnisskala genau dann metrisierbar ist, wenn M eine monotone Quasi-Ordnung über D ist und die Archimedische Bedingung erfüllt ist (s. Krantz et al. 1971, 73 ff.), welche fordert, jeder noch so große Gegenstand b muss sich durch hinreichend viele Kopien eines noch so kleinen Gegenstandes a aufwiegen lassen. Für soziale Präferenzordnungen ist diese Bedingung oft nicht erfüllt – man würde nur ungerne behaupten wollen, dass der Wert eines Menschenlebens durch hinreichend viele Euroscheine aufgewogen werden könne (s. Schurz 1997a, Kap. 11.5.2). Subjektive Einstellungsmessungen (z. B. 5 = sehr wichtig, 3 = unentschieden, 1 = unwichtig) besitzen streng genommen nur Ordinalskalenniveau (vgl. Bortz 1985, 31 f.). Dennoch werden sie in sozialwissenschaftlichen Experimenten häufig durch statistische Test ausgewertet, die Intervallskalen voraussetzen. Um sich der Harmlosigkeit einer solchen Idealisierung zu vergewissern, ist es nötig, die Ergebnisse von intervallskalenbasierten statistischen Tests durch ordinalskalenbasierte statistische Tests zu überprüfen (s. Bortz 1985, 178 ff.). Unsere bisherigen Ausführungen zu Metrisierung betrafen fundamentale empirische Metrisierung. Von abgeleiteter Metrisierung spricht man, wenn die Metrisierung eines Begriffes auf die Metrisierung anderer Begriffe zurückgeführt wird, z. B. Dichte auf den Quotienten von Masse durch Volumen, usw. Von einer theoriegeleiteten Metrisierung spricht man schließlich, wenn sich die Metrisierung auf eine bestimmte, z. B. physikalische, Theorie stützt (s. Balzer 1985, 6). In theoriegeleiteten Metrisierungen kann sich aus

3.2 Klassifikation von Sätzen nach dem Inhaltstyp

theoretischen Gründen eine Änderung des Skalenniveaus der ursprünglich empirisch metrisierten Größe ergeben. Ein Beispiel hierfür ist die absolute Temperaturskala nach Kelvin (°K), welche bei – 273,15 °C = 0 °K den absoluten Nullpunkt der Temperatur postuliert. Die Annahme des absoluten Nullpunktes bei –273,15 °C resultierte aus der Hypothese der kinetischen Gastheorie, derzufolge Wärme und Temperatur auf die Bewegung von Molekülen zurückzuführen ist, wobei das Eigenvolumen dieser Moleküle in idealen Gasen (Gasen bei hohen Temperaturen) vernachlässigbar gering ist. Bei idealen Gasen wächst das Volumen mit der Temperatur linear an (V = kNT). Verlängert man diesen Zusammenhang in einem T6V-Koordinatensystem nach links in Regionen negativer °C-Temperaturen, so stößt man auf eine Temperatur von – 273,15 °C, bei der das Volumen des idealen Gases Null werden müsste: hier würden die Gasmoleküle auf unendlich kleinem Raum zusammengedrängt sein und sich daher nicht mehr bewegen können. Diesen Punkt hat Kelvin daher als den absoluten Nullpunkt identifiziert. Seine Hypothese, dass keine noch so extreme Abkühlung den absoluten Nullpunkt unterschreiten kann, wurde seitdem vielfach bestätigt.

Theoriegeleitete Metrisierung

3.2 Klassifikation von Sätzen nach dem Inhaltstyp Auch Sätze lassen sich in mehreren Dimensionen klassifizieren. Wissenschaftstheoretisch am bedeutendsten ist die Klassifikation von Sätzen nach ihrem Inhaltstyp, sowie die nach ihrem Allgemeinheitsgrad in Kap. 3.5. Abb. 3.2-1 zeigt die Übersicht. logisch analytisch definitorisch

Beobachtungssätze (}) empirisch

empirische Allsätze (...)

deskriptiv synthetisch

theoretisch

rein theoretisch gemischt-theoretisch

normativ rein präskriptiv evaluativ gemischt-präskriptiv

Abb. 3.2-1: Klassifikation von Sätzen nach ihrem Inhaltstyp

Analytisch wahre Sätze sind solche, deren Wahrheit unabhängig von der faktischen Beschaffenheit der Welt allein durch die Gesetze der Logik oder durch extralogische Bedeutungskonventionen der Sprache bestimmt ist. Analytisch falsche Sätze sind analog solche, deren Falschheit auf diese Weise bestimmt ist, und analytisch determinierte (kurz: analytische) Sätze sind solche, deren Wahrheitswert (wahr oder falsch) auf diese Weise bestimmt

Analytische versus synthetische Sätze

79

80

3. Das analytische Instrumentarium

ist. Analytische Sätze zerfallen weiter in logische determinierte und definitorisch determinierte Sätze (Kap. 3.3 – 3.4). Synthetische Sätze sind dagegen alle Sätze, deren Wahrheitswert nicht analytisch bestimmt ist, sondern von der faktischen Beschaffenheit der Welt anhängt (bzw. im Fall von präskriptiv-synthetischen Sätzen von unserer moralischen Interpretation derselben). In Anknüpfung an unsere inhaltliche Klassifikation von Begriffsarten teilen wir die synthetischen Sätzen zunächst in deskriptive und präskriptive Sätze ein; genauer gesagt in deskriptive Sätze, rein präskriptive Sätze und gemischte Sätze mit deskriptiven und präskriptiven Satzkomponenten. Deskriptive Sätze sagen etwas über die faktische Beschaffenheit der Welt aus; präskriptive Sätze drücken dagegen Norm- oder Wertsachverhalte aus.

Subjektive Einstellungsoperatoren

3.2.1 Deskriptive versus präskriptive Sätze. Nicht jeder Satz, der einen präskriptiven Begriff enthält, ist deshalb schon präskriptiv. Sobald ein präskriptiver Begriff im Bereich eines subjektiven Einstellungsoperators liegt – d. h. eines Satzoperators, der eine subjektive Einstellung zu einem Sachverhalt ausdrückt – , verliert dieser präskriptive Begriff seine präskriptive Kraft. So sind beispielsweise die Sätze Peter glaubt, dass Stehlen schlecht ist, – oder Peter glaubt, dass man nicht stehlen soll empirisch-deskriptiv, denn sie sagen etwas über die faktische Wert- bzw. Normeinstellung Peters aus. Dasselbe gilt für alle subjektiven Einstellungsoperatoren, wie z. B. Person X glaubt, denkt, fühlt, sagt, wünscht, … (usw.), dass A. Die Bedeutung dieses Sachverhaltes für die Wertneutralitätsthese wurde in Kap. 2.5.1 herausgearbeitet.

(Def. 3.2-1) (a) Ein Satz ist deskriptiv g.d.w. jedes in ihm vorkommende präskriptive Symbol im Bereich eines subjektiven Einstellungsoperators liegt. (b) Ein Satz ist rein präskriptiv g.d.w. jeder seiner deskriptiven Teilsätze im Bereich eines präskriptiven Operators liegt. (c) Ein weder rein deskriptiver noch rein präskriptiver Satz heißt gemischt-präskriptiv. Hinweis: Gemischt-präskriptive Sätze haben sowohl deskriptive Teilsätze, die in keinem Bereich eines präskriptiven Operators liegen, als auch rein präskriptive Teilsätze. (3.2-1) Beispiele für deskriptive Sätze: (i) Peter stiehlt (de fakto). (ii) Peter glaubt, dass Stehlen verboten ist – Glaubt(peter, Verboten(Stehlen)). Beispiele für rein präskriptive Sätze: (iii) Stehlen ist verboten. – Verboten(Stehlen). (iv) Wenn eine Handlung unter allen Umständen ethisch schlecht ist, dann ist sie verboten. – 8 h(8u(Schlecht (h, u)) ! Verboten(h)).

3.2 Klassifikation von Sätzen nach dem Inhaltstyp

Beispiele für gemischt-präskriptive Sätze: (v) Wenn jemand Hunger leidet, dann darf er stehlen. – 8x (Wenn Hungerleidet(x), dann Erlaubt (x stielt)). Zwei Hinweise: (1.) Implikationszusammenhänge zwischen rein präskriptiven Sätzen wie in (iv) sind selbst rein präskriptiv. (2.) Def. 3.2-1 ist syntaktischer Natur – man kann sie mithilfe der Methode der essentiellen Satztypen gemäß Kap. 3.5 verstärken. 3.2.2 Beobachtungs- und Basissätze, empirische Sätze und theoretische Sätze: Aufbauend auf der inhaltlichen Begriffsklassifikation von Kap. 3.1.3 klassifizieren wir deskriptive Sätze nach ihrem Inhaltstyp. Hierbei sind zwei in der Literatur wenig diskutierte Subtilitäten zu bewältigen, die mit der in Kap. 3.1.3 erwähnten Tatsache zu tun haben, dass Exemplifikationen von Beobachtungsmerkmalen nur beobachtbar sind, wenn die sie exemplifizierenden Individuen ebenfalls beobachtbar sind. 1. Subtilität: Ein empirischer Satz wird üblicherweise als ein Satz definiert, der (neben logischen Begriffen) nur empirische Begriffe enthält (z. B. Kutschera 1972, 260, 292; Tuomela 1973, 23 f.). Dies ist aber ungenügend. Betrachten wir einen Allsatz „Alle Fs sind Gs“, der nur die Beobachtungsbegriffe F und G enthält. Dieser Allsatz kann nur dann als empirisch bezeichnet werden, wenn er sich nur auf wahrnehmbare Individuen bezieht. Ob dies der Fall ist, hängt von der Bedeutung des Begriffs F ab: nur wenn aus der Bedeutung von F folgt, das F nur auf wahrnehmbare Individuen zutreffen kann, ist der Allsatz ein empirischer Allsatz. Beispielsweise ist „alle Raben sind schwarz“ ein empirischer Allsatz, weil aus der Bedeutung von „Rabe“ folgt, dass nur wahrnehmbare Individuen Raben sein können. Andererseits ist „alle losen Gegenstände auf der Erde fallen nach unten“ kein empirischer Allsatz, sofern wir uns mit „Gegenständen“ auch auf mikroskopisch kleine Partikel beziehen. Derselbe Punkt betrifft auch Existenzsätze. Lewis (1970, 79) und van Fraassen (1980, 13 f.) schließen aus diesem Problem, dass man die empirische vs. theoretische Natur eines Satzes nicht aus der Bedeutung seiner Begriffe erschließen kann. Wie wir zeigen, geht dies doch. Zu diesem Zweck führen wir folgende Definition ein: Ein Quantor hat in einem Satz empirische Reichweite g.d.w. dieser Satz analytisch äquivalent ist mit seiner quantoriellen Einschränkung auf beobachtbare Individuen (bei einem Allsatz 8xA ist diese Einschränkung definiert als 8x(Bx!A), und bei einem Existenzsatz 9xA als 9x (Bx^A), mit „Bx“ für „x ist beobachtbar“). Andernfalls hat der Quantor im Satz theoretische Reichweite. 2. Subtilität: Für Beobachtungssätze ist es wesentlich, dass sie nur über endlich bzw. überschaubar viele Individuen sprechen. Denn was der Mensch beobachten kann, ist von beschränkter endlicher Komplexität. Es würde nicht genügen, Beobachtungssätze auf Singulärsätze – das sind Sätze ohne Quantoren – einzuschränken, die nur Beobachtungsbegriffe enthalten. Auch lokalisierte Existenzsätze der Form „an der Raumzeitstelle k gibt es das-und-das Beobachtbare“ sind Beobachtungssätze; z. B. „in diesem Korb ist ein roter Apfel“ (Popper 1935/76, 67, spricht von „singulären Esgibt-Sätzen“). Dies ist deshalb der Fall, weil der Existenzquantor in einem

Definitionsschwierigkeiten

Quantoren mit empirischer Reichweite

81

82

3. Das analytische Instrumentarium Lokalisierte Existenzund Allsätze

Beobachtungssätze, empirische Sätze und theoretische Sätze

solchen Satz empirische Reichweite besitzt, und weil das beschränkte Raumzeitgebiet klein genug ist, um visuell überblickt werden zu können. Aus demselben Grund müssen aber auch lokalisierte Allsätze zu den Beobachtungssätzen gerechnet werden, die folgende Form besitzen: alle (beobachtbaren) Individuen in diesem beschränkten Raumzeitgebiet haben diesund-das Beobachtungsmerkmal, z. B. „Alle Äpfel in diesem Korb sind rot“. Wieder hat der Allquantor empirische Reichweite. Zusätzlich ist hier wesentlich, dass wahrnehmbare Individuen eine gewisse Ausdehnung haben müssen und sich nicht räumlich überlappen können. Daher folgt aus der Bedeutung von „Äpfel“ und „Korb“, dass in dem Korb nur endlich bzw. überschaubar viele Äpfel Platz haben, weshalb über den Wahrheitswert dieses Satzes durch Beobachtung entschieden werden kann. Wir gelangen damit zu folgender Definition: (Def. 3.2-2) Sei S ein synthetisch-deskriptiver Satz. Dann gilt: (a) S ist ein Beobachtungssatz (i. e. S.) g. d. w. S neben logischen Begriffen nur Beobachtungsbegriffe (i. e. S.) enthält, und S entweder ein singulärer Satz ist, oder ein lokalisierter Existenzsatz bzw. Allsatz, dessen Quantoren empirische Reichweite besitzen. (b) S ist ein empirischer Satz (i. e. S.) g.d.w. S neben logischen-mathematischen Begriffen nur empirische Begriffe (i. e. S.) enthält und seine Quantoren empirische Reichweite besitzen. (c) S ist ein theoretischer Satz i. w. S. g.d.w. S auch theoretische Begriffe i. w. S. enthält oder seine Quantoren theoretische Reichweite besitzen. Falls die theoretischen Begriffe (bzw. Variablen) durch eine wissenschaftliche Theorie T eingeführt werden, heißt der Satz theoretisch i. e. S., bzw. T-theoretisch. In Analogie zu Kap. 3.1.3 sprechen wir von einem empirischen Satz im weiten Sinn, wenn er neben logisch-mathematischen Begriffen nur empirische Begriffe i. w. S. (bzw. Quantoren mit empirischer Reichweite i. w. S.) enthält. Empirische Sätze i. w. S. sind abhängig von unproblematischen Vortheorien und geben das wieder, was Wissenschaftler pragmatisch Daten nennen. Konvention: Wenn nichts hinzugesagt wird, verstehen wir Beobachtungs-, empirische und theoretische Sätze im engen Sinn. Die Klasse der empirischen Sätze ist um ein Vielfaches umfassender als die Klasse der Beobachtungssätze. Sie umfasst neben Beobachtungssätzen auch: (i) universelle empirische Allsätze wie z. B. „alle Raben sind schwarz“, (ii) singuläre empirische Dispositionsaussagen wie „dies ist wasserlöslich“, (iii) generelle empirische Dispositionsaussagen wie „Zucker ist wasserlöslich“. Eine Untergruppe der empirischen Dispositionsaussagen sind empirische Messaussagen, z. B. „dieser Baum ist (gerundet) 10,52 m hoch“, oder „alle ausgewachsenen Fichten sind mehr als 10 m hoch“. Bei den theoretischen Sätzen muss es sich nicht um Allsätze handeln. Auch Singulär- oder Existenzsätze können theoretisch sein. Z. B. drückt der Singulärsatz „dieses Mineral ist radioaktiv“ eine theoretische Hypothese

3.3 Logische Wahrheit und deduktive Logik

aus, die nur durch theoriegeleitete Messung mithilfe komplizierter Messgeräte überprüft werden kann. Theoretische Sätze sind Sätze, die auch theoretische Begriffe enthalten. Theorien sind schließlich Systeme von theoretischen Sätzen (s. Kap. 5). Wir betonen, dass Theorien auch empirische Begriffe enthalten dürfen, und um empirisch gehaltvoll zu sein, sogar müssen, weil von einigen Autoren (z. B. Putnam 1962, Quine 1995, 8 – 10, Chalmers 1994, 32) der irreführende Eindruck erweckt wird, als würde ein Begriff, sobald er in einer Theorie vorkommt, zu einem theoretischen Begriff werden. So wie wir empirische Begriffe definiert haben, ist dies nicht der Fall. Noch enger als die Kategorie der Beobachtungssätze ist die der Basissätze: Basissätze (im Sinne von Carnap 1950b, 67, und Hempel 1965, 277) sind empirische Atomsätze oder deren Negationen, z. B. dieser Baum ist (nicht) grün. Basissätze im Sinne Poppers sind lokalisierte Existenzsätze, deren syntaktischer Bereich eine Konjunktion von Basisformeln ist, z. B. „dort befindet sich ein schwarzer Rabe“. Alle Basissätze sind Beobachtungssätze, aber nicht umgekehrt. Im Wiener Kreis wollte man mit Basissätzen das unmittelbar Beobachtbare von dem abgrenzen, was bloß mittelbar beobachtbar ist, insofern es aus unmittelbar Beobachtbarem logisch folgt. Z. B. ist die Disjunktion „dort sitzt eine Krähe oder ein Rabe“ nicht unmittelbar beobachtbar; sie kann jedoch aus unmittelbaren Beobachtungen folgen. Die nähere Analyse erwies diese Abgrenzung jedoch als sehr schwierig (s. Schurz 1983, 302).

Theorien

Basissätze

3.3 Logische Wahrheit und deduktive Logik Gemäß der philosophischen Tradition versteht man unter logischen Wahrheiten denknotwendige Wahrheiten. Sowohl die Philosophiegeschichte wie die Kognitionspsychologie zeigen aber, dass unsere intuitiven Vorstellungen von Denknotwendigkeit unverlässlich sind. Die moderne Logik ersetzt den intuitiven Notwendigkeitsbegriff durch exakte Begriffe. Grundlegend für die moderne Definition von logischer Wahrheit ist die in Kap. 3.1.1 eingeführte Unterscheidung zwischen logischen und nichtlogischen Begriffen. Wir geben nun eine systemübergreifende Charakterisierung von logischer Wahrheit, die auf beliebige Logiksysteme anwendbar ist: (3.3-1) Logische Wahrheit – vorläufige Charakterisierung: Ein Satz ist logisch wahr g.d.w. seine Wahrheit allein aus seiner syntaktischen Struktur und der Bedeutung seiner logischen Begriffe logisch folgt. In einer äquivalenten Reformulierung dieser Idee sagt man: ein Satz ist logisch wahr g.d.w. seine Wahrheit schon aus seiner logischen Form folgt (diese Charakterisierung geht auf Wittgensteins Traktat 1921 zurück). Dabei erhält man die logische Form eines (natursprachlichen) Satzes, wenn man alle nichtlogischen Begriffe des Satzes durch metasprachliche Variablen ersetzt. Die logische Form eines Satzes ist also genau das, was man nach der Formalisierung des Satzes erhält (Kap. 3.1.2), allerdings ohne eine natursprachliche Interpretation seiner nichtlogischen Terme.

Logische Form

83

84

3. Das analytische Instrumentarium

Die Charakterisierung (3.3-1) ist nur vorläufig, da sie aufgrund des Auftretens des Wortes „logisch folgt“ im Definiens zirkulär zu sein scheint. Das Wesentliche einer logischen Satzform ist es, dass man ihre nichtlogischen Terme beliebig interpretieren kann. Wenn jede solche Interpretation den Satz wahr macht, dann ist der Satz logisch wahr. Wir können daher die logische Wahrheit eines Satzes aufbauend auf der Idee (3.3-1) so definieren: Logische Wahrheit

(Def. 3.3-1) Ein Satz ist logisch wahr (kurz: L-wahr) g.d.w. die ihm zugehörige Satzform unter jeder möglichen Interpretation wahr wird. Wir nennen dann auch diese Satzform logisch wahr. Definition (3.3-1) geht auf Tarski (1936b) zurück. Sie lässt sich mühelos auf spezielle Logiksysteme übertragen, indem man die zugelassenen Interpretationen spezifiziert. In der AL sind dies Wahrheitswertzuordnungen zu atomaren Sätzen. In der PL sind dies die in Kap. 3.1.2 erläuterten extensionalen Interpretationen, wobei zu beachten ist, dass dabei nicht nur die Interpretation der nichtlogischen Terme, sondern auch der Individuenbereich D variieren darf. Zunächst einige Beispiele. (3.3-2) Beispiel eines L-wahren Satzes: Wenn alle Menschen sterblich sind, dann ist kein Mensch unsterblich. Satzform: Wenn alle Fs Gs sind, dann gibt es kein F, das nicht G ist. Formalisierung: 8x(Fx ! Gx) ! : 9x(Fx ^ : Gx) Weitere Beispiele: A _ : A, A ! A _ B, 8xFx ! Fa, Fa! 9xFx, usw.

Logische Falschheit

Konsistenz und Inkonsistenz Kontingenz

Die Satzform (3.3-2) ist L-wahr, denn sie ist unter jeder (extensionalen) Interpretation von „F“ und „G“ wahr. M.a.W., sie ist in jeder möglichen Welt wahr – denn mögliche extensionale Interpretationen entsprechen möglichen Welten. In Analogie zu (Def. 3.3-1) definiert man einen Satz als logisch falsch (L-falsch) g.d.w. die zugehörige Satzform unter jeder möglichen Interpretation falsch ist. Eine Satzmenge heißt konsistent (bzw. inkonsistent), wenn mindestens eine (bzw. keine) mögliche Interpretation jede Satzform dieser Menge wahr macht. Logisch falsche Sätze bzw. Satzmengen nennt man inkonsistent bzw. widersprüchlich, weil man zeigen kann, dass sich daraus logische Widersprüche wie z. B. „p und nicht-p“ herleiten lassen. Ein weder L-wahrer noch L-falscher Satz heißt kontingent. Ein Beispiel eines logisch falschen Satzes wäre p^:p (heute regnet es und es regnet nicht). Beispiele für kontingente Sätze sind Fa (Peter ist krank), 8x(Fx ! Gx) (alle Raben sind schwarz), usw. Die allermeisten Sätze sind kontingent. Mithilfe derselben Methode definiert man die logische Gültigkeit von Argumenten bzw. Schlüssen. Dass die Konklusion aus den Prämissen logisch folgt, bedeutet, dass die Wahrheit mit Sicherheit bzw. in allen extensionalen Interpretationen (,Welten‘) von den Prämissen auf die Konklusion übertragen wird:

3.4 Bedeutungskonventionen und definitorische Wahrheit

(Def. 3.3-2) Ein Schluss ist logisch gültig (kurz: gültig) g.d.w. für seine zugehörige Schlussform gilt, dass alle möglichen (extensionalen) Interpretationen, welche sämtliche Prämissen wahr machen, auch die Konklusion wahr machen. – Äquivalent ausgedrückt: … g.d.w. es keine Interpretation gibt, welche sämtliche Prämissen wahr macht, aber die Konklusion falsch macht. Ist ein Schluss gültig, so sagt man, seine Konklusion folgt (logisch) aus den Prämissen. (3.3-2) Beispiel eines logisch gültigen Schlusses Prämisse 1: Alle Metalle leiten Strom Prämisse 2: Dein Koffer ist metallisch

8x(Fx ! Gx) Fa

Konklusion: Daher leitet dein Koffer Strom.

Ga

Ebenso wie L-Wahrheit ist logische Gültigkeit etwas, das schon aus der logischen Form eines Schlusses folgt. Daraus ergibt sich etwas Wichtiges: die Gültigkeit eines Schlusses hängt nicht von der Wahrheit seiner Prämissen ab (ein häufiges Missverständnis). Dass ein Schluss gültig ist, heißt nur: wenn seine Prämissen wahr wären, dann müsste auch seine Konklusion wahr sein. Auch ein Schluss mit falschen Prämissen kann also gültig sein, wie z. B.: „Alle Metalle sind lebendig. Dein Koffer ist metallisch. Also ist dein Koffer lebendig“. Diese Folgerungsbeziehung gilt aus logischen Gründen. Dass dagegen Metalle nicht lebendig sind, ist ein kontingentes Faktum. Freilich muss man, um einen Schluss zum Zwecke einer Begründung oder Prognose anzuwenden, auch gute Gründe dafür besitzen, dass seine Prämissen wahr sind. Allerdings kann man gültige Schlüsse nicht nur zu Begründungszwecken, sondern auch zu Falsifikationszwecken verwenden: in diesem Fall schließt man aus der Falschheit der Konklusion darauf, dass mindestens eine Prämisse falsch gewesen sein muss. Semantische Beweise der L-Wahrheit und Gültigkeit in der PL sind oft kompliziert und abstrakt. Aus diesem Grund benutzt man in der Logik zugleich ein anderes Verfahren: die deduktive Methode. Hierbei nimmt man einige grundlegende L-Wahrheiten als Axiome und einige L-gültige Schlüsse als Regeln an und bezweckt, durch Hintereinanderschaltung von Axiomen und Regelanwendungen in Form von Beweisen alle anderen L-wahren Sätze bzw. gültigen Schlüsse beweisen zu können (s. z. B. Klenk 1989). Ein volles logisches System besteht aus einer Sprache, einer logischen Semantik und einem deduktiven Beweissystem (bzw. Kalkül).

3.4 Bedeutungskonventionen und definitorische Wahrheit Definitionen wie z. B. „Ein Junggeselle ist ein bislang unverheirateter (d. h. weder geschiedener noch verwitweter) Mann“ sind ebenso wie L-wahre Sätze analytische Wahrheiten, d. h. ihre Wahrheit folgt allein aus der Bedeutung ihrer Begriffe, unabhängig von der faktischen Beschaffenheit der

Logische Gültigkeit

Deduktive Methode

85

86

3. Das analytische Instrumentarium

Welt. Doch im Unterschied zu L-wahren Sätzen wird die Wahrheit von Definitionen nicht durch die Bedeutung ihrer logischen Begriffe bestimmt, sondern sie liegt an Bedeutungskonventionen für ihre speziellen nichtlogischen Begriffe, in unserem Beispiel „Junggeselle“, „Mann“ und „bislang unverheiratet“. Die Wahrheit definitorisch wahrer Sätze ist daher nicht eine Sache ihrer logischen Form; wohl aber ist sie eine Sache der Bedeutungskonventionen der zugrunde liegenden Sprache. Wir definieren: Definitorische Wahrheit

(Def. 3.4-1) (a) Ein Satz ist definitorisch wahr (bzw.: extralogisch-analytisch wahr) g.d.w. seine Wahrheit auf gewissen Bedeutungskonventionen für seine nichtlogischen Begriffe beruht. – M.a.W., ein definitorisch wahrer Satz folgt logisch aus der Menge dieser Bedeutungskonventionen, ohne selbst L-wahr zu sein. (b) Ein Satz ist analytisch wahr g.d.w. er logisch oder definitorisch wahr ist.

Beispiele definitorisch wahrer Sätze: (3.4-1) Kein Junggeselle ist verheiratet. :9x(Jx ^ Vx) (3.4-2) Die Länge des Normalmaßstabes in Paris ist ein Meter. länge(a) = (1,m) Beispiele synthetisch wahrer Sätze: (3.4-3) Kein Eisbär ist schwarz. :9x(Ex ^ Sx) (3.4-4) Die Länge von Peters Hose ist ein Meter. länge(hose(peter)) = (1,m)

Empirische Adäquatheit von Definitionen

Den Unterschied zwischen analytischen und synthetischen Wahrheiten kann man sich so klarmachen: Wenn ein Zoologe die Wahrheit von (3.4-3) als empirisch bestätigt ansieht, so macht seine Rechtfertigung Sinn. Würde uns aber jemand berichten, trotz umfangreicher empirischer Nachforschungen sei er bisher noch auf keinen Junggesellen getroffen, der verheiratet gewesen wäre, so hat diese Person offenbar die Bedeutung von „Junggeselle“ nicht verstanden: Junggesellen sind per definitionem unverheiratet; um dies zu wissen, sind keine empirischen Forschungen notwendig. Die Beispiele machen auch deutlich, dass definitorische und synthetische Wahrheiten nicht durch ihre logische Form zu unterscheiden sind, denn (3.4-1) und (3.4-3) bzw. (3.4-2) und (3.4-4) haben jeweils dieselbe logische Form. Obwohl Definitionen nicht empirisch wahr oder falsch sein können, so können sie doch mehr oder weniger empirisch adäquat sein (s. auch Hempel 1965, Kap. 6, § 5). Beispielsweise klassifizierte man bis Linné Tiere und Pflanzen nach ihren phänomenologischen Merkmalen. Die Klassifikation nach der genetischen Abstammung, die Darwin ein Jahrhundert später begründete, stellte sich als empirisch wesentlich adäquater heraus, weil sie wesentlich mehr nomologisch relevante Merkmalszusammenhänge widerspiegelte. Niemand kann jemandem verbieten, „Fische“ als „wasserlebende Wirbeltiere“ zu definieren. Die Tatsache, dass dann auch Delphine etc. zu den Fischen zählen würden, obwohl sie von landlebenden Säugetieren ab-

3.4 Bedeutungskonventionen und definitorische Wahrheit

stammen und mit diesen wesentlich mehr Merkmale gemeinsam haben als mit den restlichen Fischen, würde diese Definition nicht widerlegen – zeigt aber ihre empirische Inadäquatheit auf. Manche Autoren bezeichnen besonders ,natürliche‘ bzw. empirisch adäquate Definitionen als empirisch wahre ,Realdefinitionen‘ (vgl. Weingartner 1996, 292). Diesem Sprachgebrauch schließen wir uns nicht an, weil damit die Abgrenzung zwischen Definitionen und synthetischen Sätzen unklar werden würde. Wir verstehen unter „Definition“ immer eine mehr oder weniger empirisch adäquate sprachliche Konvention. Es gibt drei verschiedene Arten von definitorisch wahren Sätzen, nämlich: D1) Explizitdefinitionen: In ihnen wird ein Begriff (das Definiendum) per Konvention mit einem Komplex von anderen Begriffen (das sogenannte Definiens) gleichgesetzt (vgl. von Savigny 1976, Kap. 4): (Definiendum) AFFFFFFFDFFFFFFFS Beispiel: 8x: x ist ein Junggeselle

:$

Explizitdefinitionen

(Definiens) AFFFFFFFFFFDFFFFFFFFFFS x ist ein bislang unverheirateter Mann

Logische Form: 8x(Dx1…xn $ A), wobei im Definiens A nur die Individuenvariablen x1 … xn frei sind. (Für singuläre Begriffe hat eine Explizitdefinition die Form einer Identität, z. B. a := f(b)). D2) Bedeutungspostulate (,implizite‘ Definitionen): Dabei handelt es sich um semantische Konventionen, die axiomatisch eingeführt werden, also nicht durch Ableitung gewonnen wurden. Jedoch haben diese Bedeutungspostulate nicht die Form einer Explizitdefinition, sondern andere Form, zumeist die Form einer Implikation. Beispiel: 8x (x ist rund ! x ist ohne Ecken) D3) Abgeleitete definitorisch wahre Sätze: Dabei handelt es sich um Sätze, die nicht L-wahr sind, aber aus Sätzen des Typs D1 oder D2 logisch folgen. Beispiel: 8x(x ist nicht männlich ! x ist kein Junggeselle) Echte Definitionen müssen das Kriterium der Eliminierbarkeit der definierten Begriffe erfüllen (s. Suppes 1957, 154). Damit explizite Definitionen dieses Eliminierbarkeitskriterium erfüllen, müssen sie der Forderung der Nichtzirkularität genügen, welche für eine einzelne Definition besagt, dass der Definiendumbegriff im Definiens nicht vorkommen darf. Im Gegensatz zu nichtzirkulären Explizitdefinitionen charakterisieren bloße Bedeutungspostulate einen Begriff (im Regelfall) nur unvollständig und ermöglichen nicht seine Elimination. Eine Menge von aufeinander aufbauenden Definitionen muss sich darüberhinaus in Form einer nichtzirkulären Definitionskette (D1, …, Dn) anordnen lassen: d. h., jeder Begriff eines Definiens einer Definition Di in dieser Kette ist entweder undefiniert, oder er ist das Definiendum eine früheren Definition in der Kette (vgl. von Savigny 1976, 116). Eine weitere Art von Definitionen, die in Logik und Computerwissenschaft eine wichtige Rolle spielen, sind sogenannte rekursive Definitionen, auf die

Nichtzirkularität

Definitionsketten

87

88

3. Das analytische Instrumentarium

wir hier nicht eingehen – wir erwähnen lediglich, dass auch diese trotz ihrer iterativen Struktur die Nichtzirkularitätsforderung erfüllen, denn sie sind beweisbar äquivalent mit (komplizierten) Explizitdefinitionen. Die zentrale Forderung an analytisch wahre Sätze ist ihre empirische Nichtkreativität, die wir wie folgt explizieren: Empirische Nichtkreativität

(Ms. 3.4-1) Zentralforderung an analytisch wahre Sätze: Sie müssen empirisch nichtkreativ sein, d. h., sie dürfen weder empirisch gehaltvoll sein, noch in Bezug auf das gegebene Hintergrundwissen neuen empirischen Gehalt erzeugen. Präzisierung: Eine Menge D von Sätzen kann im Hintergrundwissen W nur dann als analytisch wahr angesehen werden, wenn gilt: (a) der empirische Gehalt von D (d. h. die Menge der empirischen und nicht L-wahren Konsequenzen von D) ist leer, und (b) der empirische Gehalt von W-vereinigt-mit-D ist gleich dem von W. Beide Forderungen (a) und (b) müssen erfüllt sein, damit man sagen kann, die Wahrheit solcher Sätze beruht auf analytischer Konvention (bei Suppes 1957, 154, fehlt Forderung a, weil es dort nur um relative Nichtkreativität geht). Aus (Ms. 3.4-1) ergibt sich insbesondere: kein Begriff darf zweimal in verschiedener Weise definiert werden. Bekanntlich hat man in der Geschichte der Messung verschiedene Maßkonventionen vorgeschlagen. Zwei Beispiele:

Beispiele für empirische Kreativität

(1) 1 Meter = Die Länge des Platin-Iridium-Barrens in Paris. (2) 1 Meter = Die Länge eines Pendels in Seehöhe von 1 sec Schwingungsdauer (1) + (2) implizieren jedoch folgenden empirischen Satz: Die Länge des Platin-Iridium-Barrens in Paris = die Länge eines Pendels in Seehöhe von 1 sec Schwingungsdauer. Man darf nicht zugleich (1) und (2) als Definition der Längeneinheit 1 m ansehen. Ist (1) die geltende Definition, so ist (2) eine empirische Hypothese, und umgekehrt. Viele ,Definitionen‘, die sich in Lehrbüchern finden, wie z. B. „Wasser = H2O“, oder „Volksschule = Institution zur Vermittlung von Grundwissen für 6 – 10-Jährige“, erfüllen die Forderung der empirischen Nichtkreativität nicht. Denn die definierten Begriffe („Wasser“, „Volksschule“) wurden im Hintergrundwissen des Alltagsmenschen bereits in anderer Weise analytisch eingeführt, z. B. durch ostensive Charakterisierungen. Es handelt sich dabei eher um Begriffsexplikationen (im Sinne von Carnap, s. Kap. 2.1) als um Definitionen im engen Sinn. Auch sogenannte ,Definitionen‘ von theoretischen Begriffen wie z. B. „Kraft = Masse mal Beschleunigung“ erzeugen neuen empirischen Gehalt und sind daher keine echten Definitionen (s. Kap. 5.1). Solange ein Begriff in einem Wissenssystem nur durch eine Explizitdefinition charakterisiert wird, ist diese Explizitdefinition (nachweislich) immer empirisch nichtkreativ. Auch ein einzelnes Bedeutungspostulat für einen

3.5 Klassifikation von Sätzen nach ihrem Allgemeinheitsgrad

neuen Begriff Fx, sagen wir 8x(Fx ! A(x)), ist nicht kreativ, denn es ist ja logisch schwächer als eine Explizitdefinition 8x(Fx $ A(x)). Meistens wird aber ein ganzes System von Bedeutungspostulaten für einen (oder mehrere) Begriffe eingeführt; und hierfür gibt es keine Nichtkreativitätsgarantie. Die Bezeichnung von Systemen von Bedeutungspostulaten als ,implizite‘ Definitionen (die auf Hilbert zurückgeht) ist daher missverständlich, weil ,implizite Definitionen‘ sich ganz anders als echte Definitionen verhalten.

3.5 Klassifikation von Sätzen nach ihrem Allgemeinheitsgrad Die Übersicht ist in Abb. 3.5-1 dargestellt. Die Gehaltsstärke nimmt beim Übergang von Allsätzen über Singulärsätze zu Existenzsätzen sukzessive ab: ein Allsatz 8xFx ist logisch stärker als seine singulären Instanzen Fa, und diese sind logisch stärker als der entsprechende Existenzsatz 9xFx.

Strikte (bzw. deterministische) Allsätze

'A' für 'Antecedens'

Z.B.: Für alle x: wenn x A ist, dann ist x K

'K' für 'Konsequens'

(raumzeitlich) unbeschränkt (raumzeitlich) beschränkt Generelle Sätze Nichtstrikte Generalisierungen Statistische Generalisierungen Z.B.: q % aller A's sind K's (raumzeitlich) unbeschränkt

p(K|A) = r (mit: 0drd1; q = 100˜r) (raumzeitlich) beschränkt

Normische und ceteris paribus Generalisierungen Z.B.: A's sind normalerweise K's, bzw.: C.P. sind A's K's Gemischt-quantifizierte Generalisierungen; z.B. Allexistenzsätze, Existenzallsätze Singuläre Sätze Z.B. Dieses a ist ein A, und es ist ein (bzw. kein) K.

Existenzsätze Z.B. Es gibt (irgendwo) ein A, das ein (bzw. kein) K ist. (Gemischte Sätze)

Abb. 3.5-1: Klassifikation von Sätzen nach ihrem Allgemeinheitsgrad

89

90

3. Das analytische Instrumentarium Strikte Generalisierungen

Statistische Generalisierungen

1. (Strikte) Allsätze: Die Allimplikation 8x(Ax ! Kx) ist die einfachste Form eines Allsatzes. Das Implikationsvorderglied (Wenn-Glied) nennt man auch Antecedens(prädikat), das Implikationshinterglied (Dann-Glied) auch Konsequens(prädikat). Allgemein beginnt ein reiner Allsatz mit einem oder mehreren Allquantoren, gefolgt von einer quantorfreien Formel, die sogenannten Matrix des Allsatzes, welche typischerweise die Form einer Implikation, einer Äquivalenz, oder einer numerischen Identität besitzt. Allsätze machen eine Behauptung, die ausnahmslos gilt: die Wahrheit von 8x(Ax ! Kx) impliziert die Wahrheit von Aa ! Ka für jedes Individuum a. Man spricht daher auch von strikten bzw. von deterministischen Generalisierungen, denn gegeben Aa, ist das Eintreffen von Ka unvermeidlich determiniert. 2. Nichtstrikte Generalisierungen: Hierunter fallen in erster Linie statistische Generalisierungen der Form „q % aller As sind Ks“. Man nennt dies auch eine bedingte Wahrscheinlichkeitsaussage und schreibt dafür p(Kx|Ax) = r, mit rN100 = %. Die Wahrscheinlichkeit wird statistisch verstanden, als Häufigkeit oder als Häufigkeitsgrenzwert (s. Kap. 3.9). Statistische Generalisierungen sind zwar ebenfalls genereller Natur, doch handelt es sich bei ihnen um keine Allsätze. Die statistische Generalisierung „q % aller As sind Ks“ sagt nichts über einzelne Individuen vom Typ A aus, sondern sie sagt nur etwas über die Klasse der As aus, nämlich dass sich q % der Individuen der A-Klasse in der K-Klasse befinden. Siehe Abb. 3.5-2. A 8 12

B 4

p(Bx|Ax) = 12/20 = 3/5 p(Ax|Bx) = 12/16 = 3/4 sofern alle Individuen in A oder B liegen, gilt ferner: p(Ax) = 20/24 = 5/6, p(Bx) = 16/24 = 2/3

Abb. 3.5-2: Statistische Generalisierung

Normische Generalisierungen

Aus diesem Grund bestehen zwischen statistischen Generalisierungen und Singulärsätzen, wenn der zugrunde liegende Individuenbereich variabel bzw. potentiell unendlich ist, keine deduktionslogischen, sondern nur induktiv-probabilistische Folgerungsbeziehungen. Eine weitere Art nicht-strikter Generalisierungen sind normische Generalisierungen oder Normalfallhypothesen von der Form „As sind normalerweise Ks“ – z. B., Vögel können normalerweise fliegen (die Bezeichnung „normisch“ geht auf Scriven 1959a zurück). Wir nehmen an, dass ein normisches Gesetz eine numerisch unspezifische statistische Generalisierung der Form „die meisten As sind Bs“ analytisch impliziert, ohne jedoch damit bedeutungsgleich zu sein. Die Überprüfung normischer Gesetze wird damit zu einem Unterfall der Überprüfung statistischer Gesetze. Die Annahme, dass normische Gesetze statistische Generalisierungen implizieren, wird in Schurz (2001c) evolutionstheoretisch wie folgt begründet: normische Gesetze beschreiben das Verhalten selbstregulativer Systeme, die den Gegenstandsbereich nichtphysikalischer Wissenschaften ausmachen, von der Bio-

3.5 Klassifikation von Sätzen nach ihrem Allgemeinheitsgrad

logie aufwärts bis zu den Sozial- und Humanwissenschaften. Fast alle selbstregulativen Systeme, welche unsere Welt bevölkern, sind evolutionäre Systeme. Der für ein erfolgreiches evolutionäres System überlebenswichtige Normalfall muss aber auf lange Sicht auch der statistische Normalfall sein, denn andernfalls wäre das System ausgestorben. Eine dritte Art nicht-strikter Generalisierungen sind ceteris paribus (CP) Generalisierungen. In Schurz (2002b) werden zwei unterschiedliche Bedeutungen von „ceteris paribus“ unterschieden. (i) Komparative CP-Hypothesen besagen, dass die Zunahme des Antecedensmerkmals (oder die Zunahme seiner Häufigkeit) zu einer Zunahme des Konsequensmerkmals (oder zur Zunahme seiner Häufigkeit) führt, sofern alle anderen (relevanten) Faktoren konstant gehalten werden. Komparative CP-Gesetze können durch die Methode des randomisierten Experimentes (s. Kap. 4.3.1) empirisch überprüft werden. (ii) Exklusive CP-Hypothesen besagen dagegen, dass das Antecedensmerkmal Ax zum Konsequenzmerkmal führt (oder dessen Häufigkeit erhöht), sofern andere störende Faktoren abwesend sind. Exklusive CP-Hypothesen, kurz eCP-Hypothesen, sind weiter in definite und indefinite eCPHypothesen zu unterteilen. Bei definiten eCP-Generalisierungen sind die möglichen Typen von Störfaktoren aufgrund einer Hintergrundtheorie bekannt und lassen sich, zumindest in der theoretischen Sprache, in Form einer endlichen Liste beschreiben. Man findet solch theoretisch-definite eCP-Hypothesen in Form von Systembedingungen in physikalischen Theorien (s. Kap. 5.2, 5.6.1). In indefiniten eCP-Hypothesen ist dagegen die Liste möglicher Störfaktoren unbekannt und potentiell unendlich. Die Literatur über CP-Gesetze konzentriert sich fast ausschließlich auf indefinite eCP-Gesetze, und sie werden in der Wissenschaftstheorie kontrovers diskutiert (vgl. z. B. Cartwright 1983, 45; Gadenne 1984, Hempel 1988, Pietroski and Rey 1995; Earman et al. 2002, Hg.). Mehrere Autoren haben argumentiert, dass indefinite eCP-Gesetze gehaltleer und daher empirisch unüberprüfbar sind (s. Woodward 2002, Schurz 2001d). Denn anders als bei komperativen CP-Gesetzen existiert keine experimentelle Prozedur, durch die die Abwesenheit von beliebigen unbekannten Störfaktoren realisiert werden könnte. In Schurz (2002b) wird daher vorgeschlagen, indefinite eCP-Gesetze in den Wissenschaften evolutionärer Systeme durch die oben erläuterten normischen Gesetzeshypothesen zu ersetzen. Statistische, normische und eCP-Generalisierungen haben gemeinsam, dass sie Ausnahmen zulassen – das sind Einzelfälle, welche zwar das Antecedens, aber nicht das Konsequens erfüllen. Ihre argumentativen Anwendungen gehorchen einer nichtmonotone Folgerungsbeziehung im Sinne von Kap. 2.6.4. Strikte Gesetze verbieten solche Ausnahmen, bzw. werden dadurch falsifiziert. Sowohl strikte wie nichtstrikte Generalisierungen können raumzeitlich unbeschränkt oder beschränkt sein. Sie sind beschränkt, wenn das Antecedensprädikat eine Einschränkung auf ein endliches Raumzeitgebiet enthält – z. B. „alle Schwäne in Europa sind weiß“, im Gegensatz zu „alle Schwäne sind weiß“. Unbeschränkte strikte Allsätze heißen auch universelle Allsätze. 3. Gemischt-quantifizierte Generalisierungen enthalten sowohl All- wie Existenzquantoren. Beispiel: Jede Handlung hat ein Motiv – formal:

91

92

3. Das analytische Instrumentarium Gemischtquantifizierte Generalisierungen

Singulärsätze

Existenzsätze

Wichtige logische Folgerungsschemata

8x9y(Handlung(x) ! Motiv(y,x)). Bei gemischten Quantoren kommt es auf die Reihenfolge an: 9x 8 yRxy ist eine logisch stärkere Aussage als 8y9xRxy. Beispiel: Stehe „Rxy“ für „x ist Ursache für y“. Dann besagt 8y9xRxy soviel wie: alles hat irgendeine Ursache. Dagegen besagt 9x8yRxy: es gibt ein x, das Ursache von allem ist; m. a. W., alles hat eine gemeinsame Ursache. Bei gleichen Quantoren ist dagegen die Reihenfolge unwesentlich. 4. Singulärsätze sind alle Sätze, die keine Quantoren enthalten, und auch keinen statistischen Wahrscheinlichkeitsfunktor. Singuläre Sätze sind aus Atomsätzen (Fa, Rab, …) aussagenlogisch zusammengesetzt. Sie sagen immer etwas über eines oder mehrere Einzeldinge aus. 5. Existenzsätze: Reine Existenzsätze beginnen mit einem oder mehreren Existenzquantoren, auf die eine quantorfreie Matrixformel folgt. Im Regelfall hat die Matrix die Form einer Konjunktion 9x(Ax^lKx); z. B. es gibt einen schwarzen Schwan (,l‘ steht für ,unnegiert oder negiert‘). Solche Existenzsätze können ebenfalls unbeschränkt oder beschränkt sein, je nachdem, ob Ax eine raumzeitliche Einschränkung enthält oder nicht. 6. Gemischte Sätze sind schließlich aussagenlogische Kombinationen obiger Satztypen, wie z. B. 8x(Fx ! Gx) _ Ha. Sie spielen eine untergeordnete Rolle. Die wissenschaftstheoretisch wichtigsten logisch-deduktiven Beziehungen zwischen den angeführten Satzarten seien wie folgt zusammengestellt. (Ms. 3.5-1) Notation: “X 41 Y“ steht für „Y folgt logisch aus X“: (1) Strikter Allsatz 41 Singulärer Satz Alle A sind K 41 wenn a A ist, dann ist a K (2) Strikter Allsatz & Singulärsatz 41 Singulärsatz (Erklärungsschema) Alle A sind K, und a ist A 41 a ist K (3) Singulärsatz falsifiziert strikten Allsatz (Falsifikationsschema I) Singulärsatz 41 Negation eines strikten Allsatzes a ist A und nicht K 41 nicht alle A sind K (4) Existenzsatz falsifiziert strikten Allsatz (Falsifikationsschema II) Es gibt ein A, das kein K ist 41 nicht alle A sind K (5) Strikter Allsatz ist logisch äquivalent mit der Negation eines Existenzsatzes Alle A sind K 14 41 Es gibt kein A, das kein K ist (6) Singulärsatz 41 Existenzsatz a ist ein A 41 es gibt ein x, das ein A ist (7) Zwischen nichtstrikten Generalisierungen und singulären Sätzen gibt es keine logisch deduktiven Folgerungsbeziehungen, sondern nur statistische oder epistemisch-induktive Wahrscheinlichkeitsbeziehungen.

3.6 Generelle Sätze, Gesetzesartigkeit, Determinismus und Indeterminismus

Die obigen Charakterisierungen von Satzarten sind syntaktischer Natur. Syntaktische Charakterisierungen sind nicht immer invariant gegenüber L-äquivalenten Umformungen. Beispielsweise ist der Singulärsatz Fa L-äquivalent mit dem Allsatz 8x(x=a !Fx) (alle x, die mit a identisch sind, haben Eigenschaft F). Dieser Satz ist aber kein Allsatz im semantischen Sinn, da er nur etwas über das Individuum a besagt. Damit Charakterisierungen von Sätzen genuine Eigenschaften der damit ausgedrückten Propositionen ausdrücken, müssen sie invariant sein gegenüber L-äquivalenter Satzumformung, denn L-äquivalente Sätze drücken dieselbe Proposition aus (s. Kap. 3.1.2). Eine Standardmethode, um dieses Problem zu lösen, ist die Methode der essentiellen Satztypen (s. z. B. Hempel/Oppenheim 1948, 271). Man betrachtet einen Satz nur dann als essentiellen Allsatz, wenn er nicht L-äquivalent ist mit einem Singulärsatz. Allgemein wird definiert:

Invarianz gegenüber L-Äquivalenz und essentielle Satztypen

(Def. 3.5-1) Essentielle Satztypen (nach dem Allgemeinheitsgrad): (1.) Ein Satz S ist ein essentieller Allsatz g.d.w. S ein syntaktischer Allsatz ist und nicht L-äquivalent ist mit einem syntaktischen Singulärsatz. Analog für: Existenzsatz, gemischt-quantifizierter Satz, statistische (bzw. normische oder c.p.) Generalisierung (anstelle von Allsatz). (2.) Ein Satz S ist dagegen ein essentieller Singulärsatz g.d.w. S mit einem syntaktischen Singulärsatz L-äquivalent ist. Die Einteilung von Satzarten nach ihrem logischen Allgemeinheitsgrad ist weitgehend unabhängig von der in Kap. 3.2 aufgestellten Klassifikation von Satzarten nach ihrem Inhaltstyp: bei den einzelnen Satzarten kann es sich sowohl um empirische wie theoretische Sätze handeln, je nachdem, welche nichtlogischen Begriffe in diesen Sätzen vorkommen. Unbeschränkte Alloder Existenzsätze oder solche mit unüberschaubar großer raumzeitlicher Beschränkung können allerdings niemals Beobachtungssätze sein. Eine weitere Unterscheidung, die quer zu unseren bisherigen Unterscheidungen verläuft, ist die zwischen Evidenzen und Hypothesen. Sie betrifft den epistemischen Status unseres Wissens. Bei Evidenzen handelt es sich um sehr gut gesicherte Sätze – prototypisch dafür sind aktuale (= tatsächlich beobachtete) Beobachtungssätze. Hypothesen sind weniger gut gesicherte Sätze, die weiterer Überprüfung bedürfen – prototypisch dafür sind generelle Sätze oder potentielle Beobachtungssätze (singuläre Prognosen).

3.6 Generelle Sätze, Gesetzesartigkeit, Determinismus und Indeterminismus Strikte raumzeitlich unbeschränkte essentielle Allsätze sind prima facie Kandidaten für echte Naturgesetze, d. h.: wären sie wahr, so würden sie echte Naturgesetze ausdrücken. In der Wissenschaftstheorie nennt man solche Allsätze auch gesetzesartig. Hier zwei Beispiele:

Evidenzen und Hypothesen

93

94

3. Das analytische Instrumentarium

(3.6-1) Alle Körper ziehen sich gegenseitig an. Genauer: Für alle x und y: wenn x und y Körper sind, dann zieht x y an und y zieht x an. (3.6-2) Alle Lebewesen müssen einmal sterben. Das Problem der Gesetzesartigkeit

Allerdings hat sich herausgestellt, dass nicht jeder essentielle Allsatz auch gesetzesartig ist. Dies zeigt sich am schnellsten, wenn man essentielle, aber raumzeitlich beschränkte Allsätze betrachtet. Beispiele für raumzeitlich beschränkte Allsätze, mit zunehmender Beschränkung: (3.6-3) Alle Körper nahe der Erdoberfläche fallen mit Erdbeschleunigung nach unten (Galileis Fallgesetz). (3.6-4) Alle Säugetierarten in Polargebieten haben verglichen zu ihren Artgenossen in warmen Ländern eine rundlichere Form (Bergmanns Gesetz, s. Giere 1988, 42). (3.6-5) Bis ungefähr 10 000 v. C. lebten alle Menschen vom Jagen und Sammeln. (3.6-6) Im Mittelalter beruhte alle Landwirtschaft auf dem Lehensprinzip. (3.6-7) Alle Äpfel in diesem Korb sind rot. Die Frage, ob hier von einem wissenschaftlichen Gesetz gesprochen werden kann, hängt prima facie von der Größe des raumzeitlichen Anwendungsbereiches ab. Dies wird an der Hierarchie der Beispiele ersichtlich: (3.6-3) und (3.6-4) werden in der Physik bzw. Biologie als wissenschaftliches Gesetz bezeichnet; bei den historischen Allsätzen (3.6-5) und (3.6-6) ist der Gesetzesstatus eher zweifelhaft – viele Historiker würden hier eher von kontingenten Fakten sprechen; und im Fall (3.6-7) liegt offensichtlich kein gesetzesartiger Allsatz, sondern gemäß Kap. 3.1.3 ein lokalisierter Allsatz, d. h. ein Beobachtungssatz vor, obwohl es sich auch bei (3.6-7) logisch gesehen um einen essentiellen Allsatz handelt. Allsätze vom Typ (3.6-7) werden in der Wissenschaftstheorie auch akzidentelle Allsätze genannt (Stegmüller 1969, 275; Hempel 1965, 266). Der Begriff der Gesetzesartigkeit von Allsätzen scheint in dieser Sicht etwas Graduelles zu sein. In der Wissenschaftstheorie wurden eine Reihe von Vorschlägen gemacht, diesen Begriff logisch zu präzisieren, wobei man auf viele unvermutete Schwierigkeiten stieß. So schlugen Carnap (1947) und Hempel (1965, 266) vor, das Problem der Gradualität der Gesetzesartigkeit von raumzeitlich beschränkten Allsätzen zu vermeiden, indem man zwischen fundamentalen und abgeleiteten Allsätzen unterscheidet. Ein Allsatz ist abgeleitet, wenn er im gegebenen Hintergrundwissen aus anderen Allsätzen zusammen mit singulären Anfangs- oder Randbedingungen ableitbar ist; andernfalls ist er fundamental. Carnap und Hempel schlugen vor, nur von fundamentalen Gesetzen zu fordern, dass sie keine Individuenkonstanten oder raumzeitliche Beschränkungen enthalten dürfen; abgeleitete Gesetze dürfen solche enthalten. Dies scheint gut zu obigen Beispielen zu passen: so ist das Galileische Fallgesetz (3.6-3) aus der Newtonischen Physik und Randbedingungen über die Masse der Erde ableitbar, und das Bergmannsche Gesetz (3.6-4) ist aus der allgemeinen Evolutionstheorie sowie

3.6 Generelle Sätze, Gesetzesartigkeit, Determinismus und Indeterminismus

aus der Tatsache folgerbar, dass rundlichere Körperformen wegen ihrer geringeren Oberfläche die Körperwärme besser speichern. Doch Nagel (1961, 58) hat den Carnap/Hempelschen Vorschlag widerlegt. Nagel weist darauf hin, dass auch klare Beispiele akzidenteller Allsätze wie z. B. „Alle Schrauben an Smiths Wagen sind rostig“ abgeleitete Gesetze sein können: dieser akzidentelle Allsatz ist beispielsweise ableitbar aus der Randbedingung, dass Herr Smith seinen Wagen immer im Freien parkt und in Schottland wohnt, wo es häufig regnet, zusammen mit dem Naturgesetz, dass Eisen bei häufigem Kontakt mit Wasser rostet. Nagel schloss aus seinem Befund, dass überhaupt nur fundamentale Allsätze gesetzesartig sein können. Doch diese Beschränkung wäre zu stark und würde nicht zum wissenschaftlichen Gesetzesverständnis passen. Hempel (1977, 291 ff.) hat daher seine ursprüngliche Charakterisierung von Gesetzesartigkeit für abgeleitete Gesetze aufgegeben. Es bleibt bei unser Eingangsdiagnose: die Gesetzesartigkeit beschränkter Allsätze ist eine graduelle Angelegenheit. Aber selbst raumzeitliche universelle Allsätze sind nicht immer gesetzesartig, wie folgendes auf Reichenbach zurückgehende Beispiel zeigt (van Fraassen 1989, 27):

Nagels Problem

(3.6-8) Kein Klumpen aus radioaktivem Uran hat einen Durchmesser von mehr als einem Kilometer. (3.6-9) Kein Klumpen aus Gold hat einen Durchmesser von mehr als einem Kilometer. Bei (3.6-8) handelt es sich um ein Naturgesetz, denn jeder Klumpen aus radioaktiv angereichertem Uran von dieser Größe würde aufgrund einer atomaren Kettenreaktion explodieren. Vermutlich ist aber auch (3.6-9) wahr – aber auch wenn (3.6-9) wahr wäre, wäre es kein Naturgesetz, denn ein Klumpen Gold von dieser Größe ist physikalisch möglich. Weiterführende Lösungsvorschläge zu den diffizilen Problemen der Gesetzesartigkeit finden sich in Kap. 6.5.1. Die Frage der Gesetzesartigkeit stellt sich ebenso für statistische Generalisierungen. Zunächst wieder einige Beispiele: (3.6-10) 50 % aller Cäsium137-Atome (einer beliebigen Substanzmenge) sind nach 30 Jahren zerfallen. (3.6-11) 80 % aller Lungenkrebskranken waren schwere Raucher. (3.6-12) 70 % aller Bettnässer-Kinder haben Eltern mit gestörter Beziehung. (3.6-13) 80 % aller Afrikaner sind dunkelhäutig. (3.6-14) 60 % aller Äpfel in diesem Korb sind rot. (3.6-10, 11, 12) sind raumzeitlich unbeschränkte statistische Generalisierungen, (3.6-13, 14) sind raumzeitlich beschränkt. Beim Fall der raumzeitlich beschränkten Generalisierungen treffen wir dieselbe Gradualität der Gesetzesartigkeit an wie bei strikten Allsätzen. (3.6-14) ist das statistische Gegenstück zu (3.6-7) vorhin – eine logisch essentielle, aber eindeutig akzidentelle statistische Generalisierung. Bei der Beurteilung des Gesetzescharakters der unbeschränkten statistischen Generalisierungen (3.6-10, 11, 12) stellt sich zusätzlich die Frage von

Gesetzesartigkeit statistischer Generalisierungen

95

96

3. Das analytische Instrumentarium Determinismus und Indeterminismus

Epistemischer vs. objektiver Indeterminismus

Determinismus versus Indeterminismus. Dem deterministischen Weltbild zufolge gibt es in der Welt keinen objektiven Zufall: der gesamte Weltverlauf ist durch seine Anfangsbedingungen und durch Naturgesetze vollständig determiniert und voraussagbar. Würde man ein deterministisches physikalisches Weltbild vertreten, so wie dieses im 19. Jahrhundert von Laplace ausformuliert wurde, so könnte man (3.6-10) nicht als fundamentales Naturgesetz ansehen, sondern bestenfalls als abgeleitetes Gesetz. Denn bei vollständigem Wissen müsste dann für jedes Cs137-Atom exakt voraussagbar sein, wann es zerfällt. Die moderne Mikrophysik lehrt jedoch einen objektiven Indeterminismus – ihrem Verständnis nach gibt es objektive Zufallsprozesse. Bei (3.6-10) handelt es sich um ein objektiv indeterministisches Fundamentalgesetz: ob und wann ein Cs137-Atom zerfallen wird, ist durch keinerlei noch so vollständiges physikalisches Wissen voraussagbar. Aber die Wahrscheinlichkeit, dass ein Cs137-Atom in 30 Jahren zerfällt, beträgt genau 1/2; und daher werden nach 30 Jahren mit praktischer Sicherheit 50 % einer Cs137-Probe zerfallen sein. Wir unterscheiden daher zwischen zwei Arten von Indeterminismus: objektiver Indeterminismus, demzufolge die statistische Natur von Generalisierungen in objektiven Eigenschaften der Natur begründet ist, versus epistemischer Indeterminismus, demzufolge die statistische Natur von Generalisierungen auf die Unvollständigkeit unseres Wissens zurückzuführen ist. Während (3.6-10) ein objektiv-indeterministisches Gesetz ist, sind die Generalisierungen (3.6-11, 12) epistemisch-indeterministisch. Das heißt im Fall von (3.6-11): hätten wir genaues Wissen über eine Person, so könnten wir das Auftreten oder Nichtauftreten von Lungenkrebs mit viel höherer Wahrscheinlichkeit, nahe bei 1, voraussagen. Selbstverständlich handelt es sich auch bei den raumzeitlich beschränkten Generalisierungen (3.6-13, 14) um Fälle von epistemischem Indeterminismus. Zwischen (3.6-11) und (3.6-12) gibt es allerdings einen Unterschied: (3.6-11) ist eine klar gesetzesartige statistische Generalisierung, da unter (so gut wie) allen Umständen starkes Rauchen die Lungenkrebswahrscheinlichkeit erhöht. Bei (3.6-12) ist die Gesetzesartigkeit fraglicher: es gibt viele Umstände, unter denen gestörte Elternbeziehungen nicht zu Bettnässen bei Kindern führt. (3.6-12) enthält insofern ein höheres Maß an Kontingenz. Bei obigen Beispielen handelte es sich um numerisch-statistische Generalisierungen. Es gibt auch qualitative und komparative statistische Generalisierungen: (3.6-15) Qualitative statistische Generalisierungen – Beispiel: Die meisten Bettnässer-Kinder haben Eltern mit gestörter Beziehung p(GestörtEltern(x) | Bettnässer(x)) = hoch. (3.6-16) Komparative statistische Generalisierungen – Beispiel: Bettnässer-Kinder haben eher Eltern mit gestörter Beziehung. p(GestörtEltern(x) | Bettnässer(x)) > p(GestörtEltern(x)| nicht-Bettnässer(x)) Ersichtlicherweise ist eine komparative statistische Generalisierung sehr schwach, da sie nicht über die Höhe des statistischen Einflusses informiert, sondern nur darüber, dass irgendein wenn auch sehr geringer probabilistischer Einfluss besteht.

3.7 Gehalt von Sätzen und Gehaltsarten

3.7 Gehalt von Sätzen und Gehaltsarten (Def. 3.7-1) Der logische Gehalt eines Satzes bzw. einer Satzmenge S ist die Menge aller aus S logisch folgenden Sätze bzw. Konsequenzen. Man schreibt dafür auch C(S) (,C‘ für ,content‘). – Der empirische Gehalt eines Satzes bzw. einer Satzmenge S besteht dagegen nur aus der Menge aller aus S logisch folgenden empirischen (und somit überprüfbaren) Sätze bzw. Konsequenzen, die nicht schon analytisch wahr sind. Wir schreiben dafür E(S). Noch enger als der empirische Gehalt ist der Beobachtungsgehalt, den man als die Menge B(S) der aus einer Satzmenge S folgenden Beobachtungssätze definiert. Der Begriff des Beobachtungsgehaltes ist allerdings nur eingeschränkt brauchbar – nämlich nur für rein universelle Gesetzeshypothesen. Aus einer Allexistenzhypothese wie z. B. H:= 8x(Fx! 9 yGxy), selbst wenn sie empirisch ist, folgen nämlich keine Beobachtungssätze. Aus H folgt durch universelle Instanziierung nur Fa! 9yGay, aber dies ist kein Beobachtungssatz, denn 9 yGay ist ein raumzeitlich unbeschränkter Existenzsatz. Aus demselben Grund sind empirische Allexistenzhypothesen durch Beobachtungssätze nicht nur nicht verifizierbar, sondern auch nicht falsifizierbar. Sie sind jedoch durch Beobachtungssätze bestätigbar und schwächbar (s. Kap. 3.8). Wissenschaftstheoretisch bedeutsam ist der Begriff des relevanten logischen bzw. empirischen Gehaltes einer Satzmenge S, der in Kap. 3.10.3 erklärt wird. Ein wissenschaftstheoretisch vernachlässigter Gehaltsbegriff ist schließlich der des probabilistischen Gehaltes. Angenommen S ist eine Menge von statistischen Hypothesen. S impliziert logisch keinerlei Beobachtungssätze B, sondern verleiht ihnen bestenfalls eine gewisse Glaubenswahrscheinlichkeit. Der Vorschlag, den probabilistischen Gehalt von S als die Menge aller Sätze zu bestimmen, deren bedingte Wahrscheinlichkeit gegeben S hinreichend hoch ist, also größer ist als ein bestimmter Wert r, funktioniert aus folgendem Grund nicht: die Menge all dieser Sätze kann inkonsistent sein, auch wenn r noch so nahe bei 1 liegt. Denn die Konjunktion von sehr vielen hochwahrscheinlichen Sätzen B1, …, Bn (gegeben S) kann einen sehr niedrigen Wahrscheinlichkeitswert besitzen. Das heißt aber, dass die Negation dieser Konjunktion :(B1^…^ Bn) ihrerseits hochwahrscheinlich ist (gegeben S). In diesem Fall wäre die Menge aller hochwahrscheinlichen Konsequenzen von S inkonsistent. Anschauliche Beispiele dafür sind das Lotterie-Paradox (Kutschera 1972, 239 f.) und das Preface-Paradox (Makinson 1965). Wir schlagen daher vor, den probabilistischen Gehalt einer statistischen Hypothese S anders zu bestimmen: nämlich als die Menge aller Wahrscheinlichkeitssätze, die aus S gemäß den Axiomen der Wahrscheinlichkeit folgen.

Logischer und empirischer Gehalt

Probabilistischer Gehalt

97

98

3. Das analytische Instrumentarium

3.8 Verifikation, Falsifikation, Bestätigung und Schwächung (Def. 3.8-1) (1.) Eine Hypothese H ist verifizierbar g.d.w. es eine endliche und konsistente Menge B von Beobachtungssätzen gibt, aus der H logisch folgt. (2.) H ist falsifizierbar g.d.w. es eine endliche und konsistente Menge B von Beobachtungssätzen gibt, aus der die Negation von H logisch folgt. (3.) H ist bestätigbar respektive schwächbar g.d.w. es eine endliche und konsistente Menge B von Beobachtungssätzen gibt, die die Wahrscheinlichkeit bzw. Plausibilität von H erhöht respektive erniedrigt.

Poppers Asymmetriethese

Es sei daran erinnert, dass Veri- bzw. Falsifizierbarkeit nur mögliche Veribzw. Falsifikation meint, und daher von aktualer Verifikation bzw. Falsifikation scharf zu unterscheiden ist. Beispielsweise ist die Hypothese „morgen wird es regnen“ heute verifizierbar und falsifizierbar, aber erst morgen entweder aktual verifiziert oder falsifiziert. Dassselbe gilt für Bestätigbarkeit und Schwächbarkeit. Ist eine Hypothese verifiziert, so kann man sich ihrer Wahrheit sicher sein, gegeben dass man sich der Wahrheit der Beobachtungssätze sicher ist. Analog für Falsifikation. Wie Abb. 3.8-1 unten zeigt, sind nur wenige Hypothesen falsifizierbar, und noch weniger sind verifizierbar. Dagegen sind alle Hypothesen, sofern sie nur empirischen Gehalt besitzen, zumindest prinzipiell bestätigbar und schwächbar. Für jede Hypothese H gilt darüber hinaus: (a) H ist verifizierbar g.d.w. : H falsifizierbar ist; und (b) H ist bestätigbar g.d.w. : H schwächbar ist. Wie Abb. 3.8-1 zeigt, besitzt Poppers Asymmetriethese, wonach wissenschaftliche Gesetzeshypothesen nicht verifizierbar, aber falsifizierbar sind, nur sehr beschränkte Gültigkeit: sie ist eingeschränkt auf strikte raumzeitlich unbeschränkte empirische Allhypothesen. Dual dazu sind raumzeitlich unbeschränkte Existenzsätze wie „es gibt einen weißen Raben“ verifizierbar,

Empirische Gesetzeshypothesen Raumzeitl.

Raumzeitl. unbeschränkt

beschränkt

Strikt All

Überprüfbarkeit von Hypothesenarten

Theorien

Statistisch

Allexistenz

emp.

emp.

gehalt-

gehalt-

(normisch, c.p.) voll

los

Verifizierbar

+











Falsifizierbar

+

+









Bestätigbar

+

+

+

+

+



Schwächbar

+

+

+

+

+



Popper-Asymmetrie

Abb. 3.8-1: Überprüfbarkeit von Generalisierungsarten

3.9 Objektive und subjektive Wahrscheinlichkeit

aber nicht falsifizierbar. Raumzeitlich beschränkte Hypothesen sind ,im Prinzip‘ verifizierbar und falsifizierbar, indem die endlich vielen empirischen Individuen des betreffenden Raumzeitgebietes beobachtet werden. Keine unbeschränkte Generalisierung und kein theoretischer Satz ist verifizierbar. Allexistenzsätze und statistische Generalisierungen sind auch nicht falsifizierbar, weil sie keine Beobachtungssätze implizieren. Weder Theorien noch einzelne theoretische Hypothesen sind falsifizierbar – selbst dann, wenn sie strikt generell sind: dies hat mit dem Holismus der Theorienüberprüfung zu tun, der in Kap. 5.2+5.4 erläutert wird.

3.9 Objektive (statistische) und subjektive (epistemische) Wahrscheinlichkeit Der intuitive Begriff der Wahrscheinlichkeit involviert zugleich etwas Objektives („wahr-“) und etwas Subjektives („-scheinlich“). Die unterschiedliche Natur dieser beiden Wahrscheinlichkeitsbegriffe wurde erst im 20. Jahrhundert herausgearbeitet. Die objektiv-statistische Wahrscheinlichkeit drückt eine subjektunabhängige Eigenschaft der Realität aus. Die subjektive Wahrscheinlichkeit drückt dagegen einen Glaubensgrad eines gegebenen epistemischen Subjekts aus. Man definiert: (Def. 3.9-1) Statistische (objektive) Wahrscheinlichkeit: Die Wahrscheinlichkeit eines Ereignistyps (z. B. Fx) ist die relative Häufigkeit seines Eintretens, bzw. der Grenzwert seiner relativen Häufigkeit auf lange Sicht. Formale Notation: p( – ); z. B. p(Fx).

Objektive vs. subjektive Wahrscheinlichkeit

Subjektive (epistemische) Wahrscheinlichkeit: Die Wahrscheinlichkeit eines (bestimmten) Ereignisses bzw. Sachverhaltes (z. B. Fa) ist der rationale Glaubensgrad, in dem ein gegebenes Subjekt, oder alle Subjekte eines bestimmten Rationalitätstyps, an das Eintreten des Ereignisses glauben. Formale Notation: w( – ); z. B. w(Fa). Die relative Häufigkeit h(Fx) eines Ereignistyps Fx in einem endlichen Individuenbereich D ist die Anzahl aller F’s in D gebrochen durch die Anzahl aller D’s. Falls D dagegen unendlich ist, ist die relative Häufigkeit undefiniert. Stattdessen bezieht man sich auf eine zufällige Anordnung der Individuen in D in Form einer sogenannten Zufallsfolge (d1, d2, …), und bestimmt p(Fx) als den Grenzwert der relativen Häufigkeiten hn(Fx) von F’s in den n-gliedrigen Anfangsabschnitten dieser Zufallsfolge, für n gegen unendlich. Man schreibt: p(Fx) = limn!v hn(Fx) (s. Kap. 3.10.4.2). Hinweis zu eins- und nullwahrscheinlichen Ereignissen: Bei endlichem Individuenbereich D sind 8xFx und p(Fx)=1 gleichbedeutend. Für unendliche Individuenbereiche gilt dies nicht mehr. Gegeben eine unendliche Zufallsfolge (d1, d2, …), und ein Ereignistyp Fx, dann impliziert p(Fx)=0 keineswegs, dass es in dieser Folge kein Individuum di gibt, welches das Merkmal F hat, sondern lediglich, dass die Häufigkeiten hn(Fx) gegen Null konvergie-

Eins-Wahrscheinlichkeit und strikte Allgemeinheit

99

100

3. Das analytische Instrumentarium

ren. Beispiel: Unter den natürlichen Zahlen gibt es unendlich viele ganze 2er-Potenzen, nämlich alle Zahlen der Form 2k (für k2IN), und dennoch gilt limk!v p(Fx) = k/2k = 0, d. h. die statistische Wahrscheinlichkeit dafür, dass eine natürliche Zahl keine 2er Potenz ist, ist 1. Daher ist die statistische Hypothese p(Kx|Ax)=1 bei unendlichem Individuenbereich schwächer als die Allimplikation 8x(Ax ! Kx), denn sie lässt Ausnahmen mit gegen Null konvergierender Häufigkeit zu. Die Wahrscheinlichkeit von A unter der Annahme, dass B vorliegt, nennt man die bedingte Wahrscheinlichkeit von A gegeben B. Man schreibt dafür p(A|B) bzw. w(A|B) und definiert diesen Ausdruck gewöhnlich wie folgt: (Def. 3.9-2) Bedingte Wahrscheinlichkeit: p(A|B) : = pðA^BÞ pðBÞ , sofern p(B) > 0 Bedingte Wahrscheinlichkeit

In p(A|B) bzw. w(A|B) heißt B das bedingende Ereignis (oder Antecedens) und A das bedingte Ereignis (oder Konsequens). Im statistischen Fall koinzidiert p(A|B) mit der relativen Häufigkeit von A in der endlichen Menge aller B’s (vgl. Abb. 3.5-2), bzw. mit dem Häufigkeitsgrenzwert von A in einer unendlichen Zufallsfolge von B’s. Im subjektiven Fall ist w(A|B) der hypothetische Glaubensgrad von A unter der Annahme, dass B sicher ist. Bedeutend ist die Nichtmonotonie bedingter Wahrscheinlichkeiten: aus p(A|B) = hoch folgt keineswegs p(A|B ^ C) = hoch; vielmehr kann zugleich p(A|B ^ C) = 0 gelten. Abb. 3.9-1 zeigt ein solches Beispiel.

A

B

BšC C

Abb. 3.9-1: p(A|B) = hoch, p(A|B^C) = 0.

Statistische Wahrscheinlichkeiten beziehen sich immer auf einen wiederholbaren Ereignistyp bzw. Sachverhaltstyp, ausgedrückt in einem Prädikat bzw. einer offenen Formel. Die subjektive Wahrscheinlichkeit bezieht sich dagegen immer auf ein bestimmtes Ereignis oder einen bestimmten Sachverhalt, ausgedrückt in einem Satz, als Gegenstand des Glaubens. Ein Beispiel: Wenn z. B. gesagt wird, die Wahrscheinlichkeit dafür, dass es morgen regnet, betrage 3/4, so kann dies prima facie keine Häufigkeitsaussage, sondern nur eine subjektive Wahrscheinlichkeitsaussage sein. Denn den morgigen Tag gibt es nur einmal – entweder es regnet morgen oder es regnet morgen nicht. Dennoch gibt es zwischen beiden Wahrscheinlichkeitsbegriffen Zusammenhänge. Das bekannteste Prinzip, um statistische Wahrscheinlichkeiten auf subjektive Einzelfallwahrscheinlichkeiten zu übertragen, ist das auf Reichenbach (1949, § 72) zurückgehende:

3.9 Objektive und subjektive Wahrscheinlichkeit

(Def. 3.9-3) Prinzip der engsten Referenzklasse: Die subjektive Wahrscheinlichkeit w(Fa) eines Einzelereignisses Fa wird bestimmt als die (geschätzte) bedingte statistische Wahrscheinlichkeit p(Fx|Rx) des entsprechenden Ereignistyps Fx in der engsten (nomologischen) Bezugsklasse bzw. Referenzklasse Rx, von der uns bekannt ist, dass a in ihr liegt (dass also Ra gilt). Das Prinzip der engsten Referenzklasse findet sowohl im Alltag wie in den Wissenschaften durchgängige Verwendung. Der induktiv-statistische Spezialisierungsschluss von Kap. 2.6.1 beruht auf diesem Prinzip. Wollen wir z. B. die subjektive Wahrscheinlichkeit dafür bestimmen, dass eine bestimmte Person eine bestimmte Berufslaufbahn einschlägt (Fa), so stützen wir uns auf die uns bekannten Eigenschaften dieser Person als engste Referenzklasse (Ra), und auf die Wahrscheinlichkeit, dass eine Person x mit den Eigenschaften Rx diese Berufslaufbahn einschlägt (p(Fx|Rx)). Die obige Wetterprognose „die Wahrscheinlichkeit dafür, dass es morgen regnet, betrage 3/4“ hat gemäß Reichenbachs Prinzip die folgende statistische Deutung: die statistische Wahrscheinlichkeit dafür, dass es an einem Tag regnet, dem die gleiche Wetterentwicklung vorausgeht wie dem heutigen Tag, beträgt 3/4. Dies meinen Meteorologen, wenn sie probabilistische Wetterprognosen anstellen: die vorausgehende Wetterentwicklung der letzten Tage fungiert hier also als engste Referenzklasse. Carnap (1950b, 211) hatte ein analoges Prinzip für die Bestätigung von Hypothesen erhoben und nannte es das Prinzip des Gesamtdatums (total evidence): um die subjektive Wahrscheinlichkeit einer Hypothese H rational zu bestimmen, muss man ihre Wahrscheinlichkeit auf die gesamte empirische Evidenz hin konditionalisieren, die derzeit bekannt ist. Zu den Hauptbegründern der statistischen Wahrscheinlichkeitstheorie zählen u. a. von Mises (1964), Reichenbach (1949), und Fisher (1956) (Einführungsliteratur in Statistik: z. B. Bortz 1985). Hauptbegründer der subjektiven Theorie sind u. a. Bayes, Ramsey (1926) und de Finetti (1970) (Einführungsliteratur: z. B. Earman 1992, Howson/Urbach 1996). Carnap (1950b, Carnap/Jeffrey 1971) begründete die ,logische‘ Wahrscheinlichkeitstheorie als Spielart der subjektiven Wahrscheinlichkeitstheorie, deren Axiome jedoch starker Kritik ausgesetzt waren, weil sie über das ,Logisch-Analytische‘ weit hinausgehen (s. Kutschera 1972, 144). Vergleichende Überblicke über verschiedene Wahrscheinlichkeitstheorien findet man selten; z. B. bei Stegmüller (1973b,c), Kutschera (1972, Kap. 2), und Gillies (2000). Die gegenwärtige Wahrscheinlichkeitstheorie ist durch folgende Lagertrennung gekennzeichnet: während in den empirischen Wissenschaften fast ausschließlich von statistischer Wahrscheinlichkeit die Rede ist, versteht die breite Gruppe wissenschaftstheoretischer Bayesianer Wahrscheinlichkeit grundsätzlich im subjektiven Sinn. Von den mathematischen Wahrscheinlichkeitstheoretikern (z. B. Bauer 1978) wird dieser Interpretationskonflikt dagegen systematisch ignoriert. Denn der objektiv-statistische und der subjektiv-epistemische Wahrscheinlichkeitsbegriff gehorchen denselben ma-

Engste Referenzklasse

Gegenwärtige Situation der Wahrscheinlichkeitstheorie

101

102

3. Das analytische Instrumentarium

thematischen Grundgesetzen, die erstmals von Kolmogorov (1933) axiomatisiert wurden. Im folgenden stehen A, B, … für offene Formeln, wenn Wahrscheinlichkeit im statistischen Sinn aufgefasst wird, und für Sätze, wenn sie im subjektiven Sinn aufgefasst wird. Dass A und B disjunkt sind, bedeutet in der statistischen Lesart, dass die Extension von A ^ B faktisch leer ist; und in der subjektiven Lesart, dass A ^ B von keiner zugelassenen (extensionalen) Interpretation der Sprache wahr gemacht wird. Axiome der Wahrscheinlichkeit

(Ms. 3.9-1) Grundaxiome der Wahrscheinlichkeit – statt „p“ kann auch „w“ stehen: (A1): Für alle A: p(A) F 0 (Nicht-Negativität) (Normierung auf 1) (A2:) p(A _ :A) = 1 (A3:) Für disjunkte A, B: p(A _ B) = p(A) + p(B) (endliche Additivität). In Worten: Wahrscheinlichkeiten sind immer größer-gleich null (A1); die Wahrscheinlichkeit des gesamten Möglichkeitsraumes beträgt 1 (A2), und für disjunkte Ereignis(typen) addieren sich die Wahrscheinlichkeiten (A3).

Probabilistische Unabhängigkeit

Ein zentraler Begriff ist schließlich die probabilistische (Un)Abhängigkeit: Zwei Ereignisse A, B heißen probabilistisch unabhängig voneinander g.d.w. p(A ^ B) = p(A)Np(B). Falls p(A), p(B) > 0, so ist dies äquivalent mit p(A|B) = p(A) sowie mit p(B|A) = p(B). Die zwei Ereignisse sind also probabilistisch abhängig, wenn p(A|B) 6¼ p(A) gilt, d. h. wenn das Vorliegen von B die Wahrscheinlichkeit von A verändert.

3.10 Weiterführende Themen 3.10.1 Dispositionsbegriffe Eine permanente Disposition D eines Gegenstandes x besteht in dem zeitlich stabilen Vermögen von x, in gewissen Umständen bzw. Testsituationen (T) in charakteristischer Weise zu reagieren (R). Im Fall einer empirischen Disposition sind T und R empirische Prädikate. Carnap (1936/37, 440) hatte zum ersten Mal erkannt, dass es unmöglich ist, Dispositionsbegriffe durch extensionale Definitionen (also ohne Zuhilfenahme von modalen bzw. intensionalen Begriffen) vollständig auf Beobachtungsbegriffe zurückführen. Denn angenommen, man würde Dispositionsbegriffe wie folgt definieren: (3.10-1) Für alle x: x hat Disposition D g.d.w. (wannimmer x sich in Testumstand T befindet, reagiert x mit Reaktion R) Formal: 8x: Dx :$ 8 t (Txt ! Rxt) Beispiel: D = Wasserlöslichkeit, T = ins Wasser gegeben zu werden, R = sich darin aufzulösen Dann wäre aufgrund der Wahrheitsbedingung der materialen Implikation (!) für jeden Gegenstand a, der zu keinem Zeitpunkt der Testbedingung T

3.10 Weiterführende Themen

ausgesetzt wurde, die generelle Implikation 8t(Tat ! Rat) wahr, und somit hätte dieser Gegenstand a die Disposition D. Das Resultat ist intuitiv inakzeptabel: niemand würde z. B. mein gestern verbranntes Streichholz als wasserlöslich bezeichnen, bloß weil es zeit seiner Existenz nie ins Wasser gegeben wurde. Man nennt dieses Problem auch Carnaps Paradox (Mumford 1998, 46). Carnap (1936/37, 400) gelangte zum Schluss, dass Dispositionsbegriffe extensional nur partiell bzw. konditional definierbar seien, und zwar in Form eines sogenannten bilateralen Reduktionssatzes (s. auch Stegmüller 1970, 227 ff.):

Carnaps Paradox

(3.10-2) (BR): Für alle x, t: wenn x sich zur Zeit t im Testumstand T befindet, gilt: x besitzt Disposition D g.d.w. x Reaktion R zeitigt. Formal: 8x,t: Txt ! (Dx $ Rxt) In einem bilateralen Reduktionssatz (BR) wird das Definiendum Dx nur unter der Bedingung Txt mit einem beobachtbaren Definiens Rxt äquivalent gesetzt. Für : Txt-Kontexte lässt es der (BR) offen, ob Dx vorliegt oder nicht. Ein (BR) impliziert darüberhinaus die Permanenzeigenschaft 9 t(Txt ^ Rxt)! 8 t(Txt ! Rxt) und ist daher empirisch kreativ (s. Essler et al. 2000, 63 ff.). Eine Möglichkeit, um dennoch zu einer vollen empirischen Definition zu gelangen, ergibt sich, wenn man den intensionalen Begriff der gesetzesartigen Implikation zu Hilfe nimmt, dargestellt durch ! g. Gesetzesartige Allimplikationen sind gemäß Kap. 3.6 solche, die nicht zufällig zustande gekommen sind, z. B. nur aufgrund der Tatsache, dass das Vorderglied „Txt“ der Allimplikation 8 t(Txt ! Rxt) niemals realisiert wurde. Gesetzesartige Implikationen sind äquivalent mit kontrafaktischen Konditionalaussagen (s. Kap. 6.5.1.2) – wenn man den wasserlöslichen Gegenstand ins Wasser geben würde, dann würde er sich auflösen. Daher haben viele Autoren für folgende intensionale Definition von Dispositionsbegriffen optiert (z. B. Goodman 1955/75, 52 ff.; Pap 1978, 44; Mumford 1998, 66, spricht vom „accepted view“):

Bilateraler Reduktionssatz

Kontrafaktische Natur von Dispositionsbegriffen

(Def. 3.10-1) x besitzt Disposition D g.d.w.: wannimmer Txt eintritt (bzw. eintreten würde), dann ist (bzw. wäre) Rxt die gesetzesartigte Folge. Formal: 8x: Dx : $ 8 t(Txt ! g Rxt). Es gibt auch probabilistische oder normische Dispositionen, in denen die gesetzesmäßige Implikationsbeziehung ! g probabilistischer oder normischer Natur ist. Carnap war in (1936/37, 445) noch der Meinung, Dispositionsbegriffe und theoretische Begriffe seien gleichartig zu behandeln. In (1956, § IX, X) ging Carnap dazu über, Dispositionsbegriffe von theoretischen Begriffen scharf zu unterscheiden. Ontologisch betrachtet sind Dispositionsbegriffe funktionale Merkmale, theoretische Begriffe dagegen Strukturmerkmale (vgl. Earman 1986, 94). Strukturmerkmale sind die Ursachen von diversen Dispositionen, aber nicht

Funktionale Merkmale vs. Strukturmerkmale

103

104

3. Das analytische Instrumentarium

Dispositionsmerkmale als funktionale Merkmale

mit ihnen identisch. Semantisch betrachtet werden Dispositionsbegriffe durch eine einzige gesetzesartige Regelmäßigkeit definiert. Theoretische Begriffe äußern sich dagegen in vielen gesetzesartigen Regelmäßigkeiten bzw. Dispositionen, ohne darauf definitorisch zurückführbar zu sein. So ist polare Molekülstruktur das chemische Strukturmerkmal, welches die Ursache der Wasserlöslichkeit einer Substanz ist; zugleich ist dieses Strukturmerkmal die Ursache vieler weiterer Dispositionen, wie z. B. elektrolytische Leitfähigkeit usw. (s. Kap. 5.1). Leider wird dieser Unterschied von einigen Autoren verwischt. Quine (1960, 385; 1976, 26) meint, man solle eine Disposition besser mit einer sie verursachenden inneren Eigenschaft identifizieren, z. B. die Wasserlöslichkeit des Zuckers mit seiner inneren chemischen Struktur (s. auch Mumford 1998, 158). Dieser Vorschlag scheint aber durch folgenden Sachverhalt widerlegt zu werden: verschiedene Strukturmerkmale können dieselbe Disposition erzeugen. Beispielsweise könnte die Wasserlöslichkeit einer gewissen bislang unbekannten ,außerirdischen‘ Substanz statt durch polare chemische Bindungen durch einen anderen Mechanismus hervorgebracht werden. Die Elastizität eines Metalls wird durch eine völlig andere Molekülstruktur hervorgebracht als die Elastizität eines Gummibandes. Usw. Wenn man Dispositionsmerkmale mit Strukturmerkmalen identifiziert, müsste man in all diesen Fällen davon sprechen, dass jeweils eine andere Disposition vorliegt. Die Wasserlöslichkeit des Zuckers wäre eine andere Wasserlöslichkeit als die der außerirdischen Substanz. Aber das ist gegenintuitiv und verfehlt die Natur von Dispositionsbegriffen als Funktionalbegriffen – dass etwas wasserlöslich ist, besagt nicht weniger aber auch nicht mehr als dass es sich nach Einbringung in Wasser auflösen würde.

3.10.2 Herausforderungen an die logisch-definitorisch-synthetisch-Unterscheidung Abgrenzung logische versus nichtlogische Begriffe

Die in Kap. 3.3+3.4 getroffene Unterscheidung zwischen logischen und extralogisch-analytischen Wahrheiten setzt eine klare Abgrenzung zwischen logischen und nichtlogischen Begriffen voraus. Etchemendy (1990, 32 f.; 125 ff.) hat demgegenüber argumentiert, dass die Unterscheidung zwischen logischen und nichtlogischen Symbolen lediglich eine Frage pragmatischer Konvention sei: nichtlogische Symbole sind solche, deren (extensionale) Interpretation wir beliebig variieren lassen, während wir die Interpretation von logischen Symbolen fixieren. Um Etchemendys Herausforderung begegnen zu können, wird eine objektive und nicht-pragmatische Unterscheidung zwischen logischen und nichtlogischen Begriffen benötigt. Mittlerweile gibt es hierfür mehrere erfolgversprechende Vorschläge (z. B. Tarski 1986, Sher 1991). Aufbauend auf den Konventionen 17-1 und 17-4 von Carnap (1972, 88) wird in Schurz (1999, § 6) vorgeschlagen, ein Symbol einer interpretierten Sprache genau dann als logisches Symbol anzusehen, wenn seine extensionale Interpretation allein durch seine sprachinterne Bedeutungsfestlegung bestimmt wird, ohne Bezug auf Realtatsachen. Beispielsweise legen wir die Bedeutung wahrheitsfunktionaler Satzoperatoren durch ihre Wahrheitstafeln fest (s. Kap. 3.1.1). Diese Bedeutungsfestlegung bestimmt aber bereits voll-

3.10 Weiterführende Themen

ständig die Extension dieser Satzoperatoren, welche in ihrer Wahrheitswertefunktion besteht. Analog kann für Quantoren argumentiert werden. Extralogische Bedeutungskonventionen wie „Junggesellen sind bislang unverheiratete Männer“ legen dagegen keineswegs schon die Extension eines Begriffs fest, sondern implizieren nur gewisse mengentheoretische Beziehungen zwischen den unbekannten Begriffsextensionen (s. Etchemendy 1990, 71 f.). Schwieriger ist die zweite Herausforderung – die Quinesche Kritik in (1951) an der Unterscheidung zwischen analytischen und synthetischen Wahrheiten. Diese Kritik besagt (voralledem), dass für natürliche, empirisch vorfindliche Sprachen kein klares Kriterium existiert, um diese Abgrenzung vorzunehmen. Gemäß der semantischen Unbestimmtheitsthese von Quine (1960, § 16) ist die Bedeutung in vielen Fällen einfach unbestimmt. Nun hatte sich Carnaps logische Semantik auf konstruierte Idealsprachen bezogen (1972, 291). Für diese Sprachen ist es einfach eine Forderung, dass alle Bedeutungsbeziehungen explizit durch Bedeutungspostulate reglementiert sind. Explizitdefinitionen findet man gehäuft in den Nomenklaturen von technischen und wissenschaftlichen Sprachen. Aber man findet sie nur selten in natürlichen Sprachen: niemand hat jemals eine allgemeinverbindliche Definition von Begriffen wie „Tisch“ festgelegt – was zählt noch als Tisch?, wieviel Beine muss er haben?, etc. Darin liegt die Schwierigkeit, auf die Quine uns hinweist. Carnap (1972, Anhang D) gesteht Quine diese Schwierigkeit zu. Er gesteht Quine auch zu, dass die analytisch-synthetischAbgrenzung für theoretische Begriffe unbestimmt ist. Quines kritische Beispiele betreffen extralogische Analytizität, und zwar erstens in natürlichen Sprachen, und zweitens in wissenschaftlichen Theorien (s. dazu Kap. 5.1, 5.8.2). Die minimale Konsequenz, die aus Quines Kritik zu ziehen ist, wäre also nur, dass die analytisch-synthetisch-Unterscheidung zwar machbar ist, aber auf natürliche Sprachen und wissenschaftliche Theorien in nur geringem Umfang anwendbar ist. Quine geht allerdings weiter: er verwirft die Unterscheidung insgesamt. Stattdessen entwickelt Quine das Modell des Wissens als eines geschichteten Präferenzsystems – eines „konzentrisches Kraftfeldes“, auf welches die perzeptuellen Inputs einwirken (1951, 47 f.). Am Rande dieses Kraftfeldes liegen jene Hypothesen, die wir im Konflikt mit der Erfahrung am ehesten aufzugeben bereit sind, und in seinem Zentrum liegen logische Wahrheiten oder Bedeutungspostulate, die wir am wenigsten preiszugeben bereit sind. Ich bin wie viele andere Autoren der Meinung, dass dieses Quinesche Modell die epistemische Situation adäquat beschreibt. Doch daraus folgt nicht, dass man keinen Unterschied zwischen analytischen und synthetischen Wahrheiten machen könnte. Wie im vorigen Kapitel ausgeführt wurde, können Definitionen zwar nicht wahr oder falsch sein, sie können aber mehr oder weniger empirisch adäquat sein. Neue Erfahrungen können uns also auch dazu motivieren, Definitionen abzuändern oder aufzugeben. Selbst logische Wahrheiten können unter bestimmten Umständen revidiert werden, z. B. wenn eine mehrwertige Semantik mit den Wahrheitswerten wahr, falsch und unbestimmt verwendet wird. Das Quinesche Bild der geschichteten Präferenzordnung stimmt also auch dann, wenn die logisch-analytisch-synthetischUnterscheidung gemacht wird.

Quines semantische Unbestimmtheitsthese

Wissen als konzentrisches Kraftfeld

105

106

3. Das analytische Instrumentarium

Semantischer Holismus

Ostensive Bedeutungskonventionen

Wenn man wie Quine die analytisch-synthetisch-Unterscheidung generell verwirft, handelt man sich schwere Probleme ein. Denn der Bezug zwischen Sprache und Welt enthält notwendigerweise ein konventionelles Element. Erst wenn die Bedeutung von zumindest einigen Ausdrücken durch Konvention festgelegt ist, kann man gehaltvolle Hypothesen formulieren: solange nicht klar ist, was „Rabe“ bedeutet, ist es sinnlos, den Wahrheitswert von „dort ist ein Rabe“ herausfinden zu wollen. Das Analytische soll gerade dieses konventionelle Moment in der Benutzung von Begriffen erfassen. Wenn man aber sagt, dass es dieses konventionelle Moment gar nicht gebe, wie kann dann die Tatsache erklärt werden, dass Sprachbenutzer überhaupt kommunizieren können, d. h. ihre Begriffe in intersubjektiv übereinstimmender Weise benutzen? Eine Konsequenz, die manche Autoren aus der Quineschen Kritik gezogen haben, ist der radikale semantische Holismus, demzufolge die Bedeutung eines Begriffes durch unser gesamtes Vermutungswissen über die Extension dieses Begriffes bestimmt wird. Wie Fodor (1984, 26 f., 1990, xi) hervorhebt, würde der radikale semantische Holismus die Folge haben, dass zwei Sprecher unmöglich über denselben Gegenstand verschiedener Meinung sein können. Wenn z. B. Peter meint, alle Schwäne wären weiß, während Paul an die Existenz schwarzer Schwäne glaubt, so würden Peter und Paul mit „Schwan“ jeweils etwas anderes meinen. Doch offensichtlich können Menschen sehr wohl über etwas verschiedener Meinung sein und dennoch über dasselbe reden. Jede befriedigende Bedeutungstheorie muss auch das konventionelle Moment der Sprache erfassen, und dieses Moment soll im Begriff der Analytizität eingefangen werden. Das Quinesche Problem liegt nicht daran, dass es in natürlichen Sprachen dieses konventionelle Moment nicht gibt, sondern daran, dass dieses Moment vorwiegend in ostensiver Weise funktioniert. Wenn z. B. der Begriff „rot“ von einem Kind erworben wird, dann lernt das Kind, das Wort „rot“ mit einem gewissen perzeptuellen Wahrnehmungseindruck zu assoziieren; auf diese Weise werden im Kinde primäre Bedeutungskonventionen erzeugt. Ostensive Konventionen wie z. B.: „mit ,rot‘ ist diese Farbe gemeint“ sind die grundlegendste Schicht analytischer Sätze. Dabei handelt es sich aber um indexikalische Sätze, die man nicht in eine rein sprachliche Form kleiden kann. In wissenschaftlichen Lehrbüchern benutzt man hierfür Abbildungen oder Fotographien – z. B. die Abbildung einer Niere in einem medizinischen Lehrbuch. Zusammenfassend halten wir fest: extralogisch-analytische Wahrheiten sind auch in natürlichen Sprachen vorhanden, manifestieren sich dort jedoch vorwiegend in Form von ostensiven Charakterisierungen, und nur selten durch kontextfreie Explizitdefinitionen.

3.10.3 Relevanz und Irrelevanz in logischen Schlüssen Die klassische Definition der logisch gültigen Folgerung umfasst neben relevanten auch sogenannte irrelevante Konsequenzen. Der Haupttyp irrelevanter Konklusionen sind disjunktive Abschwächungen (sog. Additionen):

3.10 Weiterführende Themen

(3.10-3) Irrelevante Konklusion (Haupttyp) – Schlussart der Addition: (1)

H Z. B. H _ B (Daher:)

Die Erde ist hohl Analog: (1*) B Die Erde ist hohl oder heute regnet es. H_B

Irrelevante Konsequenzen haben die Wissenschaftstheorie immer wieder mit scheinbar paradoxen Resultaten konfrontiert (Weingartner/Schurz 1986). So behauptet die verbreitete Bestätigungstheorie des hypothetischen Deduktivismus (Kap. 5.8.3), dass eine Theorie T durch ihre zutreffenden empirischen Konsequenzen bestätigt wird. Würde diese These auf beliebige, also auch auf irrelevante Konsequenzen wie oben zutreffen, so käme gemäss (3.10-3 1, 1*) das folgende Resultat heraus: Gemäß (1) folgt H _ B aus H; somit würde H _ B die Hypothese H bestätigen, wenn H_B wahr wäre. Gemäß (1*) folgt H _ B aus dem wahren Beobachtungssatz B, somit ist auch H _ B wahres Beobachtungswissen. Somit wird die Hypothese H, dass die Erde hohl ist, durch das Beobachtungswissen „die Erde ist hohl oder heute regnet es“ direkt und durch den Beobachtungssatz „heute regnet es“ indirekt bestätigt – ein unsinniges Resultat. Einige Philosophen schlossen aus solchen ,Paradoxien‘, dass die deduktive Logik für wissenschaftstheoretische Rekonstruktionen ungeeignet sei (z. B. Glymour 1980). Aufbauend auf Körner (1947) wurde von Schurz und Weingartner (Schurz/Weingartner 1987, 2010; Schurz 1991, 1994) ein Weg entwickelt, das Problem irrelevanter Schlüsse mit logischen Mitteln zu lösen. Grundlegend hierfür ist die Beobachtung, dass irrelevante Schlüsse immer inessentielle Subformeln besitzen – inessentiell insofern, als sich die Subformel im Schluss durch jede beliebige andere Formel salva validitate ersetzen lässt, d. h. ohne dass sich durch die Ersetzung an der Gültigkeit des Schlusses etwas ändert. So ist im Schluss (1) die unterstrichene Subformel B der Konklusion salva validitate beliebig ersetzbar: aus H folgt „H oder X“ für beliebiges X, z. B. „H oder meine Großmutter fährt Motorrad“ (analog in 1*). Während die Schlussart (3.10-3) mit irrelevanten Konklusionen zu tun hat, gibt es auch Schlüsse mit irrelevanten Prämissen, die ebenfalls für paradoxe Resultate sorgten. Der Haupttyp prämissenirrelevanter Schlüsse sind konjunktive Abschwächungen (Simplifikationen) von der Form H ^ B 41 B, z. B. „die Erde ist hohl und heute regnet es; daher: heute regnet es“. Es wäre unsinnig, zu sagen, eine beliebige Beobachtung B bestätige die Konjunktion von B mit der Hypothese, die Erde sei hohl. Auch hier ist „H“ salva validiate durch beliebiges X ersetzbar. Statt Subformeln zu ersetzen, ist es einfacher, Prädikate zu ersetzen. Schurz (1991) gelangt zu folgender Definition: (Def. 3.10-2) Relevanz in logischen Schlüssen: (1.) Die Konklusion K eines gültigen Argumentes ist relevant g. d. w. es in K kein Prädikat gibt, das an einigen Vorkommnissen simultan durch ein beliebiges (gleichstelliges) Prädikat salva validitate ersetzbar ist. (2.) Die Prämissenmenge P eines gültigen Argumentes ist relevant g. d. w. es in P kein Prädikat gibt, das an einem einzelnen Vorkommniss durch ein beliebiges (gleichstelliges) Prädikat salva validitate ersetzbar ist.

Paradoxien der Irrelevanz

Irrelevanz als Ersetzbarkeit salva validitate

107

108

3. Das analytische Instrumentarium

In der Definition von Konklusionsrelevanz werden Mehrfachersetzungen eines Prädikates zugelassen, in der Definition von Prämissenrelevanz nur Einfachersetzungen. Dies hat mit folgendem wichtigen Unterschied zwischen Konklusionsrelevanz und Prämissenrelevanz zu tun: die Prämissen relevanter Schlüsse dürfen Begriffe enthalten, die in der Konklusion nicht auftreten; die Konklusionen relevanter Schlüsse dürfen niemals Begriffe enthalten, die in den Prämissen nicht auftreten. Einige Beispiele: (3.10-4) Beispiele („41“ für „logische Folge“, unterstrichen = s.v. ersetzbar) P-relevant, K-irrelevant: p 41 p _ q; p 41 q ! p; p 41 p ^ (q _ :q); 8x(Fx ! Gx) 41 8x(Fx ! Gx _ Hx); P-irrelevant, K-relevant: p ^ q 41 p; 8x(Fx _ Hx ! Gx) 41 8x(Fx ! Gx); P-relevant und K-relevant: p !q, p 41 q; p ! q 41 :q!:p; 8x(Fx ! Gx), Fa 41 Ga, 8xFx 41 Fa; Fa 41 9xFx; P-irrelevant und K-irrelevant: p ^ q 41 p _ r; 8x(Fx ^ Gx) 41 8x(Fx _ Hx).

Die Logik-plusRelevanz-These

Mit diesem Ansatz des relevanten Schließens lassen sich eine Reihe von Problemen der deduktionslogischen Wissenschaftstheorie befriedigend lösen, indem man an wissenschaftliche Argumentformen wie z. B. deduktive Bestätigung oder Erklärung die Forderung der Relevanz in obigem Sinne stellt (ein verwandter Ansatz stammt von Gemes 1993). Der Grundidee dieses Ansatzes zufolge besteht angewandtes Schließen, im Alltag wie in den Wissenschaften, immer aus klassischer Logik plus Relevanz. Diese Grundidee wurde auch von Grice (1975) verfolgt und wird durch kognitionspsychologische Untersuchungen bestätigt (z. B. Rips 1994, 47 ff.). Die Logik-plus-Relevanz-These besagt, dass der Gültigkeitsbegriff der klassischen Logik in sich in Ordnung ist, jedoch nicht alles erfasst, was für angewandtes Schließen wichtig ist. Die Beibehaltung des klassischen Gültigkeitsbegriffs unterscheidet diesen Ansatz vom Programm der Relevanzlogik, welches die Ersetzung der klassischen Logik durch eine nichtklassische Relevanzlogik vorschlägt (Anderson/Belnap 1975). Allerdings sind die Haupttypen irrelevanter Schlüsse (Additionen und Simplifikationen) relevanzlogisch gültig. Daher ist die Relevanzlogik kein taugliches Mittel zur Lösung wissenschaftstheoretischer Paradoxien. Für viele wissenschaftstheoretische Zwecke ist es nötig, die relevanten Konsequenzen einer Theorie bzw. eines Wissenssystems in seine kleinsten relevanten konjunktiven Teile – seine sogenannten relevanten Konsequenzelemente – zu zerlegen. Beispielsweise darf, wenn A eine wahre Konsequenz einer Theorie T und B eine falsche Konsequenz von T ist, die Konjunktion A ^ B nicht als eine neue bzw. dritte falsche Konsequenz von T zählen (s. Schurz/Weingartner 1987, § 4). Durch die Einschränkung auf relevante Konsequenzelemente werden logische Redundanzen eliminiert. Die genaue Definition (Weiterentwicklung von Schurz 1991, 423) ist folgende. Bekanntlich kann jede PL-Formel logisch äquivalent transformiert werden in eine PKN-Form (pränexe konjunktive Normalform) der Gestalt: „Quantorensequenz: Konjunktion von Disjunktionen von negierten oder unnegierten Atomformeln“.

3.10 Weiterführende Themen

(Def. 3.10-3) Definition relevanter Konsequenzelemente: (1.) Eine Formel A heißt elementar g. d. w. sie in PKN-Form ist und nicht L-äquivalent ist mit einer Konjunktion von PKN-Formeln, die allesamt kürzer sind als A. (2.) K ist ein relevantes Konsequenzelement von P g. d. w. K eine elementare relevante Konsequenz von P ist. (3.10-5) Beispiele (p ! q) ^ p: (p ^ q) _ (p ^:q): 8x(Fx _ Gx!Hx ^ Qx):

Relevante Konsequenzelemente (des linksstehenden Satzes) p, q p 8x(Fx ! Hx), 8x(Gx ! Hx), 8x(Fx ! Qx), 8x(Gx ! Qx), alle Instanziierungen dieser Gesetze (z. B. Fa ! Ha, usw.)

Für die Auswertung des Gehaltes einer Satzmenge S müssen störende irrelevante und redundante Konsequenzen eliminiert werden. Wenn man den logischen bzw. empirischen Gehalt auf die relevanten Konsequenzelemente einschränkt, erhält man folgende wissenschaftstheoretisch äußerst nützliche Gehaltsbegriffe: (Def. 3.10-4) (a) Der relevante logische Gehalt eines Satzes bzw. einer Satzmenge S ist die Menge seiner relevanten Konsequenzelemente. Wir schreiben dafür Cr(S). (b) Der relevante empirische Gehalt von S ist die Menge jener relevanten Konsequenzelemente von S, die empirische und nicht analytisch wahre Sätze sind. Wir schreiben dafür Er(S). Es lässt sich zeigen, dass jede Satzmenge L-äquivalent ist mit der Menge ihrer relevanten Konsequenzelemente, sodass durch relevante Wissensrepräsentationen keine klassische Information verloren geht (für den AL-Fall s. Schurz/Weingartner 1987, 58, Prop. 6; im allgemeinen PL-Fall ist ein Beweis bislang nur möglich, wenn man Quantoren 2. Stufe zulässt.)

3.10.4 Weiterführendes zur Wahrscheinlichkeit 3.10.4.1 Mathematische Gesetze der Wahrscheinlichkeit. Aus den Grundaxiomen der Wahrscheinlichkeit ergeben sich eine Reihe von Theoremen, von denen die wichtigsten im folgenden zusammengestellt sind. Eine Formel A in genau n freien Variablen heißt exhaustiv im statistischen Fall g.d.w die Extension von A die Menge aller n-Tupel von Individuen im Individuenbereich D umfasst, und im subjektiven Fall, g.d.w. die Menge aller A-wahrmachenden Modelle (= extensionale Interpretationen) mit der Menge aller als möglich erachteten Modelle der Sprache koinzidiert. Eine Folge von n

Relevanter Gehalt

109

110

3. Das analytische Instrumentarium

paarweise disjunkten Ai (1 A1 _ … _ An exhaustiv ist. Theoreme der Wahrscheinlichkeit



i



n) heißt Partition g.d.w. die Disjunktion

(Ms. 3.10-1) Theoreme unbedingter Wahrscheinlichkeit (anstatt „p“ kann auch „w“ stehen): (T1) p( :A) = 1 – p(A) (Komplementärwahrscheinlichkeit) (T2) p(A)  1 (obere Schranke) (T3) p(A^:A) = 0 (Kontradiktion) (T4) p(A1 _ A2) = p(A1) + p(A2) – p(A1 ^ A2) (allgem. Additionsgesetz) (T5) Wenn A1 ! A2 exhaustiv ist, dann gilt p(A1)  p(A2) (Monotonie) Theoreme bedingter Wahrscheinlichkeit (für die Formeln X in Antecedensposition ,w( – |X)‘ wird jeweils p(X) > 0 angenommen): (TB1) Wenn B ! A exhaustiv ist, dann gilt p(A|B) = 1. Die Umkehrung gilt nicht. (TB2) p(A^B) = p(A|B) N p(B) (TB3) Für jede Partition B1, …, Bn gilt: p(A) = R1in p(A|Bi) N p(Bi) (allg. Multiplikationsgesetz). Speziell folgt: p(A) = p(A|B) N p(B) + p(A|:B) N (1 – p(B)) (TB4) p(A|B) = p(B|A) N p(A) / p(B) (Bayes-Theorem, 1. Version) (TB5) Für jede Partition A1, …, An gilt: p(Ai|B) = p(B|Ai)Np(Ai) / R1in p(B|Ai) N p(Ai) (Bayes-Theorem, 2. Version) (TB6) Symmetrie der probabilistischen Abhängigkeit: p(A|B) > p(A) g.d.w. p(B|A) > p(B) g.d.w. p(B|A) > p(B| : A) (analog für F)

Wahrscheinlichkeitstheoretische Folgerung

Die Bedeutung des Bayesschen Theorems liegt in jenen Fällen, wo man vornehmlich an w(Ai|B) interessiert ist, aber nur die inverse bedingte Wahrscheinlichkeit w(B|Ai) praktisch zugänglich ist. Dies ist z. B. der Fall, wenn es sich bei den Ai um rivalisierende statistische Hypothesen, und bei B um experimentelle Stichprobenresultate handelt (s. Kap. 4.5.2). Es gibt (mindestens) vier unterschiedliche Arten des formalen Aufbaus der Wahrscheinlichkeitstheorie: den mathematisch-mengenalgebraischen Aufbau (z. B. Bauer 1978), den sprachlich-semantischen Aufbau in statistischer und in subjektiver Variante (beides z. B. bei Bacchus 1990), und den sprachlich-syntaktischen Aufbau (z. B. Carnap 1950b). Wir gehen darauf nicht näher ein. Die wahrscheinlichkeitstheoretische Folgerungsrelation lässt sich so charakterisieren: eine Wahrscheinlichkeitsaussage A folgt wahrscheinlichkeitstheoretisch aus einer Menge D von Wahrscheinlichkeitsaussagen g.d.w. A aus D und den Kolmogorov-Axiomen (plus mathematischen Definitionen) logisch folgt. Eine über die Kolmogorov-Axiome hinausgehende Annahme ist die r-Additivität: ein Wahrscheinlichkeitsmaß p heißt r-additiv g.d.w. die Wahrscheinlichkeit jeder Vereinigung von abzählbar unendlich vielen paarweise disjunkten Ereignissen Ai (i2IN) identisch ist mit der unendlichen Summe der Wahrscheinlichkeiten der Ai. Die r-Additivität ist wesentlich für die ma-

3.10 Weiterführende Themen

thematische Maßtheorie. Sie ist jedoch keine generell adäquate Forderung, da sie z. B. von Gleichverteilungen über abzählbar unendlichen Möglichkeitsräumen nicht erfüllt wird (s. de Finetti 1970, Kap. III.11.6; Earman 1992, 216, 226). Ein Zufallsexperiment ist ein Prozess, der genau eines von mehreren möglichen Ergebnissen besitzt, wie z. B. das Werfen einer Münze, oder das zufällige Ziehen eines Individuums aus einem Individuenbereich. Statistisch bedeutsam sind Kombinationen von unabhängigen identischen Zufallsexperimenten. Darunter versteht man die Mehrfachdurchführung desselben Zufallsexperimentes, wobei die Einzelausführungen physikalisch und daher auch probabilistisch voneinander unabhängig sind. Ein Beispiel wäre das Werfen von n Münzen, oder das Ziehen von n Individuen aus einem Individuenbereich. Die statistische Wahrscheinlichkeit, Zahl zu werfen, hängt nicht davon ab, was in vorausliegenden Münzwürfen geworfen wurde; darauf beruht die sogenannte Unmöglichkeit von Spielsystemen in Zufallsspielen. Aus der Unabhängigkeitsannahme folgt die statistische Produktregel: p(Fx^Gy) = p(Fx)Np(Gy), d. h. die Wahrscheinlichkeit dafür, dass in zwei unabhängigen Durchführungen desselben Zufallsexperiments, dessen Resultate durch die Individuenvariablen x und y bezeichnet werden, einmal Ergebnis F und das andere Mal Ergebnis G erzielt wird, ist gleich dem Produkt der Einzelwahrscheinlichkeiten. Für subjektive Wahrscheinlichkeiten gilt die Produktregel im allgemeinen nicht. Sobald das subjektive Wahrscheinlichkeitsmaß induktiv ist, muss unser Glaubensgrad dafür, dass das nächste Individuum b ein F ist, mit der Häufigkeit von bisher beobachteten Individuen a, die F waren, anwachsen. Es muss also w(Fa|Fb) > w(Fa) und somit w(Fa^Fb) > w(Fa)Nw(Fb) gelten, was der Produktregel widerspricht. Dieser Unterschied ist so zu erklären: In der subjektiven Wahrscheinlichkeitstheorie geht man davon aus, dass man die statistische Wahrscheinlichkeit nicht kennt. Man weiß z. B. nicht mit Sicherheit, ob es sich bei einer gegebenen Münze um eine symmetrische Münze (p = 1/2) oder um eine asymmetrische Münze mit Bias handelt, z. B. um eine magnetisierte Münze mit p = 1/3. In diesem Fall ist es induktiv sinnvoll, aus dem gehäuften Eintreten von Kopf zu schließen, dass die Münze eher Kopf als Zahl ergibt. In der statistischen Wahrscheinlichkeitstheorie geht man von der bekannten statistischen Wahrscheinlichkeit aus. Für diese gilt aufgrund der physikalischen Unabhängigkeitsannahme die Produktregel. D. h., weiß man, dass die Münze mit relativer Häufigkeit r auf Kopf landet, so weiß man auch, dass sie im zweimaligen Münzwurf mit relativer Häufigkeit r2 zweimal auf Kopf landet (usw.). Aus der statistischen Produktregel leitet sich das bekannte Binomialgesetz (oder Bernoulli Gesetz) für das (unabhängige) Ziehen von n-elementigen Zufallsstichproben ab. Bezeichne hn(Fx) die relative Häufigkeit eines Ereignisses Fx in einer n-elelementigen Zufallsstichprobe, und sei p := p(Fx), dann gilt:

Unabhängige Kombinationen von Zufallsexperimenten

Statistische Produktregel

    Binomialgesetz: p hn ðFxÞ ¼ nk ¼ nk  pk  ð1  pÞnk .   n! Dabei ist nk :¼ k!ðnkÞ! die Anzahl der Möglichkeiten, aus n Individuen k auszuwählen. Die Binomialverteilung ist in Abb. 3.10-1 über dem Einheits-

Binomialverteilung

111

112

3. Das analytische Instrumentarium p(hn) (Wahrscheinlichkeit der Stichprobe)

Abb. 3.10-1: Drei Binomialverteilungen pðhn ¼ nkÞ für p=1/2

n = 1000 n = 100 n = 10

(Normalverteilungsnäherung, d.h. die Fläche unter der Kurve ist auf 1 normiert) x

x

0

Gesetze der großen Zahlen

Das Problem des Häufigkeitsgrenzwertes

x

x

x

x

x x

x

hn= k n (Häufigkeit in der Stichprobe)

intervall [0,1] abgebildet (anschaulichkeitshalber approximiert durch eine Normalverteilung, d. h. die Flächen unter den Kurven sind jeweils auf 1 normiert, und die Funktionen erscheinen kontinuierlich, obwohl sie nur für rationale Abszissenwerte hn = k/n definiert sind). Für n!v strebt die Binomialverteilung gegen eine unendlich steilgipfelige kontinuierliche Normalverteilung (Hays/Winkler 1970, 222 ff., 609 ff.). Daraus ergeben sich die Gesetze der großen Zahlen, welche grob gesprochen besagen, dass die Häufigkeit eines Ereignisses Fx in einer unendlichen Zufallsfolge mit Wahrscheinlichkeit 1 gegen einen Grenzwert konvergiert, der mit dem Wert von p(Fx) übereinstimmt (s. Bauer 1978, 173 – 183; Stegmüller 1973b, 191 ff.). 3.10.4.2 Probleme des statistischen und des subjektiven Wahrscheinlichkeitsbegriffs. Sowohl die objektiv-statistische wie die subjektive Deutung der Wahrscheinlichkeit bergen Probleme in sich, die bis heute nicht vollständig gelöst sind. Beim statistischen Wahrscheinlichkeitsbegriff sind es voralledem Definitionsprobleme, beim subjektiven voralledem Rechtfertigungsprobleme. Die Rechtfertigung der Grundaxiome A1 – 3 ist im Fall des statistischen Wahrscheinlichkeitsbegriffs unproblematisch: sie folgt aus der Definition statistischer Wahrscheinlichkeit als Häufigkeitsgrenzwert (s. Lauth/Sareiter 2002, 250 f.). Auch die wissenschaftliche Bedeutung des statistischen Wahrscheinlichkeitsbegriffs ist offensichtlich. Die Information über statistische Häufigkeiten von alternativen Ereignistypen (z. B. {Fx, : Fx}) ermöglicht uns, die Ereignisalternative mit maximaler Häufigkeit vorauszusagen, was die Häufigkeit unserer wahren Prognosen auf lange Sicht maximiert (s. Greeno 1970, 95). Die Schwierigkeit liegt dagegen in der Frage einer adäquaten Definition von statistischer Wahrscheinlichkeit. Das Problem besteht darin, dass mithilfe eines Zufallsexperimentes potentiell unendlich viele unendlich anwachsende Ergebnisfolgen (d1, d2, … ) 2 Dv produziert werden können – warum sollten sie alle denselben Häufigkeitsgrenzwert p(Fx) besitzen, und warum sollten sie alle überhaupt einen Häufigkeitsgrenzwert besitzen? Das Problem wird dadurch verschärft, dass man aus einer gegebenen Ergebnisfolge durch willkürliche Umordnung oder Stellenauswahl der Ergebnisse jederzeit eine Folge mit beliebig abweichendem Häufigkeitsgrenzwert konstruieren kann.

3.10 Weiterführende Themen

Die (von mir sogenannte) naive statistische Theorie reagiert auf dieses Problem mit dem Gesetz der großen Zahlen: die Behauptung „p(Fx) = r“ besage gemäß diesem Gesetz nicht, dass in allen Zufallsfolgen der Häufigkeitsgrenzwert von Fx r beträgt, sondern lediglich, dass er mit Wahrscheinlichkeit 1 r beträgt. Gegen diesen Definitionsversuch haben Kritiker zurecht eingewandt, dass er zirkulär ist: im Definiens des Ausdrucks „die Wahrscheinlichkeit von Fx ist r“ kommt erneut die Phrase „mit Wahrscheinlichkeit 1“ vor – Wahrscheinlichkeiten werden also nicht auf Häufigkeitsgrenzwerte, sondern letztlich wieder auf Wahrscheinlichkeiten zurückgeführt (vgl. Kutschera 1972, 104; Stegmüller 1973c, 37; Eagle 2004, 396 f.). Ich bin wie Howson/Urbach (1996, 351) der Meinung, dass bislang nur die von Mises’sche Theorie einen Ausweg aus diesem Zirkularitätsproblem anbietet. Von Mises geht von einer Gesamtfolge von allen Durchführungen eines Zufallsexperimentes aus – z. B. die Folge aller jemals gemachten Würfe mit Würfeln desselben physikalischen Typs, angeordnet in der Zeit, und hypothetisch verlängert in die unbegrenzte Zukunft. Von Mises nennt diese Gesamtfolge ein statistisches Kollektiv. Von Mises (1964, Kap. 1) stellt nun an eine statistische Gesamtfolge (Kollektiv) g folgende zwei Bedingungen: (1.) jedes mögliche Ergebnis E besitzt in g einen Häufigkeitsgrenzwert, der mit der Wahrscheinlichkeit p(Ex) identifiziert wird, und (2.) dieser Häufigkeitsgrenzwert ist insensitiv gegenüber berechenbaren und ergebnisunabhängigen Stellenauswahlen, d. h. in allen durch solche Stellenauswahlen erzeugten Teilfolgen von g besitzt Ex denselben Häufigkeitsgrenzwert. Auf diese Weise gelingt es von Mises einerseits, den Begriff der Zufallsfolge auf natürliche Weise zu charakterisieren, nämlich als jede auf diese Weise erzeugte Teilfolge der Gesamtfolge, und zweitens folgt aus der von Mises’schen Definition nachweislich die für alle statistische Verfahren fundamentale Produktregel p(Fx1^Gx2) = p(Fx1) N p(Gx2) (s. von Mises 1964, 27). Diese beiden von Mises’schen Bedingungen gelten keineswegs apriori, sondern sind als empirisch gehaltvolle Dispositionsaussagen über die reale Natur des zugrundeliegenden Zufallsexperimentes zu verstehen. Man kann mit Popper die von Mises’sche statistische Wahrscheinlichkeit von Ex auch die generische Propensität des Zufallsexperimentes bezeichnen, das Ergebnis Ex hervorzurufen. Die Poppersche dispositionelle Wahrscheinlichkeitsauffassung liegt daher schon bei von Mises vor (s. Howson/Urbach 1996, 338). Einige Autoren, unter anderem Popper selbst, haben darüberhinaus versucht, einen objektiven Begriff der singulären Propensität bzw. Einzelfallwahrscheinlichkeit einzuführen; es gibt jedoch sehr starke Einwände gegen die Existenz singulärer Propensitäten (s. z. B. Eagle 2004; Gillies 2000, 113 ff.). Worin liegt aber der genaue empirische Gehalt einer statistischen Wahrscheinlichkeitshypothese, und wie wird sie empirisch überprüft? Es gibt keine Beobachtungsaussage, die aus dieser Hypothese logisch folgt: dass eine Zufallsfolge einen bestimmten Häufigkeitsgrenzwert r besitzt, ist für jedes noch so große n mit jedem beliebigem bis dahin erreichten Häufigkeitswert hn 6¼ r logisch verträglich. Dieser Sachverhalt, welcher von Bayesianern gerne als Einwand formuliert wird (s. Howson und Urbach 1996, 331), drückt aber lediglich die bekannte Tatsache aus, dass aus statistischen Hypothesen keine Beobachtungssätze logisch folgen. Statistische Hypothesen sind nicht

Zirkularitätsvorwurf

Der Ansatz nach von Mises

Wahrscheinlichkeit als Propensität

113

114

3. Das analytische Instrumentarium

Subjektive Wahrscheinlichkeiten als faire Wettquotienten

Probleme des Wettquotienten-Ansatzes

deduktivistisch überprüfbar, doch sie sind probabilistisch bzw. ,weich‘ überprüfbar – sie können durch endliche Stichprobenhäufigkeiten bestätigt oder geschwächt werden, vorläufig akzeptiert oder zurückgewiesen werden. Wie dies geschieht, wird in der statistischen Inferenz- und Testtheorie gezeigt (s. Kap. 4.2.3). Im Fall des subjektiven Wahrscheinlichkeitsbegriffs ist es nicht das Definitionsproblem, das Schwierigkeiten macht: subjektive Wahrscheinlichkeiten sind einfach definiert als epistemische Glaubensgrade, die die KolmogorovAxiome der Wahrscheinlichkeit A1 – 3 erfüllen. Für die subjektive Wahrscheinlichkeitstheorie stellt sich dagegen ein Rechtfertigungsproblem: aus welchen Gründen sollten rationale Glaubensgründe die Grundaxiome A1 – 3 erfüllen, und worin sollte die Bedeutung solcher Glaubensgrade in Bezug auf das Erkenntnisziel der Findung gehaltvoller Realwahrheiten liegen? Die bekannteste subjektive Rechtfertigung der Wahrscheinlichkeitsaxiome A1 – 3 ist die auf Ramsey und de Finetti zurückgehende Idee, subjektive Glaubensgrade über das Wettverhalten von rationalen Personen als faire Wettquotienten zu definieren (s. Carnap/Jeffrey 1971, 105 ff.; Howson/ Urbach 1996, 75 ff.). Die zentrale Aussage, die in dieser Methode bewiesen wird, ist folgende: die fairen Wettquotienten einer Wettperson erfüllen genau dann die Grundaxiome A1 – 3, wenn sie kohärent sind, was bedeutet, dass es kein System von fairen Wetten gibt, welches in jedem möglichen Weltzustand für die Wettperson zu einem Gesamtverlust führt. Der Wettquotienten-Ansatz ist mannigfacher Kritik ausgesetzt (s. z. B. Earman 1992, 41 ff.). Der fundamentale Einwand gegen den Wettquotiente-Ansatz ist folgender: kohärente faire Wettquotienten sind noch lange nicht rational im Sinne von wahrheitsorientiert, denn die Definition des fairen Wettquotienten nimmt nur auf die subjektiven Glaubensgrade der wettenden Person und die Wetteinsätze, aber nirgendwo auf objektive statistische Wahrscheinlichkeiten Bezug. Die reale Erfolgshäufigkeit der Wettperson wird durch die faire Wett-Rechtfertigung gar nicht berührt. Nehmen wir beispielsweise einen Subjektivisten, der begeistert 1:1 darauf wettet, dass er mit einem regulären Würfel eine Sechs würfelt, und dessen Wettquotient insofern fair ist, als er auch bereit wäre, die Gegenwette 1:1 darauf anzunehmen, dass er keine Sechs würfelt. Unser Subjektivist bleibt selbst noch dann fair und kohärent, wenn er sein ganzes Vermögen verloren hat. Er wird sich natürlich darüber wundern, dass ihm die seiner Ansicht nach ebenso fairen Gegenwetten nie abgenommen werden. Aber er kann sich nicht erklären, warum gerade er sein ganzes Vermögen verlor, während andere abgesahnt haben, solange er nicht die objektiven Häufigkeitschancen des Ereignistyps, auf den er gewettet hat, in Betracht zieht. Dies zeigt, dass die Axiome A1 – 3 bestenfalls eine Minimalbedingung für rationale Glaubensgrade liefern, die jedoch viel zu schwach ist, um aus objektiver Sicht irrationales Wettverhalten auszuschließen. Die größte Schwäche des subjektiven Wett-Ansatzes besteht also darin, dass er keinen intrinsischen Bezug zu statistischen Wahrscheinlichkeiten besitzt. Sobald man sich um einen solchen Bezug bemüht, ergibt sich ein m.E. überlegener Weg, die Grundaxiome A1 – 3 der subjektiven Theorie zu rechtfertigen: durch die Festlegung, dass rationale subjektive Glaubensgrade

3.10 Weiterführende Themen

intendieren, mit den realen statistischen Wahrscheinlichkeiten möglichst gut übereinzustimmen. Dies können sie nur, wenn sie die Grundaxiome A1 – 3 erfüllen. Ich nenne dies die häufigkeitsintendierte Rechtfertigung (ähnlich argumentierten Carnap 1950b, 167 ff., Earman 1992, 46, u. a. m.). 3.10.4.3 Principal principle und engste Referenzklasse. Einige Vertreter des subjektiven Ansatzes haben sich um zusätzliche Axiome bzw. Prinzipien bemüht, welche eine Verbindung zu objektiven Wahrscheinlichkeiten herstellen sollen. Das elementarste Prinzip dieser Art ist das sogenannte principal principle, demzufolge die subjektiven Wahrscheinlichkeiten, wenn die objektiven Wahrscheinlichkeiten gewusst werden, mit diesen übereinstimmen sollen. Dieses Prinzip kommt jedoch in zwei Versionen, die sehr unterschiedlicher Natur sind. Das auf Lewis (1980) zurückgehende singuläre principal principle wird von Subjektivisten vertreten, welche einen statistischen Wahrscheinlichkeitsbegriff ablehnen; hier werden ,objektive‘ singuläre Propensitäten einfach postuliert, und es bleibt unklar, welcher Realeigenschaft diese singulären Propensitäten entsprechen sollen. Das auf de Finetti zurückgehende statistische principal principle wird dagegen von Subjektivisten vertreten, die den statistischen Wahrscheinlichkeitsbegriff anerkennen (vgl. Strevens 2004; Howson/Urbach 1996, 345; Kutschera 1972, 82). Die Konditionalisierung auf ein beliebiges Erfahrungsdatum E(b1, …, bn) über andere Individuen b1, …, bn in (Ms. 3.10-2, StPP) ist wichtig, um daraus zwei weitere Versionen des StPP ableiten zu können: erstens das StPP für Zufallsstichproben, welches für die subjektive Rechtfertigung der statistischen Likelihood-Intuition benötigt wird (s. Kap. 4.5.2), und zweitens das konditionale StPP, welches dem Prinzip der engsten Referenzklasse und dem induktiv-statistischen Spezialisierungsschluss (Kap. 2.6.1) zugrunde liegt. („hn(Fx)“ steht für die relative Fx-Häufigkeit in einer n-elementigen Stichprobe.)

Versionen des principal principles

(Ms. 3.10-2) Statistisches Principal Principle (StPP): w(Fa | p(Fx) = r ^ E(b1, …, bn)) = r StPP für Zufallsstichproben: w(hn(Fx) = k/n | p(Fx) = r) = ðnk Þ rk  ð1  rÞnk Konditionales StPP: w(Fa | Ga ^ p(Fx|Gx) = r ^ E(b1, …, bn)) = r Das statistische principal principle ist nur für subjektive apriori-Wahrscheinlichkeiten sinnvoll: das sind hypothetische Glaubensgrade eines rationalen Subjektes, das noch keine bestimmten Erfahrungen gemacht hat. Für aktuale Glaubensgrade gilt das Prinzip nicht generell. Denn wird z. B. durch Beobachtung gewusst, dass die Münze auf Kopf fiel (= Fa), so ist der subjektive Glaubensgrad von Fa auch dann 1, wenn man weiß, dass p(Fx)=1/2 beträgt. In einer apriorischen Wahrscheinlichkeitsfunktion muss das gesamte Hintergrundwissen W des gegebenen Subjektes explizit in das Antecedens einer konditionalen Wahrscheinlichkeitsaussage w( – |W) geschrieben werden. Der Zusammenhang zwischen aktualer (personalistischer) und apriorischer Wahrscheinlichkeit wird durch die Regel der strikten Konditionalisierung bestimmt: sei w0 die apriori-Wahrscheinlichkeit bzw. die Wahrscheinlich-

Strikte Konditionalisierung

115

116

3. Das analytische Instrumentarium

Arten engster Referenzklassen

keit zu einem Ausgangszeitpunkt t0, und wt die gegenwärtige Wahrscheinlichkeit, und sei Wt das gesamte zwischen t0 und t erworbene Wissen, dann gilt für beliebige A: wt(A) = w0(A | Wt) (vgl. Carnap/Jeffrey 1971, 18). Wenig beachtet wird in der Literatur, dass das in Kap. 3.9 erläuterte Prinzip der engsten Referenzklasse aus (i) dem konditionalen StPP und (ii) der Regel der strikten Konditionalisierung folgt. Denn gegeben Ra ist unser gesamtes Wissen über Individuum a, E(b1, …, bn) unser Wissen über davon verschiedene Individuen, und wir wissen p(Fx|Rx) = r, dann gilt gemäß (i) w0(Fa |Ra ^ p(Fx|Rx)=r ^ E(b1, …, bn)) = r, woraus gemäß (ii) für die aktuale Glaubensfunktion wt(Fa) = r folgt, d. h. die Aussage des Prinzips der engsten Referenzklasse. Das Prinzip der engsten Referenzklasse lässt sich darüberhinaus auf folgende Weise rechtfertigen. Zunächst kann für ein gegebenes Ereignis Fa das Individuum a vielen verschiedenen Referenzklassen zugehören, die Fx sehr verschiedene bedingte Wahrscheinlichkeiten zuweisen. Würden wir die subjektive Einzelfallwahrscheinlichkeit auf unterschiedliche Referenzklassen beziehen, so würden wir zu widersprüchlichen probabilistischen Prognosen gelangen. Hempel (1965, Kap. 3.4) nannte dies die Ambiguität statistischer Schlüsse. Durch Bezugnahme auf eine ausgezeichnete Referenzklasse werden inkohärente Glaubensgrade und widersprüchliche Einzelfallprognosen vermieden. Warum sollte die ausgezeichnete Referenzklasse, auf die wir Einzelfallwahrscheinlichkeiten beziehen, gerade die engste Referenzklasse sein? Aus folgendem Grund: man kann beweisen, dass die Verwendung engster Referenzklassen (im Vergleich zu anderen Referenzklassen) den Häufigkeitsgrenzwert zutreffender Prognosen und den Erwartungsnutzen unserer Handlungen maximiert (s. Good 1983, 178 ff.). Das Prinzip der engsten Referenzklasse involviert ferner drei weitere Subtilitäten: (1.) Referenzklassen müssen durch nomologische Attribute (im Sinn von Kap. 6.3.2) bestimmt werden, und nicht durch rein extensional definierte Attribute. Ansonsten wäre die engste Referenzklasse eines Individuums a immer die Klasse {a} bzw. das Attribut x=a, was zum unsinnigen Ergebnis w(Fa) = p(Fx| x=a) = 1 führen würde. Auch muss das Referenzattribut vom Ereignis, dessen subjektiver Erwartungswert bestimmt wird, analytisch unabhängig sein. (2.) Wir haben den Begriff der engsten Referenzklasse in Übereinstimmung mit Reichenbach, Carnap und Hempel epistemisch bestimmt, als engste Referenzklasse Rx, von der bekannt ist, dass a in Rx liegt. Salmon (1984, 37) hat dagegen für sein Erklärungsmodell einen objektiven Begriff der engsten Referenzklasse benutzt (Kap. 6.3.2). (3.) Nützlich ist schließlich die Einschränkung auf engste relevante Referenzklassen: diese müssen nicht alle bekannten Informationen über das gegebene Individuum a erfassen, sondern nur jene Informationen, von welchen Fx probabilistisch abhängig ist. D. h., ist Rx eine relevante engste Referenzklasse von a für Fx, dann gilt p(Fx|Rx) = p(Fx|Rx^R*x) für jede weitere bekannte Information R*a. Vorschläge in dieser Richtung sind Hempels Begriff der maximal bestimmten Bezugsklasse (1965, 397) und Salmons Begriff der breitesten homogenen Bezugsklassen (1984, 37).

3.11 Zusammenfassung, einführende Literatur und Übungen

3.11 Zusammenfassung, einführende Literatur und Übungen 3.11.1 Zusammenfassung (Auswahl). Wissenschaftliche Begriffe lassen sich nach ihrem logischen Typ, nach ihrem Inhaltstyp, sowie nach ihrem Skalentyp (bzw. Abstufungstyp) klassifizieren. Darauf baut die Klassifikation wissenschaftlicher Satzarten auf. Analytisch grundlegend ist die Unterscheidung zwischen logischen, definitorischen und synthetischen (bzw. kontingenten) Sätzen, und die exakte Definition von logischer und von definitorischer Wahrheit. Wissenschaftstheoretisch subtil ist die Unterscheidung zwischen Beobachtungsbegriffen, empirischen Dispositionsbegriffen, und theoretischen Begriffen (i. e. S. sowie i. w. S.). Wissenschaftstheoretisch grundlegend ist die Unterscheidung von Satzarten nach ihrem Allgemeinheitsgrad, sowie der daran anknüpfende Begriff der Gesetzesartigkeit, die Definition der Begriffe der Verifizierbarkeit, Falsifizierbarkeit, Bestätigbarkeit und Schwächbarkeit von Hypothesen, sowie die Begriffe des logischen, empirischen sowie relevanten Gehalts von Hypothesen. Der Wahrscheinlichkeitsbegriff, der für die Formulierung nicht-strikter Merkmalszusammenhänge benötigt wird, untergliedert sich in einen statistischen Begriff von Wahrscheinlichkeit als Häufigkeitsgrenzwert, und in einen subjektiven Begriff von Wahrscheinlichkeit als rationaler Glaubensgrad. 3.11.2 Einführende Literatur. Eine kompakte Einführung in die Sprachphilosophie gibt Runggaldier (1990). Eine exzellente Logikeinführung ist Klenk (1989). Fortgeschrittene lesen z. B. Ebbinghaus (2003) zur Mengenlehre, Rautenberg (2002) zur Logik und Krantz et al. (1971) zur Metrisierungs- und Messtheorie. Stegmüllers Einführung in die Analytische Wissenschaftstheorie (1969) ist nach wie vor sehr empfehlenswert. In die philosophischen Probleme der Wahrscheinlichkeitstheorie führt Sie Stegmüller (1973b, c) ein. Sie finden darüber auch viel in Kutschera (1972) und Gillies (2000). Klassiker zu Analytisch-Synthetisch-Fragen sind Carnap (1972) und Quine (1960).

3.11.3 Fragen und Aufgaben Zu Kap. 3.1: 1) Um welche logische Art von Begriff (Term) handelt es sich in den folgenden Beispielen: (a) „Peter der Große“, (b) „der linke Fuß von …“, (c) „der linke Fuß von Peter dem Großen“, (d) „… ist verliebt in…“, (e) „… ist logisch unmöglich“, (f) „weder …, noch…“, (g) „es gibt kein …, das …“. 2) Klassifizieren Sie folgende Begriffe nach ihrem Inhaltstyp: (a) „x ist verwerflich“, (b) „x ist ein rotes Viereck“, (c) „x übt auf y eine Kraft aus“, (d) „x ist brüchig“, (e) „x versteht, was y meint“, (f) „x ist ein Fleischfresser“, (g) „x ist politisch korrekt“. 3) Was ist der Skalentyp folgender Begriffe: (a) Geschlecht, (b) Einwohnerzahl, (c) biologische Artzugehörigkeit, (d) körperliche Attraktivität, (e) Zeitpunkt, (f) Intelligenzquotient, (g) Energie, (h) Belastbarkeit.

117

118

3. Das analytische Instrumentarium

Zu Kap. 3.2: 1) Welcher der folgenden Sätze ist präskriptiv, deskriptiv oder gemischt: (a) Wirtschaftswachstum ist gut, behauptet die Wirtschaftszeitung, (b) Wenn die Wirtschaft wächst, ist der Kapitalismus gut, (c) Susi hat einen schlechten Charakter, (d) Susi ist krank. 2) Was ist der Inhaltstyp der folgenden deskriptiven Sätze: (a) Im Nebenraum wird ein politischer Umsturz vorbereitet, (b) Im Nebenraum ist das Fenster offen, (c) Meine Haut ist sehr sonnenempfindlich, (d) Im Garten von Maria steht ein Tisch, (e) Hunde bellen, (f) Magnetfelder wirken heilend. Zu Kap. 3.3 – 3.4: 1) Wie lautet die Definition von logisch wahren Sätzen und wie die Definition von definitorisch wahren Sätzen? 2) Welcher der folgenden wahren Sätze ist logisch wahr, definitorisch wahr, oder synthetisch wahr: (a) Wenn Raben schwarz sind, dann sind nichtschwarze Vögel keine Raben, (b) Jeder unzerlegbare Gegenstand ist atomar, (c) Jedes Atom besteht aus Protonen, Neutronen und Elektronen, (d) stabile Demokratien sind nicht krisenanfällig, (e) Demokratien können krisenanfällig oder nicht krisenanfällig sein, (f) Demokratien können nur stabil sein, wenn sie eine stabile Wirtschaft besitzen. Zu Kap. 3.5 – 3.6: 1) Klassifizieren Sie folgende Sätze nach ihrem Allgemeinheitsgrad: (a) 70 % aller Stundenten in München haben Wohnungsprobleme, (b) Es gibt kein Perpetuum Mobile, (c) Peter hat gestern promoviert und heute geheiratet, (d) die meisten Menschen glauben an Gott, (e) alle Bewohner dieses Heimes sind Musiker, (f) in Australien gibt es auch weiße Raben, (g) alle Säuren sind Protonendonatoren, (h) fast genau 50 % aller Münzwürfe landen auf Kopf. 2) Welcher der generellen Sätze in 1) ist gesetzesartig? Zu Kap. 3.7 – 3.8: 1) Welche Hypothesen sind verifizierbar und/oder falsifizierbar? (i) Zwischen 70 – 80 % aller Vögel können fliegen. (ii) Zwischen 70 – 80 % aller Deutschen verdienen monatlich mehr als 2000 Euro. (iii) Kraft = Masse mal Beschleunigung. (iv) Alle Schwäne sind weiß. (v) Dieser Schwan dort ist weiß. Zu Kap. 3.9: 1) Analysieren Sie folgende Aussagen nach dem Prinzip der engsten Referenzklasse: (a) So wenig, wie du geschlafen hast, wirst du morgen wahrscheinlich müde sein. (b) Wahrscheinlich wirst du dich von ihm anstecken. (c) Unwahrscheinlich, dass morgen die Berge nebelfrei sind. (d) Das Plastikspielzeug deines Kindes geht sicher nach spätestens drei Wochen kaputt. 2) Die Häufigkeiten von unverheirateten (U) vs. verheirateten (V) Frauen (F) und Männern (M) im Seminar sind so verteilt: U ^ F 20 %, U ^ M 30 %, V ^ F 27 %. Visualisieren Sie dies durch Kreisdiagramme. Wie hoch sind die Häufigkeiten von: (i) V&M, (ii) V, (iii) U, (iv) M, (v) F, (vi) U oder M, (vii) V oder F, (viii) M oder F, (ix) V&U? Wie hoch sind folgende bedingte Häufigkeiten: (x) V gegeben M, (xi) V gegeben F, (xii) M gegeben U, (xiii) F gegeben U, (xiv) F gegeben (U oder M), (xv) (V oder M) gegeben M ? Zu Kap. 3.10: 1) Charakterisieren Sie die folgenden Dispositionen durch einen bilateralen Reduktionssatz: (i) x ist hilfreich, (ii) ist reaktionsschnell. 2) Versuchen Sie, folgende Begriffe zu definieren: (i) Tisch, (ii) Stein, (iii) Ampel, (iv) Himmel, (v) Energie. Waren Ihre versuchsweisen Definitionen analytisch?

3.11 Zusammenfassung, einführende Literatur und Übungen

3) Welche der folgenden Konsequenzen sind logisch (un)gültige Konsequenzen und welche sind (ir)relevante (gültige) Konsequenzen der Prämissenmenge „Alle Menschen sind soziale Wesen, Kein soziales Wesen will allein sein“: (a) Alle weiblichen Menschen sind soziale Wesen, (b) Kein Mensch will allein sein, (c) Kein Mensch, der ein soziales Wesen ist, will allein sein, (d) Nur Menschen sind soziale Wesen, (e) Jeder Mensch will allein sein oder nicht. 4) Wie groß ist die Wahrscheinlichkeit, mit einer regulären Münze von 10 Würfen mindestens 5 mal Kopf zu erzielen?

119

4. Gesetzeshypothesen und ihre empirische Überprüfung

Wahrheitsbedingung

Relevanzbedingung

In den folgenden Abschnitten beschäftigen wir uns mit qualitativen Gesetzeshypothesen. In den meisten Fällen haben diese die folgende Form: eine Konjunktion von mehreren Antecedensfaktoren A1x, A2x (…) impliziert ein gewisses Konsequensmerkmal Kx – entweder strikt, oder sie verleiht ihm eine gewisse bedingte Wahrscheinlichkeit. Die Konjunktion aller Antecedensbedingungen bildet das komplexe Antecedensprädikat, d. h. A1x ^ A2x (^…) := A(x). (Anstelle von x können auch mehrere Individuenvariablen stehen.) Verallgemeinerungen dieser Form werden nur kurz erwähnt. Welche Bedingungen muss ein (mit nomologischen Prädikaten gebildeter) Allsatz bzw. sein statistisches Gegenstück erfüllen, um als wissenschaftliches Gesetz akzeptierbar zu sein? Voralledem zwei: (1.) Offensichtlich ist die Wahrheitsbedingung: falsche Gesetzeshypothesen liefern falsche Voraussagen und können dadurch zu schädlichen Fehlplanungen führen. Natürlich können wir uns der Wahrheit eines universellen Gesetzes niemals sicher sein, sondern dieses nur mehr oder minder gut bestätigen. Konkret einlösbar ist nicht die semantische Wahrheitsbedingung, sondern die epistemische Bestätigungsbedingung – deshalb sprechen wir im folgenden immer von „wahr bzw. bestätigt“ (s. auch Kap. 6.1). Es gibt aber noch eine zweite und oft übersehene Bedingung für wissenschaftlich akzeptable Gesetzeshypothesen, nämlich (2.) die Relevanzbedingung, die wir nun besprechen.

4.1 Die Relevanzbedingung 4.1.1 Relevanz von strikten Gesetzen Betrachten wir die folgenden zwei Allhypothesen der Form 8x(A1x ^ A2x ! Kx):

Wahre Gesetze mit irrelevanten Antecedensfaktoren

(4.1-1) (1.) Alle Männer (A1), die Antibabypillen nehmen (A2), werden nicht schwanger (K). (2.) Tollkirschen (A1), die an Vollmondnächten um Mitternacht gepflückt wurden (A2), haben halluzinogene Wirkung (K). Bei beiden Beispielen handelt es sich um universelle wahre Gesetzeshypothesen. Dennoch stimmt mit diesen Beispielen etwas nicht: ein Konjunktionsglied des Gesetzesantecedens ist jeweils irrelevant, d. h., es ist unnötig für das Eintreten der Gesetzeskonsequenz. Was (1.) betrifft, so werden Männer natürlich schon aufgrund ihrer biologischen Natur niemals schwanger – auch dann nicht, wenn sie keine Antibabypillen nehmen (das Beispiel

4.1 Die Relevanzbedingung

stammt von Salmon 1971, 34). Was (2.) betrifft, so haben Tollkirschen in jedem Fall halluzinogene Wirkung, auch wenn sie nicht an Vollmondmitternächten gepflückt werden. Dieses zweite Beispiel beruht auf mittelalterlichen Überlieferungen über „Hexensalben“ (vgl. Duerr 1978, 15 f.) und zeigt auf, was charakteristisch für viele mystische Überlieferungen ist: nämlich korrekte empirische Zusammenhänge mit wissenschaftlich irrelevanten mystischen Beifügungen zu befrachten. Aufgrund dieser Analyse können wir die Relevanzbedingung für strikte Gesetzeshypothesen so formulieren: (Def. 4.1-1) Relevanzbedingung für strikte Gesetzeshypothesen: Ein wahres bzw. bestätigtes Implikationsgesetz der Form „ 8x: wenn A1x und A2 und …, dann Kx“ ist relevant (hinsichtlich seines Antecedensprädikates) g.d.w. keine konjunktive Antecedenskomponente Aix in dem Sinn überflüssig ist, dass sie in dem Gesetz gestrichen werden kann salva veritate, d. h. unter Beibehaltung der Wahrheit bzw. Bestätigtheit. Erläuterungsbeispiel: Das Resultat der Streichung von A2x in (1:) 8x(A1x ^ A2x ^ A3x !Kx) ist (2:) 8x(A1x ^ A3x !Kx). A2 ist nur dann relevant in (1), wenn (2) falsch ist. – Spezialfall: Das Resultat der Streichung von Ax in 8x(Ax ! Kx) ist 8xKx. Ein irrelevanter Antecedensfaktor einer Gesetzeshypothese ist weder prognostisch relevant noch kausal relevant. Er ist nicht prognostisch relevant, weil seine Verifikation für die Prognose des Konsequens in jedem Einzelfall völlig unnötig ist. Er kommt nicht als Kausalfaktor in Frage, weil Kausalfaktoren bzw. Ursachen für das Eintreten der Wirkung mitverantwortlich sein müssen, und daher notwendige Teile von hinreichenden Bedingungen sein müssen. Man beachte jedoch: die Relevanz eines Antecedensfaktors im definierten Sinne ist zwar äquivalent mit seiner prognostischen Relevanz, sie stellt für kausale Relevanz dagegen nur eine notwendige, aber nicht schon eine hinreichende Bedingung dar. Damit ein prognostisch relevantes und wahres Gesetz auch als Kausalgesetz akzeptabel ist, im Sinne dass sein Antecedens A(x) als (komplexe) Ursache für Kx angesehen werden kann, sind weitere Bedingungen erforderlich, die wir in Kap. 4.3 und 6.5.2 näher besprechen. Die Relevanzbedingung hängt eng mit der auf Mackie (1975) zurückgehenden INUS-Theorie von strikten Kausalhypothesen zusammen. Grundlegend für Mackies Analyse sind zwei Begriffe: Ax heißt (nomologisch) hinreichende Bedingung für Bx g.d.w. 8x(Ax ! Bx) ein wahrer gesetzesartiger Satz ist; und Ax heißt eine (nomologisch) notwendige Bedingung für Bx g.d.w. 8x(Bx ! Ax) eine wahrer gesetzesartiger Satz ist (1975, 30 f.). Was meinen wir mit „Ursache“ in einer alltäglichen Kausalbehauptung wie „Die Ursache des Heustadelbrandes war eine brennende Zigarette?“, so lautet Mackies Ausgangsfrage. Die brennende Zigarette allein ist keine hinreichende Bedingung, denn weitere Bedingungen mussten erfüllt sein, damit es zum Heustadelbrand kam, wie z. B. dass genügend Sauerstoff vorhanden und dass das Heu trocken war. Noch bildet die brennende Zigarette eine notwendige Bedingung, denn der Heustadelbrand hätte auch durch eine an-

Prognostische und kausale Relevanz

121

122

4. Gesetzeshypothesen und ihre empirische Überprüfung

Mackies INUSTheorie der Ursache

Verallgemeinerung der Relevanzbedingung

Nichtinvarianz gegenüber L-äquivalenter Umformung

dere Ursache ausgelöst werden können, z. B. durch ein brennendes Streichholz oder durch einen Blitzschlag. Eine Ursache ist nach Mackie vielmehr ein notwendiger aber nicht (unbedingt) hinreichender konjunktiver Teil einer hinreichenden aber nicht (unbedingt) notwendigen Bedingung für die Wirkung – kurz eine INUS-Bedingung (insufficient but necessary part of an unnecessary but sufficient condition). In Anknüpfung an Mackie können wir die Relevanzbedingung auch so formulieren: für die Akzeptabilität von 8x(A1x ^ A2x!Kx) als (Kausal-)Gesetz ist nicht nur nötig, dass das gesamte Antecedensprädikat A1x ^ A2x eine hinreichende Bedingung für Kx ist; zusätzlich muss jede Antecedenskomponente Aix auch ein notwendiger Teil dieser hinreichenden Bedingung sein. Mackie ist sich bewusst, dass das Kriterium der INUS-Bedingung allein noch nicht ausreicht, um Kausalbeziehungen zu charakterisieren. Mackie betont auch, dass uns in alltäglichen Ursachenbehauptungen die Randbedingungen häufig nur unvollständig oder gar nicht bekannt sind: eigentlich formulieren wir nur Existenzbehauptungen 2. Sufe der Form „es gibt Randbedingungen ux, sodass Ax ^ ux hinreichend ist für Kx, aber weder Ax allein noch ux allein“ (1965, 24). Dieser Vorschlag ähnelt den exklusiven ceteris paribus Gesetzen von Kap. 3.5. Wir erläutern nun knapp, wie unsere Relevanzbedingung auf qualitative Gesetze von beliebig komplexer Form generalisiert werden kann. Schritt 1: Wir betrachten nun Gesetzeshypothesen der Form 8x(A1x ^… ^ Anx ! K1x _ … _ Kmx) mit Konjunktionen von Basisformeln im Antecedens und Disjunktionen von solchen im Konsequens. Wir sprechen hier von elementaren Allhypothesen. Aus Symmetriegründen ist die Relevanzbedingung nun sowohl an Antecedenskonjunkte wie an Konsequensdisjunkte zu stellen: kein Aix bzw. Kjx darf im Gesetz streichbar sein salva veritate. Schritt 2: Wie man beweisen kann, ist jeder beliebige qualitative (reine) Allsatz L-äquivalent mit einer als Konjunktion gelesenen Menge von elementaren Allhypothesen obiger Form. (Dabei lassen wir auch Allhypothesen mit leerem Antecedens zu: 8xKx). Daher können wir einen qualitativen Allsatz als relevant definieren, g.d.w. er eine L-äquivalente Zerlegung in relevante elementare Allimplikationen besitzt. (Bei gemischt-quantifizierten Hypothesen ist eine solche konjunktive Zerlegung nicht immer möglich; in diesem Fall muss die Relevanzbedingung an alle Matrixkonjunkte der PKNForm gestellt werden.) Die Relevanzbedingung für strikte Gesetzeshypothesen hängt eng mit der Definition relevanter Konsequenzen von Kap. 3.10.3 zusammen. Man kann zeigen: ein wahrer Allsatz G ist genau dann irrelevant (in obigem Sinne), wenn es einen wahren Allsatz G* gibt, der G als irrelevante Konsequenz besitzt. Dabei wird G* aus G durch Streichung von Antecedenskonjunkten bzw. Konsequensdisjunkten gewonnen. Abschließend sei ein subtiles Phänomen angesprochen: Relevanz- und Redundanzaspekte sind nicht invariant gegenüber L-äquivalenter Umformung. Aus diesem Grund hat Hempel (1977, 109) die Salmonsche Relevanzbedingung an die Gesetzeshypothesen von Erklärungen abgelehnt. Salmon hatte diese Relevanzbedingung mithilfe der von Ax und Kx bezeichneten Extensionen charakterisiert, und damit (fälschlicherweise) vorausge-

4.1 Die Relevanzbedingung

setzt, dass beliebige L-äquivalente Umformungen der sprachlichen Formulierung erlaubt seien. Hempel brachte folgendes Beispiel als Einwand: nach Salmon könnten die beiden Gesetzeshypothesen (1) „Zucker ist wasserlöslich“ (8x(Zx ! Wx)) und (2) „Salz ist wasserlöslich“ (8x(Sx ! Wx)) nicht als relevant bzw. erklärungskräftig angesehen werden, sondern nur die Hypothese (3) „alles, was Zucker ist oder Salz ist, ist wasserlöslich“ (8x(Zx _ Sx ! Wx)). Denn Zx _ Sx ist eine logische Abschwächung sowohl von Zx wie von Sx. Ob eine logische Abschwächung durch Weglassung eines Konjunktes oder durch Addition eines Disjunktes geschieht, macht nach Hempel keinen Unterschied: denn Zx ist L-äquivalent mit (Zx _ Sx) ^ (Zx _ :Sx). Somit ist (1) L-äquivalent mit (4) 8x((Zx _ Sx) ^ (Zx _ :Sx) !Wx), und darin ist wegen (3) das unterstrichene Antecedenskonjukt salva veritate streichbar. (Hinweis: Hempel gab sein Beispiel in probabilistischer Form: p(W|Z) = 0.95, p(W|S)=0.95, und p(W|S _ Z)= 0.95.) Die Kontroverse macht deutlich, dass Relevanzbedingungen selbstverständlich nicht auf beliebige L-äquivalente Umformungen bezogen werden dürfen, denn durch solche werden ja Irrelevanzen in die Formulierung hineingebracht. Vielmehr müssen wir Relevanzbedingungen (sowohl deduktiver wie probabilistischer Art) immer auf eine sogenannte irreduzible sprachliche Repräsentation der betrachteten Propositionen beziehen – das ist eine Formulierung, in denen alle Irrelevanzen durch L-äquivalente Transformation eliminiert wurden. Wenn wir das tun, so werden wir das Problem der Nichtinvarianz auf ganz natürliche Weise los. Wir definieren:

Irreduzible Repräsentation

(Def. 4.1-2) Die irreduzible Repräsentation I(S) eines Satzes S ist eine nichtredundante Konjunktion von relevanten Konsequenselementen von S (im Sinne von Def. 3.10-3), welche mit S L-äquivalent ist. (Nichtredundanz meint, dass kein Konjunktionsglied von I(S) aus den restlichen Konjunktionsgliedern logisch folgt). Beispielsweise gilt: I(p ^ (p ! q)) = p ^ q, I(p! (q^:q)) = : p, usw. Wir haben der irreduziblen Repräsentation eines qualitativen Allsatzes in „Schritt 2“ oben indirekt schon dadurch Rechnung getragen, dass wir forderten, der Allsatz muss irgendeine konjunktive Zerlegung in relevante elementare Allimplikationen besitzen. Man kann zeigen, dass jede solche Zerlegung eine irreduzible Repräsentation darstellt.

4.1.2 Relevanz von statistischen Gesetzen Eine analoge Relevanzbedingung gilt für statistische Gesetze. Wieder hat Salmon darauf aufmerksam gemacht (1971; 1989, 58 ff.). Betrachten wir folgendes Beispiel einer statistischen Generalisierung mit nur einem Antecedensfaktor, p(Kx|Ax) = r: (4.1-3) 95 % aller an einer Erkältung erkrankten Personen, die regelmäßig hohe Dosen von Vitamin C einnehmen (A), gesunden innerhalb einer Woche (K).

Beispiel für statistische Irrelevanz

123

124

4. Gesetzeshypothesen und ihre empirische Überprüfung

Als Individuenbereich bzw. Population D wählen wir hier die Menge aller an einer Erkältung erkrankten Personen. Die statistische Generalisierung (4.1-3) wurde durch empirische Befunde gut bestätigt. Weitere Untersuchungen ergaben jedoch, dass ganz unabhängig von der Einnahme hoher Dosen Vitamin C 95 % aller Erkältungserkrankten innerhalb einer Woche gesunden (s. Salmon 1971, 83, Fn. 20). Der Faktor Ax ist also statistisch irrelevant für Kx, denn es gilt schon p(Kx) = p(Kx|Ax) = p(Kx| : Ax) = 0,95.

Positive und negative statistische Relevanz

Die statistische Relevanzbedingung besagt also folgendes: das Vorliegen einer hohen bedingten Wahrscheinlichkeit p(Kx|Ax) zwischen dem Antecedensmerkmal A und Konsequensmerkmal K (das statistische Korrelat der Wahrheitsbedingung) genügt nicht für einen relevanten statistischen Zusammenhang; es muss zusätzlich die Bedingung erfüllt sein, dass das Antecedens die bedingte Wahrscheinlichkeit des Konsequens p(Kx|Ax) gegenüber der Ausgangswahrscheinlichkeit p(Kx) (bzw. gegenüber p(Kx|: Ax)) verändert. In anderen Worten, zwischen K und A muss eine probabilistische Abhängigkeit bestehen (Kap. 3.9). Wahrscheinlichkeitserhöhende Antecedensmerkmale heißen positiv statistisch relevant; wahrscheinlichkeitssenkende heißen negativ statistisch relevant. Als prognostisch relevante Merkmale für K kommen im statistischen Fall nur positiv relevante Merkmale in Frage. Negativ relevante Merkmale sind für das Gegenteil von K, also für :K, prognostisch relevant. Die Beziehung zwischen Relevanz und Kausalität ist im statistischen Fall noch komplizierter als im strikten Fall (s. Kap. 4.3, 6.5.2). Die ,Kunststücke‘ so mancher Esoteriker beruhen auf unerkannten Irrelevanzen. Ein Beispiel war die vermeintliche Fernheilungskunst von Uri Geller in den 80er Jahren: Uri Geller behauptete, über das Fernsehen durch Telekinese die kaputten Uhren seiner Zuseher wieder in Gang setzen zu können. Nach seiner Aufforderung an die Zuseher, ihre alten Uhren aus den Kästen zu holen und zu sehen, ob sie wieder gingen, riefen tatsächlich einige hundert Leute an: „wahrhaftig, die Uhr geht wieder“. Tatsächlich ist aber die Wahrscheinlichkeit, dass von einer Million alter weggelegter Uhren ei-

Abb. 4.1-1: Irrelevanz und Witz – die Titelseite von Salmon (1971).

4.1 Die Relevanzbedingung

nige Hundert durch bloßes Herumhantieren wieder zu laufen beginnen, auch ohne Gellers Heilungskunst gegeben. Erkannte Irrelevanzen liegen andererseits oft der Struktur von Witzen zugrunde – siehe Abb. 4.1-1. Eine relevante statistische Beziehung zwischen zwei Merkmalen A und K nennt man auch eine Korrelation. Das einfachste Korrelationsmaß für qualitative Merkmale ist die Wahrscheinlichkeitsdifferenz p(Kx|Ax) – p(Kx). Wir fassen dies so zusammen:

Korrelation

(Ms. 4.1-1) Statistische Relevanz und Korrelation für qualitative Merkmale: A ist (statistisch) relevant für K A ist irrelevant für K A ist positiv relevant für K

g.d.w. g.d.w. g.d.w.

A ist negativ relevant für Kx

g.d.w.

p(Kx|Ax) 6¼ p(Kx). p(Kx|Ax) = p(Kx). A K’s Wahrscheinlichkeit erhöht, d. h. p(Kx|Ax) > p(Kx). A K’s Wahrscheinlichkeit senkt, d. h. p(Kx|Ax) < p(Kx).

Einfaches Korrelationsmaß für qualitative Merkmale: Korr(A,K) = p(Kx|Ax) – p(Kx) Korr(A,K) liegt immer zwischen den Werten +1 und – 1. Gpositiv relevant für K Gpositiv A ist Jnegativ für K g.d.w. Korr(A,K) = Jnegativ girrelevant für K gnull Man beachte die grundlegende Parallelität zwischen strikter und statistischer Relevanz: A ist strikt-relevant für K, wenn 8x(Ax ! Kx), aber nicht schon 8xKx wahr ist; A ist positiv statistisch relevant für K, wenn p(Kx|Ax) hoch aber nicht schon p(Kx) hoch ist. Notation: wenn keine Verwechslungsgefahr besteht, lassen wir im folgenden die Individuenvariable ,x‘ auch weg und schreiben ,p(A)‘ statt ,p(Ax)‘, usw. Es gibt eine Reihe verwandter Korrelationsmaße, von denen hier nur drei gestreift sein sollen: (1.) Die Definition der Wahrscheinlichkeitsveränderung kann man auch so wie in Kap. 3.9 konjunktiv formulieren: falls p(B)>0, dann gilt (i) p(A|B) 6¼ p(A) g.d.w. (ii) p(A^B) 6¼ p(A)Np(B). Ebenso für < oder > anstatt 6¼. Reichenbach (1956, 158) verwendet die konjunktive Bedingung (ii). Das quantitative Korrelationsmaß, welches der konjunktiven Korrelationsdefinition entspricht, ist die Kovarianz zwischen Merkmalen, Kov(A,K) = p(A^K) – p(K)Np(A). (Für quantitive Größen X, Y ist die Kovarianz Kov(X,Y) durch l(XNY) – l(X)Nl(Y) gegeben; mit l(X) = Mittelwert von X; s. dazu Kap. 4.5.1) Beachte: Die Kovarianz ist numerisch symmetrisch, d. h. Kov(A,K) = Kov(K,A). Das Korrelationsmaß Korr(A,K) ist dagegen nur qualitativ symmetrisch, d. h. Korr(A,K) >(( 30 legitim ist), so sind die Akzeptanzintervalle durch folgende Formeln gegeben; dabei ist rs = pffiffiffiffiffiffiffiffiffiffiffiffiffiffiffiffiffiffiffiffiffi pffiffiffi r= nðmit r :¼ p  ð1  pÞÞ die Streuung der Stichprobenhäufigkeiten (s. Kap. 4.5.1): (4.2-3) Akzeptanzkoeff. 99,5 %: 95 %: 70 %:

Stichprobenumfang und abnehmender Ertrag

Akzeptanzintervall p l 2,8Nrs p l 1,96Nrs p l 1,03Nrs

Beispiel für p=0,8; n=100 [0,69 , 0,91] [0,72 , 0,88] [0,76 , 0,84]

Wie aus der Formel ersichtlich, ist bei fixiertem Akzeptanzkoeffizienten das Akzeptanzintervall umgekehrt proportional zur Quadratwurzel des Stichprobenumfangs. Das Akzeptanzintervall wird also für größere Stichprobenumfänge immer enger: je größer die Stichproben, desto schärfer werden unsere 95 %-wahrscheinlichen Prognosen für den erwarteten Stichprobenwert. Zugleich folgt daraus ein Gesetz des abnehmenden Ertrags: eine Vervierfachung des Stichprobenumfangs bringt nur eine Halbierung des Akzeptanzintervalls, usw. Hier einige 95 %ige Akzeptanzintervalle für verschiedene Stichprobengrößen n: (4.2-4) Akzeptanzintervalle für p = 0,8 (Akzeptanzkoeff. = 0,95) für variierendes n: n = 1: [0 , 1] n =10: [0,56 , 1] n = 20: [0,63 , 0,97]

n = 50: [0,69 , 0,91] n =100: [0,72 , 0,88] n = 400: [0,76 , 0,84]

n = 1600: [0,78 , 0,82] n = 10.000: [0,79 , 0,81]

Im Fall von Einfachuntersuchungen, bei denen die Stichprobe nicht weiter aufgeteilt werden muss, werden Stichprobenumfänge von größer 30 üblicherweise als groß und von kleiner 30 als klein bezeichnet. Generell sollten Stichprobenumfänge nicht kleiner als 15 oder 20 sein. Eine weitere Restriktion an Stichprobenumfänge ergibt sich, wenn die hypothetische Populationswahrscheinlichkeit nahe bei 0 oder 1 liegt; die Stichprobenumfänge müssen so groß gewählt werden, dass die Akzeptanzintervallgrenzen echt innerhalb 0 und 1 zu liegen kommen (in obigem Beispiel ist dies erst für nF 20 der Fall). Man beachte, dass die Größe der Gesamtpopulation für Fra-

4.2 Die empirische Überprüfung von Gesetzeshypothesen

gen von Stichprobenumfängen unerheblich ist; vorausgesetzt wird nur, dass sie wesentlich (mindestens 100 mal) größer ist als der Stichprobenumfang (Bortz 1985, 112). 4.2.3.2 Auffindung statistischer Hypothesen und Konfidenzintervalle. Die Hypothese p(K|A)=0,8 wird durch das Stichprobenresultat h100(K|A) = 0,75 nur schwach bestätigt. Dies ist deshalb der Fall, weil alle statistischen Alternativhypothesen, welche ein p(K|A) im Intervall 0,75 l 0,08 behaupten, durch das Stichprobenresultat h100(K|A) = 0,75 ebenso schwach bestätigt werden, bzw. beibehalten werden würden, wenn sie zur Überprüfung anstünden. Denn für alle diese Hypothesen liegt das Stichprobenresultat in dem von ihnen mit 95 % Wahrscheinlichkeit vorausgesagten Akzeptanzintervall. Man bezeichnet das Intervall aller hypothetischen Populationswahrscheinlichkeiten, für die das Stichprobenresultat gerade noch im 95 %igen Akzeptanzintervall liegt, als das Konfidenzintervall der hypothetischen Populationswahrscheinlichkeit (statt vom Akzeptanzkoeffizienten spricht man nun vom Konfidenzkoeffizienten von 95 %). Die Methode der Konfidenzintervalle geht auf Fisher und Neyman zurück (s. Stegmüller 1973c, 189 f.; Bortz 1985, 132). Sei h ein Stichprobenresultat und r eine hypothetische Populationswahrscheinlichkeit, dann gilt folgende symmetrische Beziehung zwischen Konfidenzintervall und Akzeptanzintervall:

Schwache Bestätigung

Konfidenzintervall

h 2 [r – a, r+a] g.d.w. r 2 [h – a, h + a]. Man erhält also das Konfidenzintervall, indem man das Akzeptanzintervall symmetrisch um h (statt um r) legt. Dem Konfidenzintervall entspricht die statistische Intervallhypothese p(K|A) 2 [h – a, h+a], welche in unserem Beispiel besagt: zwischen 67 und 83 % aller Bäume an Autobahnen sind krank. Diese Konfidenzintervallhypothese ist es, welche durch das Stichprobenresultat hn(K|A)=75 % (beim Konfidenzkoeff. von 95 %) stark bestätigt wird. Ist man an der Prognose enger Konfidenzintervalle interessiert, so muss die Stichprobe entsprechend groß gewählt werden. 4.2.3.3 Überprüfung auf vermutliche Relevanz – die Methode der signifikanten Unterschiede. Um zu prüfen, ob das Merkmal A für K auch relevant ist, vergleicht man – analog zur Methode des Unterschieds im strikten Fall – die Häufigkeit von K in einer A-Stichprobe (die Merkmalsgruppe) mit der Häufigkeit von K in einer A-Kontrollstichprobe (die Kontrollgruppe). Die A-Kontrollstichprobe besteht im einfachsten Fall wieder aus einer Menge von Individuen, die das Merkmal A nicht besitzen (sie könnte auch aus einer D-Zufallsstichprobe bestehen, welche einen Zufallsanteil an A-Individuen enthält). In unserem Beispiel waren von 100 Bäumen an Autobahnen 75 erkrankt. Angenommen wir bilden nun eine A-Kontrollstichprobe von 100 Bäumen aus Waldstrichen, die nicht in der Nähe von Autobahnen liegen, und stellen fest, dass darin nur 60 Bäume erkrankt sind. Heißt dies, dass Nähe zu Autobahnen die Erkrankungswahrscheinlichkeit von Bäumen erhöht, oder könnte die Abweichung zwischen A-Stichprobe und A-Kontrollstichprobe, die 15 von 100 beträgt, nur zufällig bedingt gewesen sein?

Starke Bestätigung

137

138

4. Gesetzeshypothesen und ihre empirische Überprüfung

Null- und Alternativhypothese

Signifikante Stichprobendifferenz

Wieder ist dies eine quantitative Frage, und wie oben bedient man sich einer Intervallmethode. Aufgrund der Wahrscheinlichkeitsverteilung von Zufallsstichproben lässt sich die statistische Wahrscheinlichkeit dafür berechnen, dass die Abweichung zwischen der A-Stichprobe und der A-Kontrollstichprobe rein zufällig bedingt war – dass diese Abweichung also unter der Annahme zustande kam, dass in der Population zwischen A und K kein statistischer Zusammenhang besteht: p(K|A) = p(K|:A). Man nennt diese Irrelevanzhypothese auch Nullhypothese. Die Alternativhypothese dazu ist die Relevanzhypothese und besagt, dass in der Population zwischen A und K ein statistischer Zusammenhang besteht: p(K|A) 6¼ p(K|:A). Die Wahrscheinlichkeitsverteilung der Häufigkeitsdifferenzen zweier Form einer BinoStichproben (n1, n2) aus derselben Population nimmt die q ffiffiffiffiffiffiffiffiffiffiffiffiffi 1 1 mialverteilung mit dem Mittelwert 0 und der Streuung r  n1 þ n2 an. Man berechnet damit das symmetrische 95 %-Intervall der wahrscheinlichsten positiven oder negativen Stichprobendifferenzen. Den Absolutbetrag der maximalen Häufigkeitsdifferenz, die gerade noch innerhalb des 95 %-Intervalls liegt, nennt man die signifikante Stichprobendifferenz, und der Koeffizient von 5 % heißt der Signifikanzkoeffizient. M.a.W., das 95 %-Intervall der wahrscheinlichsten Stichprobendifferenzen fungiert als Akzeptanzintervall für die Irrelevanzhypothese (Nullhypothese); und das zweiseitig-extreme 5 %-Intervall der unwahrscheinlichsten Stichprobendifferenzen fungiert als Zurückweisungsintervall der Irrelevanzhypothese und als Akzeptanzintervall der Relevanzhypothese (Alternativhypothese). Siehe Abb. 4.2-2. (Def. 4.2-2) Signifikante Stichprobendifferenz = jener Betrag, den die Differenz zwischen A-Stichprobenhäufigkeit und A-Kontrollstichprobenhäufigkeit von K mit einer Wahrscheinlichkeit gleich dem Signifikanzkoeffizienten übersteigt, gegeben es besteht in der Population kein statistischer Zusammenhang zwischen A und K.

Abb. 4.2-2: Wahrscheinlichkeitsverteilung von Stichprobendifferenzen und signifikante Stichprobendifferenz (approximiert durch Normalverteilung)

Überschreitet die tatsächlich gefundene Stichprobendifferenz die signifikante Stichprobendifferenz, dann wird die Irrelevanzhypothese verworfen und

4.2 Die empirische Überprüfung von Gesetzeshypothesen

die Relevanzhypothese akzeptiert. In diesem Fall wäre die Wahrscheinlichkeit, eine mindestens so große Stichprobendifferenz wie die tatsächlich vorgefundene zu finden, unter der Annahme der Irrelevanzhypothese kleiner als der Signifikanzkoeffizient von 5 %; die Irrelevanzhypothese wäre damit stark geschwächt, und die Relevanzhypothese stark bestätigt. Man sagt in diesem Fall, zwischen A und K besteht ein signifikanter Zusammenhang bzw. eine signifikante Korrelation. In unserem Beispiel berechnet man (bei Annahme einer approximativen Normalverteilung) eine signifikante Stichprobendifferenz von 13 von 100; der vorgefundene Unterschied von 15 von 100 ist also signifikant. Falls die Stichprobendifferenz positiv ist, d. h. falls A zu einer Erhöhung der K-Häufigkeit führt, dann ist die als signifikant befundene Korrelation positiv; falls die Stichprobendifferenz negativ ist, dann ist diese Korrelation negativ. Ist die gefundene Stichprobendifferenz dagegen kleiner als die signifikante Differenz, dann wird die Irrelevanzhypothese weiterhin akzeptiert. (Ms. 4.2-2) zeigt die Zusammenfassung. (Ms. 4.2-2) Überprüfung der statistischen Gesetzeshypothese p(K|A) = 80 % Überprüfung auf Wahrheit – Methode der Akzeptanz- und Konfidenzintervalle: ) Nimm eine A-Stichprobe: z. B. 100 A’s. Gefunden: z. B.: 75 K’s. ) Wähle den Akzeptanzkoeffizienten: z. B. 95 % Berechne aus Stichprobengröße (n=100) und Akzeptanzkoeffizienten (95 %) das ) Akzeptanzintervall: in unserem Fall: 72 – 88 K’s von 100 A’s Liegt die A-Stichprobenhäufigkeit H 3Nein: Gesetz ist stark geschwächt j 3 Ja: Gesetz ist schwach bestätigt von K im Akzeptanzintervall? h ) Stark bestätigt ist nur das (schwächere) Konfidenzintervallgesetz, das beim gegebenen Stichprobenresultat lautet: 67 %  p(K|A)  83 % Überprüfung auf Relevanz – Methode der signifikanten Unterschiede: ) Nimm eine A-Kontrollstichprobe: z. B. 100 :A’s. Gefunden: z. B. 60 K’s. )Wähle den Signifikanzkoeffizienten: z. B. 5 % Berechne aus Stichprobengröße (n=100) und Signifikanzkoeffizienten (5 %) die ) signifikante Differenz: in unserem Fall 13 von 100. Ist die tatsächliche Differenz zwischen der A- Stichprobenhäufigkeit von K und der A-Kontrollstichprobenhäufigkeit von K größer als die signifikante Differenz?

3Nein: Relevanz von A für K ist stark geschwächt 3 Ja: Relevanz von A für K ist stark bestätigt: ) signifikante Korrelation

H f f f j f f f h

Positive Differenz: Korrelation positiv

Z

T Negative Differenz: Korrelation negativ

Signifikanter Zusammenhang

Zusammenfassung

139

140

4. Gesetzeshypothesen und ihre empirische Überprüfung

Signifikante Korrelationen müssen nicht hoch sein

Das Verfahren lässt sich auch bei anders gewählten Signifikanzkoeffizienten durchführen. Eine bei einem Signifikanzkoeffizienten von 1 % signifikante Stichprobendifferenz nennt man hochsignifikant. Oft gibt man an, bei welchem Signifikanzkoeffizient die vorgefundene Differenz gerade noch signifikant ist. In unserem Beispiel ist die vorgefundene Differenz von 0,15 bei einem Koeffizient von 2,5 % gerade noch signifikant, was bedeutet, dass die Wahrscheinlichkeit, unter Voraussetzung der Nullhypothese eine Stichprobendifferenz von F 15 zu erhalten, 2,5 % beträgt. Beim Relevanztest ist die Alternativhypothese die Negation der Nullhypothese. Daher ist erstere stark bestätigt genau dann, wenn letztere stark geschwächt ist. Dies ist anders als bei der Wahrheitsprüfung, wo es zur gegebenen Hypothese p(K|A)=r potentiell unendlich viele Alternativhypothesen der Form p(K|A) = r* 6¼ r gibt. Man nennt obigen Relevanztest auch zweiseitigen Test, weil dabei positive und negative Differenzen in Betracht gezogen werden. Wenn man von vornherein weiß, dass sich der Faktor A, wenn überhaupt, dann nur auf eine Seite hin auswirken kann, wendet man den einseitigen Unterschiedstest an, in dem das einseitig-extreme 5 %-Intervall als Zurückweisungsintervall gewählt wird. Erneut nimmt die Streuung der relativen Stichprobenhäufigkeitsdifferenzen und damit die signifikante Differenz proportional zur Wurzel der Stichprobengröße (n) ab. Jede noch so kleine relative Stichprobendifferenz ist signifikant, wenn die Stichprobengröße hinreichend groß gewählt wurde. Die bloße Behauptung, dass zwischen zwei Merkmalen A und K eine signifikante Korrelation gefunden wurde, ist daher ohne eine Information über die Stichprobengröße eine sehr schwache Behauptung. Sie besagt lediglich, dass irgendeine möglicherweise nur sehr geringe Korrelation zwischen A und K vorliegt. Insbesondere bedeutet eine hochsignifikante Korrelation noch nicht, dass die Höhe dieser Korrelation besonders hoch wäre, sondern nur, dass mit sehr hoher Wahrscheinlichkeit irgendein wenn auch sehr geringer statistischer Zusammenhang zwischen A und K besteht. Dies wird in populären Darstellungen von statistischen Ergebnissen häufig verwechselt. Wird etwa berichtet, Mediziner hätten einen signifikanten Zusammenhang zwischen dem Verspeisen von Extrawurst und der Krebsrate beobachtet – um eines von vielen möglichen Beispielen zu nennen – so wird dies als sensationelles Resultat aufgefasst, ohne zu bedenken, dass dieser Zusammenhang auch praktisch vernachlässigbar klein sein könnte. Es ist daher sehr wichtig, zusätzlich zur Information über das Bestehen einer signifikanten Korrelation über die vermutliche Höhe dieser Korrelation zu informieren. Als Maß hierfür eignet sich die vermutete Häufigkeitsdifferenz in der Population, die man aus der gefundenen Stichprobendifferenz schätzt. Dividiert man diese Häufigkeitsdifferenz durch die Streuung r, so erhält man die in Kap. 4.1.2 erwähnte Effektstärke. Die statistische Relevanzprüfung wird auf Gesetzeshypothesen mit mehreren konjunktiven Antecedensfaktoren p(K|A1^…^An) verallgemeinert, indem für jeden Antecedensfaktor Ai eine Ai-Kontrollstichprobe gebildet wird und die Häufigkeitsdifferenz zwischen der A-Stichprobe und der AiKontrollstichprobe auf signifikanten Unterschied getestet wird.

4.2 Die empirische Überprüfung von Gesetzeshypothesen

4.2.3.4 Statistische Repräsentativität. Die Repräsentativitätsforderung besagt im statistischen Falle nicht nur, so wie im strikten Fall, dass die untersuchten Individuen hinsichtlich der Begleitumstände möglichst stark variieren sollen. Sie besagt spezifischer, dass alle sonstigen für das Konsequensprädikat K („krank“) relevanten Faktoren in der A-Stichprobe möglichst gleich häufigkeitsverteilt sein müssen wie in der A-Population. In unserem Beispiel wären dies andere Kausalfaktoren als Autoabgase, die Bäume krank machen, wie z. B. Industrieabgase oder Schädlingsbefall. Im Falle der statistischen Repräsentativität ist es wichtig, zwischen Definition und Kriterium zu unterscheiden. Der Definition nach ist eine Stichprobe repräsentativ, wenn alle relevanten Merkmale in ihr gleich verteilt sind wie in der Population (s. Bortz 1985, 113). Die Annahme, dass eine Stichprobe in diesem Sinne repräsentativ ist, beruht natürlich auf einem Induktionsschluss, und kann durch kein Verfahren garantiert werden. Entscheidend sind vielmehr die Kriterien für Repräsentativität, deren Erfüllung unabhängig vom induktiven Generalisierungsschritt sichergestellt werden kann. Die Kriterien für Repräsentativität ergeben sich aus den Methoden zur Erzeugung möglichst repräsentativer Stichproben. Die wichtigste Methode ist die der Zufallsstichprobe – diese Methode empfiehlt sich immer dann, wenn nichts oder nur wenig über die Verteilung der restlichen Merkmale in der Population bekannt ist. Eine Stichprobe ist eine Zufallsstichprobe im engen Sinn, wenn ein zufälliges Auswahlverfahren vorliegt, welches jedem Individuum der Population die gleiche Chance gibt, in die Stichprobe zu gelangen. Natürlich können Zufallsstichproben zufällig von der Population abweichen, aber die Wahrscheinlichkeitsverteilung ihrer zufälligen Abweichung ist statistisch berechenbar, und darauf beruhen die erläuterten statistischen Methoden. Ein zufälliges Auswahlverfahren setzt voraus, dass alle Individuen der Population dem Auswahlverfahren zugänglich und daher in irgendeiner Form erfasst sind – z. B. durch Karteikarten oder Namenslisten, aus denen blind ausgewählt wird. Diese enge Definition von Zufallsstichprobe, auf die man oft trifft, ist sowohl unnötig eng wie zu eng. Unnötig eng, weil es nur darauf ankommt, dass alle für das Konsequensprädikat relevanten Arten von Individuen dieselbe Chance haben, in die Stichprobe zu gelangen: ist dies der Fall, dann sprechen wir von einer Zufallsstichprobe im weiten Sinn. Das Auswahlverfahren darf also lediglich keine relevanten Merkmalsverteilungen verzerren (s. Mayntz et al. 1974, 69 f.). Zu eng ist die enge Definition, weil sie meist nicht realisierbar ist. In unserem Beispiel kann man schwerlich allen mitteleuropäischen Bäumen in der Nähe von Autobahnen Nummern geben, um dann aus einer großen Urne 100 Nummern zu ziehen. Worauf es nur ankommt, ist, dass die Waldstriche, aus denen man die Zufallsstichprobe der auf Erkrankung zu prüfenden Bäume zieht, keine verzerrende Merkmale besitzen (s. Kromrey 2002, 292) – beispielsweise sollte der Schädlingsbefall in diesen Waldstrichen weder höher noch niedriger liegen als der durchschnittliche Schädlingsbefall insgesamt (usw.). Wenn die Verteilung der restlichen relevanten Merkmale in der Population bekannt ist, so kann man statt einer Zufallsstichprobe auch eine sogenannte geschichtete Stichprobe bilden, um die Repräsentativität zu errei-

Definition vs. Kriterien der Repräsentativität

Methoden der Erzeugung repräsentativer Stichproben

141

142

4. Gesetzeshypothesen und ihre empirische Überprüfung

chen. Will man beispielsweise die Konsumgewohnheiten von deutschen Durchschnittsbürgern untersuchen, so ist von den Merkmalen Stadt- vs. Landbevölkerung, Alter, Familiengröße und Geschlecht anzunehmen, dass sie das Konsumverhalten beeinflussen, und es empfiehlt sich eine sogenannte proportional geschichtete Stichprobe, bei der man für jedes relevante Merkmal Mi jenen Prozentsatz von Vpn in die Stichprobe gibt, welcher der bekannten Häufigkeit in der Population entspricht (Bortz 1985, 115; Näheres zu geschichteten Stichproben z. B. in Mayntz et al. 1974, 87 ff.).

Induktive Schlüsse in Test- und Inferenzstatistik

4.2.3.5 Teststatistik, Inferenzstatistik und die Likelihood-Intuition. Akzeptanzintervalle (bzw. Zurückweisungsintervalle) sind Stichprobenintervalle; Konfidenzintervalle sind dagegen Hypothesenintervalle (s. Hays/Winkler 1970, 383). Akzeptanzintervalle gehören zur sogenannten Teststatistik, der es um die Überprüfung von gegebenen Hypothesen mit einer bereits vorhandenen Plausibilität geht. Konfidenzintervalle gehören dagegen zur sogenannten Inferenzstatistik, der es um die Auffindung der plausibelsten Hypothesen angesichts eines gegebenen Stichprobenresultates geht. In der statistischen Praxis sind beide Problemstellungen oft nicht scharf trennbar: auch wenn man bereits gewisse Hypothesen besitzt, die man einem Akzeptanzintervalltest unterzieht, so wird man dennoch an möglichst gut abgesicherten Hypothesen und daher an deren Konfidenzintervallen interessiert sein. Sowohl in teststatistische wie in inferenzstatistische Prozeduren gehen induktive Schlüsse ein. Gemäß der Differenzierung von Kap. 2.6.2 ist der Unterschied jedoch folgender: In der reinen Teststatistik wird nur vom epistemischen Induktionsprinzip Gebrauch macht, wonach im Sinn der Akzeptanzintervallmethode bisher erfolgreiche Hypothesen weiterhin beibehalten werden, und nur erfolglose Hypothesen verworfen werden. Bei der Inferenzstatistik handelt es sich dagegen um ein methodisches Induktionsverfahren, welches angesichts eines Stichprobenbefundes das 95 %-Intervall aller plausibelsten Hypothesen auffindet. Fishers Testtheorie wird gelegentlich als quasi-falsifikationistisch bezeichnet, weil sie methodologische Regeln liefert, die uns sagen, bei welchen Stichprobenresultaten eine gegebene Hypothese beibehalten, und bei welchen sie verworfen werden soll (Howson/Urbach 1996, 174). Der Unterschied zu einer echten Falsifikation liegt natürlich darin, dass die Zurückweisung der Hypothese nur mit gewisser Wahrscheinlichkeit gilt und daher grundsätzlich vorläufig ist. Stegmüller kritisiert an der statistischen Testtheorie, dass sie die epistemischen Entscheidungen auf Akzeptanz versus Zurückweisung einschränkt (1973c, 142 ff.). Doch etliche statistische Testtheoretiker (z. B. Hays/Winkler 1970, 399 f.) schließen in die Testtheorie neben den Optionen der Zurückweisung und Akzeptanz auch die Option der Zurückhaltung bzgl. eines Urteils ein – z. B. indem das zentrale 66 %-Intervall als Akzeptanzintervall, das extreme 5 %-Intervall als Zurückweisungsintervall, und das dazwischenliegende Intervall als Zurückhaltungsintervall gewählt wird (ebenso Westermann/Hager 1982, 19). Es ist wichtig, sich vor Augen zu halten, dass mithilfe des statistischen Wahrscheinlichkeitsbegriffs immer nur die Wahrscheinlichkeit von Stichprobenresultaten (gegeben gewisse Populationshypothesen) berechnet wer-

4.2 Die empirische Überprüfung von Gesetzeshypothesen

den kann, aber niemals die Wahrscheinlichkeit der Populationshypothesen selbst. Statistische Wahrscheinlichkeiten beruhen auf wiederholbaren Zufallsexperimenten, und die Entnahme von Stichproben aus einer Population ist ein wiederholbares Zufallsexperiment. Die gesamte Population bzw. die ,aktuale Welt‘ gibt es dagegen nur einmal: sie besitzt keine statistische Wahrscheinlichkeit, denn Zufallsfolgen von ,möglichen Welten‘ gibt es nicht (s. auch Hays/Winkler 1970, 328; Howson/Urbach 1996, 239). Hypothesenwahrscheinlichkeiten sind immer subjektiver Natur und gehören daher in das Gebiet der subjektiven Wahrscheinlichkeitstheorie. Aus diesem Grund wäre es eine Konfusion, das Ergebnis der Konfidenzintervallmethode so zu lesen: mit 95 %iger Wahrscheinlichkeit liegt die Populationshäufigkeit im angegebenen Konfidenzintervall. Was die Konfidenzintervallmethode sagt, ist vielmehr folgendes: für alle Hypothesen im Konfidenzintervall liegt das tatsächliche Stichprobenresultat im 95 %-Intervall ihrer wahrscheinlichsten Stichprobenresultate. In allen statistischen Test- und Inferenzverfahren geht also folgendes vor sich: die Höhe der statistischen Wahrscheinlichkeit des Stichprobenresultates E, gegeben eine statistische Hypothese H, wird als Indikator verwendet für die Plausibilität der Hypothese H, gegeben das Stichprobenresultat E. Ich nenne diese Vorgehensweise die Likelihood-Intuition (s. Kap. 4.5.2).

Hypothesenwahrscheinlichkeiten sind subjektiver Natur

Die LikelihoodIntuition

4.2.4 Fehlerquellen in der statistischen Methode Statistische Methoden bedürfen sensibler Anwendung. Es gibt darin sehr viele Fehlerquellen. Sie lassen sich in zwei Hauptgruppen unterteilen: erstens Fehlerquellen bei der Überprüfung und zweitens Fehlerquellen bei der Interpretation von statistischen Hypothesen. Die Fehlerquellen bei der Interpretation sind voralledem von zweierlei Art: erstens werden statistische Hypothesen oft vorschnell kausal interpretiert, und zweitens werden statistische Hypothesen oft vorschnell auf den Einzelfall angewandt; beides ist jedoch nur unter speziellen Voraussetzungen legitim. Wir beschäftigen uns mit diesen Interpretationsfehlerquellen in Kap. 4.3 und 4.4. In diesem Kapitel widmen wir uns Fehlerquellen bei der Überprüfung statistischer Hypothesen. In die Überprüfung einer statistischen Hypothese p(K|A) = r 6¼ p(K|:A), oder Korr(K,A) = r‘, gehen insgesamt drei Voraussetzungen ein: 1.) die AStichprobe soll möglichst repräsentativ sein, 2.) es muss eine A-Kontrollstichprobe existieren, und auch diese soll möglichst repräsentativ sein, und 3.) A-Stichprobe und A-Kontrollstichprobe sollen hinsichtlich der restlichen K-relevanten Merkmale möglichst vergleichbar sein.

Beispiele für Fehlerquellen der Repräsentativität: a) Man macht eine Telefonblitzumfrage zu einem politischen Thema auf dem Festnetztelefon. Damit erreicht man jedoch nicht die ausschließlichen Handy-Telefonierer in der jungen Generation. Die Stichprobe ist nicht repräsentativ. b) Ein Psychologe macht ein Experiment zur Partnerschaftsgestaltung. Er inseriert in der Zeitung. Versuchspersonen (Vpn) melden sich. Welcher Ty-

Nichtrepräsentative Stichproben

143

144

4. Gesetzeshypothesen und ihre empirische Überprüfung

pus von Person meldet sich auf ein solches Inserat? Wenn die Bezahlung der Versuchspersonen nicht sehr gut ist (was sie meistens nicht ist), wird die Stichprobe nur die besonderes an psychologischer Auseinandersetzung Interessierten enthalten. Die Stichprobe ist vermutlich nicht repräsentativ. c) Eine Soziologin schickt zur Erhebung der Hausarbeitsbelastung von Frauen Fragebögen aus. 20 % der Fragebögen werden zurückgeschickt. Welche Frauen füllen den Fragebogen aus und schicken ihn zurück? Womöglich die bildungsmäßig gehobeneren, engagierteren – und sicherlich eher die, die arbeitsmäßig weniger belastet sind. Die Stichprobe ist nicht repräsentativ. d) Weglassen von negativen Fällen: Ein sehr drastischer Repräsentativitätsfehler wird begangen, wenn die gegen die eigene Theorie sprechenden Fälle aus der Stichprobe einfach weggelassen werden. Dieser Fehler findet sich nicht nur in vielen ideologischen Schriften, sondern es ist auch vorgekommen, dass wissenschaftliche Testresultate unter dem Druck der Geldgeber in dieser Weise verfälscht wurden. Beispiele für Fehlerquellen der Kontrollgruppe: Fehlende Kontrollgruppe

Methode der Doppelblindtests

e) Fehlende Kontrollgruppe: Ein häufiger Fehler in nichtwissenschaftlichen Erfahrungsauswertungen ist es, dass ein Vergleich mit einer Kontrollgruppe überhaupt fehlt. Z. B. ermangeln die meisten Berichte über die Erfolge von esoterischen Heilpraktiken einer gezielten Überprüfung durch Kontrollgruppen: wenn z. B. bei einigen Personen nach Anwendung der ,Heilpendelmethode‘ die Krankheit tatsächlich später spürbar nachließ, so sagt dies wenig aus, da man nicht weiß, (i) ob das Nachlassen der Krankheit nicht auch ohne das Heilpendeln, durch normale Gesundung, zustande gekommen wäre, (ii) ob nicht ein Placebo-Effekt im Spiel war (der starke Glauben an die Heilungskraft kann u. U. selbst die Gesundung beschleunigen), sowie (iii) ob der ,Heilpendler‘ der kranken Person nicht zugleich andere Dinge verordnet hat (versteckte Variablen), z. B. gesünder zu leben etc., die er in seinem Bericht nicht erwähnt hat, die aber die eigentlich wirksamen Faktoren waren. All dies kann nur durch geeignete Kontrollgruppenvergleiche ausgeschlossen werden. Ähnliche Beispiele gab es auch in der Wissenschaft. Huff (1956, 22) berichtet über ein derartiges Beispiel: man testete ein Serum gegen Kinderlähmung bei 100 Kindern eines Dorfes, in dem kurz nach der Impfung ein Kinderlähmungsvirus auftrat. Kein Kind erkrankte, und man interpretierte dies als einen überragenden Erfolg. Tatsächlich erkrankte aber auch keines der nichtgeimpften Kinder – der Kinderlähmungsvirus zeigt nämlich nur in seltenen Fällen Wirkung. Heutzutage sind raffinierte Kontrollgruppenvergleiche in der Prüfung der Wirkung pharmazeutischer Medikamente durchgängige Praxis. Es werden sogenannte Doppelblindtests durchgeführt, worin die eine zufällig ausgewählte Hälfte der Vpn ein echtes Medikament und die andere Hälfte eine Pseudo-Pille bekommt, und weder die Patienten noch die Versuchsdurchführer wissen, welche Pillen echt und welche Pillen Placebos sind. Alle bekannten Arten von Verzerrungen der Vergleichbarkeit von Merkmalsgruppe und Kontrollgruppe werden dadurch modulo Zufallsfehlern ausgeschlossen.

4.2 Die empirische Überprüfung von Gesetzeshypothesen

f) Fehlende Vergleichbarkeit mit der Kontrollgruppe: Grundsätzlich sei bemerkt, dass die Vergleichbarkeit der beiden Stichproben bei einer bloßen Stichprobenentnahme-&-Beobachtungsmethode – einem sogenannten Quasiexperiment – nur bezüglich jener Merkmale gewährleistet werden kann, die von A statistisch unabhängig sind; auf diesem Problem beruhen nichtkausale Korrelationen. Eine Vergleichbarkeit beider Stichproben bezüglich aller von A kausal nicht beeinflussbaren Merkmale ist, wenn überhaupt, dann nur durch ein randomisiertes Experiment erreichbar (s. Kap. 4.3.1). Aber auch in diesem Fall können durch das (experimentell eingeführte) Antecedensmerkmal gleichzeitig andere versteckte Variablen in die Stichprobe eingeschleust werden und zu einer Verzerrung der Vergleichbarkeit führen. Ein Beispiel: Eine neue Unterrichtsmethode soll mit einer herkömmlichen Methode verglichen werden, und zwar durch einen Leistungsvergleich zweier Schülergruppen, von denen eine mit der neuen und die andere mit der herkömmlichen Unterrichtsmethode unterrichtet wird. Doch waren die Lehrer in beiden Gruppen auch gleich gut? Sowie: Waren die Lehrer der neuen Unterrichtsmethode vielleicht viel eher motiviert, gut zu unterrichten, als die nach der herkömmlichen Methode unterrichtenden Lehrer? Und: wussten die Schüler vom Experiment, und waren sie, gegeben dieses Wissen, dadurch unterschiedlich motiviert? Alle diese Möglichkeiten würden zu Verzerrungen der Vergleichbarkeit führen und falsche Signifikanzhypothesen zur Folge haben. Auch die Problematik der Interaktion von Untersuchungsleiter und Vpn fällt in die Kategorie der eingeschleusten Variablen. Wie Huff (1956, 15) berichtet, ergab eine Umfrage zum Thema der politischen Rechte von Schwarzen sehr unterschiedliche Ergebnisse, abhängig davon, ob ein Schwarzer oder ein Weißer der Interviewer war. Bekannt ist auch der sogenannte Rosenthal-Effekt, wonach die Beurteilung des Ergebnisses eines Experiments von den Vorerwartungen des Versuchsleiters beeinflusst wird (s. Huber 1987, 161). Rosenthal ließ Studenten (als ,Versuchsleiter‘) eine Reihe von Schülern (als ,Vpn‘) bezüglich ihrer Leistung in einem Test als erfolgreich oder nicht erfolgreich beurteilen, wobei einem Teil der Studenten vorher gesagt wurde, es handle sich um besonders leistungsstarke Schüler, und einem anderen Teil der Studenten, es handle sich um leistungsschwache Schüler. Tatsächlich waren die Schüler, wie in einem Vortest sichergestellt wurde, alle gleich leistungsstark. Die Versuchsleiter mit der Vorinformation „leistungsstark“ beurteilten die Schüler signifikant positiver als die Versuchsleiter mit der Vorinformation „leistungsschwach“. Es gibt aber Methoden, um solche Fehlerquellen möglichst wirksam auszuschalten – man muss sich nur der Mühe unterziehen, diese Methoden anzuwenden. Interaktionen zwischen Versuchsleiter und Vpn kann man ausschalten, indem die Befragung anonym durchgeführt wird. Generell am wirksamsten ist der erwähnte Doppelblindversuch, in dem weder Versuchsleiter noch Vpn wissen, wer zur Merkmalsgruppe und wer zur Kontrollgruppe gehört (Huber 1987, 166 f.).

Versteckte Variablen als Fehlerquellen

Rosenthal-Effekt

145

146

4. Gesetzeshypothesen und ihre empirische Überprüfung

4.3 Korrelation und Kausalität

Probabilistischer Ursachebegriff

Angenommen, zwischen zwei Merkmalen A und B liegt eine signifikante und hinreichend hohe Korrelation vor. Eine solche Korrelation kann zu probabilistischen Voraussagezwecken verwendet werden. Jedoch kann man daraus nicht unmittelbar schließen, dass auch ein Kausalzusammenhang zwischen A und B besteht, und zwar aus zwei Gründen: erstens, weil die Korrelation durch versteckte Variablen bewirkt worden sein kann, und zweitens, weil Korrelationen symmetrisch sind und nicht über die Richtung der Kausalbeziehung Aufschluss geben (s. Hummell/Ziegler 1979, E9 ff.). Dabei gehen wir im folgenden von einem (schwachen) probabilistischen Ursachenbegriff aus: wir nennen ein Ereignis A eine Ursache eines anderen Ereignisses B, wenn A unter Normalbedingungen kausal mitverantwortlich ist für das Eintreten von B und somit die Wahrscheinlichkeit von B erhöht.

4.3.1 Versteckte Variablen

Scheinkausalität

Eine hohe positive Korrelation Korr(A,B) kann auch eine sogenannte ,Scheinkorrelation‘ sein: zwischen A und B besteht in diesem Fall überhaupt keine kausale Beziehung, vielmehr sind beide Merkmale die Wirkung einer versteckten gemeinsamen Ursache C, und die Korrelation zwischen A und B kommt nur durch die Kausalbeziehung C ! B und C ! A zustande. Statt von einer ,Scheinkorrelation‘ wollen wir jedoch besser von einer Scheinkausalität sprechen, denn die Korrelation besteht in diesem Fall ja wirklich, und scheinbar ist lediglich der Kausalzusammenhang zwischen A und B. Die Situation ist in Abb. 4.3-1 graphisch dargestellt. Kausalbeziehungen (!) sind gerichtet, von der Ursache zur Wirkung; Korrelationen (- - -) sind dagegen ungerichtet bzw. symmetrisch.

A

Korrelation durch gemeinsame Ursachen

B

Kausalbeziehung Korrelation

C

Abb. 4.3-1: C ist gemeinsame Ursache von A und B

Reichenbach (1956, 159) nannte die Beziehung der gemeinsamen Ursache auch konjunktive Gabel (s. auch Salmon 1984, 158). Das Zustandekommen der Korrelation zwischen A und B lässt sich informell so erklären: Weil Korrelationen symmetrisch sind, erhöht das Eintreten der Wirkung A auch die Wahrscheinlichkeit, dass die Ursache C eingetreten ist. C erhöht aber zugleich Bs Eintrittswahrscheinlichkeit (unabhängig von A). Daraus folgt wahrscheinlichkeitstheoretisch, dass auch A B’s Eintrittswahrscheinlichkeit erhöht. Hier einige Beispiele: (4.3-1) Zwischen dem jähen Fall des Barometers (A) und dem baldigen Aufkommen eines Sturms (B) besteht eine hohe Korrelation (s. Grünbaum 1972, 309).

4.3 Korrelation und Kausalität

Da das Aufkommen des Sturms zeitlich nach dem Barometerfall eintritt, könnte man meinen, der Barometerfall wäre die Ursache des herannahenden Sturmes. Dies ist natürlich Unsinn: tatsächlich sind beide Ereignisse die gemeinsame Wirkung eines dritten Ereignisses C, nämlich des jähen Druckabfalls in der Atmosphäre.

Beispiele für Scheinkausalität aufgrund gemeinsamer Ursachen

(4.3-2) Zwischen dem Grad der positiven Einstellung zur Firma (A) und der psychischen Gesundheit von Arbeitern (B) wurde eine hohe positive Korrelation gemessen. Heißt dies, dass jene Arbeiter, die ständig über die Firma nörgeln, dies deshalb tun, weil sie psychische Probleme haben, die sie auf die Firma schieben, während die Firma unschuldig ist? Die statistische Analyse versteckter Variablen zeigte ein anderes Bild. Es stellte sich nämlich heraus, dass beide Eigenschaften eine gemeinsame Wirkung der versteckten Ursache der Belastung am Arbeitsplatz (C) waren: die Arbeiter mit den schlechten Arbeitsbedingungen klagten mehr über die Firma, und zugleich litt ihre psychische Gesundheit durch die fortwährende Belastung. Das Beispiel stammt von Lazarsfeld (s. Mayntz et al. 1974, 200 f.) und bezieht sich auf eine in den 1950er Jahren durchgeführte Untersuchung. Bekannt ist auch der Statistik-Kalauer über die Mär vom Storch, der die Kinder bringt. Tatsächlich besteht zwischen der Häufigkeit von Störchen und der von Neugeborenen eine positive Korrelation. Diese Scheinkausalität besitzt als gemeinsame Ursache die Variable Landregion versus städtische Region: am Land werden mehr Kinder geboren, und dort gibt es auch mehr Störche. Es gibt eine bekannte Methode, um Scheinkausalität statistisch aufzudecken, die auf Reichenbach (1956, 159) zurückgeht (s. auch van Fraassen 1980, 121; Eells 1991, 60 f.; Salmon 1984, 159 f.). Wenn die Korrelation zwischen A und B auf eine gemeinsame Ursache C zurückzuführen ist, dann muss die Korrelation zwischen A und B bei festgehaltenen Werten der Variable C verschwinden: wenn man also nur Individuen betrachtet, die das C-Merkmal besitzen, dann erhöht das zusätzliche Vorliegen von B nicht mehr die Wahrscheinlichkeit von A; und analog, wenn man nur Individuen betrachtet, die das Merkmal C nicht besitzen. In unserem Beispiel (4.3-2): wenn man nur die Arbeiter unter guten Arbeitsbedingungen untersucht, dann sollte die Korrelation zwischen Einstellung zur Firma und psychischer Gesundheit verschwinden; und dasselbe sollte passieren, wenn man nur Arbeiter unter schlechten Arbeitsbedingungen untersucht. Diese Prognose wurde in der Tat bestätigt. Reichenbach machte einen Vorschlag, wie man gemeinsame Ursachen durch rein statistische Bedingungen charakterisieren kann, welche man die Bedingungen der Abschirmung nennt, d. h. C schirmt B von einem probabilistischen Einfluss auf A ab:

Probabilistische Abschirmung nach Reichenbach

147

148

4. Gesetzeshypothesen und ihre empirische Überprüfung

(Ms. 4.3-1) Reichenbach-Bedingungen für „C ist gemeinsame Ursache für A und B“: (1) p(A|B^C) = p(A|C) H C und :C schirmen A von B ab j (2) p(A|B^:C) = p(A|:C)h bzw.: A wird durch XC von B total abgeschirmt (3) p(A|C) > p(A) H j C ist sowohl für A wie für B positiv relevant (4) p(B|C) > p(B) h Aus (1) – (4) folgt (5): p(A|B) > p(A), d. h. B korreliert positiv mit A (zum Beweis, der oben informell erklärt wurde, s. Reichenbach 1956, 160).

Intervenierende Variable

Reichenbach wählte die konjunktive Formulierung: (1‘) p(A^B|C) = p(A|C)Np(B|C), (2‘) p(A^B|:C) = p(A|:C)Np(B|:C), (3‘) p(A^C) > p(A)Np(C), (4‘) p(B^C) > p(B)Np(C), welche unter der Annahme p(B^:C), p(B^C) > 0 mit (Ms. 4.3-1) äquivalent ist. Die konjunktive Formulierung macht deutlich, dass die Abschirmung symmetrisch ist bzgl. der voneinander abgeschirmten Ereignisse: lC schirmt A von B ab g.d.w. lC B von A abschirmt. Ein bekanntes Problem der Reichenbachschen Bedingungen für Abschirmung liegt darin, dass damit nicht zwischen einer gemeinsamen Ursache C und einer kausal intervenierenden Variablen C unterschieden werden kann – in beiden Fällen sind alle Bedingungen von (Ms. 4.3-1) erfüllt. In letzterem Fall fungiert C als Mittlerursache zwischen A und B: C ist direkte Wirkung von B und direkte Ursache für A. Direkte Ursachen schirmen ihre Wirkung von indirekten Ursachen im Reichenbachschen Sinn ab. Die beiden Fälle sind daher probabilistisch ununterscheidbar. Im Fall der intervenierenden Variablen ist B jedoch eine indirekte Ursache für A. Welcher Fall vorliegt, kann nur durch zusätzliche Kausalinformation entschieden werden, A

B

A

(gemeinsame Ursache)

B (intervenierende Variable)

C C Abb. 4.3-2: Gemeinsame Ursache vs. intervenierende Variable

z. B. über ein Experiment (s. unten), oder durch Wissen über die Kausalrichtung, die im Fall der gemeinsamen Ursache von C nach B, aber im Fall der intervenierenden Variablen von B nach C geht. Ein Beispiel: (4.3-3) Zwischen dem Familienstand berufstätiger Frauen (B = verheiratet) und der Häufigkeit ihres Fernbleibens von der Arbeit (A) maß man eine hohe Korrelation. Beispiel für Intervenierende Variable

Sind verheiratete Frauen also fauler? Keineswegs: als intervenierende Variable stellte sich das Ausmaß zusätzlicher Belastung durch Hausarbeit heraus (C). B bewirkt ein hohes Ausmaß an C = zusätzliche Hausarbeit, und deshalb bleiben verheiratete Frauen häufiger im Betrieb von der Arbeit fern (hohes A). Durch diese Einsicht ändert sich die Interpretation des Beispiels schlagartig: verheiratete Frauen bleiben im Schnitt nicht deshalb

4.3 Korrelation und Kausalität

häufiger von der Arbeit fern, weil sie weniger arbeiten, sondern weil sie mehr arbeiten (das Beispiel stammt von Zeisel; s. Mayntz et al. 1974, 203 f.). Reichenbachs Bedingungen der gemeinsamen Ursache sind in der statistischen Praxis enorm nützlich, weil sie Kausalhypothesen bestätigen oder widerlegen können. Dies illustriert das folgende Beispiel von Cartwright (1979; s. Eells 1991, 62 f.). In einer amerikanischen Universität stellte man fest, dass bei weiblichen Studenten (=W) die Aufnahmequote an der Universität (=A) niedriger ist als bei männlichen (es gab hier Aufnahmeprüfungen). Es wurden folgende Kausalhypothesen aufgestellt und durch die Untersuchung von Hintergrundvariablen widerlegt: 1. Hypothese: Frauen sind weniger intelligent und fallen deshalb bei den Aufnahmeprüfungen häufiger durch. Träfe diese Hypothese zu, so müsste die Korrelation verschwinden, wenn man den IQ konstant hält. M.a.W., wenn man Frauen mit gleichintelligenten Männern vergleicht, dürfte zwischen den beiden in der Aufnahmequote kein Unterschied mehr bestehen. Tatsächlich war aber Korr(A,W| IQ=konstant) weiterhin positiv. Der Intelligenzquotient schied damit als intervenierende Variable aus. 2. Hypothese: Das Universitätssystem bevorzugt männliche Bewerber. Hier liegt der typische Fall einer makrosoziologischen Hypothese vor, die man schwer untersuchen kann, weil man die Variable der patriarchalischen Universitätsstruktur nicht experimentell variieren kann. Dennoch zeigte eine weitere statistische Information, dass auch diese zweite Vermutung nicht Ursache dieser Korrelation war, denn: 3. Hypothese: Man beobachtete, dass Frauen, verglichen zu Männern, zur Inskription in den beliebteren Fächern tendieren (wie z. B. Psychologie oder Sprachen), d. h. zur Inskription in jenen Fächern, die wegen der hohen Nachfrage strengere Aufnahmeprüfungen und niedrigere Aufnahmequoten besitzen. Als man die Variable des Studienfaches konstant hielt, verschwand die Korrelation zwischen Geschlecht und Aufnahmequote völlig – d. h. in jedem einzelnen Fach ergab sich keinerlei Unterschied mehr zwischen der durchschnittlichen Aufnahmequote von männlichen und weiblichen Kandidaten. Damit stellt sich die Wahl des Studienfachs als intervenierende Variable heraus und der Fall war geklärt: Frauen tendieren häufiger zur Inskription von gewissen überbelegten Fächern, was Ursache für die niedrigeren Aufnahmequoten ist. Trotz ihrer praktischen Leistungskraft sind die Reichenbachschen Bedingungen einer Reihe von Problemen ausgesetzt. Abgesehen vom erläuterten Ununterscheidbarkeitsproblem zwischen gemeinsamer Ursache und intervenierender Variable sind dies voralledem folgende zwei: (a) Reichenbachs Abschirmungsbedingungen schließen nicht aus, dass die Korrelation zwischen C und B, sowie zwischen C und A, selbst pseudokausaler Natur sind, also jeweils durch eine weitere gemeinsame Ursache (D bzw. E) zustande kommen. Generell verlangt die probabilistische Methode der Ausschließung von Scheinkausalitäten, dass man alle möglicherweise relevanten Hintergrundparameter untersucht und prüft, ob durch sie die gemessene Korrelation zwischen A und B zum Verschwinden gebracht wird. Für die kausale Interpretation von statistischen Korrelationen ist daher

Beispiel von Cartwright

Probleme des Abschirmungskriteriums

149

150

4. Gesetzeshypothesen und ihre empirische Überprüfung

Humes Kausalitätskritik

Ausschluss von Scheinkausalität im randomisierten Experiment

ein hohes Maß an empirischen und theoretischen Hintergrundinformationen nötig – Kausalhypothesen sind damit sehr vorläufiger Natur. (b) Reichenbachs Bedingungen liefern in deterministischen Situationen, wo alle bedingten und inversen bedingten Wahrscheinlichkeiten 1 oder nahezu 1 sind, inadäquate Resultate (dies zeigt die Kritik von Otte 1981 an Suppes 1970). Die Bedingungen würden in diesem Fall zur falschen Konklusion führen, dass sowohl C A und B voneinander ,abschirmt‘, wie auch B A und C voneinander ,abschirmt‘. Es ist eine tiefe wissenschaftsphilosophische Frage, ob und wie weit man kausale Relationen auf empirisch überprüfbare Relationen wie z. B. statistische Beziehungen zurückführen kann. Man mag sich fragen, warum man dies überhaupt versuchen sollte. Hinter diesen Versuchen steht die große Herausforderung von David Hume (1748, 82 ff.): Hume hatte argumentiert, dass hinter unseren Kausalbehauptungen nichts anderes steckt als die Beobachtung von regelmäßigen Ereignisfolgen. Ein Kausalitätsbegriff, der darüber hinaus geht, sei ein empirisch gehaltloser metaphysischer Ballast. Wissenschaftstheoretiker sind sich darin weitgehend einig, dass Kausalbehauptungen nicht einfach, so wie es die Humesche Analyse impliziert, auf strikte oder statistische Korrelationen reduzierbar sind – sie sind mehr als das. Das wissenschaftstheoretische Problem besteht darin, dieses kausale Mehr durch erfahrungswissenschaftliche Kriterien einzulösen, ohne in metaphysische Spekulationen zurückzufallen. Reichenbachs Abschirmungskriterium ist ein dahingehender Vorschlag, der allerdings nur begrenzt funktioniert. Mehr in Kap. 6.5.2. Wir erläutern nun, warum in randomisierten Experimenten Scheinkausalität modulo Zufallsfehlern aufgedeckt werden kann. In einem solchen Experiment wird eine Stichprobe von Individuen zufällig in zwei Stichproben aufgeteilt, und erst danach wird der einen (experimentellen) Gruppe der Faktor A durch externen Eingriff aufgeprägt, während in der Kontrollgruppe der Faktor A nicht oder nur zufälligerweise manchmal realisiert ist. Anschließend wird die bedingte Häufigkeit von B in der experimentellen Gruppe, he(B) = he(B|A), mit der bedingten Häufigkeit hk(B) in der Kontrollgruppe verglichen. Alle restlichen Faktoren (C, D, …, ), die nicht zur kausalen Wirkung von A gehören, sind nun in experimenteller Gruppe und Kontrollgruppe modulo Zufallsfehlern statistisch gleich verteilt, denn die Aufteilung erfolgte ja zufällig. Durch den experimentellen Eingriff wird somit der Ereignistyp A von allen anderen Variablen, die nicht zur kausalen Wirkung von A gehören, entkoppelt – man nennt diesen Eingriff auch eine kausale Intervention (s. Pearl 2000, 23; Kap. 6.5.2.3). Sollte die Korrelation zwischen A und B in der Population auf irgendeine (möglicherweise unbekannte) gemeinsame Ursache C zurückzuführen sein, dann kann es im randomisierten Experiment zu keiner Korrelation zwischen A und B mehr kommen. Man zeigt dies wie folgt. Wenn es in der Population eine gemeinsame Ursache C für A und B gäbe, welche immer diese sei, dann muss aufgrund der Reichenbach-Bedingungen (Ms. 4.3-1) (1,2) gelten (i) he(B|C) ⁄ hk(B|C) und he(B|: C) ⁄ hk(B|: C), denn bei festgehaltenem C-Wert übt A keinen kausalen Einfluss auf B aus (dabei steht ⁄ für „ungefähr gleich, modulo Zufallsfehlern“). Weil C keine

4.3 Korrelation und Kausalität

Wirkung von A ist (nur umgekehrt), muss C in experimenteller und Kontrollgruppe modulo Zufallsfehlern gleich häufig sein, d. h. es gilt (ii) he(C) ⁄ hk(C) und he( : C) ⁄ hk( : C). Aufgrund des Multiplikationsgesetzes (TB3 von Kap. 3.10.4.1) gilt: (iii) he(B) = he(B|C)Nhe(C) + he(B|:C)Nhe(:C) (iv) hk(B) = hk(B|C)Nhk(C) + hk(B|:C)Nhk(:C) Durch Einsetzen der ungefähren Gleichheiten (i) und (ii) in die Gleichungen (iii) und (iv) resultiert he(B) ⁄ hk(B). Ein randomisiertes Experiment kann (im Gegensatz zu einer statistischen Erhebung, d. h. einem Quasi-Experiment) also zeigen, dass modulo Zufallsfehler von A nach B eine Kausalbeziehung führt. Das zeigt noch nicht, ob es sich um eine direkte oder indirekte Kausalbeziehung handelt. Intervenierende Variablen, d. h. durch den Stimulus A miteingeschleuste Variablen C können durch ein Experiment nicht bzw. nur aufgrund zusätzlicher Hintergrundannahmen ausgeschlossen werden. Abschließend sei erwähnt, dass es – über Scheinkausalität hinaus – auch den Fall der kausalen Scheinunabhängigkeit und der scheinnegativen Kausalität gibt: hier misst man zunächst eine Nullkorrelation oder negative Korrelation zwischen A und B, erhält bei Einführung einer Hintergrundvariablen XC dann aber sowohl im Fall X = C wie im Fall X = :C eine positive Korrelation zwischen A und B. Diese prima facie ,paradoxale‘ Situation hat folgende qualitative Erklärung:

+ A

C

 + B

+

Ursache



Gegenursache

Abb. 4.3-3: Scheinunabhängigkeit – Korr(A,B) = 0

A hat einen direkten positiven Kausaleinfluss auf B, aber einen indirekten negativen Kausaleinfluss auf B, denn A aktiviert zugleich den Faktor C, der eine Gegenursache für B ist. Im Fall der Scheinunabhängigkeit kompensieren sich diese probabilistischen Effekte genau auf Null. Hält man aber die Variable XC fest, so wird der Kausalpfeil von A nach C unwirksam und der positive Kausalpfeil von A nach B bewirkt eine positive Korrelation. D. h., es gilt p(B|A^C) > p(B|C) und p(B|A^:C) > p(B|:C), obwohl p(B|A) = p(B) gilt. (4.3-4) Beispiel für Scheinunabhängigkeit (nach van Fraassen 1980, 149): in einer Population von US-Amerikanern findet man erstaunlicherweise keine Korrelation zwischen Rauchern (R) und Herz-Kreislaufbeschwerden (H). Es stellt sich heraus, dass Raucher, zur Kompensation ihrer Rauchsünden, dort viel häufiger Sport (S) betreiben als Nichraucher. Betrachtet man nur Sportler, oder nur Unsportliche, so ergibt sich in beiden Subpopulationen eine positive Korrelation zwischen R und H. Ist der indirekt-negative Effekt von A auf B stärker als der direkt-positive Effekt, dann liegt p(B|A) < p(B) und gleichzeitig p(B|A^C) > p(A|C) sowie

Scheinunabhängigkeit

Scheinnegative Kausalität

151

152

4. Gesetzeshypothesen und ihre empirische Überprüfung

Simpson-Paradox

p(B|A^:C) > p(B|:C) vor. Ich spreche hier von scheinnegativer Kausalität. Fälle von Scheinkausalität, Scheinunabhängigkeit und scheinnegativer Kausalität werden unter der Bezeichnung Simpson-Paradox zusammengefasst (van Fraassen 1980, 148 f.; Cartwright 1983, 24; Eells 1991, 66-69). Dennoch liegt zwischen Scheinkausalität und Scheinunabhängigkeit (bzw. scheinnegativer Kausalität) folgender Unterschied vor: bei der Scheinkausalität übt A auf B gar keinen kausalen Einfluss aus. Bei der Scheinunabhängigkeit (bzw. scheinnegativen Kausalität) übt A dagegen einen direkten positiven Kausaleinfluss zusammen mit einem indirekten negativen Kausaleinfluss auf B aus.

4.3.2 Kausalrichtung Selbst in dem eher unrealistischen Fall, in dem man weiß, dass außer A und B keine weiteren versteckten Variablen mehr im Spiel sind, kann man aus dem Vorliegen einer hohen Korrelation zwischen A und B nicht direkt eine Kausalhypothese gewinnen, weil durch die Korrelation noch nicht die Kausalrichtung festgelegt ist – es ist also noch nicht festgelegt, was als Ursache und was als Wirkung anzusehen ist. Wie wir wissen, sind Korrelationen symmetrisch. Wenn also zwischen zwei Merkmalen A und B eine positive Korrelation besteht, ohne dass diese auf den kausalen Einfluss von dritten Variablen zurückzuführen ist, dann gibt es immer noch zwei einfache Möglichkeiten und eine komplexe Möglichkeit: entweder A ist eine Ursache von B, oder B ist eine Ursache von A, oder aber drittens, A und B wirken in Form einer kausalen Rückkopplung wechselseitig aufeinander ein und verstärken sich (s. Abb. 4.3-4). Man beachte, dass auch in einer kausalen Rückkopplungsschleife die Kausalbeziehung nicht symmetrisch wird, sondern gerichtet bleibt: die Kausalwirkung von A nach B ist in diesem Fall eine andere als die von B nach A.

A

B

A

B

A

B

A

B

Abb. 4.3-4: Wie verläuft die Kausalrichtung? ................ Korrelation, fi Kausalbeziehung

Hier einige Beispiele, die aus tatsächlichen Untersuchungen stammen: Beispiele für die Frage der Kausalrichtung

(4.3-5) Die Höhe des IQ und die Höhe des sozialen Status sind positiv korreliert. Die entscheidende Frage lautet: was ist hier Ursache, was Wirkung? Ist IQ Produkt der Erziehung und Bildung, welche vom Status abhängt? Oder denkt der Verfechter der Korrelation an eine genetische Elite? Offenbar hängt die weltanschauliche Relevanz dieser Korrelation von der Hypothese über die Kausalrichtung ab, die man keinesfalls schon aus der Korrelation herauslesen kann.

4.3 Korrelation und Kausalität

(4.3-6) Aggressive Personen sehen gerne aggressive Filme. Dies ist ein bekanntes Resultat. Die wichtige Frage aber lautet: Werden Personen durch aggressive Filme aggressiv? Oder sehen aggressive Personen gerne aggressive Filme, weil sie sich dabei kompensatorisch abreagieren? Wieder hängt die normativ-politische Relevanz von der Kausalrichtung ab: im ersten Fall sollte man dem Sehen aggressiver Filme entgegenwirken, im zweiten Fall sollte man es tolerieren oder gar fördern. (4.3-7) Interesse für Computer korreliert mit Desinteresse an sozialen Beziehungen. Auch bei dieser von Pflüger und Robert Schurz (1987) belegten Beziehung lautet die wichtige Frage: Geht die Kausalrichtung von rechts nach links – versuchen die sozial Beziehungsunfähigen ihren Mangel durch intensive Beschäftigung mit technischen Dingen wie Computern auszugleichen? Das wäre die ,tiefenpsychologische‘ Deutung, welche von Pflüger/Schurz vertreten wird. Oder geht die Kausalrichtung von links nach rechts – ist die Sache einfach so, dass die meisten Computerbeschäftigten durch ihre Arbeit so erfüllt werden, dass sie aus natürlichen Gründen ihr Engagement in sozialen Beziehungen reduzieren? Dies wäre die ,rationale‘ Deutung. Man ist bei solchen Beispielen schnell geneigt, eine Korrelation in der Richtung herauszulesen, die der eigenen Auffassung entspricht. Echte wissenschaftliche Haltung kann nur in Zurückhaltung bzgl. der Kausalrichtung bestehen, sofern man nicht zusätzliche Gründe für die Kausalrichtung anführt. Solche Gründe können z. B. aus naturwissenschaftlichem Hintergrundwissen über Kausalmechanismen resultieren. Im Fall von Beziehungen zwischen zeitlich getrennten Ereignissen muss das frühere Ereignis die Ursache des späteren sein, denn Kausalbeziehungen sind zeitlich vorwärtsgerichtet. Schwieriger wird es bei zeitlich nicht lokalisierten Dispositionsmerkmalen wie in den Beispielen (4.3-5, 6, 7). Es gilt hier herauszufinden, was die unabhängige und was die abhängige Variable ist (s. Kap. 6.5.2.3), bzw. ob eine Rückkopplung vorliegt. Auch bei Kausalbeziehungen zwischen koexistenten Dispositionsmerkmalen gibt es Fälle, die einfach beantwortbar sind, z. B.: (4.3-8) Geschlecht korreliert mit gewissen geschlechtspezifischen Merkmalen. (4.3-9) Luftverschmutzung korreliert mit Häufigkeit von Atemwegserkrankungen. Im Fall (4.3-8) muss (wenn überhaupt ein Kausalzusammenhang vorliegt) das Geschlecht die Ursache sein, weil es biologisch vorgegeben ist. Im Fall (4.3-9) ist aufgrund naturwissenschaftlichem Hintergrundwissen bekannt, dass der Kausalprozess nur von Luftverschmutzung zu Atemwegserkrankung führen kann, aber nicht umgekehrt. In den Beispielen (4.3-5, 6, 7) ist jedoch die Frage der Kausalrichtung sehr fraglich, sofern überhaupt eine Kausalbeziehung besteht, und die Korrelation nicht durch eine dritte Variab-

Hintergrundwissen über Kausalmechanismen

153

154

4. Gesetzeshypothesen und ihre empirische Überprüfung

Beispiele für voreilige Kausalschlüsse in den Medien

Zeitliche Längsschnittstudien

le zustande kommt. Ein abschließendes Beispiel für eine folgenschwere Fehlinterpretation der Kausalrichtung einer Korrelation ist der Fall des Bankiers John Law, der aus der Korrelation zwischen der Menge des im Umlauf befindlichen Papiergeldes und dem Reichtum Frankreichs schloss, man könne durch In-Umlauf-Setzen von ungedecktem Papiergeld den Reichtum Frankreichs erhöhen. 1720 führte dies zum Zusammenbruch der französischen Finanzwirtschaft. Die Kritik von fragwürdigen Kausalinterpretationen statistischer Befunde hat auch eine medienkritische Komponente. In den Medien werden statistische Korrelationen häufig vorschnell als Kausalbefunde mit ,Sensationswert‘ gedeutet. Hier einige Beispiele: (1.) Langes Stillen senkt das Brustkrebsrisiko der Frau (Österreichische Krone 23. 7. 2002). Besteht ein Kausalzusammenhang? Oder ist es nur so, dass Frauen mit genetisch bedingt leistungsfähigeren Brüsten länger stillen können und zugleich ein geringeres Brustkrebsrisiko besitzen? (2.) Vor einigen Jahren wurde berichtet: häufiger Hautkontakt korreliert mit einem stärkeren Immunsystem. Es fragt sich: liegt ein direkter Kausalzusammenhang vor, oder sind bloß gesündere Menschen mit stärkerem Immunsystem durchschnittlich lustbetonter? (3.) Frauen, die viel joggen, haben eine geringere Knochendichte (Salzburger Nachrichten 3. 2. 2003). Sollte häufiges Joggen wirklich zum Abbau der Knochendichte führen? Oder ist es bloß so, dass Menschen mit genetisch bedingt geringerer Knochendichte meistens auch eine bessere Veranlagung zur Lauffähigkeit besitzen und daher häufiger laufen? (4.) Männer, die viel Schokolade essen, sind sanfter und sozial umgänglicher (Bayrisches Fernsehen 27. 2. 2003). Sollte es wirklich einen Kausalzusammenhang geben, und wenn ja, in welcher Richtung? Oder sollte nicht eher eine gemeinsame Ursache, evtl. ein genetisch bedingter Charaktertyp sowohl für Sanftheit wie für Tendenz zu mehr Süßem beim Essen wirksam sein? Sollte man so dumm sein und aus dieser Korrelation die Empfehlung ableiten, Männer sollten mehr Schokolade essen – was den Cholesterinspiegel erhöht und ihre Lebenserwartung senkt? Die Liste ließe sich fortführen. Selbstverständlich können Kausalinterpretationen von Korrelationen auch äußerst seriös sein. Es gibt aufwendige Methoden, um näher zu prüfen, ob und in welcher Richtung ein Kausalzusammenhang vorliegt – z. B. durch zeitliche Längsschnittstudien. Die meisten Untersuchungen basieren aber auf Querschnittstudien, und oft wird daraus ungerechtfertigt auf eine Kausalbeziehung geschlossen, die nicht selten Politiker veranlasst, Maßnahmen zu ergreifen, die dann etwas ganz anderes bewirken, als was sie sich davon erhoffen.

4.4 Die Anwendung statistischer Hypothesen auf den Einzelfall Aus der Nichtmonotonie bedingter Wahrscheinlichkeiten (Kap. 2.6.4) ergibt sich im Vergleich zu strikten Hypothesen ein entscheidender Unterschied für die Anwendung von statistischen Hypothesen zum Zwecke der Prognose oder Erklärung von Einzelfällen. Die Konklusion Ka eines deduktiven Schlusses mit wahren Prämissen 8x(Ax! Kx) und Aa darf man jederzeit ab-

4.4 Die Anwendung statistischer Hypothesen auf den Einzelfall

spalten – man kann von der Wahrheit der Prämissen auf die Wahrheit der Konklusion schließen, ohne sich darum kümmern zu müssen, was sonst noch wahr ist. Aus den Prämissen p(Kx|Ax) = 90 % und Aa eines induktivstatistischen Spezialisierungsschlusses darf jedoch nur dann mit subjektiver Glaubenswahrscheinlichkeit von 0,9 auf Ga geschlossen werden, wenn die in Kap. 3.9 erläuterte Bedingung der engsten (relevanten) Referenzklasse gewährleistet ist: d. h., die Antecedensinformation A muss die gesamte statistisch für K relevante Information über das fragliche Individuum a umfassen. Denn auch wenn p(Kx|Ax) hoch ist, kann p(Kx|Ax^A*x) niedrig sein, und wenn sowohl A und A* auf das fragliche Individuum a zutreffen, dann darf nur das engere Prädikat A^A* als Antecedensprädikat einer Prognose über das Individuum a herangezogen werden. Beispielsweise heilt Penicillin (A) bei den allermeisten Menschen eine schwere Erkältung (K), aber bei Menschen, die gegenüber Antibiotika allergisch sind (A*), hätte eine Penicillintherapie fatale Folgen. Dieser Zusammenhang hat für die Anwendungspraxis drastische Konsequenzen (s. Schurz 2001b, § 4). Wenn z. B. statistische Untersuchungen ergeben, dass eine Unterrichtsmethode in 80 % aller Fälle Vorteile bringt, und wir dieses Wissen dann auf den Schüler Peter anwenden, so müssen wir genau prüfen, ob es nicht spezifischere Informationen gibt, die den Wahrscheinlichkeitswert dieses statistischen Gesetzes unterlaufen. Es könnte z. B. sein, dass die neue Unterrichtsmethode nur visuellen Lerntypen Vorteile bringt, zu denen insgesamt 80 % aller Schüler zählen, dass aber Peter zufällig ein auditiver Lerntyp ist, der in die restlichen 20 % fällt. Die Anwendung des Gesetzes auf Peter wäre dann nicht nur illegitim; sie würde ihm praktisch nicht Nutzen, sondern Schaden bringen. In diesem Sinn seien alle Anwender davor gewarnt, statistische Befunde vorschnell auf die von ihnen betreuten Einzelfälle anzuwenden, ohne zuvor gründlich zu prüfen, ob es weitere relevante Merkmale ihrer Einzelfälle gibt, welche die Wahrscheinlichkeit verändern und ein ganz anderes Bild liefern. Wenn z. B. eine statistische Studie ergibt, dass Landkinder im Schnitt glücklicher sind als Stadtkinder und weniger nervöse Störungen aufweisen, so wäre dem Stadtelternpaar Meier trotzdem nicht einfach zu raten, auf das Land zu ziehen, auch wenn sie es sich finanziell leisten können, ohne genau zu prüfen, welche weiteren Folgen mit einem solchen Umzug auf das Land im speziellen Einzelfall ihrer Kinder verbunden wären. Es könnte z. B. sein, dass ihre Kinder dadurch derart nachhaltig aus ihrem sozialen Freundeskreis herausgerissen werden, dass ihnen der Umzug aufs Land trotz idyllischer Umgebung mehr schadet als nutzt. Insbesondere seien Politiker davor gewarnt, statistische Mehrheitsbefunde vorschnell in Form eines für alle Menschen bindenden Gesetzes umzuwandeln.

Die Bedeutung des Wissens über den Einzelfall

155

156

4. Gesetzeshypothesen und ihre empirische Überprüfung

4.5 Weiterführende Themen 4.5.1 Statistische Methoden für quantitative Variablen

Zufallsvariable

Eine quantitative Variable (Funktion) X ordnet jedem Individuum a der Population D einen Zahlenwert r zu, X(a) = r. Ein Beispiel für X wäre das Gewicht von Personen einer Population D. Man nennt X auch eine Zufallsvariable (Lauth/Streiter 2002, 255 – 257; Hays/Winkler 1970, 103 ff.). Über den Zahlenwerten von X existiert eine Wahrscheinlichkeitsverteilung p. Kann X nur endlich (oder abzählbar) viele Zahlenwerte einnehmen, so heißt p eine diskrete Verteilung; kann X alle Zahlenwerte eines reellwertigen Zahlenintervalls [r1,r2] := {r2IR: r1  r  r2} annehmen, so heißt p eine kontinuierlichen Verteilung. Notation: Es steht p(r) abkürzend für p(X(x)=r) und p([a,b]) für p(X(x)2[a,b]). In Kap. 3.10.4.1 haben wir die grundlegendste diskrete Verteilung der Statistik kennengelernt: die Binomialverteilung. Nun betrachten wir die grundlegendste kontinuierliche Verteilung der Statistik, die (Gauß’sche) Normalverteilung p(r) bzw. d(r) mit der typischen Glockenform, dargestellt in Abb. 4.5-1. p(r) bzw. d(r)

Mittelwert P(X) Streuung V(X)

Größe von X: r

Abb. 4.5-1: Normalverteilung p(r) (diskret) bzw. d(r) (kontinuierlich)

Kontinuierliche Verteilung und Wahrscheinlichkeitsdichte

Die Wahrscheinlichkeit p(r) verwendet man bei diskreten Normalverteilungen: ist X z. B. das Gewicht, dann steht p(50) für die Häufigkeit von Personen, deren Gewicht aufgerundet 50 kg beträgt. Kontinuierliche Verteilungen haben folgende Besonderheit: die Wahrscheinlichkeit, dass das quantitative Merkmal X genau einen von überabzählbar vielen möglichen reellen Zahlenwerten einnimmt, ist typischerweise immer null. Was interessiert, sind die Wahrscheinlichkeiten von nichtverschwindenden Intervallen, z. B. „Gewicht zwischen 50 und 51 kg“, die typischerweise positiv sind. Man kann die Wahrscheinlichkeitsverteilung über einer reellen Zahlengerade daher nicht durch die Wahrscheinlichkeiten p(r) selbst darstellen, denn dann erhielte man die triviale Nullgerade. Man behilft sich stattdessen mit der sogenannten Wahrscheinlichkeitsdichte d(r). Mathematisch ergibt sich die Wahrscheinlichkeit, den X-Wert in einem Intervall zu finden, durch das Integral der Wahrscheinlichkeitsdichte d(r) über dieses Intervall, geschrieben als Rb p([a,b]) = dðrÞdr (zur Maß- und Integrationstheorie s. z. B. Bauer 1978; Cara

nap/Jeffrey 1971, 167 – 224). Anschaulich ist dieses Integral, also die Wahrscheinlichkeit p([a,b]), durch die Fläche unter der Verteilungskurve d(r) zwi-

4.5 Weiterführende Themen

schen a und b gegeben; die Gesamtfläche unter der Verteilungskurve ist auf 1 normiert. Im diskreten Fall steht statt dem Integral die gewöhnliche Summe „R“: p([a,b]) = Raxb p(x). Diese Zusammenhänge gelten natürlich nicht nur für die Normalverteilung, sondern für beliebige Verteilungen – z. B. uniforme Verteilungen, welche jedem Zahlenwert dieselbe Dichte von 1 zuweisen, eingipfelige aber schiefe, oder mehrgipfelige Verteilungen. Die wichtigsten statistischen Parameter einer Verteilung p(X) sind ihr Mittelwert (arithmetisches Mittel) l(X) und ihre Streuung r(X). Diese Parameter sind für beliebige Verteilungen definiert. Für die Normalverteilung sind l(X) und r(X) in Abb. 4.5-1 eingezeichnet. Der Mittelwert l(X) ist der Durchschnittswert der Größe X unter Individuen der Population D. Bei symmetrisch-eingipfeligen Verteilungen koinzidiert der Mittelwert mit dem häufigsten Wert, dem sogenannten Modalwert der Verteilung. Die Streuung r(X) informiert über die durchschnittliche Abweichung der individuellen XAusprägungen vom Mittelwert; weil die Summe der gerichteten Abweichungen l(ri – l(X)) vom Mittelwert sich auf Null kompensieren würde, summiert man die Abweichungsquadrate, deren Summe man die Varianz v(X) nennt, und bildet daraus die Wurzel. Je größer die Streuung, desto flacher ist eine Verteilung. Mathematisch sind die Begriffe wie folgt definiert:

Mittelwert, Varianz und Streuung

(Def. 4.5-1) Diskreter Fall: X(x) 2 {r1, …, rn}: Kontinuierlicher Fall: X(x) 2 IR Mittelwert: l(X) =

n P

ri  pðri Þ

l(X) =

ðri  lðXÞÞ2  pðri Þ

v(X) =

i¼1

Varianz:

v(X) =

n P i¼1

þ1 R 1 þ1 R 1

r  dðrÞ dr ðri  lðXÞÞ2  dðrÞ dr

Die Streuungistdie Quadratwurzelder Varianz:r(X) =

pffiffiffiffiffiffiffiffiffiffi vðXÞ.

Die Definition des Mittelwertes subsumiert auch binäre Merkmale XF, wenn Fx durch 1 und :Fx durch 0 kodiert wird: in diesem Fall folgt l(XF) = p(Fx) pffiffiffiffiffiffiffiffiffiffiffiffiffiffiffiffiffiffiffiffiffiffiffiffiffiffiffiffiffiffiffiffiffiffiffiffiffiffiffi ffi und r(XF) = pðFxÞ  ð1  pðFxÞÞ. Die mathematische Definition einer Normalverteilung ist durch die Parameter l und r völlig bestimmt; l bestimmt ihren Mittelpunkt und r ihre Flachheit. Über dem Intervall von l – r bis l+r liegt genau 66 % der Wahrscheinlichkeit, und über [l – 2r, l+2r] liegen 95,5 % der Wahrscheinlichkeit – dies ist das übliche Akzeptanzintervall. Setzt manplffiffiffiffiffiffiffiffiffi = 0 und r = 1, so erhält man die normierte Normalverteilung 2 g(z) = ð1= 2  p ) N eðz =2Þ . Man normiert eine beliebige Normalverteilung, indem man von den X-Zahlenwerten den X-Mittelwert abzieht und durch die X-Streuung dividiert. Zur Beschreibung einer einzelnen n-elementigen Stichprobe sn = {d1, …, dn} definiert man den Mittelwert lsn und die Streuung rsn der Zufallsvariable X in sn wie im diskreten Fall von (Def. 4.5-1); für p(ri) wird hier die Häufigkeit der Werte ri in sn gesetzt. Statistisch fundamental ist die Wahrscheinlichkeitsverteilung n-elementiger Stichprobenmittelwerte. Mithilfe der statistischen Produktregel (Kap. 3.10.4.1) und der Rechengesetze für

Wahrscheinlichkeitsverteilung der Stichprobenmittelwerte

157

158

4. Gesetzeshypothesen und ihre empirische Überprüfung

Erwartungswerte (Bortz 1985, 803 ff.) berechnet man für beliebige Verteilungen folgenden Mittelwert lln ðXÞ und Streuung rln ðXÞ der X-Mittelwerte von n-elementigen Stichproben: (Ms. 4.5-1)

Zentraler Grenzwertsatz

Berechnung von Akzeptanzintervallen

lln ðXÞ = l(X).

vln ðXÞ = v(X)/n.

pffiffiffi rln ðXÞ = r(X) / n.

Der Mittelwert der Stichprobenmittelwerte ist identisch mit dem Populationsmittelwert. Die Streuung der Stichprobenmittelwerte schrumpft dagegen mit zunehmender Stichprobengröße n, umgekehrt proportional zur Wurzel von n, und geht für n!v gegen Null. Daraus folgen anschaulich die Gesetze der großen Zahlen für beliebige Zufallsvariablen, denen zufolge für n!v der Stichprobenmittelwert mit Wahrscheinlichkeit 1 gegen den Populationsmittelwert konvergiert (s. Bauer 1978, 97, 167 f., 182). Bedeutend ist der zentrale Grenzwertsatz, der über die Konvergenz der Form der Mittelwerteverteilung folgende Aussage macht: die Verteilung der Stichprobenmittelwerte einer beliebigen Verteilung über X konvergiert für wachsendes n gegen eine pffiffiffi Normalverteilung mit Mittelwert l(X) und Streuung r(X)/ n (Bauer 1978, 267 f.; Bortz 1985, 121). Der zentrale Grenzwertsatz rechtfertigt es, für größere Stichproben die Verteilung der Stichprobenmittelwerte einer beliebig verteilten Zufallsvariable X durch eine Normalverteilung zu approximieren. Die Gauß’sche Normalverteilung erweist sich damit als die mathematische Form einer Zufallsfehlerverteilung. Zugleich wird einsichtig, warum so viele statistische Standardverfahren mit der Normalverteilung arbeiten: sie beruhen auf einem Vergleich von Stichprobenmittelwert (bzw. -häufigkeit) und Populationsmittelwert (bzw. -häufigkeit), und alle Stichprobenmittelwertsverteilungen, auch die Binomialverteilung, sind für größere Stichprobenumfänge (n F 30) durch eine Normalverteilung zu approximieren. Aufbauend auf diesen Grundlagen berechnet man Akzeptanzintervalle wie folgt: Gegeben eine Hypothese H, derzufolge der Mittelwert in der Population gleich l (bzw. im binären Fall gleich p) ist, und eine n-elementige Stichprobe sn, die groß genug ist, um die Stichprobenverteilung durch die Normalverteilung zu approximieren. (1.) Man schätzt die Populationsstreuung r durch die sogenannte korrigierte Stichprobenstreuung pffiffiffiffiffiffiffiffiffiffiffiffiffiffiffiffiffiffiffiffi ðXÞ ¼ r ðXÞ n=ðn  1Þ (s. Bortz 1985, 119; dieser Schritt entfällt bei rkorr s n sn pffiffiffiffiffiffiffiffiffiffiffiffiffiffiffiffiffiffiffiffiffi der binären bzw. Binomialverteilung; hier gilt r = p p ð1ffiffiffi  pÞ ). (2.) Man berechnet die Stichprobenmittelwertstreuung: rln ¼ r= n. (3.) Man sieht in der Tabelle der Standardnormalverteilung, die in allen Statistik-Büchern zu finden ist, die Intervallgrenzen des symmetrischen 95 %-Intervalls nach; diese betragen l1,96. (4.) Durch Umkehrung der z-Transformation rechnet man diese Intervallgrenzen auf die Stichprobenmittelwertsverteilung um: l l 1,96 rln . Damit hat man das Akzeptanzintervall für H gefunden. Für kleine Stichproben sieht man in der Tabelle der t-Verteilung bzw. der Binomialverteilung nach. – Zur Prüfung auf signifikante Unterschiede zweier unabhängig gezogener Stichproben sn und sm verwendet man den sogenannten t-Test für unabhängige Stichproben. Die Wahrscheinlichkeitsverteilung der Stichprobenmittelwertsdifferenz D = lsn – lsm ist unter der Annahme der Nullhypothese eine Normalverteilung mit dem Mittelwert 0 und der Streu-

4.5 Weiterführende Themen

pffiffiffiffiffiffiffiffiffiffiffiffiffiffiffiffiffiffiffiffiffiffiffiffiffiffiffiffiffiffi ung r(D) = rN ð1=nÞ þ ð1=mÞ (Bortz 1985, 166 ff.; r wird mithilfe beider Stichproben geschätzt). Man berechnet wie oben das 95 % Akzeptanzintervall der Nullhypothese, an deren Grenzen die kleinsten gerade noch 95 %signifikanten Stichprobenmittelwertsdifferenzen liegen. Wir haben uns oben nur mit den statistischen Methoden für Populationsmittelwerte (bzw. -mittelwertsunterschieden) von Variablen beschäftigt. Darüber hinaus interessiert man sich dafür, ob sich aus Messungen zweier Variablen X und Y, z. B. Größe und Gewicht von verschiedenen Personen, ein zugrundeliegender funktionaler Zusammenhang herauslesen lässt, der zusätzlich durch eine Streuung aufgrund weiterer Faktoren statistisch verschmiert ist. Diese Fragestellung behandeln die Methoden der Kurvenanpassung. Gemäß Glymour (1981, 322) besteht dieses Verfahren in folgender zweistufiger Vorgangsweise: (1.) Man bestimmt erstens den mathematischen Typ der Funktion (linear, polynomisch, …), von dem man vermutet, dass er die funktionale Tendenz hinter den Messwerten wiedergibt. (2.) Man bestimmt die am besten approximierende Funktion innerhalb des gewählten Funktionstyps mithilfe der Methode der kleinsten Abweichungsquadrate. Während Schritt 2 ein wohlverstandenes statistisches Verfahren ist (s. Bortz 1985, 219 – 221, 241), wird Schritt 1 von philosophisch schwierig zu rechtfertigenden Intuitionen geleitet. Das Problem liegt darin, dass sich eine gegebene Menge von Messwertpaaren {(x1,y1, …, (xn,yn)} durch eine Kurve beliebig genau approximieren lässt, wenn man den Kurventyp in Schritt 1 nur kompliziert genug wählt (z. B. ein hochgradiges Polynom). Wie Forster & Sober 1994 und Hitchcock & Sober 2004 herausarbeiten, würde sich eine solche Kurve aber häufig nicht dem wahren, aber statistisch verschmierten Zusammenhang zwischen X und Y anpassen, sondern stattdessen den Zufälligkeiten der vorliegenden Messungen – man spricht hier von einem Overfitting. Aber auch das gegenteilige Kriterium der einfachsten Kurve ist nicht immer zielführend (s. Glymour 1981, 324 ff.). Das Kurvenanpassungsproblem lässt sich jedoch durch das Kriterium des Voraussageerfolges lösen: eine durch Fitten auf eine Datenmenge gewonnene Kurve ist nur dann bestätigt, wenn sich ihre Voraussagen anhand neuer Datenmengen bestätigen. Nur wenn dies der Fall ist, kann induktiv geschlossen werden, dass beim Fitten auf die ursprüngliche Datenmenge kein Overfitting passiert ist. Dies zeigt Abb. 4.5-2: die alten Daten (weiß) werden von einem Polynom, der geschlängelten Kurve, natürlich beser approximiert Y

Y Alte Daten: weiß

Das Kriterium des Voraussageerfolges

Neue Daten: grau

X

Abb. 4.5-2: Die Gerade wird bestätigt

Kurvenanpassung

X

Abb. 4.5-3: Das Polynom wird bestätigt

159

160

4. Gesetzeshypothesen und ihre empirische Überprüfung

als von der Gerade. Liegen die neuen Daten (grau) so wie in Abb. 4.5-2, so spricht dies dafür, dass der Erfolg des Polynoms auf einem Overfitting beruhte und die Gerade dem wahren Zusammenhang entspricht. Kommen die neuen Daten so zu liegen wie in Abb. 4.5-3, dann spricht dies dafür, dass der wahre Zusammenhang tatsächlich ein Polynom ist. Dieses Kriterium zeigt zugleich, dass Bestätigungen aufgrund neuer Voraussagen beim Kurvenanpassen ein entscheidendes Surplus besitzen gegenüber Bestätigungen durch ex-post-fakto Erklärungen.

4.5.2 Die Rechtfertigung der Likelihood-Intuition innerhalb der subjektiven Wahrscheinlichkeitstheorie

Likelihood-Intuition als übergreifendes induktives Prinzip

Statistisch definiert sind nur die Wahrscheinlichkeiten unserer Erfahrungen bzw. Stichprobenresultate E, gegeben gewisse statistische Populationshypothesen H; wir schreiben für diese Wahrscheinlichkeiten p(E:H). Die Wahrscheinlichkeiten unserer Hypothesen H, gegeben unsere Erfahrungen E, sind dagegen grundsätzlich subjektiver Natur. Wie in Kap. 4.2.3.5 ausgeführt, beruhen die Methoden der Inferenz- und Teststatistik auf der Likelihood-Intuition: dieser Intuition zufolge ist die inverse Wahrscheinlichkeit p(E:H) das Basiskriterium für die Beurteilung der Plausibilität einer Hypothese H bei gegebenem E. Man nennt p(E:H) auch das Likelihood von E gegeben H (so auch Hays/Winkler 1970, 316, Earman 1992, 34; die Terminologien sind leider unterschiedlich). Die Likelihood-Intuition ist nicht zu verwechseln mit der Likelihood-Methode, sondern sie ist viel grundlegender. Sie ist das grundlegende induktive Prinzip der schließenden Statistik. Die Likelihood-Intuition kennt verschiedene Ausprägungen. In der Methode der Likelihood-Maximierung wird angenommen, die Stützung einer Hypothese H durch eine Evidenz E ist umso größer, je höher das Likelihood von E gegeben H ist. In der Methode der Likelihood-Erwartung wird angenommen, die Stützung einer Hypothese H durch eine Evidenz E ist umso größer, je näher E dem mit den Likelihoods von E gegeben H gebildeten Erwartungswert von E kommt. Darüber hinaus kann man unterscheiden zwischen Likelihood-Punktmethoden, bei denen Punktevidenzen mit Punkthypothesen verglichen werden, und LikelihoodIntervallmethoden, bei denen Evidenzintervalle mit Punkthypothesen verglichen werden, so wie in der in Kap. 4.2.3.1 erklärten Methode der Akzeptanz- und Zurückweisungsintervalle, oder Punktevidenzen mit Intervallhypothesen, so wie in der in Kap. 4.2.3.2 erklärten Methode der Konfidenzintervalle. Auch die Intervallmethoden beruhen auf der Likelihood-Intuition. Der Unterschied liegt darin, dass man nun das 95 %-wahrscheinliche Intervall von Stichprobenmittelwerten Er mit dem durchschnittlich größten Likelihood p(Er: H) betrachtet. Weil Stichprobenmittelwerte normalverteilt sind, ist dieses Intervall das symmetrisch um den Populationsmittelwert zentrierte 95 %-Intervall. Die Fisherschen Intervallmethoden sind heutzutage statistische Standardpraxis. Das philosophische Problem der statistischen Inferenz- und Testmethoden liegt tiefer: diese Methoden sind nur dann gut begründet, wenn man die Likelihood-Intuition als gerechtfertigt ansieht. Warum sollte die inverse

4.5 Weiterführende Themen

Wahrscheinlichkeit p(E:H) als Maß der Plausibilität der Hypothese H bei gegebener Evidenz E herangezogen werden? Innerhalb der statistischen Theorie gibt es auf diese Frage keine Antwort – es handelt sich um eine Basisintuition. Denn die Plausibilität der Hypothese H gegeben Evidenz E ist eine subjektiv-epistemische Wahrscheinlichkeit w(H|E), über welche die statistische Theorie keine Aussagen macht. Die subjektive Wahrscheinlichkeitstheorie besitzt eine Antwort auf die Frage, wie die Likelihood-Intuition zu rechtfertigen ist. Die Grundlage dieser Antwort ist das in Kap. 3.10.4.3 erläuterte statistische principal principle (StPP), welches für eine Stichprobenerfahrung E und eine Populationshypothese H ganz einfach besagt: w(E|H) = p(E:H). Es gilt nun: (4.5-9)

w(H|E) = w(E|H)Nw(H) / w(E) = p(E:H)Nw(H) / w(E)

(gemäß der Bayes-Regel) (gemäß dem StPP).

Grundprinzip des Bayesianismus

D. h., die subjektive Wahrscheinlichkeit von H, geben das Stichprobenresultat E, ist gleich der statistischen Wahrscheinlichkeit von E unter der Annahme H, multipliziert mit dem Verhältnis der Ausgangswahrscheinlichkeiten von H und von E. Es ist das Charakteristikum der subjektiv-Bayesianischen Statistik, dass immer von gewissen Ausgangswahrscheinlichkeiten für Hypothesen ausgegangen werden muss. Die Ausgangswahrscheinlichkeit von E kann jedoch eliminiert werden, indem man sich auf komparative Hypothesenbewertungen beschränkt. Seien H1, H2 zwei konkurrierende statistische Hypothesen, dann bestimmt sich das Verhältnis ihrer subjektiven Wahrscheinlichkeiten (gegeben E) wie folgt: (4.5-10)

wðH1 jEÞ pðE : H1 Þ wðH1 Þ ¼  . wðH2 jEÞ pðE : H2 Þ wðH2 Þ

Das Verhältnis wird nur durch das Verhältnis ihrer beiden Likelihoods (die sogenannte Likelihood-Ratio) und das Verhältnis ihrer beiden Ausgangswahrscheinlichkeiten bestimmt. Die Maximierung des Likelihoods p(E:Hi) unter gegebenen Alternativhypothesen H1, …, Hn ist somit genau dann ein Indikator der subjektiv wahrscheinlichsten Hypothese, wenn die Ausgangswahrscheinlichkeiten der verglichenen Hypothesen gleich sind, wenn also in unserem Beispiel w(H1) = w(H2) gilt. Man nennt die Annahme gleicher Ausgangswahrscheinlichkeiten für konkurrierende Hypothesen auch das Indifferenzprinzip: in Ermangelung weiteren Wissens werden konkurrierende Möglichkeiten als gleichwahrscheinlich angenommen. Wir halten also fest:

Likelihood-Ratio

(Ms. 4.5-2): Subjektive Rechtfertigung der Likelihood-Intuition: unter der Annahme des Indifferenzprinzips ist die Höhe des Likelihood von E gegeben H ein Indikator für die subjektive Wahrscheinlichkeit von H gegeben E. Die subjektive Rechtfertigung der Inferenz- und Teststatistik durch das Indifferenzprinzip krankt daran, dass das Indifferenzprinzip selbst einer Reihe bekannter Probleme ausgesetzt ist. Beispielsweise ist die Indifferenz von

Probleme des Indifferenzprinzips

161

162

4. Gesetzeshypothesen und ihre empirische Überprüfung

Bayes-Statistik

Verteilungsunabhängige Konvergenzresultate

Induktive Annahmen der Bayes-Statistik

Ausgangsverteilungen sprachabhängig: so ist etwa eine für die Wellenlänge (X) indifferente Verteilung nach Umrechnung auf die Wellenfrequenz (Y:=1/ X) nicht mehr indifferent (s. Howson/Urbach 1996, 60; Earman 1992, 15). Die weithin akzeptierte Konsequenz dieser und anderer Probleme ist folgende: keine Ausgangsverteilung ist informationslos, auch nicht die Gleichverteilung. Einen Ausweg aus diesem Rechtfertigungsdilemma sehe ich lediglich in der Tatsache, dass Gleichverteilungen dazu dienen, den induktivstatistischen Generalisierungsschluss zu rechtfertigen; für diesen gibt es aber, wie in Kap. 2.6.1 erwähnt, unabhängige Rechtfertigungsmethoden. Jüngere Bayesianer haben sich aus den erwähnten Gründen vom Indifferenzprinzip verabschiedet. Mithilfe der Bayes-Statistik lässt sich die Endverteilung der subjektiven Wahrscheinlichkeitsdichte dw(Hr|E) für eine Partition von möglichen Punkthypothesen Hr: l=r gegeben E berechnen, auch wenn die Ausgangsverteilung dw(Hr) keine Gleichverteilung ist (s. Hays/Winkler 1970, 233 ff., 461). Unabhängig von der Ausgangsverteilung bewirkt die Konditionalisierung auf ein Stichprobenresultat E: hn(Fx)=k/n eine Verschiebung des Verteilungsmaximums in Richtung der Stichprobenhäufigkeit k/n, wobei die Verteilung zunehmend steilgipfeliger wird – vorausgesetzt, dass die Ausgangsverteilung undogmatisch ist, was grob gesprochen bedeutet, dass keinem nichtverschwindenden Hypothesenintervall eine subjektive Nullwahrscheinlichkeit zugeordnet wird. Man spricht hier von verteilungsunabhängigen Konvergenzresultaten (Earman 1992, 58). Auf diese Weise modelliert der moderne Bayesianismus induktives Lernen aus Erfahrung. Dieses Lernmodell hat jedoch den Haken, dass es nur „im Grenzwert“ funktioniert. Für jede noch so große Stichprobengröße n kann durch eine hinreichend extreme aber dennoch undogmatische Ausgangsverteilung das Lernen aus Erfahrung verhindert werden. Auch die Bayes-Statistik ohne Indifferenzprinzip beruht auf induktiven Annahmen. Diese induktiven Annahmen umfassen (a) das statistische principal principle und (b) die Annahme der Existenz eines Häufigkeitsgrenzwertes – beides wird von der Bayes-Statistik vorausgesetzt. Beide Annahmen zusammen sind nachweislich gleich stark wie die sogenannte Annahme der Vertauschbarkeit bzw. Symmetrie von subjektiven Wahrscheinlichkeiten (s. Carnap/Jeffrey 1971, 117 ff.; Earman 1992, 89; Kutschera 1972, 74). Die Vertauschbarkeit subjektiver Wahrscheinlichkeiten besagt (vereinfacht), dass der Glaubensgrad w(Fai) dafür, dass ein Individuum ai eine Eigenschaft F besitzt, für beliebige Individuen ai in Raum und Zeit prima facie gleich ist. Stellen wir uns die Individuen in der Zeit geordnet vor, so ist dies nichts anderes als eine induktive Gleichförmigkeitsannahme in der Zeit. Die Vertauschbarkeitsannahme impliziert für nichtdogmatische Ausgangswahrscheinlichkeiten ein gleichförmiges induktives Lernen in dem Sinne, dass w(Fan|Fa1^ … ^ Fan – 1) > w(Fan) für beliebige n>1 und a1, …, an gilt (s. Kutschera 1972, 74 ff.; Earman 1992, 108).

4.6 Zusammenfassung, einführende Literatur und Übungen

4.6 Zusammenfassung, einführende Literatur und Übungen 4.6.1. Zusammenfassung (Auswahl). Gesetzeshypothesen müssen neben der Bedingung der (vermutlichen) Wahrheit auch die Relevanzbedingung erfüllen. D. h., ihre Antecedensfaktoren müssen für das Eintreten des Konsequens im strikten Fall notwendig und im statistischen Fall wahrscheinlichkeitsverändernd sein. Die Überprüfung einer strikten Gesetzeshypothese auf Wahrheit bzw. Relevanz erfolgt mithilfe der auf Mill zurückgehenden Methoden der Übereinstimmung bzw. des Unterschieds, welche in der modernen statistischen Testtheorie in Form der Methode der Akzeptanzintervalle und signifikanten Unterschiede konkretisiert werden. Aus diesen Überprüfungsmethoden ergeben sich auf natürliche Weise auch Auffindungsmethoden für Gesetzeshypothesen aus gegebenen Daten. Die statistischen Überprüfungsmethoden enthalten eine Reihe von möglichen, aber bei entsprechender Kenntnis vermeidbaren Fehlerquellen. Insbesondere darf aus signifikanten statistischen Korrelationen aus mehreren Gründen nicht direkt auf eine Kausalbeziehung geschlossen werden. Sämtliche statistischen Test- und Inferenzmethoden basieren auf der Likelihood-Intuition, welche innerhalb der subjektiven Wahrscheinlichkeitstheorie eine kontroverse Rechtfertigung mithilfe des Indifferenzprinzips besitzt. 4.6.2 Einführende Literatur. Eine gut lesbare Einführung in Wahrscheinlichkeitstheorie und induktive Überprüfungsmethoden ist Skyrms (1989). Sie finden darüber auch viel in der klassischen Einführung Hempel (1974). Eine hervorragende Statistik-Einführung ist Bortz (1985), und mit Fokus auf Forschungsmethoden Bortz/Döring (2002). Zum Bayesianismus s. Earman (1992) und Howson/Urbach (1996). Eine Standardreferenz zu Korrelation und Kausalität ist immer noch Hummel/Ziegler (1976).

4.6.3 Fragen und Aufgaben Zu Kap. 4.1: 1) Rekapitulieren Sie die Definition der Relevanz von strikten Allsätzen. 2) In einem Individuenbereich D mit Merkmalen F, G, H kommen folgende Individuenarten vor: F^G^H, F^G^:H, F^:G^:H, :F^G^:H, :F^:G^:H. Welche strikten Allimplikationen (der Form lX^lY ! lZ) sind in D wahr? Welche davon sind relevant? 3) Zu Aufgabe 2) zu Kap. 3.9: welche statistischen Relevanzbeziehungen liegen hier zwischen Geschlecht und Familienstand vor? Wie hoch sind die Korrelationen gemäß dem Differenzmaß? Zu Kap. 4.2: 1) Betrachten Sie folgende drei strikte Gesetzeshypothesen: (G1) Alle männlichen Stammesmitglieder sind tätowiert. (G2) Alle männlichen Stammesmitglieder über 40 sind tätowiert. (G3) Alle Stammesmitglieder über 40 sind tätowiert. Es wurden drei Stichproben von Stammesmitgliedern untersucht, mit folgendem Resultat („+/ – “ bedeutet, dass alle Individuen bzw. kein Individuum der Stichprobe das Antecedensmerkmal aufwies(en):

163

164

4. Gesetzeshypothesen und ihre empirische Überprüfung

1. Stichprobe: 2. Stichprobe: 3. Stichprobe:

männlich + + –

über 40 + – +

Resultat alle tätowiert alle tätowiert nicht alle tätowiert

Erläutern Sie anhand dieses Beispiels die Methoden der Übereinstimmung und des Unterschieds. Welche der Hypothesen (G1 – 3) werden wodurch bestätigt, und welche darunter wodurch als relevant bzw. irrelevant erwiesen? 2) (a) Die Hypothese lautet: 60 % aller Biertrinker sind Raucher. Gegeben eine Stichprobe von 500 Biertrinkern, für die Sie ein Akzeptanzintervall von 282 – 318 berechnen. Bei welchem Stichprobenergebnis wäre die Hypothese schwach bestätigt, und bei welchen Stichprobenergebnissen wäre sie stark geschwächt? – (b) Angenommen, Sie finden in Ihrer Stichprobe 315 Raucher. Wie lautet das 95 % Konfidenzintervall für die Häufigkeit von Rauchern unter den Biertrinkern in der Population? 3) Gegeben eine Stichprobe von 100 Kaffeetrinkern mit 48 Rauchern, und eine Kontrollstichprobe von 100 Nicht-Kaffeetrinkern mit 32 Rauchern. In welchem Prozentintervall müsste die signifikante Stichprobendifferenz liegen, damit aus diesen Befunden auf eine signifikante Korrelation geschlossen werden kann, bzw. nicht auf eine solche geschlossen werden kann? 4) Geben Sie Beispiele für Fehlerquellen in der statistischen Methode. 5) Kuno bringt eine Pflanze aus Guatemala mit, die angeblich hervorragend gegen Erkältung wirkt. Er gibt sie 7 Bekannten, die schwer erkältet sind, und erzählt ihnen begeistert von der Pflanze. 4 davon rufen später an und berichten, sie wären nach Einnahme der Pflanze schnell wieder genesen (die anderen 3 melden sich nicht). Kuno ist daraufhin von der Heilkraft der Pflanze überzeugt. Kuno hat dabei aber mindestens drei voneinander unabhängige methodische Fehler begangen. Welche? Zu Kap. 4.3: 1) Angenommen, eine Korrelation zwischen zwei Merkmalen F und G wird durch ein drittes Merkmal H total abgeschirmt. Wann handelt es dabei um eine Scheinkausalität, und wann handelt es sich dabei um eine intervenierende Variable? Geben Sie jeweils ein Beispiel. 2) Aus welchem Grund kann man aus dem Vorliegen einer Korrelation zwischen zwei Merkmalen selbst dann, wenn keine versteckten Variablen im Spiel sind, nicht eindeutig auf eine Kausalbeziehung schließen? Geben Sie ein Beispiel. 3) Jemand berichtet über folgende statistische Zusammenhänge: (a) Personen, die mehr Fleisch essen, sind aggressiver. (b) Gesündere Personen haben mehr Geld. (c) Personen, die häufiger ins Kino gehen, haben häufiger Autounfälle. Diskutieren Sie für jedes Beispiel alle möglichen kausalen Erklärungen dieser Korrelationen. Zu Kap. 4.5: 1) Ihrer Hypothese H zufolge liegt das durchschnittliche Alter der ersten Liebschaft von Mädchen bei 15 Jahren. Sie ziehen zwecks Überprüfung eine 25-elementige Stichprobe. Die korrigierte Streuung dieser Stichprobe betrage 2,5 Jahre. Berechnen Sie das 95 %ige Akzeptanzintervall von H (Information: in der Standardnormalverteilung liegen dessen Grenzen bei l 1,96).

4.6 Zusammenfassung, einführende Literatur und Übungen

2) Fortsetzung von 1): Sie ziehen eine 30-elementige Stichprobe von Jungen und vergleichen diese mit der Mädchenstichprobe. Die aus beiden Stichproben ermittelte korrigierte Streuung betrage 2,8 Jahre. Berechnen Sie die signifikante Mittwelwertsdifferenz beim Signifikanzniveau von 5 %. 3) Was ist die Likelihood-Intuition, und wie lautet ihre subjektiv-probabilistische Rechtfertigung?

165

5. Theorien und ihre empirische Bewertung 5.1 Theoretische Begriffe und multiple Zuordnungsgesetze Beispiele für theoretische Begriffe

Zuordnungsgesetz

Theoretische Begriffe und empirische Dispositionen

Theoretische Begriffe referieren auf Dinge oder Eigenschaften, die wir Menschen nicht mit den Sinnen wahrnehmen können. Theoretische Individuenbegriffe der Naturwissenschaften bezeichnen z. B. Individuen, die zu klein sind, um gesehen zu werden, von Bakterien bis zu Molekülen, Atomen und Elementarteilchen. Theoretische Prädikate der Naturwissenschaften bezeichnen Eigenschaften oder Wirkungsarten, für deren Wahrnehmung uns kein Sinnesorgan zur Verfügung steht, wie z. B. elektrische oder magnetische Kraftfelder, oder sie bezeichnen innere (chemische, biologische, neuronale) Strukturen eines Systems, von dem wir nur die äußeren InputOutput-Relationen beobachten können. In den Human- und Sozialwissenschaften können theoretische Individuenbegriffe komplexe soziale Gebilde wie z. B. den Staat Deutschland bezeichnen. Theoretische Prädikate der Psychologie und Sozialwissenschaften bezeichnen z. B. mentale oder emotionelle Zustände von Personen, oder komplexe Zustände von Gesellschaften. Wie in Kap. 3.10.1 erläutert, besteht zwischen theoretischen Begriffen und empirischen Dispositionsbegriffen folgende Beziehung. Empirische Dispositionen sind funktionale Merkmale und ihre Bedeutung wird durch einen einzelnen Zuordnungssatz qua Bedeutungspostulat analytisch charakterisiert. Theoretische Merkmale bezeichnen dagegen Strukturmerkmale, welche die Ursachen verschiedener empirischer Dispositionen sind, aber mit diesen nicht analytisch äquivalent sind. Theoretische Begriffe werden daher immer durch mehrere Zuordnungsgesetze charakterisiert, deren Liste offen und potentiell unendlich ist. Ganz allgemein verstehen wir unter einem (empirischen) Zuordnungsgesetz oder Indikatorgesetz für einen theoretischen Begriff s ein Gesetz, in dem der von s bezeichneten theoretischen Eigenschaft unter gewissen empirisch identifizierbaren Umständen ein empirisches Korrelat zugeordnet wird. Erläutern wir dies an dem Beispiel von Kap. 3.10.1, der empirischen Disposition der Wasserlöslichkeit. Die Ursache der Wasserlöslichkeit ist bei allen bekannten chemischen Molekülarten ihre Dipolstruktur, d. h., die Moleküle besitzen ein positiv und ein negativ geladenes Ende, und da auch Wassermoleküle solche Dipolstruktur besitzen, können letztere sich leicht zwischen die Substanzmoleküle einlagern, Plus-Pol an Minus-Pol orientiert und umgekehrt, und so die Substanz im Wasser molekular verteilen, d. h. auflösen. Diese Dipolstruktur ist aber nicht nur die Ursache der Wasserlöslichkeit, sondern von vielen anderen empirischen Dispositionsmerkmalen, die in Abb. 5.1-1 zusammengefasst sind (vgl. Mortimer 1973, 83):

5.1 Theoretische Begriffe und multiple Zuordnungsgesetze Theoretisches

Empirische Dispositionen Dix

Zuordnungsgesetze für Di

Merkmal W(x)

(Speziell:) x ist wasserlöslich (D1)

T1xto (D1x l R1xt)

Substanzmoleküle

Speziell:) x ist unlöslich in Öl (D2)

T2xt o (D2x l R2xt)

von x haben

(Allgemein:) x ist löslich in allen polaren

Dipolstruktur

Lösungsmitteln (Wasser, Ammoniak, })

}

(Allgemein:) x ist unlöslich in allen unpolaren Lösungsmitteln (Öl, Benzol,})

}

x hat erhöhten Schmelzpunkt

}

x-Lösungen sind stromleitend (Elektrolyse)

}

x absorbiert und emittiert elektromagnetische Strahlung in bestimmtem Spektralbereich u.a.m. }

} Tnxt o (Dnx l Rnxt)

Abb. 5.1-1: Ein theoretisches Merkmal verursacht viele empirische Dispositionen

Logisch gesehen liegt im einfachsten Fall folgender Zusammenhang vor. Aus der Hintergrundtheorie T folgt, dass das theoretische Merkmal s(x) durch eine Reihe von permanenten Dispositionsmerkmalen Dix empirisch identifiziert werden kann: (5.1-1)

(Für alle x:) s(x) g. d. w. Dix für 1 i  n

(Formal: 8x: s(x) $ Dix)

Jedes Dispositionsmerkmal wird durch ein bilaterales Zuordnungsgesetz in Form eines bilateralen Reduktionssatzes semantisch charakterisiert: (5.1-2) Zi(D): (Für alle x, t:) Wenn Tixt, dann (Dix g.d.w. Rixt) für 1  i  n (Formal: 8x,t: Tixt ! (Dix $ Rixt)) Dabei ist Ti die empirische Testbedingung und Ri die empirische Reaktion. Z. B.: Wenn x zur Zeit t ins Wasser gegeben wird (=Tixt), dann ist x wasserlöslich (=Dix) g.d.w. x sich kurz nach t in Wasser auflöst (=Rixt). Wie in Kap. 3.10.1 erläutert, kann man anstelle des Zuordnungsgesetzes Zi(D) die Disposition auch durch eine nomologische Implikation definieren, Dix $ (Tixt !g Rixt). Wir ziehen im folgenden aber Zuordnungsgesetze vor, weil nur sie die empirischen Identifikations- bzw. Messbedingungen liefern, welche sich auf den theoretischen Term übertragen. Aus den n theoretischen Gesetzen in (5.1-1) und den n Bedeutungspostulaten in (5.1-2) folgen logisch die folgenden n Zuordnungsgesetze für einund-dasselbe theoretische Merkmal s(x): (5.1-3) Zi(s): (Für alle x, t:) Wenn Tixt, dann (s(x) g.d.w. Rixt) (Formal: 8x,t: Tixt ! (s(x) $ Rixt))

für 1  i  n

Z. B.: wenn x zu t in Wasser gegeben wird, dann besitzen die Moleküle von x Dipolstruktur g.d.w. x sich kurz nach t in Wasser auflöst. (Hinweis: wenn die Beziehung zwischen s(x) und den Dix bzw. Rixt bloß einseitig-implikati-

167

168

5. Theorien und ihre empirische Bewertung

ver Natur ist, spricht man von unilateralen Zuordnungsgesetzen: Tixt ! (s(x)!( ) Rixt).) Die Zuordnungsgesetze in (5.1-3) können aber nicht als analytisch wahr, als partielle Definitionen bzw. Bedeutungspostulate für das theoretische Merkmal s(x) angesehen werden. Dies ist deshalb unmöglich, weil zwei (oder mehrere) Zuordnungsgesetze zusammen empirischen Gehalt besitzen, wie folgendes Argument zeigt: Mehrere Zuordnungsgesetze haben empirischen Gehalt

Synthetische Natur von Zuordnungsgesetzen

(5.1-4) (Z1:) Wenn eine Substanz x (zu t) ins Wasser gegeben wird, dann besitzen x’s Moleküle Dipolstruktur g.d.w. x sich darin auflöst. ( T1xt ! (s(x) $ R1xt)) (Z2:) Wenn eine Substanz x (zu t) in Öl gegeben wird, dann besitzen x’s Moleküle Dipolstruktur g.d.w. x darin ungelöst bleibt. (T2xt ! (s(x) $ R2xt)) Daraus folgt: Wenn sich eine Substanz x irgendwann in Wasser aufgelöst hat, dann wird die (rückgewonnene) Substanz in Öl immer ungelöst bleiben. (9t(T1xt^R1xt)! 8t(T2xt ! R2xt)) Mehrere Zuordnungsgesetze für denselben theoretischen Begriff können also nicht als analytisch wahr aufgefasst werden, denn schon zwei zusammen haben empirischen Gehalt. Man dürfte maximal ein Zuordnungsgesetz unter allen anderen als analytisch wahr auszeichnen (gegeben T’s Permanenzhypothese bzgl. s). Aber dies wäre pure Willkür. Warum sollte man „Wasserlöslichkeit“ als analytische Wahrheit für molekulare Dipolstruktur auszeichnen? Dies würde nur dazu führen, dieses Zuordnungsgesetz immun zu machen gegenüber möglichen empirischen Korrekturen, und wäre daher inadäquat. Wir gelangen damit zu folgender Konsequenz: (Ms. 5.1-1) Zuordnungsgesetze für theoretische Begriffe sind grundsätzlich synthetischer Natur – es handelt sich bei ihnen um Bestandteile der jeweiligen Hintergrundtheorie, die wie alle anderen Bestandteile hypothetischer Natur und somit bewährbar bzw. korrigierbar sind.

Vereinheitlichung durch Zuordnungsgesetze

Die empirischen Konsequenzen, die aus mehreren Zuordnungsgesetzen für denselben theoretischen Begriff folgen, zeigen zugleich auf, wie die empirische Vereinheitlichungsleistung von qualitativen Theorien zustande kommt. Unter empirischer Vereinheitlichung ist ganz allgemein zu verstehen, dass viele empirische Regelmäßigkeiten auf einige wenige theoretische Gesetze zurückgeführt werden. Aus den n Zuordnungsgesetzen der Form Zi(s): Tixt ! (s(x)$ Rixt) (1in) ergeben sich nämlich n2 empirische Konsequenzen der Form (Kij:) 9t(Tixt^Rixt) !8t(Tjxt!Rjxt), für 1i, jn. Für i 6¼ j drückt eine solche empirische Konsequenz aus, dass man aus der empirischen Feststellung einer Disposition Di auf das Vorliegen einer anderen Disposition Dj schließen kann (im Fall i=j wird ausgedrückt, dass die Disposition Di permanenter Natur ist). Durch die Einführung des theoretischen Merkmals s(x), welches allen n Dispositionen gemeinsam zugrunde liegt, werden somit n2

5.1 Theoretische Begriffe und multiple Zuordnungsgesetze

empirische Gesetze auf nur n theoretische Zuordnungsgesetze zurückgeführt (Schurz/Lambert 1994, § 2.3; Schurz 2008a, § 4.4.1). Wenn Zuordnungsgesetze die Bedeutung von theoretischen Begriffen nicht ausschöpfen, wodurch wird deren Bedeutung dann festgelegt? Die bewährte Antwort der modernen Wissenschaftstheorie lautet: durch das gesamte Netz der für diesen Begriff zuständigen Hintergrundtheorien. Hier ist jedoch eine Differenzierung angebracht. Bei speziellen theoretischen Begriffen wie „Dipolstruktur“ stammen die empirischen Zuordnungsgesetze nicht aus derselben Theorie wie jene Gesetze, mittels derer wir die Bedeutung dieses Begriffes charakterisieren würden. Die empirischen Indikatorgesetze für Dipolstruktur, soweit sie das Löslichkeitsverhalten betreffen, stammen aus der qualitativen anorganischen Chemie. Für die Explikation der Bedeutung des Begriffs „Dipolstruktur“ würden wir dagegen die chemische Molekültheorie und die Theorie der Elektrostatik heranziehen. Wir führen damit die Bedeutung spezieller theoretischer Begriffe wie Dipolstruktur auf die Bedeutung allgemeiner theoretischer Begriffe zurück – in unserem Beispiel Elektron, Bindung und elektrische Ladung. Damit ist die Bedeutungsfrage natürlich nur verschoben, denn wir müssen weiterfragen: wodurch wird die Bedeutung der fundamentalsten theoretischen Begriffe charakterisiert, wie z. B. Masse und Kraft in der Mechanik, oder Ladung in der Elektrostatik, usw.? Wieder lautet die Antwort: durch die zuständige Hintergrundtheorie – im Fall von Masse und Kraft ist dies die Mechanik. Hier liegt also der Fall vor, dass die empirischen Zuordnungsgesetze für Masse und Kraft aus derselben Theorie stammen, welche die Bedeutung dieser Begriffe insgesamt charakterisiert. Wir illustrieren dies nun am Beispiel des Massebegriffes – dabei beschränken wir uns einfachheitshalber auf die klassische (Newtonische) Physik. Prätheoretisch gesprochen ist die Masse die Menge an Materie einer Substanz. Sie ist Ursache verschiedener empirischer Dispositionen von materiellen Gegenständen, welche jeweils durch einen Zuordnungssatz charakterisiert werden. Zunächst lassen sich Massen im alltäglichen Gebrauch einerseits auf einer Balkenwaage und andererseits auf einer Federwaage bestimmen:

Die Bedeutung theoretischer Begriffe

(5.1-6) T1xt uuuuuuuuuuA (Mx auuuuuuuuuuuuuA R1xt) (Z1:) x wird (zu t) auf Masse von x = k x wird durch k MasseBalkenwaage gegeben einheiten ausbalanciert T2xtuuuuuuuuuuA (Mx auuuuuuuuuuuuuA R2xt) (Z2:) x wird (zu t) auf Masse von x = k Die Feder wird um k Einheiten Federwaage angebracht gedehnt bzw. komprimiert Bei diesen beiden Zuordnungsgesetzen handelt es sich um Messgesetze. Man schreibt Messgesetze üblicherweise in die kompakte funktionale Form 8x,t: T1xt ! (m(x) = bal(x,t)),

Zuordnungsgesetze für Masse

169

170

5. Theorien und ihre empirische Bewertung

wobei „bal“ die empirische Funktion bezeichnet, welche dem Gegenstand x die Anzahl der x aufwiegenden Masseeinheiten zuordnet. Ein funktionales Gesetz dieser Form ist aber logisch äquivalent mit dem bilateralen Reduktionssatz 8x,t,r: T1xt ! (m(x)=r $ bal(x,t)=r). Wenn wir „Mx“ für die Formel „m(x)=r“ und „R1xt“ für die Formel „bal(x,t)=r“ setzen, ist dies die logische Form obigen Zuordnungsgesetzes (Z1). Dies zeigt, dass bilaterale Reduktionssätze eine sehr weite Klasse von Zuordnungsgesetzen umfassen: alle empirischen Indikatorgesetze und Messgesetze für theoretische Begriffe. Balkenwaage-Messung und Federwaage-Messung beruhen auf unterschiedlichen physikalischen Mechanismen. Das Ergebnis der BalkenwaageMessung ändert sich auch dann nicht, wenn sich die Schwerkraft ändert; das Ergebnis der Federwaage-Messung ändert sich jedoch dadurch. Die Balance der Balkenwaage kommt dadurch zustande, dass rechts und links die gleiche Schwerkraft wirkt; die Ausdehnung oder Kompression der Feder hängt dagegen vom Betrag der Schwerkraft ab. Balkenwaage-Messungen würden auch am Mond oder Jupiter richtige Ergebnisse liefern, Federwaage-Messungen nicht. Siehe Abb. 5.1-2.

KG

KG KG

}

}  KG

%RDE

*UPITER

Abb. 5.1-2: Balken- und Federwaage auf Erde und Jupiter

Schwere vs. träge Masse

Sowohl Balkenwaage wie Federwaage beruhen auf dem Gewichtseffekt der Masse, d. h. auf der Existenz einer auf das Objekt wirkenden Gravitationskraft – man spricht daher auch von „schwerer“ Masse. Daher ist der Anwendungsbereich der Zuordnungsgesetze Z1 und Z2 begrenzt: wo keine Gravitationskraft vorhanden ist, also im schwerelosen Raum des Weltalls, sind beide Zuordnungsgesetze nicht anwendbar. Will man Massenmessungen im schwerelosen Raum, z. B. in einem Space-Shuttle auf der Umlaufbahn durchführen, so muss man sich eines anderen Effektes der Masse bedienen, nämlich ihrer Trägheit in Stoßexperimenten. Mithilfe des Gesetzes von der Erhaltung des Impulses (= Masse mal Geschwindigkeit) vor und nach einem Stoß kann man Massenverhältnisse durch Stoßexperimente bestimmen. Beim sogenannten plastischen Stoß bleiben die aufeinanderstoßenden Massen nach dem Stoß aneinander haften; hierfür ergibt sich das folgende Zuordnungsgesetz:

5.1 Theoretische Begriffe und multiple Zuordnungsgesetze

(5.1-7) Z3: Wenn x auf eine ruhende Einheitsmasse mit Geschwindigkeit v plastisch stößt, dann gilt: (m(x) = v*/(v – v*)) $ (nach dem Stoß betrug die gemeinsame Geschwindigkeit v*). Man sagt auch, Z3 messe die träge Masse. Die physikalische Diskussion, ob schwere und träge Masse wirklich identisch seien, entstammt einer operationalistischen Perspektive. Sie wird durch die Hintergrundtheorie der Mechanik einfach deshalb mit „ja“ beantwortet, weil diese Theorie nur über ein theoretisches Merkmal namens „Masse“ spricht, und alle drei Zuordnungsgesetze Z1 – Z3 folgen aus dieser Hintergrundtheorie (s. Kap. 5.2). Es gibt zahlreiche weitere Zuordnungsgesetze bzw. Messmethoden für Massenwerte, die in speziellen Kontexten benötigt werden. Beispielsweise ist die Masse von Körpern auch aus ihrem Volumen und ihrer Dichte bestimmbar; Newton hatte auf diese Weise die Masse der Erde geschätzt. Erneut ergibt sich, dass zwei Zuordnungsgesetze zusammen empirischen Gehalt besitzen. Die empirische Prognose 9t(T1x^R1x) ! 8t(T2x!R2x), welche von Z1 und Z2 für die Massenmessung impliziert wird, lautet in Worten wie folgt: hat ein Gegenstand auf einer Balkenwaage k Masseeinheiten aufgewogen, so wird er eine geeichte Federwaage um k Längeneinheiten stauchen oder komprimieren. Keines dieser Zuordnungsgesetze kann als analytische Definition von Masse angesehen werden. Erneut ergibt sich zwingend: der Massebegriff wird durch die gesamte Hintergrundtheorie bestimmt, in der dieser Begriff vorkommt. Wie wir im nächsten Kapitel zeigen, werden alle grundlegenden Axiome der Mechanik benötigt, um die obigen Zuordnungsgesetze abzuleiten. Daraus folgt, dass sich mit der Änderung der Hintergrundtheorie auch die Bedeutung eines theoretischen Begriffs ändert. Masse bedeutet in der klassischen Mechanik etwas anderes als in der speziellen Relativitätstheorie. Diese Konsequenz wurde im Verlauf der Kuhn-Kontroverse sehr hervorgehoben, aber teilweise auch übertrieben: denn obwohl die Einsteinsche Masse geschwindigkeitsabhängig ist und die Newtonsche nicht, so haben sie immer noch vieles gemeinsam: z. B. gelten die obigen Zuordnungsgesetze auch in der speziellen Relativitätstheorie, wenn man sie in einem relativ zum gemessenen Körper ruhenden Bezugssystem betrachtet (vgl. Kanitscheider 1981, 13 f.). Die Einsicht, dass es unmöglich ist, in Theorien eine lokale analytischsynthetisch-Unterscheidung auf der Ebene ihrer einzelnen Axiome bzw. Theoreme durchzuführen, brachte eine tiefgreifende Umwälzung der aus dem logischen Empirismus hervorgegangenen Wissenschaftstheorie mit sich, die sich nur langsam durchsetzte. In (1936/37) hatte Carnap zwar erkannt, dass mehrere Reduktionssätze, die einen theoretischen Begriffe charakterisieren, empirischen Gehalt erzeugen (Vol. 3, 451), doch mit dem Begriff „Reduktionssatz“ blieb er dem semantischen Reduktionsprogramm treu (s. Kap. 1.2.2). In (1939) wechselt Carnap von der Bezeichnung „Reduktionssatz“ zur Bezeichnung „Interpretativer Satz“, aber so wie Hempel (1958, 208 ff.) separiert Carnap das interpretative System J von der Theorie T, ohne dabei J als analytisch zu bezeichnen. Dasselbe gilt für Carnap (1956), worin Zuordnungsgesetze ,Korrespondenzregeln‘ (K) genannt wer-

Abhängigkeit der Bedeutung von der Hintergrundtheorie

Langsame Abkehr vom semantischen Reduktionsprogramm

171

172

5. Theorien und ihre empirische Bewertung

Ockhamsches Rasiermesser

Empirische Signifikanz

den. Noch in (1963, 961) spricht Carnap von „T^K“ als der ,Gesamttheorie‘. Wir arbeiten hier und im nächsten Kapitel dagegen heraus, dass Zuordnungsgesetze gewöhnliche Konsequenzen einer Theorie T sind, die aus dem Zusammenwirken aller Axiome der Theorie folgen. In (1963, 964) resümiert Carnap, dass er bislang vergebens versucht hat, einen funktionierenden Analytizitätsbegriff für wissenschaftliche Theorien zu finden. Als Ausweg schlägt er erstmals die globale Zerlegung einer Gesamttheorie in einen synthetischen ,Ramsey‘-Satz und einen analytischen ,Carnap‘-Satz vor, welche in Kap. 5.8.2 erläutert wird. Dieser Vorschlag ändert aber nichts an der Einsicht, dass eine analytisch-synthetisch-Unterscheidung unter den einzelnen Axiomen oder Konsequenzen einer Theorie unmöglich ist. Auch Nagel gelangt in (1961) zur Einsicht, dass der analytische Gehalt der Mechanik nicht lokalisierbar ist (s. Stegmüller 1970, 111 ff., 124). Die Einführung eines theoretischen Begriffs ist gemäß dem Prinzip des Ockhamschen Rasiermessers nur dann gerechtfertigt, wenn dieser Begriff eine essentielle Funktion in der empirischen Systematisierungsleistung der Theorie besitzt, d. h. wenn dieser Begriff zur Erklärung und Voraussage empirischer Phänomene benötigt wird. Einen solchen theoretischen Begriff nennen wir empirisch signifikant. Die positivistische Wissenschaftstheorie sprach hier von kognitiver Signifikanz (z. B. Hempel 1951); aber die Identifikation von empirischer mit kognitiver Signifikanz scheint aus den schon in Kap. 2.5.3 angeführten Gründen zu eng zu sein: auch theoretische Begriffe, die (noch) keine empirische Funktion besitzen, können kognitiv bedeutungsvoll sein. Die (post-)positivistische Wissenschaftstheorie sah es als eine wichtige Aufgabe an, Kriterien dafür anzugeben, wann ein theoretischer Begriff empirisch signifikant bzw. wann er empirisch überflüssig ist (s. dazu Hempel 1951, Carnap 1956, 51 ff.; Stegmüller 1970, Kap. V). Es gab viele Rückschläge in diesem Unternehmen, die auch mit dem Problem irrelevanter Konklusionen von Kap. 3.10.3 zu tun hatten (s. Schurz 1991, § 5.1.4). Ohne auf die Hintergründe näher einzugehen, schlagen wir vor, einen theoretischen Begriff s genau dann als empirisch signifikant zu bezeichnen, wenn das gesamte Theoriennetz mindestens ein (möglichst bilaterales) Zuordnungsgesetz für s relevant impliziert. Zuordnungsgesetze können nicht nur für primitive, sondern auch für komplexe theoretische Begriffe bzw. theoretische Beschreibungen bestehen. Dieser Fall tritt in der Wissenschaftsgeschichte häufig auf: beispielsweise verfügte die ältere chemische Phlogistontheorie der Verbrennung über kein akzeptables Zuordnungsgesetz für die empirische Identifikation von Phlogiston; sie verfügte jedoch über bilaterale Zuordnungsgesetze für die komplexen Begriffe der Abgabe bzw. der Aufnahme von Phlogiston (s. Schurz 2009, § 4). In diesem Fall ist zwar der komplexe Begriff empirisch zugeordnet, aber der Status des primitiven Begriffs ist empirisch unterbestimmt. In einem direkten Zuordnungsgesetz bezeichnen Testbedingung T und Reaktion R empirische Merkmale. Oft ist es der Fall, dass T oder R selbst theoretische Bedingungen enthalten – in diesem Fall sprechen wir von einem konditionalen Zuordnungsgesetz: ein solches Gesetz fungiert als Zuordnungsgesetz für einen theoretischen Begriff s1(x) unter der Bedingung,

5.1 Theoretische Begriffe und multiple Zuordnungsgesetze

dass der Wert eines zweiten theoretischen Begriffs s2(x) schon bestimmt wurde. Carnap sprach hier von Reduktionsketten (1936/37, 434 – 436; 456 f.), und Balzer (1985, Kap. IV) arbeitete denselben Gedanken in Form sogenannter Messketten aus. Ein formales Beispiel einer Zuordnungskette:

Messketten

T1x ! (s1(x) $ R1x) (5.1-8) Direktes Z-Gesetz für s1: Konditionales Z-Gesetz für s1: T2x^s1(x) ! (s2(x)$R2x) Daraus folgt: direktes Z-Gesetz für s2: T1x^R1x^T2x ! (s2(x)$R2x)). Ein historisch bedeutendes Beispiel ist die Thomson-Millikansche Messkette zur Bestimmung von Elektronenmasse und Elektronenladung (Mortimer 1973, 12 f.). Thomson gelang es 1897 in seinem Kathodenstrahlexperiment, das Verhältnis von Elektronenladung und Elektronenmasse (e/m) zu bestimmen. Millikan gelang es 1909 in seinem Öltröpfchenexperiment, die Elektronenladung (Elementarladung) e zu bestimmen. Daraus ergab sich schließlich die Elektronenmasse (Elementarmasse) als 9,11N10 – 28 Gramm. Auch in den Human- und Sozialwissenschaften sind die meisten Begriffe theoretischer Natur. Die Zuordnungsgesetze, welche diese theoretischen Begriffe mit empirischen Begriffen verbinden, werden in den Human- und Sozialwissenschaften auch Indikatorbeziehungen bzw. Indikatorgesetze (oder Operationalisierungen) genannt. Sie sind ebenfalls hypothetische Bestandteile der jeweiligen Theorie, und keinesfalls per Definition wahr. (5.1-9) Human- und sozialwissenschaftliche Beispiele: Theoretische Begriffe: Indikatoren: Einstellung von Personen H G Antworten auf diverse Interviewfragen; j J Verhalten in experimentellen Situationen hg Kognitives Entwicklungsniveau eines Kindes

H G Resultat in verschiedenen kognitiven LeisjJ h g tungstests

Charaktertendenz von Personen, z. B. Aggressivität, emotionelle Labilität

H G Verhalten in gewissen experimentell vorff j J gegebenen Situationen; oder: Resultat f f eines Persönlichkeitstests (etc.) hg

Die politische Einstellung einer sozialen Schicht

H G Z. B. ihr durchschnittliches WählerverhaljJ h g ten; oder Umfrageergebnis

Die Indikatoren rechts sind spezifische empirische Dispositionen, aus deren Bedeutungscharakterisierung sich Zuordnungsgesetze bzw. Indikatorgesetze für die theoretischen Begriffe links ergeben. Die Form solcher Indikatorgesetze ist in den Human- und Sozialwissenschaften jedoch nur selten strikter Natur. Eine Person, die eine überwiegend konservative politische Einstellung besitzt, muss deshalb nicht bezüglich aller politischen Fragen konservativ antworten, sondern wird dies nur mit hoher probabilistischer Tendenz tun. Eine hochintelligente Person muss nicht in jedem Intelligenztest gut abschneiden, sondern nur, wenn sie in guter psychischer Verfassung ist, usw. Die Zuordnungsgesetze für theoretische Begriffe dieser Disziplinen sind also im Regelfall hochwahrscheinlicher oder normischer Natur und haben die folgende Form (dabei steht „A ) B“ für „wenn A, dann normalerweise B“; weil ,)‘ nicht verschachtelbar ist, wird konjunktiv zerlegt):

Indikatorgesetze in den Sozialwissenschaften

173

174

5. Theorien und ihre empirische Bewertung Probabilistische Zuordnungsgesetze

Wahrscheinlichkeitslogik

Homogenität eines theoretischen Merkmals

Likert-Skala

(5.1-10) Probabilistische und normische Zuordnungsgesetze: p(Rixt | Tixt^s(x)) = hoch, sowie p(s(x) | Tixt^Rixt) = hoch, bzw. Tixt^s(x) ) Rixt, sowie Tixt^Rixt ) s(x) Die probabilistischen bzw. normischen Konsequenzen solcher Zuordnungsgesetze folgen wahrscheinlichkeitstheoretisch, d. h. aufgrund der Wahrscheinlichkeitsaxiome (s. Kap. 3.9). Eine einfache Methode, diese Konsequenzen zu erhalten, ist die auf Adams (1975) zurückgehende Wahrscheinlichkeitslogik. Aus der Tatsache, dass mehrere Indikatoren mit einem theoretischen Begriff korrelieren, folgt jedoch nicht schon logisch, dass sie auch untereinander korrelieren, denn Korrelationen sind nicht unbedingt transitiv. Man benötigt hierzu probabilistische Irrelevanzannahmen, die per default generiert werden (s. Schurz 1997c). Beispielsweise schließt man mithilfe der default-Annahme, dass T2 für das Zuordnungsgesetz (Z1): p(s | T1^R1) F 1 – e1 nicht negativ relevant ist, auf (Z1*): p(s | T1^R1^T2) F 1 – e1. Analog schließt man vom Zuordnungsgesetz (Z2): p(R2 | T2^s) F 1 – e2 auf (Z2*): p(R2 | T1^R1^T2^s) F 1 – e2. Daraus folgt wahrscheinlichkeitslogisch gemäß der Regel der vorsichtigen Transitivität die empirische Hypothese (Z3:) p(R2 | T1^R1^T2) F 1 – e1 – e2. Es gibt ein semantisch vollständiges System wahrscheinlichkeitslogischer Regeln: das System P (s. Schurz 1998c). Auf diese Weise ergeben sich aus n probabilistischen Zuordnungsgesetzen für denselben theoretischen Begriff n2 empirische statistische Hypothesen, die zum probabilistischen Gehalt der Theorie zählen, so wie er in Kap. 3.7 expliziert wurde. Im Unterschied zu den Naturwissenschaften sind die Indikatorbeziehungen human- und sozialwissenschaftlicher Theorien in geringerem Grade durch die Theorie vorgegeben und gelegentlich mit willkürlichen Entscheidungen behaftet. Daraus ergeben sich spezielle Probleme, die nun erläutert seien. Für theoretische Begriffe der Human- und Sozialwissenschaften gibt es prima facie immer viele mehr-oder-weniger plausible Indikatoren. Die entscheidende Frage ist aber, ob diese Indikatoren tatsächlich ein homogenes theoretisches Merkmal messen. Zum Beispiel könnte man den konservativen Charakter von Vpn einerseits durch weltanschauliche Einstellungsfragen testen wie (a) „Sind Ihnen christliche Werte wichtig?“, oder durch praktische Einstellungsfragen wie (b) „Meiden Sie in Ihrem Leben häufige Veränderungen?“ Wenn Untersuchungen ergeben, dass viele Vpn Fragen vom Typ (a) positiv, aber solche vom Typ (b) negativ beantworten, bzw. umgekehrt, dann muss man schlussfolgern, dass diese Fragen kein homogenes theoretisches Merkmal messen. Die Eigenschaft eines Indikators, jenes theoretische Merkmal zu messen, das er messen soll, nennt man auch seine Konstruktvalidität (Huber 1987, 132; Bortz/Döring 2002, 57). Da sich theoretische Merkmale nicht direkt empirisch messen lassen, ist die Beurteilung der Konstruktvalidität eines Indikators nur indirekt möglich. Man misst beispielsweise, wie stark eine ganze Gruppe von Indikatoren für ein theoretisches Merkmal s untereinander korreliert; ist diese Korrelation hinreichend hoch, so schließt man daraus, dass alle Indikatoren dasselbe homogene theoretische Merkmal s messen. Raffinierte Verfahren dieses Typs sind die Likert- und die Guttman-Skala.

5.1 Theoretische Begriffe und multiple Zuordnungsgesetze

Diese Skalen werden in einem Vortest bestimmt, bevor der Haupttest durchgeführt wird. Pars pro toto sei die Likert-Skala erläutert (Mayntz et al. 1974, 55). Gehen wir davon aus, dass für ein theoretisches Merkmal, z. B. der Grad der patriarchalischen Einstellung einer Vp, verschiedene prima facie plausible Testfragen vorliegen, deren Antworten intervallskaliert sind auf einer Skala von 0 bis 4; dabei entspricht 4 maximaler patriarchalischer Einstellung. (Z. B. „Sind Frauen besser geeignet, Haushalt zu führen als Männer?“ 0: eindeutig nein, 1: eher nein, 2: unentschieden, 3: eher ja, 4: eindeutig ja.) Man führt mit diesen Fragen einen Vortest durch und bestimmt die 25%-Gruppe der Vpn mit dem höchsten Mittelwert auf alle Fragen, und die 25%-Gruppe mit dem geringsten Mittelwert auf alle Fragen. Dann wird für jede einzelne Frage der Unterschied zwischen dem Mittelwert in der oberen 25%-Gruppe und dem Mittelwert in der unteren 25%-Gruppe mit einem t-Test auf Signifikanz überprüft. Man wählt schließlich nur solche Fragen für den anschließenden Haupttest aus, deren Mittelwertsunterschied hochsignifikant ist: die Antworten auf diese Fragen korrelieren am stärksten untereinander, woraus man schließt, dass sie das fragliche theoretische Merkmal am besten messen. Dieser Schluss ist zwar plausibel, aber nicht zwingend: die interne Korrelation eines empirischen Indikatorbündels macht es zwar plausibel, dass damit ein homogenes zugrundeliegendes theoretisches Merkmal erfasst wird; ob dieses aber jenes ist, welches der Forscher ursprünglich im Sinn hatte, ist letztlich eine Frage, die nur im Kontext der gesamten Hintergrundtheorie beantwortbar ist. Das Ergebnis einer Untersuchung über ein theoretisches Merkmal hängt in hohem Maß davon ab, welchen Indikator man für das Merkmal wählt. In vielen Fällen legt man die Indikatoren für theoretische Merkmale nur nach Gutdünken fest – in solchen Fällen besteht die Gefahr zweierlei Arten von Indikatorverzerrungen: Verzerrte Indikatoren durch Einseitigkeit: Ein Beispiel hierfür sind die in Kap. 5.5 näher besprochenen kognitiven Leistungstests von Piaget. Zu gewissen kognitiven Entwicklungsniveaus von Kindern, die Piaget in einer hochtheoretischen Sprache beschreibt, sind viele verschiedene Indikatoren denkbar, und es zeigte sich, dass man bei Wahl anderer Indikatoren als jener von Piaget zu ganz anderen Ergebnissen gelangt. Zur Vermeidung von einseitigen Indikatoren ist daher folgende Regel einzuhalten:

Verzerrte Indikatoren durch Einseitigkeit

(Ms. 5.1-2) Eine theoretische Hypothese ist nicht nur anhand von einem, sondern anhand von so viel als möglichen gleichermaßen plausiblen Indikatoren zu testen. Wegen der erwähnten Unterbestimmtheit von Indikatoren in den Humanund Sozialwissenschaften ist die Befolgung dieser Regel wichtig. Verzerrte Indikatoren durch versteckte Variablen: Gelegentlich werden in Indikatoren versteckte Variablen eingeschleust, die mit dem theoretischen Begriff, der gemessen werden soll, wenig zu tun haben. Ein Beispiel ist die Interpretation von Antworten auf Einstellungsfragen. Häufig werden solche Antworten als direkte Indikatoren für die Einstellung von Personen verwen-

Verzerrte Indikatoren durch eingeschleuste Variablen

175

176

5. Theorien und ihre empirische Bewertung

det. Z. B. dient die Frage „wie wichtig ist für Sie Umweltschutz, im Vergleich zu Wirtschaftswachstum?“ als Maß für Ökologiebewusstsein. Was aber in Interviews gemessen wird, ist zunächst nur die verbale Selbsteinstellung, also die Selbstbeurteilung der eigenen Einstellung, welche mit der tatsächlichen Einstellung im praktischen Verhalten nicht unbedingt übereinstimmen muss (psychologische Studien belegen, dass die meisten Vpn zu übertrieben positiver Selbsteinschätzung neigen, s. Krebs 1998). Die versteckte Variable ist hier also der Wahrhaftigkeitsgrad der Selbstbeurteilung. Daraus können wir folgende methodische Regel gewinnen: (Ms. 5.1-3) Für jeden Indikator ist zu prüfen, ob und welche möglichen versteckten Variablen durch ihn eingeschleust werden können.

5.2 Das Beispiel der Newtonischen Physik Die Newtonische Physik ist ein Musterbeispiel einer (guten) wissenschaftlichen Theorie. An ihrem Beispiel erläutern wir in diesem Kapitel dreierlei: (1.) den Systemcharakter und Überprüfungsholismus wissenschaftlicher Theorien, (2.) die Unterscheidung zwischen den Axiomen einer Theorie und den daraus abgeleiteten Konsequenzen, und (3.) innerhalb der Axiome einer Theorie die graduelle Unterscheidung zwischen dem Kern einer Theorie und ihrer Peripherie. Während der Theoriekern jene Gesetze umfasst, die für die Theorie von entscheidender Bedeutung sind und ihre Identität definieren, enthält die Peripherie spezielle Hypothesen von geringerer Bedeutung, welche ohne die Identität der Theorie zu gefährden abgeändert oder preisgegeben werden können. Die grundlegenden empirischen bzw. vortheoretischen Grundbegriffe der klassischen Physik sind die singulären Terme für physikalische Körper (x2D) und Zeitpunkte (t2T) als unabhängige Variable, und die Ortsfunktion (s), Geschwindigkeit (v) und Beschleunigung (a) als abhängige Variable (zum Begriff ,vortheoretisch‘ s. Kap. 5.3). Dabei sind Geschwindigkeit und Beschleunigung als erste bzw. zweite Ableitung des Ortes nach der Zeit definiert. Die grundlegenden theoretischen Begriffe der klassischen Physik sind die Masse- und die Kraftfunktion. Eine Vielzahl weiterer Begriffe, wie z. B. Impuls, Arbeit und Energie, werden mithilfe dieser Begriffe definiert. Das wichtigste Kernaxiom der Newtonschen Mechanik ist folgendes: Erstes Kernaxiom der newtonischen Physik

(5.2-1) (N1) Gesamtkraft = Masse mal Beschleunigung Genauer: Die Summe aller auf einen Körper x zu einer Zeit t ausgeübten Kräfte ist gleich dem Produkt der Masse von x und der Beschleunigung von x zu t. Formal: Für alle x in D und t in T: Ri2I fi(x,t) = m(x) N a(x,t). Notation: D = Menge physikalischen Körper; T = Menge der Zeitpunkte; m(x) = die zeitunabhängige skalare Masse von x, a(x,t) = der zeitabhängige

5.2 Das Beispiel der Newtonischen Physik

Beschleunigungsvektor von x, und fi = der auf x wirkende zeitabhängige Kraftvektor vom Typ i. Beachte: I ist eine variable, d. h. durch (N1) nicht fixierte Indexmenge, die alle auf x zu t wirkenden Einzelkräfte indiziert. (N1) ist das zweite Axiom von Newtons Principia; sein erstes Axiom „jeder kräftefreier Körper verharrt in Ruhe oder gleichförmiger Bewegung“ ist eine Folge von (N1), denn aus a(x,t) = 0 folgt Ri2I fi(x,t) = 0. Beachte: (N1) macht gleich zwei Aussagen. Erstens, dass die Gesamtkraft die Summe aller Einzelkräfte ist, dass also die Kräfte in ihrem Zusammenwirken nicht interagieren. Dies ist entscheidend für das Prinzip der vektoriellen Zusammensetzung von Gesamtkräften aus Einzelkräften. Zweitens besagt (N1), dass die derart vektoriell zusammengesetzte Gesamtkraft gleich dem Produkt von Masse und Beschleunigung ist. Axiom (N1) ist ein gemischtes Axiom, da es sowohl theoretische wie nichttheoretische Begriffe enthält. (N1) liefert allein jedoch noch kein Zuordnungsgesetz für Masse oder Gesamtkraft. Nur das Verhältnis der beiden ist durch (N1) als die Beschleunigung empirisch bestimmt. Aus demselben Grund hat das Axiom (N1) allein keinen empirischen Gehalt, d. h., es folgen daraus keine gehaltvollen Prognosen über die Bewegungen von Körpern. Auf Sachverhalte dieser Art hat insbesondere Sneed (1971, 118, 127) aufmerksam gemacht. Er benutzt dabei die modelltheoretische Sprechweise: dass (N1) in Isolation betrachtet empirisch leer ist, besagt modelltheoretisch, dass sich für jedes nichttheoretische Modell (D,T,s) Masse- und Kraftfunktionen m und F(x,t):=Ri2Ifi(x,t) finden lassen, welche dieses zu einem (N1) erfüllenden theoretischen Modell (D,T,s,m,F) ergänzen – man wähle die Funktionen F und m einfach so, dass ihr Quotient mit der Beschleunigungsfunktion zusammenfällt. Das zweite Kernaxiom (N2) von Newtons Theorie ist das Aktio=ReaktioPrinzip (das dritte Axiom der Principia). Im Gegensatz zu (N1), welches über die Gesamtkraft spricht, macht (N2) eine Aussage über einzelne Kräfte: (5.2-2) (N2) Wirken geradlinig gerichtete Kräfte zwischen zwei Körpern x und y, so gilt Kraft = Gegenkraft, d. h. der von x auf y wirkenden Kraft entspricht eine von y auf x wirkende betragsgleiche Gegenkraft. Verallgemeinert: die Summe aller Kräfte, die in einem abgeschlossenem System wirken, beträgt null. Wenn also zwei Körper durch Stoßwirkung oder durch Gravitation aufeinander eine Kraft ausüben, so ist die vom ersten auf den zweiten Körper ausgeübte Kraft betragsmäßig gleich der vom zweiten auf den ersten Körper ausgeübte Kraft, und von entgegengesetzter Richtung. (N2) enthält nur theoretische Begriffe und ist also ein rein theoretisches Gesetz. Es ist ebenfalls ein Kerngesetz, aber nicht mehr ganz so allgemein wie (N1) (vgl. Balzer et al. 1987, 182). Kombiniert man (N1) mit (N2), so gewinnt man gemäß der Standardinterpretation, wie man sie insbesondere bei Sneed (1971, 129 ff.) findet, bereits empirischen Gehalt. Diese Interpretation ist strenggenommen unrichtig: ein solcher empirischer Gehalt folgt nur, wenn zusätzlich sogenannte Systembe-

Zweites Kernaxiom der newtonischen Physik

177

178

5. Theorien und ihre empirische Bewertung Die Rolle von Systembedingungen

dingungen (oder Randbedingungen im weiten Wortsinn) angenommen werden, welche besagen, dass nur die-und-die Kräfte im betrachteten System anwesend sind – anders ausgedrückt, wenn die Indexmenge I in (N1) definitiv bestimmt wird. Unter der Systembedingung, dass keine äußeren Kräfte anwesend sind, ergibt sich aus (N1 + N2) das Impulsgesetz für den plastischen und elastischen Stoß zweier Körper, aus dem auch das Zuordnungsgesetz Z3 (5.17) von Kap. 5.1 für die Massenmessung durch plastischen Stoß folgt. Sodann gewinnt man aus (N1 + N2), zusammen mit der Definition des Drehmomentes, das allgemeine Hebelgesetz, Kraft mal Kraftarm = Last mal Lastarm, und damit das Zuordnungsgesetz Z1 (5.1-6) zur Massenbestimmung auf der Balkenwaage (Sneed 1971, 131 ff.); erneut aber nur unter der Systembedingung, dass auf die Objekte auf beiden Waagarmen die gleiche nach unten gerichtete Gravitationskraft wirkt, und sonst keine. Aus der Kombination dieser zwei Zuordnungsgesetze ergibt sich empirischer Gehalt, der die Voraussage einiger qualitativ neuartiger Phänomene ermöglicht: so kann man beispielsweise die Geschwindigkeitsverhältnisse von Körpern beim plastischen Stoß im schwerelosen Raum vorhersagen, deren Massen auf einer Balkenwaage auf der Erde gemessen wurden. Allgemeiner folgt aus (N1) und (N2) zusammen mit den Definitionen von ,Energie‘ und ,Impuls‘ sowie der Systembedingung, dass das gegebene System von Körpern geschlossen ist, d. h. keine äußeren Kräfte einwirken, der Energieerhaltungssatz und Impulserhaltungssatz der Mechanik (s. z. B. Gerthsen/Kneser 1971, 28 – 30). (N1 + N2) liefern zusammen mit Systembedingungen Zuordnungsgesetze für die Massenmessung und die wirkende Gesamtkraft, aber sie sagen noch nichts über die Natur einzelner Kraftarten aus. In den Ableitungen der Massenmessgesetze aus (N1 + N2) und Systembedingungen kürzen sich die wirkenden Kräfte mithilfe des Aktio-Reaktio-Prinzips (N2) heraus. Aus diesem Grund ist der empirische Gehalt von (N1 + N2) plus Systembedingungen noch vergleichsweise bescheiden. Eine wahrhafte Explosion des empirischen Gehaltes wird erzeugt, indem zu den Kerngesetzen (N1 + N2) spezielle Kraftgesetze, also Zuordnungsgesetze für einzelne Kraftarten hinzugefügt werden; wie z. B. das von Newton formulierte Gesetz der Gravitationskraft:

Gravitationskraft

(5.2-3) (N3) Die Gravitationskraft, die ein Körper x auf einen Körper y ausübt, ist betragsmäßig proportional zum Produkt der beiden Massen, dividiert durch ihr Abstandsquadrat, und von y nach x gerichtet (x zieht y in gerader Linie zu sich heran). (N1) – (N3) liefern diverse Differentialgleichungen der klassischen Gravitationsmechanik, deren exakte oder approximative Lösung es ermöglicht, die Bewegung von beliebigen Körpern in beliebigen Schwerefeldern vorauszusagen – in Abhängigkeit von den Systembedingungen, die für den gesamten betrachteten Zeitraum spezifizieren, welche anwesenden Körper eine nichtvernachlässigbare Gravitationskraft ausüben, sowie von den Anfangsbedingungen, welche den empirischen Zustand des betrachteten Systems zu einem beliebig gewählten ,Anfangs‘zeitpunkt spezifizieren. Systembedingungen liefern ein theoretisches Modell des betrachteten Systems im betrachteten Gesamtzeitraum, und Anfangsbedingungen liefern eine empiri-

5.2 Das Beispiel der Newtonischen Physik

sche Momentaufnahme des Systems zu einem Anfangszeitpunkt. Eine Vielzahl von qualitativ unterschiedlichen Phänomenen werden damit voraussagbar bzw. erklärbar, wie z. B.: 1) die Bewegungen von starren Körpern auf der Erdoberfläche bei konstanter Schwerkraft, wie der freie Fall, die parabolische Flugbahn von Wurfgeschossen, Bewegung auf schiefer Ebene, das Gravitationspendel, Rotationsbewegungen um Achsen, Hebelwirkungen, Kreisel, etc.; 2) die Bewegung von Himmelskörpern in Schwerefeldern des Weltraums, z. B. die Planetenbahnen um die Sonne, die Erdrotation, die Mondbahn um die Erde, verbunden damit der Wechsel der Jahreszeiten, Tag-und-Nacht, Ebbe und Flut, bis zu Kometenbahnen und zur Voraussage der Bahnen von Raumschiffen. 3) Durch Übertragung der Mechanik von Massenpunkten und Massenkugeln auf aus Atomen zusammengesetzt gedachte Stoffe ergeben sich die mechanischen Gesetze der Feststoffe, Flüssigkeiten und insbesondere der Gase, in Bezug auf Temperatur- und Druckverhältnisse, Reibungsphänomene, Strömungslehre, usw. Zahlreiche bislang nur durch empirisch-induktive Generalisierung bekannte Phänomene wurden so theoretisch einheitlich erklärbar (Archimedes‘ Hebelgesetz, Descartes‘ Stoßgesetze, Galileis Fallgesetz, Keplers Planetengesetze, usw.). Dies ist noch längst nicht alles, denn bislang wurde nur ein einziges Kraftgesetz, das Gravitationsgesetz, herangezogen. Es gibt eine Reihe weiterer spezieller Kraftgesetze, die andere physikalische Kraftarten beschreiben, z. B. Stoffkräfte wie die Hookesche Federkraft oder die Reibungskraft; über diese Kraftgesetze werden alle periodischen Schwingungsund Wellenvorgänge in mechanisch-elastischen Medien erklärbar. Durch Miteinbezug der Coulombschen elektrostatischen Kraft sowie deren elektrodynamische Verallgemeinerung werden schließlich elektrostatisch oder elektrodynamisch verursachte Bewegungsvorgänge erfasst, von elektrischen Stromkreisen bis hin zum Bohrschen Atommodell. Es darf nicht unerwähnt bleiben, dass in der Verbindung von klassischer Mechanik und Elektrodynamik eine Inkohärenz bestehen blieb, deren Lösungsversuche in Bezug auf Lichtausbreitungsphänomene zur speziellen Relativitätstheorie und in Bezug auf Atommodelle zur Quantenmechanik führten. Die skizzierte Vielfalt der empirischen Anwendungen der klassischen Mechanik ist in Abb. 5.2-1 übersichtlich dargestellt. Abb. 5.2-1 macht zugleich deutlich, dass sich physikalische Theorien in Form von hierarchischen Theorienverbänden organisieren. An der Spitze stehen gewisse allgemeine Kerngesetze, angereichert durch spezielle Kraftgesetze, welche in Anwendung auf spezifische Systeme hin durch Systembedingungen spezialisiert werden. Die Einheiten solcher Theorieverbände bezeichnen wir (wie die Strukturalisten) als Theorienelemente; die von oben nach unten gerichtete Relation zwischen diesen Elementen ist die Spezialisierungsrelation. (Hinweis: „Theorienverbände“ werden von den Strukturalisten „Theoriennetze“ genannt, während wir die Bezeichnung „Theoriennetz“ für Beziehung zwischen Theorienverbänden reservieren.) Damit haben wir am Beispiel der Newtonischen Theorie drei wesentliche Eigenschaften guter wissenschaftlicher Theorien kennengelernt, nämlich (i)

Explosion des empirischen Gehalts

Weitere spezielle Kraftgesetze

Theorienverband Theorienelemente

Spezialisierungsrelation

179

180

5. Theorien und ihre empirische Bewertung

Kernaxiome der Mechanik Addition

F = m a & Aktio = Reaktion

spezieller

(direkte Konsequenz: Impuls-, Energieerhalt.)

Kraftgesetze Gravitationskraft (plus Reibungskraft)

Hookesche Federkraft (plus Reibungskraft)

Elektromagnetische Kraft (plus Reibungskraft)

Einsetzen spezieller Systembedingungen Stabile Planetenbahnen

Bewegungen auf der Erdoberfläche

Schwingungs- und Wellenvorgänge

Stoßvorgänge, Hebelwirkung

Einsetzen spezieller Anfangsbedingungen Prognose einzelner Bewegungsvorgänge oder Phänomene Einz. Planetenbahn, Monde, Komet, Rakete, Jahresz., Gezeiten

Freier Fall, Schiefe Ebene Wurfgeschosse

Massenmess. per Balkenw., Stoß

Pendel, freie u. stehende Welle Reflexion, Brechung, Massenmessung durch Federwaage

Abb. 5.2-1: Theorienverband der klassischen Mechanik und seine Anwendungsvielfalt

Qualitativ neue Voraussagen

Bestätigungssurplus von (qualitativ neuen) Voraussagen

ihren Systemcharakter, d. h. der empirische Gehalt kommt erst durch das Zusammenwirken einzelner Axiome zustande, die in Isolation betrachtet empirisch gehaltlos sind, (ii) verbunden damit die empirische Kreativität solcher Theorien, sowie (iii) ihre Globalität und Vereinheitlichungskraft, d. h., qualitativ unterschiedlichste Phänomene werden durch dieselben theoretischen Mechanismen erklärt. Insbesondere die dritte Eigenschaft (iii) bildete die Grundlage des überwältigenden Erfolges der klassischen Physik. So gelang es der Newtonschen Mechanik, die im Aristotelischen Weltbild zuvor getrennte Physik der irdischen Körper und die der Himmelskörper unter eine gemeinsame Theorie zu subsumieren. Verbunden mit der dritten Eigenschaft ist auch die Fähigkeit von guten wissenschaftlichen Theorien, qualitativ neuartige Phänomene vorauszusagen, von denen bislang noch keine einzige individuelle Instanz beobachtet wurde. Pars pro toto erwähnen wir die Voraussage der Newtonischen Mechanik, dass die Gravitationskraft auch zwischen gewöhnlichen irdischen Körpern wirken sollte. Da derartiges im Alltag nirgendwo beobachtbar ist, war diese Voraussage so unglaublich, dass sie die Mehrheit der Zeitgenossen Newtons anzweifelten. Erst als Henry Cavendish 1798 die Massenanziehung zwischen zwei Stahlkugeln mithilfe seiner Torsionswaage demonstrierte, konnte auch der Common Sense von der Richtigkeit des Newtonischen Gravitationsgesetzes überzeugt werden. In Kap. 4.5.1 haben wir am Beispiel der Kurvenanpassung gesehen, dass neue Voraussagen, wenn sie zutreffen, einen ungleich höheren Bestätigungswert für Gesetzeshypothesen besitzen als ex-post-fakto Erklärungen. In noch höherem Maß trifft dies für die Bestätigung von Theorien durch qualitativ neue Voraussagen zu, wie dies von Worrall (1997, 153 f.) und Carrier (2003, § 4) u. a. betont wird. Beispielsweise liegt keine über Keplers empirische Planetengesetze hinausgehende Bestätigung der Newtonschen Himmelsmechanik vor, wenn zum 100sten Mal die elliptische Bahn des Mars

5.2 Das Beispiel der Newtonischen Physik

beobachtet wurde. Aber als die Voraussage bestätigt wurde, dass selbst eine von der Erde geeignet hochgeschossene Raumkapsel in eine stabile Umlaufbahn geraten wird, war das ein gewaltiger Bestätigungserfolg der Newtonischen Theorie. Qualitativ neue Voraussagen können unmöglich durch empirische Induktion gewonnen werden, da empirische Induktion immer nur bereits beobachtete Phänomene typengleich generalisieren kann. In Schurz (2009, § 5) wird argumentiert, dass für eine qualitativ neue Voraussage immer ein theoretisches Systemmodell benötigt wird. Aufgrund ihrer überragenden Erfolge wuchs die Newtonsche Physik im 18. und 19. Jahrhundert zu einem universalen Erklärungsparadigma heran, das mit dem Erklärungsversprechen bzw. der Programmatik einherging, dass sich auf seiner Grundlage alle Naturphänomene erklären lassen müssten: dazu schien es bloß nötig zu sein, die für einen gewissen Phänomenbereich verantwortlichen speziellen Kraftgesetze zu finden (s. Kitcher 1990 f., 203). Das Newtonsche Programm ist ein typisches Beispiel für das, was Kuhn mit seinem Paradigmenbegriff im Auge hatte (s. Kap. 1.2.4), und was Lakatos mit seinem Begriff des Forschungsprogrammes beschrieb (Kap. 5.6.1). Zwar wurde die klassische Physik im 20. Jahrhundert durch Relativitätstheorie und Quantenmechanik abgelöst; doch für Gegenstände mittlerer Größenordnung ist sie nach wie vor approximativ gültig, und in Anwendung auf komplexe Systeme ist sie noch heute ein progressives Forschungsprogramm. Wir haben in diesem Kapitel auch zwei wichtige Besonderheiten physikalischer Theorien kennengelernt. Erstens, dass ihre grundlegenden Gesetze als Differentialgleichungen formuliert sind (s. Lauth/Sareiter 2002, 64). In Differentialgleichungen wird die Zustandsfunktion des betrachteten Systems, im mechanischen Fall die Ortsfunktion s(x,t), als Funktionsvariable behandelt, und es wird ein Zusammenhang hergestellt zwischen dem Zustand des Systems zu einem variablen Zeitpunkt t und der Veränderungsrate des Zustandes zu t. D. h., die Funktion s(x,t) und ihre Ableitungen nach t stehen in einem gesetzesmäßigen Zusammenhang. Beispielsweise erhält man die Differentialgleichung des um eine ruhende Sonne kreisenden Planeten, indem man die variable Kraftfunktion im Kernaxiom (N1) durch die Gravitationskraft zwischen Sonne und Planeten gemäß (N3) ersetzt. Das Lösen einer Differentialgleichung bedeutet, konkrete Zustandsfunktionen s(x,t) aufzufinden, welche den von ihr geforderten Zusammenhang erfüllen. Jede Differentialgleichung besitzt eine ganze Menge möglicher Lösungen, die alle möglichen Bewegungsabläufe (Trajektorien) des betrachteten Systemtyps beschreiben; durch Einsetzen von empirischen Anfangsbedingungen gewinnt man daraus eine eindeutige Lösung. Differentialgleichungen leisten eine besondere Vereinheitlichungsfunktion. Die Vereinheitlichungsfunktion qualitativer Theorien kommt dadurch zustande, dass derselbe theoretische Begriff durch verschiedene Zuordnungsgesetze operationalisiert wird. Im Fall von theoretischen Differentialgleichungen kann bereits durch ein einziges Zuordnungsgesetz wie z. B. das Gravitationsgesetz eine Subsumtion qualitativ unterschiedlicher Phänomene erzeugt werden, weil die Variation von System- und Anfangsbedingungen zu ganz unterschiedlichem Systemverhalten führt.

Das newtonische Forschungsprogramm

Differentialgleichungen

181

182

5. Theorien und ihre empirische Bewertung

Systembedingungen und exklusive CP-Klauseln

Idealisierung

Approximative Wahrheit

Die zweite Eigentümlichkeit physikalischer Theorien ist das erläuterte Zusammenspiel von fundamentalen Naturgesetzen wie (N1 – 3) und Systembedingungen. Systembedingungen sind typische theoretische Hilfshypothesen im Sinne von Lakatos (1974), die an der Peripherie der Gesamttheorie angesiedelt sind: sie listen alle nichtvernachlässigbaren Kräfte auf, und involvieren daher eine exklusive ceteris-paribus (eCP) Klausel, die besagt, dass außer den erwähnten Kräften keine sonstigen anwesend sind. Die eCP-Klausel physikalischer Theorien ist jedoch theoretisch definit im Sinne von Kap. 3.5. Denn in Axiom (N1) sind, gemäß gängiger physikalischer Interpretation, nicht beliebige, sondern nur physikalische Kräfte gemeint (s. Schurz 2002b, § 6.1). Der Begriff der Gesamtkraft ist zwar nicht innerhalb einer einzelnen physikalischen Theorie definiert, aber das gesamte Theoriennetz der Physik liefert eine theoretische Klassifikation aller Kraftarten und verbinden damit einen Vollständigkeitsanspruch: nach gegenwärtigem Stand sind dies die vier fundamentalen Wechselwirkungskräfte (Gravitationskraft, elektromagnetische Kraft, starke und schwache Kernkraft) sowie die bei Stoßvorgängen auftretenden Trägheitskräfte. Freilich kann das Vorliegen theoretischer Systembedingungen aus empirischen Systembeschreibungen immer nur unsicher erschlossen werden. Insbesondere involviert ein theoretisches Systemmodell immer ein gewisses Maß an Idealisierung, da für kein reales physikalisches System strikt alle Kraftwirkungen berücksichtigt werden können, sondern nur die hauptsächlichen; die geringfügigen werden als vernachlässigbar betrachtet. Beispielsweise wird bei der Vorhersage einer elliptischen Planetenbahn nur die Kraft der Sonne berücksichtigt; in einem komplexeren Ansatz auch eine eventuelle Störung durch die Gravitationskraft anderer Planeten; aber niemand kann alle winzigen Einflüsse (z. B. die Kraft des Sonnenwindes oder kleiner kosmischer Stäube) auch noch in die Differentialgleichung mit aufnehmen. Man muss daraus keineswegs so radikale Konsequenzen ziehen wie Cartwright (1983), derzufolge physikalische Theorien im Grunde lügen, oder wie van Fraassen (1989, Kap. 8) und Giere (1999), denen zufolge es physikalische Gesetze im strikten Sinn gar nicht gäbe. Vielmehr folgt daraus lediglich die bekannte Tatsache, dass die quantitativen Vorhersagen physikalischer Theorien niemals strikt wahr, sondern immer nur approximativ wahr sind – je näher die Daten an der vorausgesagten Kurve liegen, desto besser die Approximation. (Davon abgesehen enthält auch jede Messung eine Ungenauigkeit – ein zweiter Grund für diese Approximationsbeziehung.) Aus dem Systemcharakter (guter) wissenschaftlicher Theorien folgt schließlich auch der Holismus ihrer empirischen Überprüfung, dem wir uns abschließend zuwenden. Das Argumentschema (5.2-4) zeigt in qualitativer Darstellung, wie sich aus der Newtonischen Theorie eine konkrete empirische Prognose gewinnen lässt. Trifft die Prognose (approximativ) zu, so hat sich das Gesamtsystem der Prämissen bewährt. Wir nennen das Gesamtsystem aller zu diversen Prognosen verwendeten Prämissen auch die aktuelle Version einer Theorie. Trifft die Prognose dagegen nicht zu, liefern also die Beobachtungen der Planetenbahn eine signifikante Abweichung von der vorausgesagten Bahn, so lässt sich daraus nur schließen, dass mindestens eine der Prämissen der ak-

5.2 Das Beispiel der Newtonischen Physik

(5.2-4) Kernaxiom N1 (Gesamtkraft = Masse mal Beschleunigung) Kernaxiom N2 (Kraft = Gegenkraft) Spezifisches Gesetz der Gravitationskraft (Semi-Kern) Hilfshypothese = Systembedingung: Auf den Planeten p wirkt nur die Gravitationskraft der Sonne.

2 Prognose trifft zu: Gesamtsystem ist bewährt!

2

Prognose: Die Bahn des Planeten p ist die-und-die Funktion von der Zeit und von den Anfangsbedingungen von p. Prognose trifft nicht zu: Überprüfungsholismus – wo liegt der Fehler? Ad-hoc-Kernschutz: In Hilfshypothesen!

tuellen Theorieversion falsch gewesen sein muss, aber man weiß nicht, welche Prämisse den Fehler enthält. Dies ist der sogenannte Holismus der Überprüfung. Lakatos (1974, 98 ff.) hat überzeugend demonstriert, dass im Fall eines Konflikts einer aktuellen Theorienversion mit der Erfahrung die Theorievertreter zunächst dazu tendieren, nicht den Theoriekern anzuzweifeln oder gar preiszugeben, sondern den Theoriekern dadurch vor Falsifikation schützen, dass die Hilfshypothesen an der Peripherie der Theorie aufgegeben oder abgeändert werden (s. Kap. 5.6.1). Das heißt, dass neue bisher unbeobachtete Störfaktoren postuliert werden, welche in den Systembedingungen noch nicht berücksichtigt wurden und die beobachteten Abweichungen von den theoretischen Voraussagen erklären sollen. Dies klingt wie ein gewaltiges ad-hoc-Manöver, aber stellt sich bei genauerer Betrachtung als ein plausibles Vorgehen heraus, welches nur dann als ad-hoc-Manöver kritisierbar ist, wenn der postulierte unbekannte Störfaktor sich nicht unabhängig beobachten oder bestätigen lässt. Hier zwei wissenschaftsgeschichtliche Beispiele (vgl. Grünbaum 1976, 332-335, 358): (1.) Als J. Adams und U. Le Verrier 1846 eine beträchtliche Abweichung des Planeten Uranus von der vorausgesagten Bahn entdeckten, postulierten sie einen bisher unbekannten Planeten Neptun, dessen Gravitationswirkung auf den Uranus für dessen Abweichung von der vorausgesagten Bahn verantwortlich sei. Sie berechneten aufgrund der beobachteten Ablenkung des Uranus in etwa Größe und Bahn dieses Planeten und erklärten die Tatsache, dass ihn bislang noch niemand beobachtet hatte, damit, dass er zu klein sei, um von den bisherigen Teleskopen gut gesehen zu werden. In der Tat konnte der Planet Neptun ein wenig später mit stärkeren Teleskopen beobachtet werden, was als glänzender Erfolg der Newtonischen Physik gewertet wurde. (2.) Andererseits beobachtete Le Verrier auch eine Abweichung des Planeten Merkur von seiner vorausgesagten Bahn und postulierte zur Erklärung dieser Abweichung ca. 1856 einen noch kleineren Planeten namens Vulkan. Dieser Planet konnte trotz hartnäckiger Versuche nicht teleskopisch aufgefunden werden. Erst wesentlich später gelang es, die Abweichungen des Merkur von seiner vorausgesagten Bahn zu erklären – aber nicht durch

Holismus der Überprüfung

Störfaktoren und Ad-hoc-Hypothesen

183

184

5. Theorien und ihre empirische Bewertung

Abänderung einer Hilfshypothese, sondern durch Übergang zu einem neuen Theoriekern: die Abweichungen konnten erst innerhalb der allgemeinen Relativitätstheorie erklärt werden. Eine Hypothese H nennt man ad hoc, in Relation zu einem epistemischen Hintergrundsystem W zu einem gegebenen Zeitpunkt t, wenn H nur deshalb angenommen wurde, um einen Theoriekern in W mit einer speziellen empirischen Beobachtung in W verträglich zu machen. H besitzt in W zum Zeitpunkt t also keine unabhängigen Bestätigungen. Die beiden Beispiele lehren uns zweierlei: die momentane ad-hoc-Artigkeit einer Hypothese darf noch nicht als positive oder negative Wertung für die Zukunft aufgefasst werden. Alles hängt von der zukünftigen Entwicklung ab – speziell von zwei Fragen: 1) produziert die Hypothese H im Hintergrundwissen W neuen empirischen Gehalt, durch den sie sich überprüfen lässt?, und 2) führt diese Überprüfung, sofern sie möglich ist, zu einer Bestätigung von H? Sofern 1) eintritt, wird die Hypothese entweder zu einer empirisch bewährten oder (im negativen Überprüfungsfall) zu einer empirisch falsifizierten oder geschwächten Hypothese; in beiden Fällen verliert sie ihren ad-hoc-Charakter. Nur wenn Fall 1) nie eintritt, wenn also H keinen neuen empirischen Gehalt produziert, schlägt der ad-hoc-Charakter von H zum Permanenzzustand und damit ins Negative um. Über den Begriff der ad-hoc-Artigkeit gab es breite wissenschaftstheoretische Kontroversen (z. B. Popper 1935/76, 51 f.; Hempel 1974, 44 – 47; Grünbaum 1976; Lakatos 1974, 121 f., 169).

5.3 Grundlegendes zur Theorienstatik: Struktur wissenschaftlicher Theorien

Untergliederungen von Theorien

Gemäß unseren bisherigen Ausführungen können wir wissenschaftliche Theorien T auf viererlei Weise in Bestandteile gliedern (s. Abb. 5.3-1 unten). (1.) Hinsichtlich der Sprache der Theorie: da gibt es einmal die theoretische Gesamtsprache, welche alle Begriffe von T und alle damit bildbaren Ausdrücke enthält; und als Teilsystem davon die empirische bzw. vortheoretische Teilsprache von T, welche neben den logisch-mathematischen Begriffen von T nur empirische oder vortheoretische Begriffe von T sowie die damit bildbaren Ausdrücke enthält. (2.) Hinsichtlich der logischen Natur der Sätze von T: hier unterschieden wir zwischen den Axiomen von T, und den logischen (oder probabilistischen) Konsequenzen von T. Die Axiome von T sollen natürlich sein: darunter verstehen wir elementare nichtredundante Gesetzeshypothesen, aus der alle anderen Behauptungen von T als logische (oder probabilistische) Konsequenzen folgen. Wir identifizieren die Theorie T mit der Menge ihrer Axiome, während wir die Konsequenzen im logischen (oder probabilistischen) Gehalt C(T) von T zusammenfassen (bzw. im relevant repräsentierten Gehalt Cr(T) gemäß Kap. 3.10.3). (3.) Hinsichtlich der begrifflichen Natur der Sätze von T – diese Unterscheidung gilt sowohl für die Axiome wie für die Konsequenzen von T: hier unterscheiden wir zwischen rein theoretischen Gesetzen von T, welche neben logisch-mathematischen Begriffen nur theoretische Begriffe enthalten,

5.3 Grundlegendes zur Theorienstatik: Struktur wissenschaftlicher Theorien

gemischten Sätze von T, welche sowohl empirische/vortheoretische und theoretische Begriffe enthalten, und empirischen/vortheoretischen Konsequenzen von T, welche neben logisch-mathematischen Begriffen nur empirische/vortheoretische Begriffe enthalten. Eine Untermenge der gemischten Sätze von T sind die Zuordnungsgesetze von T. Im Normalfall enthalten die Axiome von T nur rein theoretische Gesetze oder Zuordnungsgesetze, während die empirischen/vortheoretischen Sätze sich als Konsequenzen ergeben. (4.) Hinsichtlich des epistemischen Status der Sätze von T: hier unterscheiden wir graduell zwischen dem Kern und der Peripherie von T. Diese Unterscheidung ist nur für die Axiome von T unproblematisch. Die Kernaxiome von T definieren die historische Identität einer Theorie – werden Kernaxiome verändert, so handelt es sich um eine andere Theorie. Periphere Hypothesen von T (z. B. Systembedingungen) charakterisieren nur die aktuelle Version einer Theorie zu einem bestimmten Zeitpunkt bzw. in einer bestimmten Anwendung – werden sie geändert, so ändert sich die aktuelle Theorieversion, aber nicht die Theorie selbst. Dazwischen liegen kernnähere bzw. kernentferntere Spezialgesetze von T. Man beachte, dass auch periphere Hypothesen den logischen Status von ,Axiomen‘ besitzen, weil sie nicht aus anderen Axiomen ableitbar sind. Die Unterscheidung der Konsequenzen von T in kernnahe und kernentfernte ist zwar grundsätzlich möglich, aber schwierig, denn es kann sein, dass eine Konsequenz S aus einem Kernaxiom und einer peripheren Hilfshypothese folgt, und dann ist es willkürlich, ob man S als eher kernnahe oder eher peripher ansieht. Nähere Erläuterungen: Ad (1.) Zur sprachlichen Untergliederung: Wir bezeichnen die theoretischen Begriffe einer Theorie allgemein mit s1, … ,sn. Die empirischen/vortheoretischen Begriffe bezeichnen wir mit p1, … ,pm. Wir schreiben für eine Theorie T auch T(p1, … ,pm; s1, … ,sn). Ausgewählte theoretische oder empirische Begriffe bezeichnen wir mit ausgewählten Buchstaben; z. B. „m“ für „Masse“. Ad (2.) Natürliche Axiomatisierung – irreduzible Repräsentationen: Theorien können in verschiedenen, aber logisch äquivalenten Weisen axiomatisiert werden. Von einer Axiomatisierung Ax(T) von T verlangen wir, dass sie natürlich ist, und verstehen darunter, dass es sich bei Ax(T) um eine irreduzible Repräsentation von T im Sinne von (Def. 4.1-2) handelt. Im Begriff der natürlichen Axiomatisierungen schließen wir einerseits irrelevante oder redundante Bestandteile in den Axiomen aus, und verstehen andererseits unter Axiomen nur konjunktiv unzerlegbare Bestandteile, was für Fragen der Bestätigung, Wahrheitsnähe und Vereinheitlichung zentral ist (s. Kap. 5.6.2, 5.8.3). Normalerweise ist die natürliche Axiomatisierung logisch eindeutig bestimmt. Wenn mehrere natürliche Axiomatisierungen möglich sind, drücken sich darin keine logischen Trivialitäten, sondern interessante Umordnungen aus. Beispielsweise wurden zur Newtonschen Punktmechanik zwei alternative Axiomatisierungen entwickelt: die Lagrangesche Mechanik und die Hamiltonsche Mechanik. Ad (3.) Gemischte Sätze und Zuordnungsgesetze: Nicht jeder gemischte Satz ist ein Zuordnungsgesetz, d. h., etabliert eine relevante Verbindung zwischen empirischen/vortheoretischen und theoretischen Begriffen. Z. B.

Theorien als historische Entitäten

Theoriekern und Theorieversion

Natürliche Axiomatisierung

185

186

5. Theorien und ihre empirische Bewertung Axiome von T Theoretische Gesamtsprache

Rein theoretische Sätze Zuordnungsgesetze

Empirische Teilsprache

Gemischte Sätze Empirische Konsequenzen von T alle Konsequenzen von T

Abb.5.3-1: Untergliederung von T nach der Sprache

Abb. 5.3-2: Untergliederung von T nach der log. und begriffl. Natur ihrer Sätze

Kern: definiert die Identität von T im historischen Wandel Peripherie: definiert die aktuelle Version von T Abb. 5.3-3: Untergliederung der Axiome von T nach ihrem epistemischen Status

ist die Konjunktion Fa^s(a) oder der Existenzsatz 9x(Fx^s(x)) essentiell gemischt, aber kein Zuordnungsgesetz. (5.) Empirische, vortheoretische und T-theoretische Begriffe – Präzisierung: In Kap. 3.1.3 haben wir folgende Begriffsarten einer Theorie T eingeführt: (a) ihre empirischen Begriffe, (b) ihre theoretischen Begriffe im weiten Sinn, d. h. ihre nichtempirischen Begriffe, und (c) als Untermenge von (b) ihre T-theoretischen Begriffe, deren Bedeutung durch T erklärt wird. Letztere Begriffsart präzisieren wir nun wie folgt: T-theoretische Begriffe

(Def. 5.3-1) Ein theoretischer Begriff s heißt T-theoretisch g.d.w. aus T mindestens ein Zuordnungsgesetz für s relevant folgt. Anders als Sneed (1971, s. unten) schließen wir in (Def. 5.3-1) nicht aus, dass es nicht auch andere Theorien T‘ gibt, die ein Zuordnungsgesetz für s liefern. Für die Rekonstruktion von konkreten Theorien sind die Unterscheidungen (a, b, c) noch zu grob: (d) Einerseits werden in den meisten wissenschaftlichen Theorien gewisse Begriffe als vortheoretisch aufgefasst, weil sie durch unproblematische Vortheorien von T empirisch erklärt werden. So sieht man die Ortsfunktion als vortheoretischen Begriff der Mechanik an, der in der Kinematik erklärt wird, welche wiederum die Axiome der Längenmetrisierung, Zeitmetrisierung und euklidischen Geometrie als Vortheorie enthält (s. die Rekonstruktion in Balzer 1982, 141 – 170). (e) Andererseits gibt es in Theorien oft theoretische Begriffe, deren Bedeutung und empirische Signifikanz von übergeordneten Theorien im Theorien-

5.3 Grundlegendes zur Theorienstatik: Struktur wissenschaftlicher Theorien

netz erklärt wird, die keine Vortheorien von T sind. Beispielsweise enthält die chemische Atomtheorie den Begriff der „elektrischen Ladung“, welcher im Theorienverband der Elektrodynamik erklärt wird. Wollten wir nur mit den Unterscheidungen (a, b, c) auskommen, so müssten wir uns immer auf das gesamte Theoriennetz beziehen, was enorm unpraktisch ist. Daher führen wir Unterscheidungen ein, die (d) und (e) Rechnung tragen. Unsere Unterscheidungen sind relativ zu einer gegebenen Theorie T in einem gegebenen Theoriennetz N. In Erweiterung einer Idee von Balzer/Mühlhölzer (1982, 32) erklären wir zunächst die Relation der Vortheoretizität zwischen Theorieelementen: (Def. 5.3-2): T1 ist eine Vortheorie von T2 im gegebenen Theoriennetz N g.d.w. gilt: a) alle Begriffe von T1, die keine T2-Begriffe sind, sind empirisch, b) es gibt theoretische Begriffe von T2, die keine T1-Begriffe sind, c) es gibt nichtempirische Begriffe von T2, die nicht T2-theoretisch sind, aber die T1-theoretisch sind: diese Begriffe heißen vortheoretisch in T2 bzgl. T1. Die (deskriptiven) Begriffe einer Theorie T im Netz N zerfallen damit in: (i) die empirischen Begriffe von T, (ii) die vortheoretischen Begriffe von T – das sind alle nichtempirischen Begriffe von T, die vortheoretisch in T sind bezüglich irgendeiner Vortheorie T‘ von T im Theoriennetz N, und (iii) die theoretischen Begriffe von T im weiten Sinn – das sind alle T-Begriffe, die weder empirisch noch vortheoretisch in T sind. Die Menge (iii) enthält folgende Untermengen: (iii-1) Die theoretischen Begriffe von T im engen Sinn sind jene theoretischen Begriffe von T i.w.S., die T*-theoretisch sind bzgl. irgendeiner Theorie T* im Netz N, welche keine Vortheorie von T ist. Die Menge (iii-1) enthält als weitere Untermenge (iii-1-1) die T-theoretischen Begriffe, für welche T selbst ein Zuordnungsgesetz bereitstellt. (iii-2) Die empirisch insignifikanten theoretischen Begriffe von T – für sie liefert nicht einmal das gesamte Theoriennetz N ein Zuordnungsgesetz. Eine Theorie T eines Theoriennetzes N nennen wir eine basale Theorie, wenn T in N keine Vortheorie besitzt. Ein Theoriennetz ist empirisch fundiert, wenn seine basalen Theorien nur theoretische Begriffe enthalten, die durch direkte Zuordnungsgesetze (s. S. 172) charakterisiert sind, andernfalls wären diese ohne empirische Zuordnung und daher nicht empirisch signifikant. Die Forderung der empirischen Fundiertheit ist aus unserer Perspektive eine wissenschaftliche Adäquatheitsforderung an Theoriennetze. Dagegen optieren für die strukturalistischen Wissenschaftsphilosophen in (Balzer et al. 1987, 416 ff.) für eine kohärentistische Position, wonach die Begriffe von basalen Theorien eines Theoriennetzes allesamt theoretischer Natur wären, da es ja keine Vortheorien mehr gibt, welche diese erklären könnten. Wie in Kap. 2.7.2 erläutert, handelt sich eine solche kohärentistische Sichtweise schwerwiegende Zirkularitätsprobleme ein.

Vortheoretische Begriffe

Empirisch fundierte Theoriennetze

187

188

5. Theorien und ihre empirische Bewertung

Strukturalistische Explikationen von T-Theoretizität

Der vorgeschlagene Theoretizitätsbegriff unterscheidet sich von jüngeren Theoretizitätsbegriffen des Strukturalismus dadurch, dass wir einen unabhängig explizierten Empirizitätsbegriff benutzen. Für uns ist ein Begriff nur dann theoretisch (in irgendeinem Sinn), wenn er nicht empirisch ist (und evtl. weitere Bedingungen erfüllt). Der Strukturalismus will einen solchen Empirizitätsbegriff vermeiden – denn alles ist ,irgendwie‘ theoriegeladen – und daraus ergeben sich Probleme. Gemäß dem Vorschlag von Sneed (1971, 31; s. auch Stegmüller 1973a, 51; Balzer 1985, 140) ist ein Begriff w einer Theorie T T-theoretisch g.d.w. in jeder Messung von w anhand einer Anwendung a die Theorie T (bzw. ihr Zutreffen auf gewisse Anwendungen) vorausgesetzt werden muss. Sneeds Kriterium wurde aus zwei Gründen als zu eng kritisiert: erstens bezieht sich das Kriterium auf den gesamten Wissensstand der scientific community, und zweitens gibt es gemäß Punkt (e) oben viele Fälle, wo ein T-theoretischer Begriff zugleich durch eine andere Theorie T* gemessen werden kann (s. Balzer/Moulines 1980, § III, IV). Daher hat Balzer (1985, 144), grob gesprochen, folgenden Alternativvorschlag gemacht: w ist ein T-theoretischer Begriff g.d.w. es T-Messmodelle gibt, welche die Bestimmung des Wertes von w aufgrund der Werte der anderer Begriffe von T in der gegebenen Anwendung erlauben. Balzers Kriterium ist aber zu weit, da nach diesem Kriterium auch gewöhnliche empirische Begriffe in geeigneten Messmodellen als T-theoretisch zu bezeichnen wären (s. Schurz 1990b, Schurz 2013b).

5.4 Methodische Merkmale (guter) wissenschaftlicher Theorien 5.4.1 Systemcharakter, empirische Kreativität, Globalität und Vereinheitlichung

Empirische Kreativität

(1.) Unter dem Systemcharakter von Theorien verstehen wir die Tatsache, dass man eine Theorie nicht verstehen kann, wenn man die einzelnen Gesetzeshypothesen der Theorie in Isolation betrachtet, sondern nur, wenn man das Zusammenwirken ihrer Gesetzeshypothesen betrachtet. Insbesondere: während eine einzelne Hypothese bzw. Axiom im Regelfall noch gar keinen empirischen Gehalt besitzt, wird durch die konjunktive Vereinigung mehrerer Hypothesen vielfältiger empirischer Gehalt erzeugt. (2.) Mit (1.) hängt das zweite Merkmal von wissenschaftlichen Theorien zusammen – ihre empirische Kreativität. Darunter ist zu verstehen, dass zwei oder mehrere theoretische Hypothesen neuen empirischen Gehalt erzeugen, der über die Summe bzw. Vereinigung der empirischen Gehalte der einzelnen Gesetzeshypothesen hinausgeht (s. Kutschera 1972, 304 f.). Beachte: die empirische Kreativität besagt nicht nur, dass der empirische Gehalt der Konjunktion H1^H2 zweier theoretischer Sätze H1, H2 mehr enthält als die Vereinigung ([) der empirischen Einzelgehalte, E(H1)[E(H2) (dies ist meist trivial erfüllt), sondern darüber hinaus: dass der empirische Gehalt von H1^H2 auch mehr enthält als die empirische Konsequenzenmenge von E(H1)[E(H2). Ein einfaches Beispiel für empirische Kreativität: ange-

5.4 Methodische Merkmale (guter) wissenschaftlicher Theorien

nommen H1 = 8x(Fx!s(x)), H2 = 8x(s(x)!Gx), und Fx und Gx sind empirische Prädikate. Dann gilt E(H1) = leer, E(H2) = leer, somit E(E(H1)[E(H2)) = leer, aber E(H1^H2) = C(8x(Fx!Gx)). So lässt sich erklären, wie es möglich ist, dass eine theoretische Spekulation H, die keinen empirischen Gehalt besitzt, in einem späteren Stadium im Kontext einer dann entwickelten Theorie T neuen empirischen Gehalt erzeugen kann. Denn obwohl E(H) leer ist, kann E(T^H) wesentlich größer sein als E(T). (3.) Verbunden mit (1.) und (2.) ist das dritte Merkmal: Globalität und Vereinheitlichungsleistung. Mit Globalität ist gemeint, dass eine gute wissenschaftliche Theorie qualitativ verschiedene empirische Phänomene erklärt, und mit Vereinheitlichungsleistung, dass sie eine Reihe solcher qualitativ verschiedener Phänomene einheitlich erklärt und damit viele empirische Gesetzeshypothesen auf wenige theoretische Prinzipien zurückführt. Wir haben beide Merkmale im letzten Kapitel eingehend illustriert. Die Bedeutung der Vereinheitlichungsfunktion von Theorien für Erklärung und Bestätigung wurde von vielen Autoren herausgearbeitet (Friedman 1990, Kitcher 1990, Schurz/Lambert 1994, Maxwell 1998). Allerdings waren bisherige Versuche, den Vereinheitlichungsbegriff zu präzisieren, vielen Einwänden ausgesetzt (vgl. z. B. Barnes 1992, Humphreys 1993). 5.4.2 Holismus der Bedeutung, des empirischen Gehalts, und der Theorienüberprüfung (die Duhem-Neurath-Quine-These) Aus den geschilderten Merkmalen guter Theorien folgen drei Arten des Holismus: (1.) Unter dem Holismus der Bedeutung theoretischer Begriffe verstehen wir die erläuterte Tatsache, dass die Bedeutung theoretischer Begriffe durch das gesamte Netz der dafür zuständigen Hintergrundtheorien bestimmt wird. Somit ist es in wissenschaftlichen Theoriensystemen nicht mehr möglich, unter den Sätzen, Axiomen oder Theoremen, zwischen analytischen und synthetischen Bestandteilen zu unterscheiden. In diesem beschränkten Bereich ist die Quinesche These (1951) von der Ununterscheidbarkeit von Analytisch-Synthetisch also bestätigt. (2.) Unter dem Holismus des empirischen Gehalts verstehen wir die damit zusammenhängende Tatsache, dass wir immer nur ein ganzes System von theoretischen Hypothesen mit der Erfahrung konfrontieren können, weil nur dieses empirischen Gehalt erzeugt. (3.) Aus dem Holismus des empirischen Gehaltes folgt unmittelbar der Holismus der Theorienüberprüfung: wenn ein System von theoretischen Hypothesen T = {H1, … ,Hn}, aus der die empirische Voraussage P folgt, mit einer Erfahrung E in Konflikt gerät, d. h. E 41 :P, dann weiß man zwar mit Sicherheit, dass im Hypothesensystem T irgendwo eine falsche Hypothese enthalten sein muss, es ist aber logisch gesehen unbestimmt, welche der Hypothesen Hi falsch ist bzw. verworfen werden soll. Auf diese Tatsache hatte als erster Duhem (1908) hingewiesen, und er hatte sie auf die berühmte Formel gebracht: es gibt kein experimentum crucis für eine Theorie, d. h., der Ausgang eines Experimentes kann niemals endgültig dazu zwingen, den Theoriekern oder andere Einzelbestandteile von Theorieversionen preiszu-

Arten des Holismus

189

190

5. Theorien und ihre empirische Bewertung

Logischer Hintergrund des Überprüfungsholismus

geben. Dieselbe Tatsache wurde später von Neurath (1914/15) und von Quine herausgearbeitet. Während Duhem und Neurath vorwiegend den Systemcharakter wissenschaftlicher Hypothesensysteme im Auge hatten, arbeitet Quine (1951, 47) die daraus folgende empirische Unterbestimmtheit von Theorien heraus: unterschiedliche aber empirisch äquivalente Änderungen des theoretischen Systems sind angesichts konfligierender Erfahrungen möglich. Der Holismus der Theorienüberprüfung bewegt sich völlig im Rahmen der deduktiv-monotonen Logik: (Ms. 5.4-1) Deduktionslogischer Hintergrund des Überprüfungsholismus: Wenn die Prämissenmenge {H1, … ,Hn} die Prognose P logisch impliziert, dann impliziert (per Modus Tollens) die Negation der Prognose, :P, die Negation der Konjunktion aller Prämissen, also :(H1^…^Hn). Letztere ist logisch äquivalent mit der Disjunktion aller negierten Prämissen, also :H1_…_:Hn, was soviel besagt wie: mindestens eine der Prämissen Hi ist falsch. Wenn P eine probabilistische bzw. nichtmonotone Konsequenz des Hypothesensystems H1, … ,Hn ist, so ergibt sich für den Holismus der Theorienüberprüfung der folgende wesentliche Unterschied, auf den in Schurz (2001b, § 8) hingewiesen wird: um die Konsequenz P zu vermeiden, muss nicht unbedingt eine Hypothese Hi verworfen oder abgeändert werden, so wie das für deduktiv-monotone Konsequenzen zwingend ist. Es würde auch genügen, einfach eine neue Hilfshypothese H* hinzuzufügen, welche die bisherige Folgerung von P blockiert. Bei allen hier erläuterten Spielarten des Holismus handelt es sich um einen gemäßigten Holismus: es wird weder behauptet, dass alles mit allem zusammenhängt, noch dass die Bedeutung aller Begriffe theoriebestimmt ist.

5.4.3. Theoriennetze

Arten von intertheoretischen Relationen

Ein Theoriennetz besteht aus unterschiedlichen Theorienverbänden. Zwischen den Theorieelementen eines Theoriennetzes bestehen (mindestens) drei Arten von Relationen (s. auch Balzer et al. 1987): 1.) Durch die Relation der Spezialisierung entstehen, wie in Kap. 5.2 erläutert, hierarchische Theorienverbände. 2.) Die in Kap. 5.3 erläuterte Relation der Vortheoretizität kann sowohl innerhalb wie zwischen Theorienverbänden bestehen. Die Relationen der Spezialisierung und Vortheoretizität verbinden nur theoretisch homogene Theorieelemente miteinander; ausdrucksgleiche Begriffe der so verbundenen Theorieelemente werden miteinander identifiziert. So verbundene Theorieelemente können niemals miteinander in Konkurrenz treten. Dies ist anders für die dritte Kategorie intertheoretischer Beziehungen: 3.) Zwischen verschiedenen Theorieverbänden gibt es intertheoretische Querverbindungen. Im einfachsten Fall handelt es sich ebenfalls um Identitätsverbindungen. Wird beispielsweise in einem mechanischen Modell eine

5.4 Methodische Merkmale (guter) wissenschaftlicher Theorien

elektrostatische Kraft verwendet, so wird dieser Kraftbegriff aus dem Theorienverband der Elektrostatik entlehnt. Eine zweite wichtige Art von intertheoretischen Querverbindungen sind Brückenprinzipien. Das prototypische Beispiel ist die Brückenbeziehung zwischen phänomenologischer Thermodynamik und statistischer Mechanik, derzufolge die empirisch metrisierte absolute Temperatur einer Substanz proportional ist zur mittleren kinetischen Energie der Substanzmoleküle. Schurz (2009) zeigt, dass unter bestimmten Bedingungen solche intertheoretischen Brückenprinzipien aus der Vereinigung beider Theorien folgen. Eine dritte Art intertheoretischer Querverbindungen sind approximative Theorienreduktionen. Beispielsweise folgt die klassische Mechanik approximativ aus der speziellen Relativitätstheorie, wenn die Partikelgeschwindigkeiten sehr klein gegenüber der Lichtgeschwindigkeit sind, und sie folgt approximativ aus der allgemeinen Relativitätstheorie, wenn das Schwerefeld sehr gering ist. Gemäß den klassischen Definitionen von Nagel und Hempel ist eine Theorie T1 auf eine Theorie T2 strikt reduzierbar, wenn es eine Menge B von Brückenprinzipien gibt, welche T1-Begriffe mithilfe von T2-Begriffen charakterisieren, sowie eine Menge A von einschränkenden Anwendungsbedingungen, sodass T1 aus A, B und T2 folgerbar ist (s. Nagel 1970; Hempel 1969). Eine solche Reduktion heißt approximativ, wenn nicht T1 selbst, sondern eine approximative T1-Annäherung T1* aus A, B und T2 folgerbar ist – d. h., die empirischen Gesetzesprognosen von T1* kommen denen von T1 quantitativ sehr nahe (Nagel 1970, 121, 125; Hempel 1969, 193). Jüngere Autoren haben andere Reduktionsbegriffe vorgeschlagen, die in Rott (1992, Kap. 1 – 2) im Detail diskutiert werden; Rotts Diskussion legt nahe, dass die Grundidee des klassischen Hempel-Nagelschen Reduktionsbegriffs nach wie vor als gut bewährt gelten kann. Die Theorien von Physik und Chemie sind zu einem dichten Theoriennetz verbunden. Auch Teile der Biologie und Psychologie (wie z. B. Neuropsychologie) lassen sich zwanglos in das skizzierte Netz eingliedern. Geht man in die Evolutionsbiologie, so hat man ein relativ autonomes Theoriennetz vorliegen, das nicht viele theoretische Begriffe aus dem physikalischchemischen Theoriennetz importiert, das andererseits aber zentrale Begriffe in Psychologie, Human- und Sozialwissenschaften exportiert. In diesen ,höheren‘ Disziplinen findet man statt vereinheitlichenden Fundamentaltheorien eine zunehmende Konkurrenz von Theorieansätzen in lockerer Kombination mit Faktenwissen (s. Schurz/Weingartner 1998). Trotz dieser Unterschiede, die viel mit der hohen Komplexität der Gegenstände dieser Disziplinen zu tun haben, zeigen wir in den nächsten Kapiteln anhand ausgewählter Theorien dieser Gebiete, dass dieselben wissenschaftstheoretischen Einsichten zutreffen, welche wir bisher anhand naturwissenschaftlicher Beispiele erarbeitet haben.

Approximative Theorienreduktion

Theoriennetze in den Wissenschaften

191

192

5. Theorien und ihre empirische Bewertung

5.5 Das Beispiel der Piagetschen kognitionspsychologischen Entwicklungstheorie

Kognitive Entwicklungsstufen nach Piaget

Die zentrale Kernthese von Piagets kognitionspsychologischer Entwicklungstheorie besagt, dass die kindliche Intelligenzentwicklung auf der stufenförmigen Ausbildung von allgemeinen logisch-strukturellen Fähigkeiten beruht (Brainerd 1978, 33; Schurz 1985, 342). Sind diese einmal ausgebildet, so können sie innerhalb kurzer Zeit überall angewendet werden. Vorher dagegen werden entsprechende kognitive Aufgabenstellungen einfach „nicht begriffen“. Eine weitere Kernthese besagt, dass die sogenannte konkret-operationelle Stufe vom Kind mit ca. 6 – 7 Jahren erreicht wird. Daneben gibt es noch die sensomotorische Stufe, die mit 2 Jahren erreicht wird, und die formal-operationelle Stufe, die der Jugendliche mit 13 – 14 Jahren erreicht. Wir konzentrieren uns hier auf die konkret-operationelle Stufe. Die konkret-operationelle Stufe ist durch die Ausbildung folgender Fähigkeiten gekennzeichnet: (1.) Der Perspektivenwechsel, d. h. die Fähigkeit, die eigene Sehperspektive von jener anderer Personen zu unterscheiden. Damit zusammenhängend findet der zeitgleiche Übergang vom egozentrischen zum soziozentrischen Denken statt. (2.) Die Fähigkeit zu erkennen, dass bestimmte Operationen, die das Aussehen von bestimmten Dingen verändern, auch wieder rückgängig gemacht werden können, d. h. die Einsicht in die sogenannte Reversibilität dieser Operationen. Damit zusammenhängend bildet sich voralledem die Einsicht heraus, dass bestimmte Eigenschaften der Dinge unter solchen Operationen erhalten bleiben, d. h., es entsteht die Fähigkeit zur Invariantenerkennung. Reversibilität und Invariantenerkennung untersuchte Piaget voralledem in zwei Bereichen: einerseits die Invarianz der Anzahl von Gegenständen unter ihren verschiedenen Anordnungsmöglichkeiten; und andererseits die Invarianz der Menge einer Substanz unter ihren verschiedenen Verformungsmöglichkeiten bzw. Wandlungsmöglichkeiten ihrer Gestalt. Alle diese Fähigkeiten hat nun Piaget mithilfe gewisser Tests überprüft, welche empirische Zuordnungsgesetze zur Verfügung stellen. Die Fähigkeit des Perspektivenwechselns überprüfte Piaget anhand eines dreidimensionalen Modells einer Gebirgslandschaft: die Kinder durften das Gebirge zunächst rundherum umgehen und sich ansehen; dann setzten sie sich auf eine Seite und sollten anhand von vorgezeigten Bildern sagen, wie das Gebirge von verschiedenen anderen Positionen aussieht. Die Fähigkeit zur Invarianz der Zahl testete Piaget und seine Mitarbeiter mit einem Zahltest, in dem eine bestimmte Anzahl von Münzen auf vorgezeigten Abbildungen einmal weit auseinanderliegend angeordnet wurde, das andere Mal eng beisammen, und es wurde gefragt „sind auf diesem Bild mehr, gleich viel oder weniger Münzen als auf jenem?“ Die Invarianz der Substanzmenge wurde schließlich mit Piagets berühmten Tonkugelexperiment getestet, in dem eine Tonkugel zu einer langen Wurst auseinandergezogen wurde, und gefragt wurde: „Ist der Ton nun mehr geworden, oder gar weniger geworden, oder gleichviel geblieben?“

5.5 Piagetsche kognitionspsychologische Entwicklungstheorie

Bei allen diesen Tests ergab sich, dass die entsprechende Fähigkeit von Kindern vor 6 Jahren überwiegend nicht beherrscht wurde, nach 7 Jahren dagegen durchwegs schon. Piaget sah durch diese Ergebnisse seine Stufentheorie der kognitiven Intelligenzentwicklung als sehr gut bewährt an. Wir fassen die skizzierte kognitionspsychologische Entwicklungstheorie von Piaget sowie ihre zentrale empirische Prognose wie folgt zusammen: (5.5-1) K1:

K2 – 4:

S1: S2 – 4:

I1 – 3:

P:

Die kindliche Intelligenzentwicklung beruht (primär) auf der stufenförmigen Entwicklung von allgemein anwendbaren logischstrukturellen Fähigkeiten. Die konkret-operationelle Stufe ist gekennzeichnet durch die Ausbildung der Fähigkeit zu: (K2) Perspektivenwechsel, (K3) Erkenntnis der Reversibilität von Operationen, und (K4) Invariantenerkennung. Mit 6 – 7 Jahren erreichen fast alle Kinder die konkret-operationelle Stufe. Typische Beispiele der in K2 – 4 genannten Fähigkeiten sind: für den Perspektivenwechsel (S2) die Fähigkeit zur Koordination von verschiedenen Sehperspektiven; für die Reversibilitäts- und Invariantenerkennung erstens (S3) die Fähigkeit, die Invarianz der Anzahl gegenüber unterschiedlicher Anordnung zu erkennen, und zweitens (S4) die Fähigkeit, die Invarianz der Substanzmenge gegenüber Formveränderung zu erkennen. Selektive Tests (Indikatoren) für die Fähigkeiten in (S2 – 4) sind: (I1) zum visuellen Perspektivenwechsel der Piagetsche Gebirgstest, (I2) zur Invarianz der Anzahl der Piagetsche Zahltest, und (I3) zur Invarianz der Substanzmenge der Piagetsche Tonkugeltest.

Kern und Peripherie der Piagetschen Theorie

Empirische Prognose: Fast alle Kinder scheitern an Piagets Tests vor Erreichen des 6. Lebensjahrs, beherrschen Piagets Tests jedoch nach dem Erreichen des 7. Lebensjahrs.

K1 – 4 sind Kerngesetze, mit ihnen steht und fällt die Piagetsche Theorie. K1 – 4 sind hochtheoretische Gesetze; der einzige in ihnen vorkommende empirische Begriff ist der des Alters eines Kindes. S1 – 4 sind Spezialgesetze, die ebenfalls einen zentralen Status besitzen: sie exemplifizieren die hochabstrakten Fähigkeiten in K2 – 4 durch konkretere Beispiele. Dennoch sind auch S2 – 4 immer noch theoretische Gesetzeshypothesen, denen die genauen empirischen Zuordnungsgesetze fehlen. Diese werden durch die Indikatorgesetze I1 – 3 nachgetragen, welche für jede der drei Fähigkeiten jeweils einen von Piaget entwickelten empirischen Test anführen. Die Indikatorgesetze I1 – 3 liegen am weitesten außen in der Peripherie der Piagetschen Theorie. Aus K1 – 4, S1 – 4 und I1 – 3 folgt logisch die empirische Prognose. Übungshalber sei dieser Folgerungszusammenhäng kurz erläutert. Das Kerngesetz K1 ist für die Folgerung nicht direkt notwendig; sein über S1 und

Erläuterung des Folgerungszusammenhangs

193

194

5. Theorien und ihre empirische Bewertung

Replikationsstudien zu Piaget

K2 – 4 hinausgehender Gehalt liegt in einer Aussage über den Gesamtverlauf der kognitiven Intelligenzentwicklung. Dennoch ist K1 für die logische Interpretation der Spezialgesetze S2 – 4 wichtig. Stehe Fix für die drei allgemeinen in K2 – 4 genannten Fähigkeiten, und stehe BFix für die drei in S1 – 3 genannten beispielshaften Fähigkeiten. Aus K1 geht hervor, dass die Gesetze S2 – 4 nicht als bloße Implikationsgesetze Fix ! BFix, sondern als Äquivalenzgesetze zu rekonstruieren sind (die Individuenvariable x laufe über Kinder): Fix $ BFix, für i = 1,2,3. Wann immer also ein Kind die beispielshafte Fähigkeit nicht beherrscht, beherrscht es auch nicht die allgemeine Fähigkeit, welche die strukturelle Stufe der Kognition definiert. Nur unter dieser Annahme kann man auch den ersten Teil der Prognose folgern, dass vor dem 6. Jahr die entsprechenden Tests nicht bewältigt werden. In analoger Weise sind auch K2 – 4 als Äquivalenzgesetze zu rekonstruieren: KS(x) $ Fix (mit ,KS‘ für ,konkret-operationale Stufe‘). Die Indikatorgesetze I1 – 3 haben die Form: Tix ! (BFix $ Kix), mit ,Tix‘ für ,an x wird der Test Ti durchgeführt‘, und ,Kix‘ für ,x beantwortet den Test Ti korrekt‘. Zusammen mit dem Spezialgesetz S1: Unter6(x) ! :KSx und Über-7(x) ! KSx, folgt daraus deduktiv die Prognose Pi: Unter-6(x) ! (Tix!:Kix) und Über-7(x) ! (Tix!Kix). Allerdings nur, wenn man sämtliche Gesetzeshypothesen als strikt auffasst. Nun muss aber zumindest S1 als statistische bzw. normische Generalisierung formuliert werden (es gibt Kinder in Ausnahmesituationen, für die S1 nicht zutrifft): Unter-6(x) ) :Ksx, und: Über-7(x) ) KSx. Bei dieser Rekonstruktion folgt daraus wahrscheinlichkeitslogisch (s. 5.1-10) die normische Prognose Pi‘: Unter-6(x)) (Tix!:Kix) und Über-7(x) ) (Tix!Kix). Zu Piagets kognitiver Entwicklungspsychologie gab es in den 60er und 70er Jahren zahlreiche Replikationsstudien, an deren sich dieselben holistischen Phänomene der Theorienüberprüfung studieren lassen, die wir bereits anhand physikalischer Theorien kennengelernt haben. Eine wichtige Voraussetzung der Piagetschen Theorie ist natürlich, dass ihre Tests die gemeinten kognitiven Fähigkeiten adäquat messen, d. h., dass diese Tests wirklich selektiv sind für die in S2 – 4 genannten beispielhaften Fähigkeiten, und für die in K2 – 4 genannten dahinterstehenden allgemeinen Fähigkeiten. Dies wurde von etlichen Autoren kritisch hinterfragt. Gemäß der methodischen Regel (Ms. 5.1-2) muss Piagets Theorie, wenn sie als bewährt gelten soll, auch für andere ebenso plausible Indikatoren der kognitiven Fähigkeiten des Kindes Geltung besitzen. Z. B. sollte es egal sein, ob man den Sehtest zum Perspektivenwechsel an der einen oder der anderen dreidimensionalen Gestalt durchführt. So ersetzte man im Piagetschen Sehtest die Gebirgslandschaft durch ein viel einfacheres Beispiel: eine Schachtel, die an allen vier Seiten eine jeweils andere Farbe hatte. Man fragte die Kinder, nachdem man sie ein oder mehrmals um die Schachtel herumgeführt hatte, und dann auf eine bestimmte Seite niedersetzen ließ, wieder: „welche Farbe hat die Schachtel, wenn man sie von dieser oder jener anderen Seite aus betrachtet?“ Das überraschende Ergebnis war, dass die Kinder diesen einfacheren Sehtest zum Perspektivenwechsel nun bereits mit nur 4 Jahren beherrschten (s. Brainerd 1978, 103 – 106). Dies brachte aber die Entwicklungspsychologen, getreu der Lakatos-Devise, nicht gleich dazu, die Piagetsche Theorie in ihrem Kern preiszugeben.

5.5 Piagetsche kognitionspsychologische Entwicklungstheorie

Man schloss daraus vielmehr, dass Piagets Sehtest nicht wirklich selektiv war für die Fähigkeit zum Perspektivenwechsel beim Sehen, sondern zusätzliche versteckte Schwierigkeitsvariablen involvierte (Brainerd 1978, 80; Schurz 1985, 352). Piagets Sehtest, so argumentierte man, testet nicht nur die prinzipielle Fähigkeit zum Perspektivenwechsel, sondern erfordert zugleich ein hohes räumliches Vorstellungsvermögen: immerhin mussten die Kinder ein kompliziertes Gebirge in ihrer geistigen Anschauung rotieren lassen, um das rotierte Gebirge auf dem Bild wiederzuerkennen, und diese Fähigkeit geht weit über die simple Fähigkeit der Unterscheidung und Koordination von Eigen- und Fremdperspektive hinaus. Man schlug also zunächst vor, nur zwei periphere Gesetze der Piagetschen Theorie zu ändern, nämlich S1 und I1: man vermutete also einerseits, dass die konkret-operationelle Stufe doch schon früher erreicht wird als von Piaget vermutet, und änderte andererseits das Indikatorgesetz für einen adäquaten Test der Fähigkeit zum visuellen Perspektivenwechsel. Analoges fand man beim Zahltest heraus. Als zusätzliche Schwierigkeitsvariable, die im Piagetschen Test involviert war, stellt sich hier die Beherrschung der Sprache heraus. Hatten die Kinder wirklich verstanden, was mit „mehr Münzen“ genau gemeint war – mehr im Sinne der Anzahl, und nicht mehr im Sinne der von den Münzen beanspruchten Fläche? Man führte den Zahltest daher mithilfe von nonverbalen Lernexperimenten durch: man belohnte die Kinder, wenn sie Bilder mit der gleichen Anzahl von Münzen darauf einander zuordnen konnten. Und siehe da, nach einigem Training wurde der Zahltest bereits von 4 – 5 jährigen Kindern beherrscht (Siegel 1978, 56 f.). Dies gab derjenigen Fraktion, welche nur die Peripherie der Piagetschen Theorie ändern wollte, zwischenzeitlich starken Auftrieb. Man sah darin weitere Evidenz dafür, dass die Erreichung der konkret-operationellen Stufe schon zwischen 4 und 5 Jahren erfolgt, also das Spezialgesetz S1 zu ändern war, zusammen mit einer Änderung des Indikatorgesetzes I2. Man versuchte auch, im Fall des dritten Piagetschen Tests, der Verformung von Tonkugeln, durch Testmodifikationen zu einer Avancierung des Erreichens der konkret-operationellen Stufe zu gelangen, was in diesem Fall allerdings von nur geringerem Erfolg gekrönt war (Siegel 1978, 59 f.). Andere Autoren brachten daraufhin jedoch Experimente oder Befunde vor, die man nicht mehr mit einer solchen Peripherieänderung in Einklang bringen konnte. Ein Beispiel ist die Aussagenlogik. Gemäß Piaget müssten die Gesetze des aussagenlogisch korrekten Schließens mit 13 – 14 Jahren, nämlich bei Erreichen der formal-operationalen Stufe, beherrscht werden. Tatsächlich aber werden gewisse aussagenlogische Gesetze, z. B. der Modus Ponens, schon mit drei Jahren beherrscht, während andere aussagenlogische Gesetze wie z. B. Modus Tollens von etlichen Personen überhaupt nie, d. h. nicht einmal im Erwachsenenstadium, wirklich beherrscht werden (s. Brainerd 1978, 248 f.; Evans 1982, Kap. 8 – 9). Piaget zufolge gehören aber Modus Ponens und Modus Tollens derselben abstrakt-strukturellen Fähigkeit zu. Dieser Konflikt lässt sich keinesfalls mit Änderungen eines Indikator- oder Spezialgesetzes beheben. Er greift direkt den Kern der Piagetschen Theorie an. Eine analoge Schwierigkeit ergab sich bei der Erkenntnis der sogenannten Erhaltung des Objekts. Dieses kognitive Prinzip besagt folgendes: Objekte,

Versteckte Schwierigkeitsvariablen

Änderung der Peripherie

Probleme, die Piagets Theoriekern angreifen

195

196

5. Theorien und ihre empirische Bewertung

Änderung des Kerns

die zunächst sichtbar sind und danach durch eine Transformation unsichtbar werden, hören nicht plötzlich auf zu existieren, sondern existieren weiter. Eine gewisse Variante dieser Fähigkeit wird nach Piaget bereits nach Vollendung der sensomotorischen Stufe mit 2 Jahren beherrscht: wenn man einen Ball hinter einer Wand versteckt, so suchen die Kinder nach dem Ball (Brainerd 1978, 46 – 70). Eine andere Variante dieser Fähigkeit, nämlich ihre Anwendung auf die Auflösung von Zucker in Wasser, wird jedoch noch nicht einmal mit 6 Jahren beherrscht: löst man den sichtbaren Zucker in Wasser auf und fragt dann die Kinder, nachdem auch die kleinsten Zuckerkrümel im Wasser verschwunden sind: „was ist nun mit dem Zucker passiert – ist er nicht mehr da?“, so geben die Kinder sehr unterschiedliche Antworten, und eine schwache Mehrheit behauptet in der Tat, der Zucker wäre nun nicht mehr da (Piaget 1975, 136 – 138; Schurz 1985, 352). Derartige Befunde führten etliche Psychologen dann dazu, die Kernannahme K1 der Piagetschen Theorie preiszugeben. Die alternative Theorie, die entwickelt wurde, besagt, dass die Entwicklung der Intelligenz nicht über die Ausbildung allgemein-abstrakter Strukturen bzw. Fähigkeiten vor sich geht, sondern auf der Entwicklung inhaltsspezifischer und inhaltsgebundener Fähigkeiten beruht, welche zunächst nur in einem bestimmten Anwendungsbereich beherrscht werden, und erst später durch Differenzierung und Generalisierung auf andere Bereiche übertragen werden können (s. Ausubel et al. 1978; Novak 1980, 20; Schurz 1985, § 6.1). Mit anderen Worten schlugen diese Autoren vor, das Piagetsche Kerngesetz K1 durch folgendes alternative Kerngesetz zu ersetzen: K1*. Die kindliche Intelligenzentwicklung beruht auf der Entwicklung von inhaltsspezifischen Fähigkeiten, die sukzessive differenziert und generalisiert werden. Durch diese Wendung wurde ein alternatives Paradigma bzw. Forschungsprogramm der kognitiven Entwicklungspsychologie kreiert. Unsere wissenschaftstheoretische Analyse hat gezeigt, dass der bisher entwickelte wissenschaftstheoretische Rahmen auch auf Theorien der Psychologie hervorragend anwendbar ist. Bevor wir diesen Rahmen in Kap. 5.7 auf ein Beispiel der Soziologie anwenden, werden im nächsten Kapitel einige grundlegende Einsichten zur Dynamik wissenschaftlicher Theorien systematisch dargestellt.

5.6 Grundlegendes zur Theoriendynamik 5.6.1 Das Lakatos-Modell der Theorienrevision Die von Lakatos (1974, 129 ff.) entwickelte Methodologie wissenschaftlicher Forschungsprogramme geht von zwei Annahmen aus: (L1): Es ist immer möglich, den Kern einer Theorie im Falle eines Konfliktes mit der Erfahrung dadurch zu retten, dass Anpassungen in der Peripherie der Theorie vorgenommen werden (s. auch Rott 1994, 38). Wir nennen (L1)

5.6 Grundlegendes zur Theoriendynamik

die Lakatossche Immunisierungsthese – sie wird jedoch nicht so heiß gegessen wie gekocht. (L2): Jede (physikalische) Theorie benötigt spezielle Hilfshypothesen, nämlich exklusive Ceteris-Paribus Hypothesen (eCP-Hypothesen), um zu empirischen Prognosen zu gelangen. Diese lagern sich wie ein Schutzgürtel in der äußeren Peripherie um Mitte und Kern (1974, 99). Im Falle eines Konfliktes mit der Erfahrung ist es immer möglich, den Konflikt zu beseitigen, indem die eCP-Hypothese fallengelassen oder ersetzt wird. Wir nennen (L2) die Lakatossche Ceteris-Paribus-These. Offensichtlich liefert (L2) eine starke Stütze für (L1). Lakatos ist sich über die Natur dieser eCP-Hypothesen nicht im Klaren (seine Rekonstruktion auf S. 106 nimmt er auf S. 180, Anm. 364, wieder zurück). Mit dem in Kap. 5.2 erklärten Begriff der Systembedingung besitzen wir folgende Antwort auf Lakatos‘ Problem: es sind die Systembedingungen, welche in physikalischen Theorien immer eine eCP-Klausel implizieren: die-und-die Kräfte wirken auf das System, und sonst keine. Ein Beobachtungsdatum, das dem theoretischen Gesamtsystem widerspricht, nennt Lakatos eine Anomalie (1974, 108). Es wird nur dann zu einer Falsifikation der Theorie, wenn an der Systembedingung bzw. an der darin enthaltenen eCP-Hypothese festgehalten wird. Im Regelfall wird der Verfechter einer Theorie jedoch die ursprüngliche eCP-Hypothese fallen lassen und stattdessen kompliziertere Systembedingungen annehmen, in denen unbekannte Störfaktoren postuliert werden. Solche Postulate sind typische ad-hoc-Hypothesen in dem in Kap. 5.2 erläuterten Sinn. Durch eine ad-hoc-Hypothese, die unbekannte Störfaktoren postuliert, wird lediglich der Widerspruch zwischen dem widerspenstigen Datum E und der Gesamttheorie T bereinigt, ohne jedoch das abweichende Datum befriedigend zu erklären. Daher bleibt das abweichende Datum auch nach dieser ad-hoc-Anpassung eine Anomalie bzw., wie wir im nächsten Kapitel sagen werden, ein Misserfolg der Theorie. Darüber hinaus ist eine solche ad-hoc-Anpassung gemäß Lakatos nur zulässig, wenn sie wissenschaftlich progressiv ist. Lakatos beruft sich dabei auf Popper (1935/76, § 20) und schlägt folgende Bedingungen der Progressivität vor. Er nennt einen durch eine Anomalie erzwungenen Übergang von einer Gesamttheorie T1 zu einer anderen T2 theoretisch progressiv, wenn T2 den gesamten empirischen bewährten Gehaltsanteil der Vorgängertheorie ebenfalls enthält, aber darüber hinaus neuen empirischen Überschussgehalt besitzt, der noch nicht unbedingt überprüft worden sein muss. Und er nennt einen solchen Übergang empirisch progressiv, wenn ein Teil des empirischen Überschussgehaltes von T2 auch bestätigt wurde (1974, 115 f.). Der Übergang heißt schließlich progressiv, wenn er zumindest theoretisch progressiv ist; andernfalls heißt er degenerativ. Lakatos‘ Ansatz betrachtet an der Stelle von einzelnen Theorieversionen Ti ganze historische Folgen solcher Theorieversionen (T1,T2, … ), die durch Übergänge aufgrund widerspenstiger Daten auseinander hervorgehen. Im Sinne von Kap. 5.2 stelle man sich unter einer einzelnen Theorieversion Ti nicht ein einzelnes Theorienelement, sondern einen Theorienverband vor. Besitzen die Theorieversionen einer solchen Folge einen gemeinsamen

Lakatos’ Immunisierungsthese Lakatos’ CeterisParibus-These

Theoretische und empirische Progressivität nach Lakatos

197

198

5. Theorien und ihre empirische Bewertung

Keine Sofortrationalität

Positive und negative Heuristik nach Lakatos

Rivalisierende Forschungsprogramme

Theoriekern, so handelt es sich um die historische Entwicklung eines Theorienparadigmas im Kuhnschen Sinn, bzw. eines Forschungsprogrammes im Lakatosschen Sinn. Der (sogenannte) raffinierte Falsifikationismus von Lakatos besteht in zwei Grundgedanken: (RF1): Eine Theorieversion Ti ist angesichts von widerspenstigen Daten (Misserfolgen) erst dann als gescheitert bzw. ,weich‘ falsifiziert anzusehen, wenn eine bessere Theorieversion Ti+1 entwickelt wurde, d. h. eine solche, die gegenüber Ti progressiv ist (1974, 117). In der Wissenschaft gibt es, so Lakatos, keine Sofortrationalität (keine rationale Sofortentscheidung): erst aufgrund der historischen Entwicklung gegenüber einem permanenten Input von neuen Daten kann eine Entscheidung über den Erfolg eines Theoriekerns bzw. Forschungsprogrammes getroffen werden (ebd., 150). (RF2): Die Entwicklung eines Theoriekernes in Form einer Reihe von Theorieversionen ist nur solange rational akzeptabel, solange sie nicht degenerativ ist, d. h., solange die ad-hoc-Anpassungen neuen empirischen Gehalt erzeugen (1974, 116) – wobei Lakatos an späterer Stelle jedoch hinzufügt, dass der empirische Überschussgehalt, den eine progressive Theorienfolge produziert, sich zumindest gelegentlich auch empirisch bestätigen muss (ebd., 131). Unter einem Forschungsprogramm versteht Lakatos einen harten Theoriekern zusammen mit einer negativen und einer positiven Heuristik (1974, 131 ff.). Die negative Heuristik besagt, dass Theorieanpassungen nicht im Kern, sondern immer im Schutzgürtel der Peripherie vorgenommen werden sollen. Lakatos schränkt diese Behauptung aber ein, indem er sagt, dass im Fall einer degenerativen Entwicklung sich die Modus-Tollens-Treffer der widerspenstigen Daten auch gegen den Theoriekern selbst richten können und sollen (1974, 131). Die positive Heuristik skizziert ein Programm, wie durch immer komplexere theoretische Modelle bzw. Systembedingungen der Theoriekern mit widerspenstigen Daten fertig werden könnte. Lakatos führt als Beispiel Newton selbst an, der wusste, in welcher Richtung seine anfänglichen Idealisierungen durch realistischere Systembedingungen zu ersetzen wären: in einem ersten Schritt behandelte er Planeten und Sonne nicht mehr als Massenpunkte, sondern als Massenkugeln, danach zog er die Rotation der Planetenkugeln in Betracht, und schließlich entwickelte er die Anfänge einer Störungstheorie zur Berücksichtigung interplanetarischer Gravitationswirkungen (ebd., 132). Die Lakatosschen Maximen (RF1) und (RF2) können allerdings auch miteinander in Konflikt geraten. Angenommen nämlich, der Übergang von Theorieversion Ti zu Ti+1 ist degenerativ, aber eine bessere Theorieversion Ti+2 ist nicht in Aussicht. Gemäß RF2 ist Ti+1 dann nicht akzeptabel. Gemäß RF1 ist Ti+1 jedoch nicht fallenzulassen, weil keine bessere Alternative vorhanden ist. Lakatos‘ Maxime RF2 ist also nicht immer strikt durchführbar: eine kurzfristige Degeneration einer Theorienfolge kann auch hingenommen werden, wenn keine bessere Theorie in Aussicht ist (s. Keuth 1978). Ein Beispiel dafür ist die Entwicklung des Rutherford-Bohrschen Atommodells (s. Schurz/Lambert 1994, Kap. 4.3). In der Wissenschaftspraxis löst sich das Problem im Regelfall wie folgt: je stärker degenerativ eine Theorienfolge wird, desto mehr Wissenschaftler werden sich nach alternativen

5.6 Grundlegendes zur Theoriendynamik

Theoriekernen umsehen und versuchen, alternative Forschungsprogramme zu entwickeln. Die entsprechende Disziplin tritt dann in eine ,revolutionäre‘ Phase im Sinne von Kuhn ein. In der Geschichte der Naturwissenschaften wurden in solch revolutionären Phasen meistens neue überlegene Theoriekerne gefunden: im Falle des Bohrschen Atommodells war dieser alternative Theoriekern die Quantenmechanik. Im Bereich der Sozial- und Humanwissenschaften zeigt sich ein anderes Bild: viele dieser Disziplinen befinden sich in einem Zustand der anhaltenden Koexistenz von rivalisierenden Forschungsprogrammen, von denen keines die Vorherrschaft erringt (s. Schurz 1998a). Der Zustand, den Kuhn als ,revolutionär‘ bezeichnete, ist hier der Normalzustand. Lakatos‘ Methodologie ist ebenso auch auf Folgen von Theorieversionen anzuwenden, in denen eine Kernveränderung oder ,Revolution‘ stattfindet – seine Kriterien der theoretischen bzw. empirischen Progressivität finden auch hierfür Anwendung, sofern davon ausgegangen wird, dass die rivalisierenden Theorien über eine gemeinsame empirische oder vortheoretische Sprache verfügen. Lakatos illustrierte sein Wissenschaftsmodell anhand einer eindrucksvollen Reihe von Beispielen aus der Physik und Chemie. Wie wir in Kap. 5.5 und 5.7 zeigen, ist sein Ansatz ebenso auf Theorien der Psychologie und Sozialwissenschaften anwendbar.

Kernveränderungen

5.6.2 Theorienbewertung, Theorienvergleich und Theorienfortschritt Unter einer Theorieversion verstehen wir im folgenden einen Theoriekern zusammen mit seiner gesamten Peripherie und schreiben dafür KiVj – die j.te Version des Theoriekerns Ki. Eine Theorieversion enthält also alle Bestandteile, die nötig sind, um aus der Theorie gehaltvolle empirische Konsequenzen zu gewinnen. Wie Lakatos betrachten wir historische Folgen von Theorieversionen. Eine Theorie kann im Laufe ihrer Entwicklung verschiedene Versionen durchlaufen, sie kann in ihrer Peripherie angereichert, modifiziert oder reduziert werden, usw. Solange nur die Peripherie geändert wird, handelt es Wandlungen derselben ,Theorie‘. Eine Änderung des Theoriekerns führt dagegen zu einer neuen Theorie. Man kann die theoriendynamische Entwicklung also schematisch wie in Abb. 5.6-1 darstellen. Dabei können, wie erläutert, in einer Disziplin solche rivalisierenden Theorienfolgen qua konkurrierende Forschungsprogramme auch zeitlich nebeneinander existieren. Gemäß dem Zweiebenenmodell von Kap.2.3 werden Theorien im Regelfall anhand ihrer generellen empirischen Konsequenzen überprüft. Gene-

Normalwiss. Phase K1

K1

K1V1

K1V2

Revol. Phase

Normalwiss. Phase K2

K2

K2V1

K2V2

} Theorie 1

Abb. 5.6-1: Historische Theorienfolge

} Theorie 2

Historische Theorienfolgen

199

200

5. Theorien und ihre empirische Bewertung

Intendierter Anwendungsbereich einer Theorie

relle empirische Sätze werden gemäß den Methoden von Kap. 4 anhand von singulären Beobachtungssätzen überprüft, welche wir hier als unproblematisch voraussetzen. Es gibt auch spezielle Theorien, die direkt über Einzelindividuen sprechen, wie z. B. astronomische Theorien über bestimmte Galaxien, usw.; wir lassen auch diese Fälle zu. Einen generellen oder singulären empirischen Sachverhalt, der im intendierten Anwendungsbereich einer Theorie steht, nennen wir im folgenden ein empirisches bzw. experimentelles Phänomen. Wir sprechen davon, dass gewisse Phänomene aus einer Theorie folgen (gemeint sind natürlich die Phänomenbehauptungen). Die Folgerungsbeziehung zwischen der Theorie und den aus ihr folgenden Phänomenen ist häufig deduktiver Natur; sie kann aber auch statistischer oder normischer Natur sein. Wir entwickeln die zentralen Begriffe empirischer Überprüfung zunächst für Theorieversionen und dann, darauf aufbauend, für Theoriekerne. (Ms. 5.6-1) Eine Theorieversion KiVj ist falsifiziert, g. d. w. einige aus ihr deduktiv folgenden Phänomene durch aktuale Beobachtungssätze falsifiziert wurden. Normalerweise führt dies zur Konstruktion einer neuen Theorieversion KiVj+1. Volle Theorieversionen mit deduktiven empirischen Konsequenzen sind also nach wie vor falsifizierbar; nur Theorieversionen mit probabilistischen Konsequenzen oder Theorien qua Theoriekerne sind es nicht (rekapituliere Kap. 3.8). (Ms. 5.6-2) Ein Erfolg einer Theorieversion KiVj ist ein empirisches Phänomen P, das von KiVj korrekt vorausgesagt oder erklärt wird.

Erfolge und Misserfolge von Theorieversionen

Wurde P korrekt vorausgesagt, so handelt es sich um einen erfolgreichen Überprüfungsversuch der Theorie. Erfolgreiche Voraussagen zählen mehr als erfolgreiche ex-post-Erklärungen (s. Kap. 4.5.1, 5.2). (Ms. 5.6-3) Ein Misserfolg einer Theorieversion KiVj ist ein widerspenstiges Phänomen, worunter ein empirisch gut bestätigtes Phänomen P verstanden wird, welches eine der folgenden Eigenschaften besitzt: (a) P steht in logischem Widerspruch zu KiVj, oder (b) P wird durch KiVj sehr unwahrscheinlich gemacht, oder (c) P widersprach einer Vorgängerversion KiVj – 1 von KiVj, und KiVj entstand aus KiVj – 1 durch Anpassung mithilfe einer Hilfshypothese H, welche dazu dient, Falsifikation von KiVj durch P zu vermeiden, und welche im erläuterten Sinn ad hoc ist: außer der Vermeidung der Konsequenz non-P produziert H in KiVj keinen bestätigten empirischen Überschussgehalt (was weder impliziert noch ausschließt, dass H in KiVj noch unüberprüften empirischen Überschussgehalt produziert).

5.6 Grundlegendes zur Theoriendynamik

Es ist wesentlich, dass der Begriff des Misserfolges nicht nur logisch widersprechende oder probabilistisch konfligierende Phänomene umfasst, sondern auch solche Phänomene, die durch ad-hoc-Anpassungen der Theorieversion in nicht mehr konfligierende Phänomene verwandelt wurden. Solche Phänomene enttäuschen das von der Theorie gegebene Erklärungsversprechen; und ihr Charakter als Misserfolge wird dadurch erhärtet, dass die ad-hoc-Hypothesen, die ihren Konflikt mit der Theorieversion verhindern, im gegebenen Hintergrundwissen niemals hohe Plausibilität genießen können, denn bis dato gibt es für sie keine unabhängigen Bestätigungen (s. auch Schurz/Lambert 1994, § 3.4). Um logische Irrelevanzen und Redundanzen zu vermeiden, setzen wir voraus, dass die Menge aller Erfolge einer Theorie(version) bzw. die Menge ihrer Misserfolge jeweils durch die Mengen ihrer relevanten Konsequenzelemente bzw. durch deren Negationen repräsentiert werden. Den Grad der Bewährung oder Schwächung einer Theorieversion beurteilen wir nun über die Mengen ihrer Erfolge und Misserfolge. Dabei beschränken wir uns auf nichtfalsifizierte Theorieversionen: (Ms. 5.6-4) Eine nichtfalsifizierte Theorieversion KiVj ist umso bewährter, je mehr Erfolge und je weniger Misserfolge sie hat. Sie ist umso geschwächter, je weniger Erfolge und je mehr Misserfolge sie hat. Die Erfolge bzw. Misserfolge einer Theorieversion KiVj sind immer als relativiert zu denken auf einen gegebenen Wissensstand der scientific community, insbesondere auf eine gegebene Menge an empirischem Wissen. Man kann die Explikationen in (Ms. 5.6-1 – 5.6-4) auch dahingehend erweitern, dass man zusätzlich ein Hintergrundsystem W von unproblematischen Annahmen annimmt, und die Begriffe der deduktiven bzw. probabilistischen Konsequenz auf W hin relativiert bzw. konditionalisiert (so wie dies z. B. Glymour 1981, 35, vorschlägt; s. auch Lakatos 1974, 104). Die Beschränkung auf nichtfalsifizierte Theorieversionen hat folgenden Grund: von einer bewährten bzw. bestätigten Theorie(version) spricht man nur dann, wenn die Erfahrungen die Wahrscheinlichkeit der Theorie erhöhen; eine falsifizierte Theorie hat aber Wahrscheinlichkeit null, gegeben die Erfahrungen, und kann daher nicht als bestätigt bzw. bewährt gelten. Diese Sichtweise wird allerdings von Vertretern des Begriffs der Wahrheitsnähe angezweifelt. Dazu einige Erläuterungen. Gemäß der Grundidee von Popper (z. B. 1974, 60 – 142) ist eine Theorie(version) umso wahrheitsnäher, je mehr wahre und je weniger falsche Konsequenzen sie besitzt; wobei das „mehr“ bzw. „weniger“ im mengentheoretischen Inklusionssinn zu verstehen ist. (Hinweis: Für empirische Wahrheitsnäheauswertungen bezieht man sich, statt auf „wahre bzw. falsche“, auf „empirisch bestätigte bzw. widerlegte“ Konsequenzen. Poppers Wahrheitsnähedefinition war bekanntlich einigen Paradoxien ausgesetzt, welche auf irrelevanten Konsequenzen oder redundanten Konjunktionen beruhen, und von Schurz/Weingartner 1987 einer Lösung zugeführt wurden, für die sich später auch Popper 1992, 469,

Bewährung und Schwächung von Theorieversionen

Wahrheitsnähe

201

202

5. Theorien und ihre empirische Bewertung

Dynamische Wahrheitsnähe

ausspricht.) Wahrheitsnähevertreter argumentieren nun so: Theorien, die falsche Konsequenzen besitzen, müssen nicht verworfen werden, wenn sie hinreichend wahrheitsnahe sind, bzw. wahrheitsnäher als die vorhandenen Alternativtheorien (s. Kuipers 2000, Kap. 6.2.1). Diese Kritik ist berechtigt gegenüber der naiv-falsifikationistischen Sofortrationalität. Anders als Kuipers (ebd.) denke ich aber, dass das Wahrheitsnäheprogramm gegenüber der raffiniert-falsifikationistischen Strategie keinen Vorteil besitzt: ein Wissenschaftler, der ein der Theorie logisch widersprechendes Datum nicht gleich als Anlass zur Verwerfung der Theorie nimmt, wird normalerweise auch gewisse ad-hoc-Anpassungen an der Peripherie seiner Theorie anpeilen, um nicht der logischen Falsifikation ausgesetzt zu sein. Der Wahrheitsnäheansatz und der raffinierte Falsifikationsansatz laufen beide darauf hinaus, dass die Theorie angesichts widerspenstiger Daten nicht verworfen wird. Der Begriff des Misserfolges einer Theorie hat gegenüber dem Wahrheitsnäheansatz jedoch einen entscheidenden Vorteil. Man beachte nämlich, dass unter Misserfolgen im Sinn von (Ms. 5.6-3) nicht die erfahrungskonfligierenden Konsequenzen der Theorie verstanden werden, sondern die Phänomene, die mit diesen Konsequenzen konfligieren. Nur weil wir diesen Misserfolgsbegriff verwenden, können wir Misserfolge vom Typ (c) in (Ms. 5.6-3) berücksichtigen. Der Wahrheitsnähebegriff geht dagegen lediglich von den Konsequenzen einer Theorie aus: er kann zwar Misserfolge im Sinne von (a+b) einfangen, weil die Theorie die Negationen solcher Misserfolge impliziert oder wahrscheinlich macht; er kann aber nicht Misserfolge vom Typ (c) einfangen, weil die Theorie in keiner logischen oder probabilistischen Beziehung zu solchen Misserfolgen mehr steht – nur ihre Vorgängerversion tat dies. Man kann allerdings ein modifiziertes Wahrheitsnähekonzept entwickeln, welches sich direkt auf Erfolge und Misserfolge von Theorien (im oben definierten Sinn) bezieht. Wir sprechen hier von dynamischer Wahrheitsnähe, weil der Begriff des Misserfolges auf Vorgängertheorien Bezug nimmt: (Ms. 5.6-4*) Die dynamische Wahrheitsnähe einer Theorieversion KiVj (in Bezug auf die gegebenen Daten) ist umso höher, je mehr Erfolge und je weniger Misserfolg sie hat. – Sie ist umso geringer, je weniger Erfolge und je mehr Misserfolge sie hat. (Ms. 5.6-4*) unterscheidet sich von (Ms. 5.6-4) nur dadurch, dass nun auch falsifizierte Theorieversionen zugelassen werden. Die Kriterien (Ms. 5.6-4) bzw. (Ms. 5.6-4*) geben uns die Möglichkeit, vergleichende intertheoretische Bewertungen vorzunehmen. Die verglichenen Theorieversionen können dabei auch einen unterschiedlichen Kern besitzen; vorausgesetzt ist nur, dass es eine gemeinsame empirische Grundsprache gibt, in der Phänomene und empirische Konsequenzen formuliert werden. Das einfachste intertheoretische Vergleichskriterium, das ähnlich verfährt wie Poppers Wahrheitsnähebegriff, ist folgendes:

5.6 Grundlegendes zur Theoriendynamik

(Ms. 5.6-5) Intertheoretischer Vergleich und Theorienfortschritt: K1V1 ist empirisch erfolgreicher als K2V2 g.d.w. entweder K1V1 mehr Erfolge und nicht mehr Misserfolge hat als K2V2, oder wenn K1V1 weniger Misserfolge und nicht weniger Erfolge hat als K2V2. Die Entwicklung von K1V1 nach K2V2 ist ein empirischer Theorienfortschritt, wenn K1V1 empirisch erfolgreicher ist als K2V2.

Empirischer Theorienvergleich

Das „mehr“ und „weniger“ ist dabei wieder im Sinn einer Mengeninklusion zu verstehen, und nicht im numerischen Sinn des Abzählens. Dies ist in Abb. 5.6-2 dargestellt:

T2 Erfolge Ungeprüfter empirischer Gehalt Misserfolge T1

Abb. 5.6-2: T2 ist empirisch erfolgreicher als T1

Es wäre unsinnig, den empirischen Erfolg von Theorien durch Abzählen ihrer Erfolge bzw. Misserfolge vergleichen zu wollen – also z. B. zu sagen, die Piagetsche Entwicklungstheorie sei empirisch erfolgreicher als das Bohrsche Atommodell, weil erstere 5 Erfolge und 4 Misserfolge besitzt, aber letzteres nur 4 Erfolge und 5 Misserfolge hat. Ein intertheoretischer Vergleich macht überhaupt nur Sinn, wenn die Theorien über denselben Bereich empirischer Phänomene sprechen. Daher scheint die mengentheoretische Inklusionsrelation das adäquate Fortschrittskriterium zu sein. Daraus folgt freilich, dass die Beziehung des größeren empirischen Erfolges im Sinne von (Ms. 5.6-5) nur eine partielle Ordnung unter diversen Theorieversionen herstellt. Wann immer zwei Theorieversionen sich so zueinander verhalten, dass die eine sowohl einige Misserfolge wie einige Erfolge enthält, welche die andere nicht enthält, dann ist mit diesem Kriterium keine eindeutige Aussage mehr in Bezug auf empirische Erfolgspräferenz möglich. (Auch Poppers Wahrheitsnähekriterium liefert nur eine partielle Ordnung in diesen Sinn; s. Schurz/Weingartner 2010.) Dies bringt uns zu einer weiteren wichtigen Beziehung zwischen Theorien hinsichtlich ihres empirischen Erfolges, nämlich die Beziehung der empirischen Komplementarität: zwei Theorieversionen T1 und T2 sind empirisch komplementär, wenn beide jeweils Erfolge in Bereichen besitzen, wo die andere Theorieversion Misserfolge oder neutrale Nicht-Erfolge besitzt; zusätzlich können sich beide auch in einigen Erfolgen und Misserfolgen überschneiden. Siehe Abb. 5.6-3.

Empirisch komplementäre Theorien

203

204

5. Theorien und ihre empirische Bewertung E2

E1

U2

U1 1

M1

Notation wie Abb. 5.6-2: E = Erfolge U = ungeprüfter emp. Gehalt

M2 4

3

M = Misserfolge

2 Theorie T1

Theorie T2

Abb. 5.6-3: Empirische komplementäre Theorieversionen Feld 1: gemeinsamer Erfolg von T1 und T2; Feld 2: gemeinsamer Misserfolg; Feld 3: T2-Erfolge, die T1-Misserfolge sind; Feld 4: T1-Erfolge, die T2-Misserfolge sind. Die Komplementärfelder 3 und 4 sind signifikant belegt, verglichen zu Übereinstimmungsfeldern 1 und 2.

Einfachheit von Theorien

Wissenschaftsgeschichtliches Fallbeispiel

Ein Beispiel sind Aggressionstheorien im Bereich der Psychologie (vgl. Nolting 1978, Schurz 1998a, 33): hier sind die Frustrationstheorie, die Lerntheorie und die Triebtheorie der Aggression wohletablierte Ansätze, ohne dass eine klare Ablösung stattfand. Dies liegt daran, dass es für alle drei Theorien prototypische Beispiele gibt, die von den jeweils anderen Theorien nicht befriedigend erklärbar sind. Es gibt Fälle von durch Frustration ausgelöster Aggression, die nicht auf Nachahmung oder Lernen zurückführbar sind; es gibt Fälle von aggressivem Verhalten durch Lernen, also durch Nachahmung eines ,role models‘, denen kein frustrierender Anlass vorausgeht, und es gibt schließlich spontan-aggressives Verhalten bei Kindern, dem weder eine Frustrations- noch eine Lerngeschichte vorausgeht. Man kann die Explikation (Ms. 5.6-5) der empirischen Theorienpräferenz in verschiedener Weise verfeinern. Z. B. kann man Phänomene zwar nicht abzählen, aber man kann sie unterschiedlich gewichten (vgl. Schurz/Lambert 1994, § 2). Zusätzlich kann man auch die Einfachheit einer Theorie als ceteris-paribus-Präferenzkriterium vorschlagen. Doch Einfachheit ist schwer zu präzisieren, weil sie verschiedene Dimensionen besitzt: Einfachheit in der Sprache, in den Axiomen, im komputationellen Rechenaufwand, usw. (Bunge 1961, 147 f.; Kutschera 1972, 328). Überdies ist fraglich, warum ein ästhetisches Kriterium wie Einfachheit für die Beurteilung des objektiven Bestätigungsgrades einer Theorie herangezogen werden sollte. Wie Forster und Sober (1994) argumentieren, ist Einfachheit ein epistemisches Epiphänomen, d. h.: dort, wo Einfachheit einen berechtigten Vorzugsgrund abgibt, ist dies deshalb der Fall, weil Einfachheit auf einen objektiven Bevorzugungsgrund verweist, wie z. B. höhere Wahrscheinlichkeit aufgrund weniger ad-hoc-Annahmen. Eine wesentliche Stütze unseres Bewertungsmodells ist, dass sich Wissenschaftler in ihren de-fakto-Theorienbewertungen genau an solchen Erfolgsund Misserfolgsmengen zu orientieren scheinen. Bunge (1967, Bd.II, 47) stellt beispielsweise die empirischen Erfolge und Misserfolge von verschiedenen Theorien der Optik zusammen, so wie man sie in einschlägigen Lehr-

5.6 Grundlegendes zur Theoriendynamik

Ray optics (HERO von Alexandrien)

Hydrodynamic aether theory (DESCARTES)

Corpuscular theory (NEWTON)

Longitudinal wave theory (HUYGENS)

Transversal wave theory (FRESNEL, CAUCHY, GREEN)

Electromagnetic theory (MAXWELL)

Electromagnetic theory without aether

Electromagnetic theory with curved space

Electromagnetic theory and electron theory

Quantum electrodynamics and nonrelativistic Q.M.

Quantum electrodynamics and relativistic Q.M.

büchern findet, in historischer Aufreihung von links nach rechts. Dies ist in Abb. 5.6-4 dargestellt.

1. Rectilinear propagation

X

X

X

X

X

X

X

X

X

X

X

2. Reflection

X

X

X

X

X

X

X

X

X

X

X

3. Refraction

X

X

X

X

X

X

X

X

X

X

X

4. Extremal travel time

X

X

X

X

X

X

X

X

X

X

X

5. Dispersion

X

X

X

X

X

X

X

X

X

X

X

6. Superposition

X

X

X

X

X

X

X

X

7. Double refraction

X

X

X

X

X

X

X

X

8. Decrease of speed intransparent media

X

X

X

X

X

X

X

X

9. Diffraction

X

X

X

X

X

X

X

X

10. Interference

X

X

X

X

X

X

X

X

11. Doppler effect

X

X

X

X

X

X

X

X

X

X

X

X

X

X

X

X

X

X

X

X

X

X

X

X

Theories of Light !

Empirical Phenomena #

12. Polarization 13. Radiation pressure

X

14. Anomalous dispersion 15. Invariant speed light 16. Change of frequency in gravitational field 17. Light scattering

X

X

X

X X

X

X

18. Blackbody spectrum

X

X

19. Photoelectric effect

X

X

20. Compton effect

X

X

Abb. 5.6-4: Theorien der Optik nach Bunge (1967, 47) – „x“ steht für „Erfolg“, ein fehlendes „x“ steht für „Misserfolg“ oder für „damals noch unbekannt“.

205

206

5. Theorien und ihre empirische Bewertung

Aus diesen Beispielen geht auch klar hervor, dass unerfasste Phänomene im intendierten Anwendungsbereich einer Theorie von Wissenschaftlern immer als Misserfolge gezählt werden, unabhängig davon, ob ad-hoc-Einschränkungen des empirischen Gehalts der Theorieversion vorgenommen werden oder nicht. Die Frage, ob eine gegebene Theorieversion rational zu akzeptieren oder besser aufzugeben ist, hängt wie erläutert nicht nur vom Stand des empirischen Erfolges, sondern auch vom Stand der Alternativtheorien ab: Akzeptanz und Zurückweisung von Theorieversionen

Das Klebeparadox

(Ms. 5.6-6) (1.) Eine Theorieversion ist rational akzeptierbar, solange sie hinlänglich bewährt ist und keine alternative Theorieversion existiert, die deutlich empirisch erfolgreicher ist. (2.) Eine Theorieversion ist zurückzuweisen, wenn sie stark geschwächt ist und zumindest eine deutlich empirisch erfolgreichere alternative Theorieversion existiert. Bewusst ist (Ms. 5.6-6, 2.) nicht die Negation von (Ms. 5.6-6, 1.) (das wäre nur dann der Fall, wenn statt dem „und“ in Ms. 5.6-6, 2. ein „oder“ stünde). Wir lassen also den Fall zu, wo sich eine Theorieversion in einem Zwischenstadium zwischen rationaler Akzeptanz und Zurückweisung befindet – sie hat sich einerseits nicht sonderlich bewährt, aber es steht andererseits auch keine bessere Alternativtheorie zur Verfügung. Wie in Abb. 5.6-2 eingezeichnet, enthält der empirische Gehalt einer Theorie immer auch einen empirisch ungeprüften Gehaltsanteil. Gemäß dem Kriterium der theoretischen Progression von Lakatos sollte ein größerer ungeprüfter empirischer Gehalt einer Theorieversion ceteris paribus ebenfalls als Vorzugskriterium bzw. als Fortschritt angesehen werden. Dass Kühnheit von Theorien im Sinne eines großen ungeprüften Gehaltes ein Präferenzkriterium ist, hat auch Popper betont (1935/76, 225 f., 373). Bei dieser Idee ist jedoch Vorsicht geboten: nicht jede beliebige ungeprüfte Gehaltsvermehrung zählt als Fortschritt. Lakatos erwähnt selbst das tacking paradox (Klebeparadox) – das ist die Möglichkeit, durch die bloße konjunktive Hinzufügung irgendeiner empirisch ungeprüften Behauptung den empirischen Gehalt einer Theorieversion zu erhöhen (1974, 128). Beispielsweise würde man es wohl nicht als empirischen Gehaltsfortschritt, sondern lediglich als Sensationshascherei ansehen, wenn ein Wissenschaftler die etablierte Version der Quantenmechanik behauptet und hinzufügt, seine Theorie behaupte auch die Existenz von Gedankenübertragung, ohne eine echte theoretische Verbindung herzustellen (manch Physiker ist solchen publizistischen Versuchungen erlegen). Lakatos fordert, die Verbindung einer neuen empirischen Gehalt erzeugenden Hilfshypothese mit der bisherigen Theorie muss inniger sein als die einer bloßen Konjunktion, ohne jedoch genau zu sagen, was er damit meint (1974, 128). Das Klebeparadox ist auch in der Bestätigungsliteratur intensiv diskutiert worden, und es wird hier durch den Begriff der relevanten Bestätigung gelöst (s. Kap. 3.10.3, 5.8.3.1). Zur Lösung des Klebeparadoxes in Bezug auf

5.6 Grundlegendes zur Theoriendynamik

empirische Gehaltserweiterungen stellen wir an eine Theorie T die Forderung, dass T hinsichtlich ihres empirischen Gehaltes homogen ist in folgendem Sinn: eine Faktorisierung von T in Bezug auf E(T) ist nicht möglich. Dabei verstehen wir unter einer Faktorisierung von T in Bezug auf E(T) eine Unterteilung von T und von E(T) in je zwei disjunkte (relevant repräsentierte) Teilmengen, T1[T2 = T und E1[E2 = E(T), sodass T1 alle Phänomene in E1 impliziert und T2 alle Phänomene in E2 impliziert (,[‘ für ,Vereinigung‘). Ist umgekehrt eine solche Faktorisierung möglich, so ist T eine heterogene Theorie in Bezug auf E(T). Jede durch irrelevante Verstärkung gewonnene Theorie T^H ist in diesem Sinn faktorisierbar. In der Terminologie von Kap. 5.4.1 gilt dann: die Verbindung von T und H ist nicht empirisch kreativ. Wir können Lakatos‘ Idee des empirischen Gehaltsüberschusses als ceteris-paribus-Präferenzkriterium adäquat explizieren, indem wir uns auf homogene Theorieversionen beziehen. Wir nennen zwei Theorieversionen K1V1 und K2V2 empirisch erfolgsäquivalent, wenn ihre Erfolgsmengen und ihre Misserfolgsmengen übereinstimmen.

(Ms. 5.6-7) Empirischer Gehaltsüberschuss im intertheoretischen Vergleich: Von zwei homogenen und empirisch erfolgsäquivalenten Theorieversionen K1V1 und K2V2 ist K1V1 rational vorzuziehen, wenn K1V1 mehr ungeprüften empirischen Gehalt besitzt als K2V2 („mehr“ wieder im mengentheoretischen Inklusionssinn).

Faktorisierung von Theorien

Empirischer Gehaltsüberschuss

Nachdem wir die wichtigsten Kriterien für die Bewährung, Präferenz bzw. Akzeptanz von Theorieversionen entwickelt haben, kommen wir zur Frage der Bewährung von Theoriekernen. Der Grad der Bewährung eines Theoriekernes hängt aufgrund unserer Ausführungen nicht nur von seiner aktuellen Theorieversion, sondern auch von der Geschichte seiner Theorieversionen ab.

(Ms. 5.6-8) Ein Theoriekern ist umso bewährter – je bewährter seine aktuelle Version ist im Sinne von (Ms. 5.6-4), – je mehr empirischen Gehalt diese hat, vorausgesetzt sie ist homogen, und – je weniger seiner früheren Versionen falsifiziert wurden.

(Ms. 5.6-9) Ein Theoriekern ist umso geschwächter, – je geschwächter seine aktuelle Version ist im Sinne von (Ms. 5.6-4), – je weniger empirischen Gehalt diese hat, und – je mehr seiner früheren Versionen falsifiziert wurden.

Bewährung und Schwächung von Theoriekernen

207

208

5. Theorien und ihre empirische Bewertung

Vergleich von Theoriekernen

(Ms. 5.6-10) Ein Theoriekern K1 ist einem Theoriekern K2 rational vorzuziehen wenn: (a) entweder die aktuelle Version von K1 empirisch erfolgreicher ist als die aktuelle Version von K2, oder (b) wenn hinsichtlich (a) Gleichwertigkeit gegeben ist und die aktuelle Version von K1 homogen ist und einen größeren ungeprüften empirischen Gehalt besitzt als die aktuelle Version von K2, oder (c) wenn hinsichtlich (a) und (b) Gleichwertigkeit gegeben ist, aber die Geschichte von K1 weniger falsifizierte Vorgängertheorien enthält als die Geschichte von K2. Die Frage, ob ein Theorieprogramm bzw. ein Theoriekern rational zu akzeptieren, beizubehalten oder aufzugeben ist, hängt wie im Fall von Theorieversionen vom Stand der Alternativtheorien ab.

Akzeptanz und Zurückweisung von Theoriekernen

(Ms. 5.6-11) Ein Theoriekern ist rational akzeptierbar, solange er hinlänglich bewährt ist und kein alternativer Theoriekern existiert, der empirisch erfolgreicher ist (gemäß Ms. 5.6-10, a). (Ms. 5.6-12) Ein Theoriekern ist zurückzuweisen, wenn er hinlänglich geschwächt ist und zumindest ein rational vorzuziehender alternativer Theoriekern existiert. Man beachte wieder, dass (Ms. 5.6-11) nicht die Negation von (Ms. 5.6-12) ist; Theoriekerne können in einem unbefriedigenden Zwischenstadium zwischen Akzeptanz und Zurückweisung verharren. Kriterium (Ms. 5.6-11,12) lässt daher auch einen Paradigmenpluralismus zu, also eine Situation, in der verschiedene konkurrierende Theoriekerne rational akzeptierbar sind.

5.7 Das Beispiel der Adorno-Milgramschen Theorie des autoritären Charakters Die Adorno-Milgramsche Theorie des autoritären Charakters ist eine sozialwissenschaftliche Theorie. Sie wurde von Adorno et al. in „The Authoritarian Personality“ (1950) als eine makrosoziologische These entwickelt, mit der Adorno und später Milgram (1974) u. a. erklären wollten, wie es zu Nationalsozialismus und Faschismus kommen konnte. Die soziologische Kernthese dieser Theorie lautet: Kern der AdornoMilgram-Theorie

(K): Unsere Gesellschaft ist in ihrem Innern autoritär strukturiert, das heißt, die Individuen haben autoritäre gesellschaftliche Strukturen in Form eines stark autoritätsorientierten Charakters (aC) verinnerlicht. Wir wollen nun zeigen, dass unser Theorienmodell auch auf eine solche Theorie, die von empirischen Wissenschaften weit entfernt zu sein scheint,

5.7 Adorno-Milgramsche Theorie des autoritären Charakters

anwendbar ist. Der springende Punkt in der Weiterentwicklung dieses Theoriekerns war die folgende Idee von Milgram (1974), den verinnerlichten Autoritarismus empirisch zu prüfen: Menschen mit stark autoritätsorientiertem Charakter müssten gegenüber von ihnen anerkannten Autoritäten eine starke Tendenz zu bedingungsloser Unterwerfung aufweisen. Wir formulieren dies als Spezialgesetz der Theorie – es spezifiziert die Bedeutung von „autoritärem Charakter“, ist aber noch kein volles Zuordnungsgesetz, weil es noch keine definitive Verbindung mit beobachtbaren Begriffen erzeugt, sondern diesen nur näher kommt: (S1): Menschen mit autoritätsorientiertem Charakter (aC) tendieren zur bedingungslosen Unterwerfung (bU) gegenüber von ihnen anerkannten Autoritäten (aA). Die weitere Idee von Milgram, die ihn zu seinem in den frühen 1960er Jahren durchgeführten Experiment führte, war nun folgende: man nehme eine gesellschaftlich anerkannte Autorität, z. B. einen wissenschaftlichen Experten. Dieser Wissenschaftler soll die Vp zu einer Handlung auffordern, die in seinen Kompetenzbereich fällt, z. B. an einem wissenschaftlich gutgeheißenen Experiment mitzuarbeiten. Zugleich soll die Handlung so geartet sein, dass sie den allgemein und somit auch von der Vp anerkannten Moralvorstellungen stark widerspricht. Konkret ließ Milgram die Vp von einem Wissenschaftler im weißen Kittel auffordern, in einem wissenschaftlich gutgeheißenem Experiment eine andere Person, die bzgl. ihrer Lernfähigkeit trainiert werden sollte, durch Elektroschocks negativ zu konditionieren. Natürlich wurde der Wissenschaftler sowie die scheinbar gequälte Person in Wirklichkeit durch Schauspieler gespielt, und der zugefügte Schmerz war nicht echt. Die Szene wurde jedoch so gespielt, dass die Vp glaubte, sie wäre echt: die Vp sah die scheinbar gequälte Person entweder auf einem Videoschirm oder hörte ihr Schreien und glaubte wirklich, dass die Person Schmerzen leidet. Die Vp stand somit in einem inneren Konflikt: sollte sie gemäß ihrer moralischen Einstellung handeln, oder sollte sie sich an der Autorität des ,weißen Kittels‘ orientieren, welcher versichert, das Experiment sei trotz der Schmerzzufügung ethisch vertretbar und äußerst wichtig? Dabei hatte die Vp die Freiheit, das Experiment abzubrechen; sie wurde vom Wissenschaftler im weißen Kittel nicht zum Weiterführen gezwungen, sondern nur verbal dazu zu überreden versucht. Personen, die in einer solchen Situation gegen ihre ansonsten akzeptierten Moralvorstellungen sich der Autorität freiwillig unterwerfen, sind gemäß Milgram sichere Belege für eine Tendenz zur bedingungslosen Unterwerfung. Damit können wir folgendes weitere Spezialgesetz der Adorno-Milgramschen Theorie formulieren, das uns der Ebene der Beobachtung noch näher bringt:

Das MilgramExperiment

(S2): Personen P, die zur bU gegenüber aA tendieren, zeigen folgendes Verhalten: wenn eine aA P zu einer Handlung H auffordert aber nicht zwingt, wobei (i) H in den Kompetenzbereich der aA fällt und

Spezialgesetz: empirische Operationalisierung

209

210

5. Theorien und ihre empirische Bewertung

(ii) H mit den sonstigen von P akzeptierten Fundamentalnormen in Konflikt steht, dann wird P dennoch mit hoher Wahrscheinlichkeit H ausführen. Schließlich kommen an der Peripherie der Adorno-Milgramschen Theorie noch die speziellen Indikatorgesetze hinzu, die Milgram für seine empirische Prognose benutzte: (I1): Ein Beispiel für eine allgemein (und daher auch von P) anerkannte Autorität ist ein Wissenschaftler. (I2): Ein Beispiel für eine Handlung H, die die Bedingungen (i) und (ii) von Spezialgesetz S2 erfüllt, ist die Durchführung eines wissenschaftlich gutgeheißenen Experiments, worin einer Person Schmerzen zugefügt werden. K+S1+S2+I1+I2 liefern zusammen die Prognose P des Milgram-Experimentes: Empirische Prognose der Adorno-MilgramTheorie

Mögliche Indikatorvariationen in der Adorno-MilgramTheorie

(P) Ein hoher %-Satz der Menschen wird tatsächlich das im Milgram-Experiment vorgetäuschte ,wissenschaftliche Experiment‘ durchführen. Tatsächlich erbrachte das Milgram-Experiment genau dieses für viele Menschen schockierende Ergebnis. Milgram sah dies als einen starken Erfolg der Theorie des autoritären Charakters an. Erläuterung zur Folgerungsbeziehung: Es fällt auf, dass beide Spezialgesetze S1+2 nur statistische Trendaussagen machen. Realistischerweise wird auch das Kerngesetz nicht als strikt aufzufassen sein. Sowohl K, S1 wie S2 sind also als normische Hypothesen zu konstruieren. Um aus diesen normischen Hypothesen die Prognose wahrscheinlichkeitslogisch zu gewinnen, müssen zusätzliche Irrelevanzannahmen gemacht werden (wie unterhalb von 5.1-10 erläutert), die jedoch völlig harmlos sind. Um den Theoriekern der Adorno-Milgramschen Theorie wirklich als empirisch bewährt anzusehen, müsste man entsprechend der methodischen Devise von (Ms. 5.1-2) weitere Tests mit variierenden Indikatorgesetzen anstellen. Der Theoriekern macht nämlich eine sehr generelle Aussage; beim Milgramschen Experiment könnte es sich jedoch auch nur um ein Spezialphänomen der wissenschaftlichen Expertokratie handeln, welches nicht auf Autoritäten oder Handlungen beliebiger Art übertragbar ist. Andere solche Indikatoren bzw. Überprüfungsbeispiele wären etwa folgende: Indikatorvariation 1: Ein österreichischer Offizier sagt zu seinen Soldaten: die euch im folgenden aufgegebene Aktion zerstört zwar große Teile des Wienerwaldes, ist aber für die österreichische Sicherheit unerlässlich. Werden dann die meisten Soldaten mitmachen? Indikatorvariation 2: Ein Priester oder religiöser Sektenführer sagt zu seinen Anhängern: zur Vertreibung des Bösen aus diesem Menschen ist es nötig, ihn in einer bestimmten Art zu foltern – werden dann die gläubigen Anhänger mitmachen? Meines Wissens sind Überprüfungen der Adorno-Milgramschen Theorie mit variierten Beispielen bzw. Indikatoren nie gemacht worden. Würden

5.8 Weiterführende Themen

solche variierten Experimente erneut eine positive Bestätigung der variierten Theorienversion K+S1+S2+I1*+I2* liefern, so wäre das ein guter Grund, auch die Kernthese K selbst als hinlänglich bewährt und als vermutlich zutreffende Erklärung für den Faschismus anzusehen. Würde ein solches Experiment mit variiertem Indikator ein negatives Ergebnis erbringen, so wären gemäß dem Holismus der Theorienüberprüfung folgende Interpretationsmöglichkeiten gegeben: (1.) die gemäß (I1*) bestimmte Person ist keine anerkannte Autorität; d. h. das variierte Indikatorgesetz (I1*) wird verworfen; oder, die gemäß (I2*) bestimmte Handlung H fällt nicht in den Bereich, für den die anerkannte Autorität als kompetent gehalten wird, oder sie widerspricht nicht wirklich den ansonsten anerkannten Moralvorstellungen, d. h. das variierte Indikatorgesetz I2* wird verworfen. Beispielsweise könnte der Priester keine von den betroffenen Personen wirklich anerkannte Autorität sein, oder das Verwüsten des Wienerwaldes könnte für die ausführenden Soldaten keine wirklich moralisch verwerfliche Handlung sein. (2.) Falls aber I1* und I2* sehr plausibel sind, wird man dazu übergehen, z. B. S1 zu verwerfen: man könnte dann vermuten, dass die Unterwerfungstendenz nur gegenüber bestimmten Autoritäten auftritt, z. B. nur gegenüber scheinbar objektiven Experten im weißen Kittel, nicht aber gegenüber Politikern etc. Und (3.) schließlich könnte man auch die Kernthese selbst bezweifeln.

5.8 Weiterführende Themen 5.8.1 Instrumentalismus und Realismus Der wissenschaftstheoretische Realismus geht davon aus, dass die theoretischen Begriffe wissenschaftlicher Theorien referieren, d. h. reale aber unbeobachtbare Individuen, Eigenschaften oder Prozesse bezeichnen. Der wissenschaftstheoretische Instrumentalismus bestreitet dies: ihm zufolge können nur empirische Begriffe reale Referenz besitzen. Theoretische Begriffe besitzen lediglich instrumentelle Funktion in der Organisation unseres empirischen Wissens, also z. B. die Funktion, eine Vielzahl zusammenhängender empirischer Regelmäßigkeiten auf einfachste Weise darzustellen und zu systematisieren. Die Auseinandersetzung zwischen Instrumentalismus und Realismus hat eine lange Tradition (Losee 1977, 28). In den 1950er und frühen 1960er Jahren dominierte die vom logischen Empirismus beeinflusste instrumentalistische Position (z. B. Hempel 1958, Carnap 1956, Sneed 1971); sie wird heute prototypisch von van Fraassen bezogen. Für van Fraassen (1980, 8 – 12) ist die Frage, ob Theorien wahr wären, sinnlos; es stellt sich lediglich die Frage der empirischen Adäquatheit von Theorien. Ab den späten 1960er Jahren wurden vermehrt realistische Positionen entwickelt (Putnam 1962, Maxwell 1962, Bhaskar 1978, Leplin 1984). Ein Realist muss von einer Theorie, deren theoretische Begriffe nicht referieren, sagen, dass sie strenggenommen falsch ist, denn aus einer Theorie T(s) mit theoretischem Begriff s folgt logisch die Existenzbehauptung 9sT(s). Es muss zwischen verschiedenen Arten des wissenschaftstheoretischen Realismus unterschieden werden (s. Kuipers 2000, Kap. 1). Der metaphysi-

Realismus vs. Instrumentalismus

Arten des Realismus

211

212

5. Theorien und ihre empirische Bewertung

Das Wunderargument nach Putnam

Quines empirische Unterbestimmtheitsthese

sche Realismus geht davon aus, dass die Annahme der realen Referenz theoretischer Begriffe eine unerlässliche Annahme ist, um sinnvoll Wissenschaft betreiben zu können; andernfalls wären Theorien weder bedeutungsvoll noch rational vergleichbar (z. B. Putnam 1978, 110 ff.; Bunge 1974, 45). Diese Position scheint unhaltbar zu sein. Es gibt viele theoretische Begriffe vergangener Theorien, von denen man heute weiß, dass sie keine Referenz besitzen, obwohl diese Theorien eine Zeitlang erfolgreich waren: ein bekanntes Beispiel ist das Phlogiston (der Feuerstoff) und das Kalorium (der Wärmestoff) chemischer Theorien im 18. und frühen 19. Jahrhundert (Carrier 2003). Auch in zahlreichen Theorien der Gegenwart wird die Frage, ob gewisse theoretische Konzepte wirklich reale Referenz besitzen oder nur nützliche Fiktionen sind, nachhaltig diskutiert (vgl. van Fraassen 1980, 11). Dem in Kap. 2.7.1 erläuterten hypothetisch-konstruktiven Realismus zufolge kann die Frage, ob und inwieweit ein theoretischer Begriff reale Referenz besitzt, nicht a priori entschieden werden – die Plausibilität der Referenzhypothese hängt vom Erfolg dieses Begriffs in der Erfahrungserkenntnis ab. In dieser Sichtweise konvergiert die realistische Begründungsfrage mit der instrumentalistischen Bedeutungsfrage: die instrumentalistische Fragestellung nach der empirischen Adäquatheit einer Theorie ist in dieser Sicht unerlässlich für die Beantwortung der realistischen Begründungsfrage (s. Kuipers 2000, Kap. 6.2.1). Es gibt ein vieldiskutiertes Standardargument zugunsten des wissenschaftstheoretischen Realismus: das Wunderargument (Putnam 1978, 19 ff.; Musgrave 1988). Dieses Argument besagt, dass es so unwahrscheinlich wäre wie ein Wunder, wenn Theorien, die über lange Zeit empirisch erfolgreich waren, nicht auch im realistischen Sinne wahr wären. Das Gegenargument ist die empirische Unterbestimmtheitsthese von Quine (z. B. 1960, 141 ff., 1995, § 41), derzufolge es immer möglich ist, zu einer gegebenen Theorie T empirisch äquivalente Theorien T* mit stark unterschiedlichem und sogar inkompatiblen theoretischem Überbau zu konstruieren, sodass unmöglich T und T* zugleich wahr sein können. Derartige empirisch äquivalente Theorietransformationen sind jedoch immer post-hoc. Worrall (1997, 153 f.) und Carrier (2003, § 4) haben herausgearbeitet, dass das Wunderargument, wenn überhaupt, nur dann Geltung besitzen kann, wenn man unter dem empirischen Erfolg von Theorien die Fähigkeit versteht, qualitativ neue Voraussagen zu machen (im Sinne von Kap. 5.2). Keine posthoc konstruierte Quine-Theorie T* hat das jemals gekonnt. Laudan (1997, 121 f.) und Carrier (2003, § 7) geben jedoch historische Beispiele für Theorien, die sogar qualitativ neue empirische Voraussagen machten, obwohl ihre theoretischen Begriffe aus heutiger Sicht komplette Fiktionen sind – die Phlogistontheorie und die Theorie des Kalorium. Es scheint, dass diese Beispiele das Wunderargument widerlegen. In Schurz (2009) wird dagegen wie folgt argumentiert: wann immer eine Theorie T erfolgreich qualitativ neue Voraussagen gemacht hat, und später von einer noch erfolgreicheren Theorie T* mit anderem theoretischen Überbau abgelöst wird, dann muss es im theoretischen Überbau von T einen Begriff geben, welcher zu einem möglicherweise komplexen theoretischen Begriff von T* in einer Brückenbeziehung steht, und daher indirekte realistische

5.8 Weiterführende Themen

Referenz besitzt, sofern man annimmt, dass T* realistische Referenz besitzt. In diesem Sinn besaß der Begriff der Phlogistonierung folgende indirekte Referenz: die Phlogistonierung einer Substanz (Aufnahme von Phlogiston) entspricht in der modernen Chemie der Abgabe von Elektronen an den Bindungspartner.

Indirekte Referenz durch Brückenbeziehung

5.8.2 (Nicht-)Eliminierbarkeit und (Un-)Definierbarkeit theoretischer Begriffe In den 1950er und 1960er Jahren wurde die aus instrumentalistischer Sicht vorrangige Frage behandelt, inwieweit theoretische Begriffe empirisch unentbehrlich oder entbehrlich seien (s. Hempel 1958, 186). Ein logisches Theorem von Craig, von dem sich einige Autoren die Eliminierbarkeit theoretischer Begriffe versprachen, erwies sich schnell als Enttäuschung (Hempel 1958, § 9; Kutschera 1972, 298 – 300). Aussichtsreicher erschien eine auf Ramsey (1931, 212 – 215) zurückgehende Methode, die darin besteht, die theoretischen Begriffe einer Theorie nicht gänzlich zu eliminieren, sondern über sie existentiell zu quantifizieren. Gegeben sei eine Theorie, welche wir nun als einen einzigen Satz T(s1, … sn) anschreiben (die Konjunktion aller Axiome von T), mit den theoretischen Begriffen s1, …, sn, sowie mit diversen nichttheoretischen Begriffen pi, die nicht extra angeschrieben werden. Dann lautet ihr Ramsey-Satz R(T): (5.8-1)

R(T): 9X1, …, Xn: T(X1, … Xn),

in Worten, es gibt irgendwelche theoretischen Entitäten X1, …, Xn, welche die Behauptungen der Theorie erfüllen. Man beweist leicht, dass ein empirischer bzw. nichttheoretischer Satz genau dann aus T folgt, wenn er aus R(T) folgt (s. z. B. Tuomela 1973, 57; Ketland 2004, 293, Th. 3). Es gilt also (5.8-2)

Ramsey-Satz einer Theorie

E(R(T)) = E(T),

d. h., eine Theorie und ihr Ramsey-Satz haben denselben empirischen Gehalt. Da im Ramsey-Satz keine theoretischen Begriffe mehr vorkommen, sondern nur mehr ,theoretische‘ Variablen, sahen viele Autoren, einschließlich Ramsey, den Ramsey-Satz einer Theorie als empirischen bzw. nichttheoretischen Satz an (Stegmüller 1986, 46; Ketland 2004, 294 f.; u. a. m.). Also glaubte man, im Ramsey-Satz eine empirisch äquivalente nichttheoretische Axiomatisierung der Theorie gefunden zu haben. Diese Sichtweise ist jedoch problematisch, da der Ramsey-Satz die Existenz von gewissen Entitäten behauptet, die wir als „theoretisch“ bezeichnen (s. auch Maxwell 1962, 17; Hempel 1958, 216). Die Interpretation des Ramsey-Satzes als theoretisch oder nichttheoretisch hängt davon ab, ob man eine instrumentalistische oder realistische Interpretation der Quantoren 2. Stufe vornimmt. In der instrumentalistischen Interpretation von R(T) nimmt man an, der Individuenbereich D besteht aus empirisch zugänglichen Individuen, und man lässt die Variablen Xi über beliebige Teilmengen von D laufen – ob diese Extensionen gewissen theoretischen Realeigenschaften entsprechen

Instrumentalistische Deutung des Ramsey-Satzes

213

214

5. Theorien und ihre empirische Bewertung

Realistische Deutung des Ramsey-Satzes

Carnap-Satz einer Theorie

oder nicht, ist dabei belanglos (diese Interpretation teilen sich viele Autoren, von Sneed 1971 bis zu Ketland 2004, 291). In diesem Fall ist der Ramsey-Satz modelltheoretisch gesehen in der Tat ein empirischer Satz. Denn die Modelle, welche den Wahrheitswert von R(T) bestimmen, sind dann rein empirische Modelle (D,e1, …, em); dabei sind die „ei“ die Extensionen der empirischen Begriffe „pi“ von T (diese empirischen Modelle werden im Strukturalismus auch partielle Modelle genannt; s. Balzer et al. 1987, 57). Man kann leicht zeigen, dass jedes empirische Modell von R(T) zu einem vollen Modell (D,e1,…, em, t1, …, tn) von T erweiterbar ist (hier sind die „ti“ die Extensionen der theoretischen Begriffe „si“; s. Ketland 2004, 293, Th. 4). (Hinweis: dies bedeutet noch nicht, dass R(T) mit E(T) logisch äquivalent ist, denn R(T) ist ein Satz 2. Stufe und E(T) enthält Sätze 1. Stufe. Wenn es einen zum Ramsey-Satz R(T) L-äquivalenten empirischen Satz 1. Stufe gibt, so nennt man die theoretischen Begriffe der Theorie Ramsey-eliminierbar; s. Sneed 1971, 53; Tuomela 1973, 60; u. a. m.) In realistischer Interpretation, die man z. B. bei Lewis (1970) und Papineau (1996) vorfindet, nimmt man an, dass die existenzquantifizierten Variablen reale theoretische Entitäten bezeichnen. Die Modelle des realistisch interpretierten Ramsey-Satzes sind nun nicht mehr simple empirische Modelle (D, e1, …, em). Erstens werden zum Individuenbereich der Theorie neue theoretische Individuen hinzugefügt, welche zum erweiterten Individuenbereich Dt führen. Zweitens korrespondiert nicht jede Teilmenge von Dt einer realen Eigenschaft. Im einfachsten Fall muss man also eine Menge Et von Extensionen von ,genuinen‘ theoretischen Eigenschaften annehmen, über welche die Variablen 2. Stufe laufen. Nun ist nicht mehr jedes empirische Modell des instrumentalistisch interpretierten Ramsey-Satzes zu einem Modell des realistisch interpretierten Ramsey-Satzes erweiterbar, denn die Quantoren 9Xi von R(T) können Erfüllungen in der Potenzmenge von De, aber keine Erfüllungen in Et besitzen. In philosophischen Worten: einem empirischen Modell, das den Ramsey-Satz instrumentalistisch erfüllt, ist nicht ablesbar, ob die jeweiligen theoretischen Entitäten, dessen Existenz von R(T) postuliert werden, bloß nützliche Fiktionen sind oder real existieren (s. dazu Schurz 2005a; Melia & Saatsi 2006). In realistischer Interpretation enthält R(T) mehr als nur den empirischen Gehalt einer Theorie. Er enthält vielmehr den gesamten synthetischen Gehalt einer Theorie. Denn wenn wir annehmen, dass die Bedeutung der theoretischen Begriffe der Theorie T(s1, …, sn) durch nichts anderes als durch diese Theorie selbst bestimmt wird, so scheint die Behauptung, die T über die Welt macht, genau die von R(T) zu sein: es gibt unbeobachtbare Entitäten X1, …, Xn, welche die Gesamtbehauptung der Theorie T(X1, …, Xn) erfüllen. Dies brachte Carnap (1963, 965) auf die Idee, den Ramsey-Satz einer Theorie durch folgenden analytischen Satz zu ergänzen, den ich den Carnap-Satz C(T) von T nenne: (5.8-3)

C(T): R(T)!T.

Die Bedeutungscharakterisierung der theoretischen Begriffe von T, welche C(T) liefert, besagt m. a. W. folgendes: das n-Tupel von T-Termen (s1, …, sn)

5.8 Weiterführende Themen

bezeichnet ein n-Tupel (X1, …, Xn) von Entitäten, welche die theoretische Behauptung T(X1, …, Xn) erfüllt, sofern es ein solches n-Tupel von Entitäten gibt. Damit ist die These, dass die Bedeutung von theoretischen Begriffen durch die Theorie selbst bestimmt wird, auf ihren logischen Begriff gebracht. Es ist offensichtlich, dass die Konjunktion von R(T) und C(T) mit T L-äquivalent ist, d. h. (5.8-4)

41 R(T)^C(T) $ T,

und man kann unschwer beweisen, dass C(T) keinen nichttautologischen empirischen Satz L-impliziert (Tuomela 1973, 59), d. h. (5.8-5)

E(C(T)) = E(Ø),

was dazu berechtigt, C(T) als analytisch anzusehen. Somit ist es Carnap gelungen, eine globale Auftrennung des Gehalts einer Theorie in einen synthetischen und einen analytischen Anteil vorzunehmen. Auf der Ebene der einzelnen Axiome und Theoreme von T ist eine analytisch-synthetisch-Auftrennung weiterhin unmöglich. Der Carnap-Satz C(T) liefert keine lokale Bedeutungscharakterisierung für die einzelnen si’s, sondern nur eine für alle si zusammen. Und er liefert lediglich eine partielle Bedeutungscharakterisierung der theoretischen Begriffe. Eine volle Bedeutungscharakterisierung im Sinne einer Explizitdefinition (Kap. 3.4) müsste die Bedingung erfüllen, dass in allen möglichen Modellen bzw. Welten die Extension des Definiendums eindeutig durch die Extension der Definiensbegriffe festgelegt ist. Der CarnapSatz legt die Extension der Begriffe si jedoch nur in jenen möglichen Welten fest, worin es genau ein n-Tupel von Entitäten (X1, …, Xn) gibt, welches T(X1, …, Xn) erfüllt. Gibt es mehrere solcher n-Tupel in einer Welt, so ist die Referenzbeziehung in dieser Welt mehrdeutig; und gibt es gar kein solches n-Tupel, so ist die Theorie T falsch und ihre Terme si sind denotationslos. Lewis (1970, 83 – 85) hat vorgeschlagen, den Carnapschen Ansatz zu verstärken, indem der Theorie unterstellt wird, sie würde implizit postulieren, dass die Referenz ihrer theoretischen Begriffe in der realen Welt eindeutig bestimmt sei. Durch diese Annahme gelingt es Lewis, den analytischen Gehalt einer Theorie folgende lokale ,Definitionen‘ mithilfe definiter Deskriptionen der theoretischen Begriffe darzustellen. Seine Definitionen lauten in Worten einfach (s. 1970, 87 f.): der theoretische Begriff si bezeichnet das i.te Glied des eindeutigen n-Tupels von Entitäten, welches in unserer Welt die Behauptung T(X1, …, Xn) der Theorie erfüllt. Die These von Lewis, dass wissenschaftliche Theorien mit Existenz- und Eindeutigkeitsbehauptungen für die Referenz ihrer theoretischen Begriffe einhergehen, ist allerdings selbst dann zweifelhaft, wenn man den Ramsey-Lewis-Satz realistisch interpretiert (interpretiert man diesen Satz instrumentalistisch, so ist diese These unhaltbar; s. auch Papineau 1996, 6, Fn. 5). Davon abgesehen sind ,Definitionen per definiter Deskription‘ jedenfalls keine vollwertigen, sondern nur partielle Definitionen, weil diese Definitionen die Extension der Terme si nur in solchen möglichen Modellen bzw. Welten bestimmen, in denen die ihnen zugrundeliegende Existenz- und Eindeutigkeitsannahme erfüllt ist.

Globale analytischsynthetischAuftrennung

Die Lewis-Definitionen theoretischer Begriffe

215

216

5. Theorien und ihre empirische Bewertung

Auflösung des semantischen Zirkularitätsproblems

Theorienübergreifende Relevanz

Die Ramsey-Carnap-Methode löst das semantische Zirkularitätsproblem. Fodor (1987, 73 – 94) und andere Gegner des semantischen Theorienholismus haben die Befürchtung geäußert, die Bestimmung der Bedeutung theoretischer Begriffe wäre zirkulär. Denn einerseits ist ja, gemäß der Maxime semantischer Kompositionalität, die Bedeutung von T(s1, …, sn) durch die Bedeutung der theoretischen Begriffe si (1in) (nebst der Bedeutung der anderen Begriffe in T) bestimmt. Andererseits folgt aus dem semantischen Theorienholismus, dass die Bedeutung der theoretischen Begriffe durch die Bedeutung der Theorie bestimmt ist. Und das sei ein Zirkel. Diese scheinbare Zirkularität wird durch die Ramsey-Carnap-Analyse vollständig zum Verschwinden gebracht. Denn der Ramsey-Satz R(T) lässt sich verstehen, ohne eine unabhängige Bedeutungskenntnis der theoretischen Begriffe vorauszusetzen, und der Carnap-Satz bzw. die Lewis-Definitionen fügen hinzu, die Bedeutung der theoretischen Begriffe (s1, …, sn) liegt darin, jene Entitäten (X1, …, Xn) zu bezeichnen, welche die Behauptung der Theorie T(X1, …, Xn) erfüllen. Mit der Ramsey-Analyse löst sich auch das von Kuhn (1967) aufgeworfene semantische Inkommensurabilitätsproblem, welches besagt, dass die theoretischen Begriffe verschiedener konkurrierender Theorien, auch wenn sie wortgleich sind, semantisch unvergleichbar seien. Vorausgesetzt aber, dass sich die Vertreter zweier rivalisierender Theorien T1, T2 zumindest die empirischen bzw. vortheoretischen Begriffe teilen, können beide Theorienvertreter den Ramsey-Satz der jeweils anderen Theorie in ihrer gemeinsamen Sprache formulieren, verstehen und empirisch überprüfen (s. Papineau 1996, 9 f.). Es gibt jedoch einen Grund, warum Theorien in der wissenschaftlichen Praxis schlecht durch ihren Ramsey-Satz ersetzbar sind, und dieser Grund hängt mit der in Kap. 5.4.1 erläuterten empirischen Kreativität von Theorien zusammen: wenn T(s) und H(s) sich einen theoretischen Begriff s teilen, dann ist R(T^H) logisch stärker als R(T)^R(H), denn die letztere Konjunktion impliziert nicht, dass in T und in H über dieselbe Entität existenzquantifiziert wird. Wenn man also Theorien durch ihre Ramsey-Sätze ersetzt, so muss man dies immer für die gesamte Theorieversion tun. Man darf die Ramsey-Methode nicht auf konjunktive Teile von Theorien anwenden, sonst käme die Interaktion zwischen den theoretischen Begriffen in den einzelnen Theorieteilen nicht zustande. Kutschera (1972, 304) spricht in diesem ZusammenhangvondertheorieübergreifendenRelevanztheoretischer Begriffe.

5.8.3 Allgemeine Theorien der Bestätigung In Kap. 4 und 5.6 haben wir spezifische Überprüfungsmethoden für spezifische Hypothesenarten entwickelt. In der Wissenschaftstheorie wurden auch allgemeine Bestätigungstheorien entwickelt (s. z. B. Lenzen 1974; Kutschera 1972, Kap. 5; Kuipers 2000, Teil I). Das Problem dieser Bestätigungstheorien besteht meist darin, dass sie zu unspezifisch sind. Als notwendige Bedingungen für gewisse Bestätigungsarten sind sie jedoch brauchbar. 5.8.3.1 Hypothetisch-deduktive Bestätigung. Der Grundidee des hypothetischen Deduktivismus (kurz: H-D) zufolge werden Hypothesen bzw. Theo-

5.8 Weiterführende Themen

rien durch ihre zutreffenden (empirischen) Konsequenzen bestätigt. Der Begriff der H-D-Bestätigung wurde von mehreren Autoren in (naiver) Anwendung des klassisch-logischen Folgerungsbegriffs wie folgt präzisiert (s. z. B. Lenzen 1974, 25 – 30; Glymour 1981, 35; Stegmüller 1971, 32; Kuipers 2000, § 2.1.1): (5.8-9) (H-D-Best): Evidenz E bestätigt eine Hypothese H, kurz Best(H,E), g.d.w. (a) H 41 E, (b) nicht 41 :H, (c) nicht 41 E, und (d) E ist wahr (semantische Fassung) bzw. als wahr akzeptiert (epistemische Fassung). (,X 41 Y‘ steht für ,Y folgt logisch aus X‘, und ,41 Y‘ für ,Y ist L-wahr‘.) Zusatz: Eine Relativierung auf Hintergrundannahmen S ist möglich, indem „logische Folge“ (X 41 Y) durch „Folge aus S“ (X, S 41 Y) ersetzt wird (dabei kann X leer sein); wir sagen dann: E bestätigt H relativ zu S. Beispiel: (Fa!Ga) bestätigt 8x(Fx!Gx). Ga bestätigt 8x(Fx!Gx) relativ zu Fa. Wie in Kap. 3.10.3 erläutert, stellte sich in der Folgediskussion heraus, dass dieser Bestätigungsbegriff zu ,Paradoxien‘ führte, die überwiegend mit irrelevanten Deduktionen zu tun haben. In Schurz (1991, 1994) wird vorgeschlagen, die Probleme des H-D-Bestätigungsbegriffs durch die Relevanzforderung an die Prämissen und Konklusion der Folgerungsrelation zu lösen. Um die Invarianz der Bestätigungsrelation gegenüber L-äquivalenter Umformung ihrer Argumente zu gewährleisten, wird eine irreduzible Repräsentation von Prämissen und Konklusion der Folgerungsrelation im Sinne von Def. 4.1-2 angenommen. Jene Paradoxien des H-D-Bestätigungsbegriffs, die mit Irrelevanz und Redundanz zu tun haben, werden durch den so verstärkten Begriff der relevanten H-D-Bestätigung gelöst (ein ähnlicher Ansatz wurde von Gemes 1993 entwickelt). Auf die Bestätigung probabilistischer Hypothesen ist der relevante H-DBegriff nicht anwendbar, da diese keine logischen, sondern nur probabilistische Konsequenzen besitzen. Für die Bestätigung strikter Hypothesensysteme gibt der Begriff der relevanten H-D-Bestätigung zwar eine interessante notwendige Bedingung ab, aber noch keine hinreichende Bedingung. Beispielsweise kann man von einer Bestätigung der empirischen Gesetzeshypothese 8x(Ax!Kx) nur sprechen, wenn die Evidenz eine repräsentative AStichprobe (und für die Relevanz eine A-Kontrollstichprobe) umfasst – wir haben diese feineren Bedingungen in Kap. 4 ausgeführt. Auch für die Bestätigung von Theorien genügt nicht die bloße relevante Herleitbarkeit einiger empirischer Phänomene – entsprechend feinere Bestätigungsbegriffe für Theorien haben wir in Kap. 5.6.2 entwickelt. 5.8.3.2 Subjektiv-probabilistische Bestätigung. Vertreter des Bayesianismus und der subjektiven Wahrscheinlichkeitstheorie haben probabilistische Bestätigungsbegriffe entwickelt, die noch allgemeiner sind als der H-D-Begriff, und vor ähnlichen Problemen stehen. Der Grundidee zufolge wird eine Hypothese H durch eine Evidenz E inkrementell bestätigt, wenn E die subjektive Wahrscheinlichkeit von H gegenüber der Ausgangswahrscheinlichkeit erhöht. Dies wird wie folgt definiert (s. Carnap 1950b, 463; Kutschera 1972, 427; Kuipers 2000, § 2.1.2; Howson/Urbach 1996, 117 ff.):

Hypothetischdeduktive (H-D) Bestätigung

Relevante H-D Bestätigung

217

218

5. Theorien und ihre empirische Bewertung

(5.8-10)

Bestätigung als Wahrscheinlichkeitserhöhung

Probleme des subjektiv-probabilistischen Bestätigungsbegriffs

(W-Best): E bestätigt H, kurz Best(H,E), g.d.w. w(H|E) > w(H). Daraus folgt (s. Kap. 3.10-1): Best(H,E) $ w(E|H) > w(E).

D. h., der inkrementelle Bestätigungsbegriff (W-Best) hängt nur vom Likelihood von E gegeben H und von der Ausgangswahrscheinlichkeit von E ab. Zusatz: Eine Relativierung auf Hintergrundannahmen S ist möglich, indem w auf S konditionalisiert wird. Dieser Bestätigungsbegriff ist u. a. folgenden drei Problemen ausgesetzt: Problem 1: Es gibt im Grunde nur zwei Fälle, in denen w(E|H) ohne subjektive Willkür bestimmt ist. Erstens der Fall, wo es sich bei H um eine statistische Hypothese handelt, und wo w(E(a)|H) mit dem statistischen Likelihood p(E(x):H) identifiziert wird. Dieser Fall wurde in Kap. 4.5.2 abgehandelt. Zweitens der Fall, wo es sich bei H um eine Theorie handelt, die E logisch impliziert: dann gilt nämlich H 41 E und somit w(E|H) = 1, und das Vorliegen einer Bestätigungsrelation Best(H,E) reduziert sich auf die (triviale) Bedingung w(E) < 1. Problem 2: Für den Fall H 41 E fällt der Bestätigungsbegriff (W-Best) nachweislich mit dem klassischen HD-Bestätigungsbegriff (H-D-Best) zusammen, sofern angenommen wird, dass w(E) < 1 und w(H) > 0 gilt (die Regularitätsbedingung; s. Carnap/Jeffrey 1971, 111; Earman 1992, 41; Kutschera 1972, 72). Diese Tatsache wird von Bayesianern oft als Erfolg angesehen (Earman 1992, 54; Howson/Urbach 1996, 119 ff.; Kuipers 2000, § 2.1.2). Angesichts der oben erläuterten Paradoxien des klassischen H-D-Begriffs ist dieser Erfolg sehr fraglich: genau dieselben Paradoxien ergeben sich nämlich auch für den subjektiven Bestätigungsbegriff. Auch für den probabilistischen Bestätigungsbegriff ist also eine Lösung des Relevanzproblems nötig. Problem 3: Es wird gelegentlich behauptet, das Faktum, dass jede aus H folgende nicht L-wahre Evidenz E die Hypothese H im Sinne von (W-Best) bestätigt, würde zeigen, dass bereits die Grundaxiome der Wahrscheinlichkeit induktive Konsequenzen besitzen. Dies ist unrichtig. Eine subjektive Wahrscheinlichkeitsfunktion w ist induktiv, wenn die Beobachtung von verifizierenden Instanzen Ha einer Hypothese 8xHx die Wahrscheinlichkeit dafür erhöht, dass H auf andere unbeobachtete Individuen b zutrifft: w(Hb|Ha) > w(Hb) (s. Kap. 4.5.2). Der Bestätigungseffekt von (W-Best) kommt im Fall H 41 E auch für nichtinduktive Wahrscheinlichkeitsfunktionen zustande, für die w(Hb|Ha) = w(Hb) gilt, und zwar aus dem folgenden trivialen Grund: wenn H 41 E gilt, dann bestätigt E einen Teilgehalt von H, nämlich E selbst (s. Schurz 2005b, § 4). Von einer genuinen probabilistischen Bestätigung von H durch E würde ich dagegen nur sprechen, wenn E auch die Wahrscheinlichkeit jener Hypotheseninstanzen erhöht, die noch nicht beobachtet wurden – m. a. W., wenn w induktiv ist (s. hierzu Schurz 2013a). 5.8.3.3 Die Goodman-Paradoxie. Goodman zeigte, dass die Anwendung induktiver Schlüsse auf beliebige Prädikate rational unmöglich ist, weil sie in logische Widersprüche führt. Seine berühmte Definition (1955/75, 97 ff.) ist die des Prädikates „grot“ (G*x).

5.8 Weiterführende Themen

Gegeben eine Stichprobe {ai:1in} von vor to beobachteten grünen Smaragden, so sind alle diese Smaragde auch grot. Genauer gesagt sind die Behauptungen Sai^Baito^Gai und Sai^Baito^G*ai definitorisch äquivalent. Wenden wir den induktiven Verallgemeinerungsschluss sowohl für „grün“ wie für „grot“ an, so ergibt unser Sample die beiden Allhypothesen H := „Alle Smaragde sind grün“ und H* := „Alle Smaragde sind grot“. H und H* implizieren aber für alle nicht vor to untersuchten Smaragde widersprüchliche Prognosen (grün versus rot). Zwischen der Goodman-Paradoxie und subjektiv-induktiven Vertauschbarkeitsannahmen (s. Kap. 4.5.2) besteht folgender Zusammenhang: für reguläre (undogmatische) Wahrscheinlichkeitsfunktionen kann die Vertauschbarkeitsannahme nicht zugleich für ein Prädikat (Gx) und sein Goodmansches pathologisches Gegenstück (G*x) Geltung besitzen (s. Kutschera (1972, 139 ff.). Für die Lösung der Goodman-Paradoxie stellt sich die Frage, nach welchen rationalen Kriterien wir entscheiden, welche Prädikate wir als vertauschbar bzw. als induktiv projezierbar ansehen (Goodman 1955/ 75). Viele Kriterien für die Induzierbarkeit wurden vorgeschlagen und erwiesen sich bei näherer Betrachtung als untauglich (s. Kutschera 1972, 145 – 156). Ein von Carnap (1947, 146; 1976, 211) stammender Vorschlag besagt, dass nur die qualitativen Prädikate induzierbar sind – das sind solche, die keinen definitorischen Bezug auf Individuenkonstanten enthalten. „Grot“ ist dagegen ein sogenanntes positionales Prädikat, weil es in seiner Definition auf den Zeitpunkt t0 Bezug nimmt. Dagegen hatte Goodman (1955/75, 105) das folgende Problem der Sprachabhängigkeit ins Feld geführt: durch wechselseitige Umdefinitionen kann man von unserer gewöhnlichen Sprache, in der „grün“ und „rot“ (Gx, Rx) als (undefinierte und insofern qualitative) Grundbegriffe fungieren, zu einer in ihrer Ausdrucksstärke äquivalenten Sprache übergehen, in der „grot“ und „rün“ (G*x, R*x) als Grundprädikate fungieren: (5.8-12) Sprachabhängigkeit der Goodman-Paradoxie: Sprache L (Gx, Rx primitiv) Definitionen in L: G*x : $ ((Bxto!Gx)^(:Bxto!Rx)). R*x : $ ((Bxto!Rx)^(:Bxto!Gx)).

Sprache L* (G*x, R*x primitiv) Definitionen in L*: Gx : $((Bxto!G*x) ^ (:Bxto!R*x)). Rx : $((Bxto!R*x)^(:Bxto!G*x)).

Viele Autoren haben aus dem Problem der Sprachabhängigkeit stark erkenntnisskeptische Konsequenzen gezogen – Goodman (1978, 138) behauptet: „Choosing ,green‘ rather than ,grue‘ as projectible … may seem like raindancing.“ In Schurz (1997b, § 4) wird vorgeschlagen, zwischen qualitativen und positionalen Beobachtungsbegriffen in sprachunabhängiger Weise durch Bezug auf ihre ostensive Erlernbarkeit zu unterscheiden:

Goodmans ,pathologisches Prädikat

,

(5.8.-11) (Def-Grot): Gegeben ein konstanter in der Zukunft liegender Zeitpunkt to, sagen wir das Jahr 3000, so heiße ein Gegenstand x grot (G*) g.d.w. x grün ist, falls er vor to beobachtet wurde (Bxto), und andernfalls rot ist. In Formeln: G*x: $ ((Bxto ! Gx) ^ (:Bxto !Rx)).

Goodmans Rätsel der Induktion

Qualitative Prädikate nach Carnap

Das Problem der Sprachabhängigkeit

219

220

5. Theorien und ihre empirische Bewertung

die ostensive Erlernbarkeit von qualitativen Prädikaten hängt nicht von irgendwelchen positionalen Informationen ab, die von positionalen Prädikaten dagegen schon. Damit ist noch nicht die Frage beantwortet, warum sich Induktionen auf qualitative und nicht auf positionale Prädikate stützen sollen. Induktion besteht genau darin, Merkmalsmuster, die bisher konstant beobachtet wurden, in derselben konstanten Weise in die Zukunft (bzw. auf andere Bereiche hin) zu verlängern. Um Induktionsregeln sinnvoll formulieren zu können, müssen wir wissen, was in den Beobachtungen konstant blieb. Dies hängt natürlich von den qualitativen, und letztlich von den primitiv-qualitativen Merkmalen ab. Positionale Merkmale sind in dieser Hinsicht Pseudomerkmale: dass Individuen ,konstant‘ das positionale Merkmal „grot“ besitzen, heißt, dass diese Individuen zur Zeit to ihre beobachtete Farbe von grün nach rot verändern. Wenn wir also „grot“ von Instanzen vor to auf Instanzen nach to übertragen, dann haben wir dabei Antiinduktion und nicht Induktion betrieben. Aus diesem Grunde ist es für die Formulierung von Induktionsregeln unerlässlich, sie mithilfe von Grundprädikaten zu formulieren, die qualitative Merkmale bezeichnen.

5.9 Zusammenfassung, einführende Literatur und Übungen 5.9.1 Zusammenfassung (Auswahl). Zuordnungs- bzw. Indikatorgesetze verbinden theoretische mit empirischen Begriffen. Weil theoretische Merkmale die Ursachen vieler empirischer Dispositionen sind, gibt es für sie (unbegrenzbar) viele Zuordungsgesetze, die zusammen empirischen Gehalt erzeugen. Eine Auftrennung einer Theorie in analytische und synthetische Bestandteile auf der Ebene ihrer Axiome ist daher unmöglich. Die Bedeutung von theoretischen Begriffen wird durch die gesamte dafür zuständige Hintergrundtheorie bestimmt. Mit diesem semantischen Holismus hängt auch die empirische Kreativität von Theorien sowie der Duhem-Quinesche Überprüfungsholismus zusammen. Konflikte mit der Erfahrung werden gemäß der Lakatos-These solange als möglich durch ad-hoc-Modifikationen der Peripherie einer Theorie zu bereinigen versucht, bevor der Theoriekern angezweifelt oder verändert wird. Dieses theoriendynamische Vorgehen wird anhand von Theorienbeispielen der Physik ebenso wie der Psychologie und Soziologie im Detail nachvollzogen. Während wissenschaftstheoretische Instrumentalisten die zentrale Funktion von theoretischen Begriffen in einer möglichst ökonomischen Systematisierung der empirischen Fakten sehen, glauben Realisten daran, dass theoretische Begriffe reale unbeobachtbare Merkmale bezeichnen. Ein zentrales Argument zugunsten des wissenschaftstheoretischen Realismus ist das Wunderargument, demzufolge es so unwahrscheinlich wäre wie ein Wunder, wenn eine Theorie nachhaltigen Voraussageerfolg besäße, obwohl ihren theoretischen Begriffen in der Wirklichkeit nichts entspricht.

5.9 Zusammenfassung, einführende Literatur und Übungen

5.9.2 Einführende Literatur. Allgemeines zur Struktur wissenschaftlicher Theorien finden Sie in Hempel (1974), Stegmüller (1970) und Kutschera (1972). Eine gute Einführung ist Ladyman (2002), und eine umfassende Textsammlung ist Curd/Cover (1998, Hg.). Ein Klassiker für Theoriendynamik und Theorienbewertung ist Lakatos (1974). Eine wichtige Textsammlung zu Theorienfortschritt und Realismus ist Papineau (1997, Hg.). Information zu Ramsey- und Carnap-Sätzen findet sich in Tuomela (1973) und Carnap (1976). Goodman stellt seine Bestätigungsparadoxie in (1955/75) dar.

5.9.3 Fragen und Aufgaben Zu Kap. 5.1: 1) Weshalb kann man theoretische Begriffe nicht empirisch definieren? 2) Geben Sie Zuordnungsgesetze (Indikatorgesetze) für folgende theoretische Begriffe: (a) „Person x ist ängstlich“, (b) „x ist metallisch“. Zeigen Sie, welcher empirische Gehalt durch Kombination von Zuordnungsgesetzen folgt. 3) Im World Value Survey Projekt wird u. a. die Lebenszufriedenheit von Menschen in verschiedenen Ländern untersucht. Dabei werden u. a. folgende Indikatoren benutzt: (a) Würden Sie sagen, dass Sie alles-in-allem glücklich sind? (b) Wie würden Sie ihren Gesundheitszustand (oder: ihren Wohlstand) einschätzen? Diskutieren Sie die beiden methodischen Indikatorregeln anhand dieses Beispiels. Zu Kap. 5.2: 1) Erläutern Sie anhand der Prognose einer Planetenbahn durch die Newtonsche Physik den Holismus der Theorienüberprüfung. 2) Inwiefern enthält eine Systembedingung eine exklusive CP-Klausel? 3) Was ist eine ad-hoc-Hypothese? Geben Sie ein Beispiel. Zu Kap. 5.3 – 5.4: 1) Was sind die strukturellen Bestandteile, und was die methodischen Merkmale guter wissenschaftlicher Theorien? Erläutern Sie diese anhand eines Beispiels. 2) Welches sind die wichtigsten Beziehungen zwischen den Theorieelementen von Theorienverbänden und von Theoriennetzen? Zu Kap. 5.5 – 5.7: 1) Unter welcher Bedingung ist nach Lakatos die Einführung einer ad-hoc-Hypothese in eine Theorienversion erlaubt? 2) Erläutern Sie den Holismus der Theorienüberprüfung anhand der Piagetschen kognitiven Entwicklungstheorie, oder anhand der Adorno-Milgramschen Theorie des autoritären Charakters. 3) Geben Sie ein Beispiel einer ad-hoc Hypothese, um den Kern der Piagetschen Theorie gegenüber der Modus-Tollens-Anomalie zu retten. 4) Gemäß einem bekannten Experiment von Asch handeln Menschen unter starkem sozialen Konformitätsdruck oft gegen ihre Überzeugung. Versuchen Sie damit, eine alternative Erklärung für das Milgram-Experiment zu liefern. 5) Was ist ein Misserfolg einer Theorie – welche Arten von Misserfolgen gibt es? 6) Eine Theorieversion ist umso bewährter (bzw. geschwächter), je …? Ergänzen Sie. 7) Ein Theoriekern ist umso bewährter (bzw. geschwächter), je … ? Ergänzen Sie.

221

222

5. Theorien und ihre empirische Bewertung

Zu Kap. 5.8: 1) Erläutern Sie die Positionen des wissenschaftstheoretischen Instrumentalismus und Realismus. 2) Was besagt das Wunderargument? Warum muss es auf Bestätigungen durch qualitativ neue Voraussagen eingeschränkt werden? 3) Warum lässt sich einer Theorie T mit dem synthetischen Gehalt von T, und der Carnap-Satz mit dem analytischen Gehalt von T identifizieren? 4) Welchen Problemen sind die klassischen Definitionen der (i) hypothetisch-deduktiven Bestätigung, und (ii) der subjektiv-probabilistischen Bestätigung ausgesetzt? 5) Definieren Sie anhand der Gesetzeshypothese „alle Schwäne sind weiß“ die Goodmanschen Prädikate „wün“ und „greiß“ und erläutern Sie das Sprachabhängigkeitsproblem, indem Sie „weiß“ und „grün“ durch „wün“ und „greiß“ definieren.

6. Erklärung – und was alles dazu gehört 6.1 Das deduktiv-nomologische Modell der Erklärung Wie in Kap. 2.3 ausgeführt, werden empirische Sachverhalte erklärt bzw. vorausgesagt, indem sie aus generellen Hypothesen zusammen mit singulären Sätzen abgeleitet werden. Hempel hat den Erklärungsbegriff logisch zu explizieren versucht. Die Schwierigkeiten, denen sein Versuch ausgesetzt war, lösten eine breite wissenschaftstheoretische Debatte aus (Übersichten s. Stegmüller 1969, Schurz 1983, Salmon 1989, Schurz 1990, Hg.). Hempel (1942) und Hempel/Oppenheim (1948) haben das folgende Modell der deduktiv-nomologischen (DN) Erklärung entwickelt: (Ms. 6.1-1) (DNEE) Eine DN-Erklärung eines Ereignisses bzw. einer singulären Tatsache E besteht in einem deduktiven Argument der Form „G1, …, Gn, A1, …, Am / E“, sodass gilt: (1) Formbedingung: (i) G1, …, Gn ist eine nichtleere Menge von strikt-generellen Sätzen, (ii) A1, …, Am (das Antecedens) ist eine nichtleere Menge von singulären Sätzen, und (iii) E ist ein singulärer Satz. Die gesamte Prämissenmenge heißt Explanans („das Erklärende“) und die Konklusion E das Explanandum („das zu Erklärende“). (2) Folgerungsbedingung: E ist deduktive Konsequenz von G1, …, Gn, A1, …, Am. (3a) Wahrheitsbedingung – semantische Modellversion: Die Explanansprämissen sind wahr; ergo ist auch das Explanandum wahr. Die Erklärung heißt dann wahr. (3b) Akzeptanzbedingung – epistemische Modellversion: Die Explanansprämissen sind im gegebenen epistemischen Hintergrundsystem W empirisch bestätigt, und das Explanandum ist durch Explanans-unabhängige Evidenz gesichert. Die Erklärung heißt dann gut bestätigt. Einfaches Beispiel:

G: A:

Alle Metalle leiten Strom Diese Vase ist metallisch

8x(Mx!Sx) Ma

E:

Deshalb leitet sie Strom

Sa

In der semantischen Modellversion wird die Wahrheit der Explanansprämissen verlangt. Natürlich sind die erklärenden Gesetzesprämissen niemals definitiv verifizierbar ist. Praktisch wichtiger ist daher die epistemische Modellversion. Sie ist auf ein epistemisches Hintergrundsystem (oder Hintergrundwissen) W bezogen, worunter alle zu einem gewissen Zeitpunkt von einer Person oder scientific community rational akzeptierten Sätze

Das Hempel-Oppenheim-Modell der DN-Erklärung

Epistemisches versus semantisches Erklärungsmodell

224

6. Erklärung

Covering Law

Beschreibung und Erklärung

bzw. Propositionen zu verstehen sind. Sowohl das Explanans wie das Explanandum einer gut bestätigten DN-Erklärung müssen Bestandteile von W sein. Neben dem semantischen und epistemischen Erklärungsbegriff führt Hempel (1965, 273, 338) auch den Begriff der potentiellen Erklärung ein: hier wird statt Bedingung (3a) bzw. (3b) die bloße logische Konsistenz der Explanansprämissen verlangt. Der Begriff der potentiellen Erklärung ist wichtig für die Bewertung von bislang unbestätigten Hypothesen aufgrund ihrer Erklärungskraft, gemäß dem Schluss auf die beste Erklärung von Kap. 2.6.3. Das einfachste Beispiel einer DN-Erklärung hat die Form 8x(Ax!Ex), Aa / Ea. Weil hierbei Antecedens und Explanandum durch ein einzelnes Gesetz implikativ verbunden sind, sprach Dray (1957) vom covering law Modell der Erklärung – ein Ausdruck, der sich später einbürgerte. Hempel (1965, 355) wies Drays Etikettierung zurück, da sein DN-Modell auch wesentlich komplexere Erklärungen einschließt, in denen das Draysche covering law nur indirekt aus der Gesetzesmenge mathematisch-deduktiv folgt. Ein Beispiel ist die DN-Erklärung einer Planetenposition aus Anfangsbedingungen, Systembedingungen und den Naturgesetzen (N1 – 3) der Newtonschen Mechanik gemäß Kap. 5.2. Hempel betont, dass in alltäglichen Erklärungen „Ea weil Aa“ die Gesetzesprämissen oft weggelassen werden, aber implizit angenommen werden müssen. Wie der Titel „The Function of General Laws in History“ verrät, möchte Hempel (1942) entgegen dem methodendualistischen Programm (Kap. 1.2.5.6) aufzeigen, dass auch die Geistes- und speziell die Geschichtswissenschaften auf allgemeine Gesetzeshypothesen angewiesen sind, sobald sie etwas zu erklären bzw. zu verstehen suchen. Um diese These zu untermauern, gibt Hempel mehrere Beispiele von DN-Erklärungen in den Geschichts- und Sozialwissenschaften (1942, 235 f.), und er gesteht bereits in (1942, 237) zu, was ihm seine Kritiker später vorwerfen sollten, dass es nämlich in diesen Disziplinen kaum strikte Gesetzesbeziehungen gibt – statt dessen gäbe es ,weiche‘ Gesetzesbeziehungen, die man als statistische Hypothesen auffassen könnte. Das probabilistische Gegenstück zu seinem deduktionslogischen Erklärungsmodell hat Hempel jedoch erst 20 Jahre später entwickelt (s. Kap. 6.3.1). Historisch bedeutsam ist auch, dass das DN-Modell Beschreibung und Erklärung in keinen grundsätzlichen Gegensatz stellt. Idealistische Philosophien (wie z. B. Hegel) hatten Erklärung als etwas prinzipiell über Beschreibung Hinausgehendes, als eine Art Wesensschau aufgefasst. Solche metaphysischen Erklärungskonzeptionen waren es, die Duhem (1908, 20 f.), Wittgenstein (1921, 6.371) und andere zu der Behauptung veranlassten, die Wissenschaft könne grundsätzlich nur beschreiben und nichts (im Sinne einer Wesensschau) erklären. Hier wirkte Hempels antimetaphysisches Modell wie ein Befreiungsschlag, insofern Erklärungen darin in keiner prinzipiellen Weise über Beschreibungen hinausgehen. Nicht nur Explanandum und Antecedens, sondern auch die Gesetze sind dem DN-Modell zufolge Beschreibungen – eben generelle Beschreibungen. Worin Erklärungen über Beschreibungen hinausgehen, das ist dem DN-Modell zufolge die Herstellung eines logischen Zusammenhangs.

6.2 Erklärung versus Voraussage und Begründung

Aus demselben Grund gibt es in der Wissenschaft keine absoluten Letzterklärungen: jede Erklärung muss irgend etwas Unerklärtes voraussetzen. Jede noch so lange Aneinanderreihung von Erklärungen muss einen Anfang haben, an dem gewisse nicht weiter erklärte Prämissen angenommen werden – in der Physik etwa das Gravitationsgesetz oder die Tatsache, dass es einen Urknall gab. Man kann diese These auch so ausdrücken: Selbsterklärungen, so wie wir sie z. B. in Theologie und Metaphysik finden (Beispiel: „Warum gibt es Gott? Gott trägt die Ursache seiner selbst in sich!“) sind wissenschaftlich inakzeptable Pseudoerklärungen. Fast wichtiger als Ereigniserklärungen im Sinne von (DNEE) ist die wissenschaftliche Erklärung von Gesetzen durch übergeordnete Theorien, im Sinne der 2. Ebene von Kap. 2.3 (s. Nagel 1961, 79 ff.; Friedman 1990; Alston 1971, 14; u. a. m.). Hempel intendierte, im DN-Erklärungsbegriff auch Gesetzeserklärungen zu berücksichtigen, und der einzige Grund, warum er dies nicht explizit tat, waren Schwierigkeiten, die mit Irrelevanz und Redundanz im Sinne von Kap. 3.10.3 zu tun haben (Hempel/Oppenheim 1948, 273, Fn. 33). Im Sinne von Hempels Intention sei das analoge DN-Modell der Gesetzeserklärung (DNGE) wie folgt dargestellt: (Ms. 6.1-2) (DNGE): Eine DN-Erklärung eines Gesetzes G besteht in einem deduktiven Argument der Form „T [ A / G“, sodass gilt (,[‘ für ,Mengenvereinigung‘): (1) Formbedingung: (i) T ist eine nichtleere Menge von Gesetzen oder Axiomen von Theorien, von denen alle essentiell quantifiziert und einige essentiell generell sind, (ii) A (das Antecedens) ist eine eventuell leere Menge von singulären Sätzen oder lokalisierten Existenzsätzen; (iii) G ist ein essentiell genereller Satz. (2) Folgerungsbedingung, (3a) Wahrheitsbedingung bzw. (3b) Akzeptanzbedingung lauten wie im Modell (DNEE) der DN-Ereigniserklärung. Beispiel: die theoretische Erklärung von Planentenbahnen in Kap. 5.2, oder die theoretische Erklärung des Piagetschen Entwicklungsgesetzes von Kap. 5.5.

Keine Letzterklärungen

Gesetzeserklärungen

Die in den 1950er Jahren einsetzende Erklärungsdebatte führte zu vielen unterschiedlichen Problemfeldern, die im folgenden knapp dargestellt sind.

6.2 Erklärung versus Voraussage und Begründung Allgemein betrachtet ist eine Erklärung eine Antwort auf eine Warum-Frage (Hempel 1965, 354; Bromberger 1965; van Fraassen 1980, 134). Schon Hempel (1965, 354 f.) sah jedoch klar, dass zwischen zwei Arten von Warum-Fragen unterschieden werden muss – er nannte sie erklärungssuchende (explanation-seeking) versus gründesuchende (reason-seeking) WarumFragen. Erstere fragen danach, aufgrund welcher Ursachen bzw. Realgründe das fragliche Ereignis eintrat. Letztere fragen dagegen nach Glaubensgrün-

Realgründe versus Glaubensgründe

225

226

6. Erklärung

Strukturgleichheit von Erklärung und Voraussage?

DN-Begründung

den, also danach, warum es vernünftig ist zu glauben, dass das fragliche Ereignis eintrat bzw. eintreten wird (s. auch Stegmüller 1969, 172, 198). Realgründe sind Ursachen in einem weiten (nicht auf zeitlich gerichtete Kausalprozess eingeengten) Sinn; Glaubensgründe sind dagegen Rechtfertigungsgründe. Die Unterteilung ist keinesfalls disjunkt: im Gegenteil, in den meisten Fällen liefert uns die Kenntnis der Realgründe eines Ereignisses auch Glaubensgründe, und umgekehrt. Aber ist dies immer so? Dies war in der Tat die Auffassung des frühen und mittleren Hempel, der die These der Strukturgleichheit von Voraussage und Erklärung vertrat (Hempel 1942, 234 f.; 1965, 367; Stegmüller 1969, 153 ff.). Popper (1935/76, 31) hatte dieselbe Strukturgleichheitsthese vertreten und sie so formuliert: Kausalität = Prognosededuktion. Der Strukturgleichheitsthese zufolge unterscheiden sich Erklärung und Voraussage nur in den pragmatischen Zeitumständen des Bekanntwerdens von Prämissen und Konklusion. Eine Erklärung liegt vor, wenn das Explanandum zuerst beobachtet bzw. bekannt wurde, und die Explanansprämissen erst hinterher dazu gewonnen wurden. Eine Voraussage liegt dagegen vor, wenn uns zuerst nur die Explanansprämissen bekannt waren, und das Explanandum daraus durch Deduktion erschlossen wurde. Man beachte, dass es sich bei solchen Voraussagen um Voraussagen im epistemischen Sinn handelt, die nicht unbedingt Voraussagen im Zukunftssinn sein müssen. Auch sogenannte Retrodiktionen, also hypothetische Rückschlüsse auf historisch vergangene Ereignisse, sind ,Voraussagen‘ in diesem epistemischen Sinn. Um dem Rechnung zu tragen, hat Stegmüller (1969, 760; 1983, 976) eine differenziertere Terminologie entwickelt, die wir übernehmen. Ein DN-Argument, dessen Antecedens Glaubensgründe für das Explanandum liefert, nennen wir eine DN-Begründung. Sind die Prämissen des DNArgumentes zuerst bekannt und wird die Konklusion daraus nachträglich abgeleitet, so sprechen wir von einer ex-ante-Begründung. Eine ex-ante-Begründung ist eine Voraussage (im zeitlichen Sinn), wenn das Antecedensereignis zeitlich vor dem Explanandumereignis eintritt; sie ist eine Retrodiktion, wenn das Antecedensereignis erst danach eintritt. Wenn andererseits die Konklusion des Argumentes bereits bekannt war, bevor die begründenden Prämissen gefunden waren, dann sprechen wir von einer ex-post-Begründung. Wenn schließlich ein DN-Argument ex-post-Charakter hat und sein Antecedens Realgründe für das Explanandum liefert, dann endlich sprechen wir von einer DN-Erklärung. Die Strukturgleichheitsthese trifft offenbar auf das Verhältnis von ex-ante- und ex-post-Begründung zu. Hempel behauptete aber die Strukturgleichheit von ex-ante-Begründung und Erklärung. Trifft diese These zu? Hempel (1965, 367) untergliedert seine These in zwei Teilthesen: Teilthese 1: Jede Voraussage (ex-ante-Begründung) ist eine potentielle Erklärung. Teilthese 2: Jede Erklärung ist eine potentielle Voraussage (ex-ante-Begründung). Eine umfangreiche Kontroverse hat gezeigt, dass beide Teilthesen unhaltbar sind.

6.2 Erklärung versus Voraussage und Begründung

6.2.1 Nicht jeder Glaubensgrund ist ein Realgrund (Einwände gegen Teilthese 1). Gemäß Kap. 4.3 gibt es zwei Hauptfälle von pseudokausalen Gesetzesbeziehungen, die sowohl im probabilistischen wie im hier besprochenen strikt-deterministischen Fall auftreten: 1.) Pseudokausalität durch gemeinsame Ursachen. In DN-Erklärungen mit zeitlichen Sukzessionsgesetzen geht das Antecedensereignis A eines DN-Argumentes dem Explanandumereignis E zeitlich voraus. Dies garantiert noch nicht, dass A eine Ursache für E ist – A und E können auch Wirkungen einer gemeinsamen Ursache im Sinne von Kap. 4.3.1 sein. In diesem Fall kommt A nur als DN-Indikator, aber nicht als DN-Ursache für E in Frage. Beispiel: Das plötzliche Fallen des Barometerstandes ist ein sicherer Voraussagegrund für einen herannahenden Sturm, aber nicht seine kausale Erklärung (Grünbaum 1972, 30; Hempel 1965, 374 f.). 2.) Gerichtetheit (Asymmetrie) der Ursache-Wirkungs-Beziehung. Zeitliche Sukzessionsgesetze „wenn A, dann wird E der Fall sein“ sind L-äquivalent mit ihrer Kontraposition „wenn non-E, dann war non-A der Fall“. Aber non-E kann keine Ursache von non-A sein, denn Kausalbeziehungen zwischen zeitlich getrennten Ereignissen sind immer vorwärtsgerichtet. Beispiel: Dass Herr Corsi (auf den ein Mafia-Anschlag geplant ist) heute vor dem Kiosk steht, ist ein DN-Glaubensgrund dafür, dass er gestern nicht erschossen wurde, aber kein Realgrund. In zeitlichen Koexistenzgesetzen (8x,t: Axt$ Ext) werden zwei Merkmalsparameter A und E zum selben Zeitpunkt t korreliert. Auch hier liegen gerichtete Ursache-Wirkungs-Beziehungen in einem allgemeineren (nicht zeitlich-prozessualen) Sinn vor (Schurz 2001a, Kap. 3). Obwohl die strikte Regelmäßigkeit in beide Richtungen besteht, ist nur eine Richtung kausaler Natur. Beispiel: Bei konstantem Sonneneinstrahlungswinkel ist die Höhe eines Fahnenmastes h proportional zu seiner Schattenlänge (Bromberger 1966, 92; Hempel 1965, 352 f.). Die Höhe des Mastes ist die Ursache seiner Schattenlänge, aber nicht umgekehrt. Daher kann nur diese Implikationsrichtung für eine Erklärung verwendet werden. Die Implikationsrichtung von der Schattenlänge auf die Höhe liefert nur einen Voraussagegrund für die Höhe des Mastes. 6.2.2 Kausalität und Gesetzesartigkeit im Erklärungsrahmen. Um DN-Erklärungen von bloßen DN-Begründungen abzugrenzen, benötigt man eine Theorie der Kausalität. Eine solche Theorie muss auch eine Theorie der Gesetzesartigkeit mit umfassen. Denn von einer kausalen Erklärung kann man nur dann sprechen, wenn das Antecedens- und das Explanandumereignis durch einen naturgesetzlichen Zusammenhang verbunden sind (Hempel/ Oppenheim 1948, 66). So würde man das folgende DN-Argument „G: Alle Äpfel in diesem Sack sind rot, A: Dieser Apfel stammt aus diesem Sack / E: Daher ist dieser Apfel rot“ nicht als Erklärung dafür ansehen, warum dieser Apfel rot ist. Bei G handelt es sich um einen akzidentellen Allsatz im Sinne von Kap. 3.6 – dass sich dieser Apfel in diesem Sack befindet, ist akzidentell und hat ursächlich nichts mit seiner roten Farbe zu tun. Andererseits ist für Voraussagen bzw. Begründungen Gesetzesartigkeit nicht erforderlich: um vorauszusagen, dass der Apfel, den ich aus dem Sack nehme, rot sein

Nichtkausale DN-Voraussagen

Brombergers Fahnenmastbeispiel

227

228

6. Erklärung

Erklärung und Kausalität

wird, genügt es völlig, dass der Allsatz „alle Äpfel in diesem Korb sind rot“ induktiv hinreichend abgesichert ist. Gesetzesartigkeit und Kausalität sind höchst schwierige Probleme. Die Mehrheit der vorgeschlagenen Erklärungsmodelle versuchte zunächst, den Erklärungsbegriff unabhängig von Kausalitätsfragen zu explizieren. Bromberger (1965, 73), Stegmüller (1969, 760) und andere Autoren stellten daher kritisch fest, dass die vorgeschlagenen Erklärungsmodelle eigentlich nicht den Erklärungsbegriff im engeren Sinn explizierten (diese Kritik trifft auch auf Stegmüllers spätere ,Kehrtwendung‘ in 1983, 633 ff., zu). Wie der spätere Hempel (1977, Nachwort) halten wir daran fest, dass das Liefern von Realgründen zum Bedeutungskern des Begriffs der Ereigniserklärung gehört. Auf DN-Gesetzeserklärungen durch übergeordnete Theorien lässt sich die Kausalitätsforderung jedoch nicht oder nur indirekt anwenden. Ein Gesetz ist kein raumzeitlich lokalisierter Sachverhalt und kann daher kein direkter Gegenstand einer Kausalbeziehung sein. Es wäre auch zu stark, würde man von jeder Gesetzeserklärung durch Theorien verlangen, dass sie Kausalmechanismen liefern müsse: viele Gesetzeserklärungen höherer Wissenschaften, wie z. B. evolutionstheoretische Erklärungen, liefern keine Kausalmechanismen. Van Fraassen (1980, 124 – 127) hebt hervor, dass es auch im Bereich der Physik zahlreiche nichtkausale Erklärungen gibt, z. B. Erklärungen aufgrund von Symmetrieprinzipien, oder Erklärungen in der Quantenmechanik. Gemäß van Fraassen (1980, 124) ist der Begriff des Kausalprozesses theorieabhängig. Dem schließen wir uns an. In Schurz (1990a, 277) wird vorgeschlagen, die Kausalitätsbedingung durch Bezug auf das maximal vollständige kausale Modell M(A,E|W) über die Natur der Beziehung zwischen dem Antecedensereignis A und dem Explanandumereignis E zu explizieren, welches im gegebenen epistemischen Hintergrundsystem W verfügbar ist. M(A,E|W) besteht aus drei Klassen von Aussagen: erstens eine Liste aller in W bekannten möglicherweise kausal relevanten Ereignisse in der räumlichen Umgebung und zeitlichen Vorgeschichte von A und E, zweitens alle Informationen über die strikten oder statistischen Zusammenhänge dieser Ereignisse, sowohl untereinander wie mit A und E, und drittens schließlich alle in W akzeptierten theoretischen Prinzipien über die Natur kausaler Prozesse. Nur aufgrund eines solchen Kausalmodells lässt sich beurteilen, ob A als Ursache für E angesehen werden kann. In M(A,E|W) darf es z. B. keine Ereignisse geben, die A von E im Sinne von Kap. 4.3.1 abschirmen, und gemäß den in M(A,E|W) akzeptierten Kausalprinzipien muss zumindest ein nicht allzu unwahrscheinlicher Kausalprozess von A nach E führen, wenn dessen Details auch nicht bekannt sind. – Zusammengefasst: (Ms. 6.2-1) In einer DN-Ereigniserklärung müssen (i) die generellen Prämissen gesetzesartig sein, und (ii) die Konjunktion A von Antecedensprämissen muss eine im epistemischen Hintergrundsystem W akzeptable Ursache für E sein, d. h.: aufgrund von M(A,E|W) führt ein plausibler Kausalprozess von A nach E.

6.2 Erklärung versus Voraussage und Begründung

6.2.3 Die Bedingung der prognostischen Funktion (Einwände gegen Teilthese 2). Viele Erklärungstheoretiker haben Teilthese 2, derzufolge jede Erklärung eine potentielle Vorausage sein müsse, für probabilistische Erklärungen zurückgewiesen (dazu Kap. 6.3.3). Aber auch für DN-Erklärungen hatte Scriven (1959a, 468 – 469) folgenden gewichtigen Einwand vorgebracht: oft kommt es vor, dass wir einen Effekt E mit einem Realgrund A erklären, aber unser einziger Glaubensgrund dafür, dass die Ursache A eintrat, liegt darin, dass wir den Effekt E beobachtet haben. In einer solchen Situation kann A niemals als Voraussagegrund oder Glaubensgrund für E fungieren. Hempel (1965, 372) nannte solche Erklärungen selbstbestätigende Erklärungen. So erklären wir beispielsweise die Rotverschiebung im Spektrum der Sterne (E) mithilfe der Expansion des Universums (A), aber der einzige Grund, warum wir die Hypothese der Expansion des Universums für bestätigt halten, ist die spektroskopische Beobachtung der Rotverschiebung. Um derartige Bestätigungszirkel zu vermeiden, ist von einer DN-Begründung bzw. potentiellen DN-Voraussage folgendes zu fordern:

Selbstbestätigende Erklärungen Hempels Rotverschiebungsbeispiel

(Ms. 6.2-2) Eine DN-Begründung muss im gegebenen Hintergrundsystem W prognostische Funktion besitzen, d. h.: es muss in W eine Menge Best(A) von Evidenzen geben, welche A bestätigen, ohne E logisch zu implizieren. Auf diese Weise wird in Schurz (1982, 328 f.; 1983, 265 ff.) die Bedingung der prognostischen Funktion expliziert, welche für DN-Begründungen zu fordern ist. Im Beispiel der Rotverschiebung ist die Bedingung der prognostischen Funktion verletzt. Für DN-Erklärungen ist diese Bedingung jedoch nicht zu fordern – hier genügt es, dass A Ursachen für E liefert, unabhängig von der Frage, wie das Wissen um A in W bestätigt wurde. Selbstbestätigende DN-Erklärungen ohne prognostische Funktion treten gehäuft im Bereich der Erklärung instabiler oder chaotischer Prozesse auf (s. Schurz 1996b). Beispielsweise lässt sich nicht genau voraussagen, wann eine Lawine abgehen wird. Ist sie aber heute mittag um halbzwei Uhr abgegangen, so lässt sich das selbstredend damit erklären, dass genau zu diesem Zeitpunkt die erwärmungsbedingte Verminderung der Reibungskraft zwischen verschiedenen Schneeschichten so groß wurde, dass die obere Schneeschicht ins Rutschen geriet. Eine perfekte DN-Ereigniserklärung – aber eine strukturgleiche DN-Voraussage erzielen zu wollen, wäre aussichtslos. 6.2.4 Irrelevanz und Redundanz – die logischen Probleme der DN-Erklärung. In (Hempel/Oppenheim 1948, 273 ff.) erkannte Hempel, das die Folgerungsbedingung (2) seines DN-Modells (Ms. 6.1-1) zu schwach war, um triviale oder unsinnige DN-Argumente, sogenannte Pseudoerklärungen, auszuschließen. In der sich daran anschließenden DN-Erklärungsdebatte wurde eine Vielfalt von solchen Beispielen von Pseudoerklärungen (bzw. -begründungen) entdeckt und durch stärkere Bedingungen zu eliminieren versucht (Übersicht s. Stegmüller 1969, Kap. X; 1983, Kap. XI.1; Schurz

Instablität, Chaos, und Nichtvoraussagbarkeit

229

230

6. Erklärung Irrelevanz und Redundanz

1983, 225 – 254). Nicht alle, aber viele dieser Beispiele hatten mit Irrelevanz und Redundanz im Sinne von Kap. 3.10.3 zu tun. Durch Einsatz der Begriffe der relevanten Deduktion und der irreduziblen Repräsentation von Kap. 3.10.3 und 4.1.1 können wir diese Probleme wie folgt bereinigen (näheres s. Schurz 1982, 1983, Kap. IV): (Ms. 6.2-3) Es muss eine irreduzible Repräsentation I(G,A) der Explanansprämissen und eine irreduzible Repräsentation I(E) der Konklusion des DN-Argumentes geben, sodass für jedes elementare Konjunktionsglied Ei von I(E) das DN-Argument „I(G,A) / Ei“ relevante Prämissen und eine relevante Konklusion besitzt. Darüber hinaus muss jede Gesetzesprämisse die Relevanzbedingung von Kap. 4.1.1 erfüllen. Die Relevanzbedingung (Ms. 6.2-3) ist für DN-Erklärungen wie DN-Begründungen zu fordern. Salmon (1984, 92 f.) hat darauf aufmerksam gemacht, dass Relevanzbedingungen für Erklärungen versus Begründungen einen unterschiedlichen Stellenwert besitzen. Im Begründungs- oder Voraussagefall ist der Defekt einer redundanten Prämisse harmlos: er verhindert nicht die Voraussagekraft des DN-Argumentes. Im Begründungsfall ist die Relevanzfrage also eine Frage der kognitiven Effizienz. Im Erklärungsfall handelt es sich bei einer redundanten Erklärung tatsächlich um eine falsche Erklärung, da vom Antecedens der Erklärung postuliert wird, dass es die Ursachen für das Explanandum anführt, und ein deduktiv redundantes Antecedenskonjunkt kann keine kausale Relevanz besitzen. Aufgrund des Gesagten können die Modelle der DN-Ereignisbegründung (DNB), Gesetzesbegründung (DNGB), Ereigniserklärung (DNEE) und Gesetzeserklärung (DNGE) wie folgt verbessert werden: Für (DNEB) und (DNEE) gelten die Bedingungen von (Ms. 6.1-1, 1 und 3); für (DNGB) und (DNGE) die Bedingungen von (Ms. 6.1-2, 1 und 3). Zu allen vier Modellen gesellt sich Bedingung (Ms. 6.2-3) hinzu. Zu (DNEB) und (DNGB) addiert sich Bedingung (Ms. 6.2-2), zu (DNGE) addiert sich Bedingung (Ms. 6.2-1, i), und zu (DNEE) die Bedingung (Ms. 6.2-1, i+ii).

6.3 Probabilistische Erklärungsmodelle 6.3.1 Induktiv-statistische Erklärungen nach Hempel. Das induktiv-statistische (IS) Modell der Erklärung wurde von Hempel in (1965, 381 ff.) entwickelt, in (1968) verbessert, und von Niiniluoto (1981, 441 f.) wie folgt verallgemeinert: Hempels Modell der induktiv-statistischen Erklärung

(Ms. 6.3-1) (ISE) Eine induktiv-statistische Erklärung ist ein Quasi-Argument der Form „G1, …, Gn, A //r E“, für das gilt: (1) Formbedingung: G1, …, Gn ist eine Menge von gesetzesartigen statistischen Sätzen, A und E sind Singulärsätze der Form A(a) und E(a), respektive.

6.3 Probabilistische Erklärungsmodelle

(2) Folgerungsbedingung: {G1, …, Gn} impliziert wahrscheinlichkeitstheoretisch ein statistisches Minimalgesetz der Form p(E(x)|A(x)) = r, sodass r nahe bei 1 liegt. Daraus folgt die induktive Stützungsbedingung:Für alle subjektive Wahrscheinlichkeitsfunktionen, die das statistische principal principle von Kap. 3.10.4.3 erfüllen, beträgt die subjektive Wahrscheinlichkeit des Explanandums, gegeben das Explanans, gleich r: w(E(a) | A(a)^G1^…^Gn) = r. (3) Akzeptanzbedingung: Die Explanansprämissen sind im gegebenen epistemischen Hintergrundsystem W empirisch bestätigt, und das Explanandum ist durch Explanans-unabhängige Evidenz gesichert. (4) Maximale Bestimmtheit: Das Antecedens A ist im Hintergrundsystem W in Bezug auf E maximal bestimmt. Beispiel (Hempel 1968, 177): G: 95% von mit Plasmodium Vivax infizierten Personen erkranken an Malaria, A: Jones hat sich mit Plasmodium Vivax infiziert. [95%] E: Jones ist an Malaria erkrankt. Die von Niiniluoto vorgenommene Verallgemeinerung besteht darin, dass in Hempels IS-Modell die Gesetzesmenge G auf das Minimalgesetz p(E(x)|A(x)) = r beschränkt wird, wogegen in (1) nur die wahrscheinlichkeitstheoretische Herleitbarkeit des Minimalgesetzes aus der Gesetzesmenge verlangt wird. Zudem hatte Hempel die induktive Stützungsbedingung als unabhängige Bedingung gefordert; sie folgt aber für jedes subjektive Wahrscheinlichkeitsmaß, welche das statistische principal principle erfüllt. Formbedingung 1), Folgerungsbedingung 2) und Akzeptanzbedingung 3) sind das jeweilige Gegenstück zum DN-Erklärungsbegriff (Ms. 6.1-1, DNEE). Die Bedingung 4) der maximalen Bestimmtheit ist dagegen völlig neu: sie hängt mit der Nichtmonotonie von IS-Argumenten zusammen und ist der Grund dafür, warum IS-Erklärungen nur Quasi-Argumente sind. Man beachte: die wahrscheinlichkeitstheoretische Folgerung in (2) ist monoton; nur die induktiv-statistische Folgerung „G1, …, Gn, A //r E“ ist nichtmonoton bzgl. A. Die Unterscheidung zwischen Erklärungen und Voraussagen bzw. expost-Begründungen, die wir anhand des DN-Erklärungsbegriffs erläutert haben, trifft in gleicher Weise auf IS-Argumente zu. Im Sinne von Bedingung (Ms. 6.2-1) ist ein IS-Argument nur dann eine IS-Erklärung, wenn aufgrund des maximal vollständigen Kausalmodells M(A,E|W) A ein Ursachenfaktor für E ist. Da Hempel vom Wahrscheinlichkeitswert r fordert, nahe bei 1 zu liegen, kommen IS-Erklärungen auch als Voraussagen bzw. ex-post-Begründungen in Frage, allerdings nur dann, wenn die Bedingung (Ms. 6.2-2) der prognostischen Funktion erfüllt ist. An die statistischen Gesetze eines ISArgumentes ist ferner die statistische Relevanzbedingung von Kap. 4.1.2 zu stellen. Zudem wird für die wahrscheinlichkeitstheoretische Folgerungsrelation in (2) eine logische Relevanzbedingung an Prämissen und Konklusion

Niiniluotos Verallgemeinerung

231

232

6. Erklärung

des Argumentes benötigt, welche analog ist zur Bedingung (Ms. 6.2-3) von Kap. 6.2. Im folgenden nehmen wir an, dass der Begriff der IS-Erklärung (ISE) und der IS-Begründung (ISB) in diesen Hinsichten verbessert wurde und wenden uns nun jenen Fragen zu, die spezifisch sind für probabilistische Erklärungen: (i) die Bedingung der maximalen Bestimmtheit, sowie (ii) die Frage der erforderlichen Höhe des Wahrscheinlichkeitswertes r.

Maximale Bestimmtheit nach Hempel

6.3.2 Die Bedingung der maximalen Bestimmtheit. In einem IS-Argument wird die statistische Wahrscheinlichkeit des Minimalgesetzes als subjektive Erwartungswahrscheinlichkeit auf den Einzelfall des Explanandums übertragen. Gemäß Kap. 3.9 muss hierzu die Reichenbachsche Bedingung der engsten Referenzklasse erfüllt sein. Hempels Bedingung der maximalen Bestimmtheit von (1968) ist eine schwach relevanz-verstärkte Version dieser Bedingung, die sich in äquivalenter Umformulierung so ausdrücken lässt. Wie nennen ein Gesetz „p(Ex|Ax) = r“ einen Erklärungskandidat für Ea im Hintergrundsystem W, wenn (a) Aa in W bekannt ist, und (b) A ein nomologisches (gesetzesartiges) und von E logisch unabhängiges Prädikat ist. (Ms. 6.3-2) (MB) Aa ist maximal bestimmt für Ea im epistemischen Hintergrundsystem W g.d.w. (MB1) Aa in W ist und es gibt in W ein statistisches Gesetz p(Ex|Ax) = r, sodass für jedes Gesetz p(Ex|A*x) = r* in W, für das A*a und 8x(A*x ! Ax) in W sind (d. h. A*x ist in W mindestens so bestimmt wie Ax), gilt: r = q; und (MB2) für jeden alternativen Erklärungskandidaten A**a in W, der ebenfalls Bedingung (MB1) erfüllt, muss aus W folgen, dass p(Ex|A**x) = p(Ex|Ax) gilt. Bedingung (MB1) besagt m. a. W., dass Aa die gesamte in W über das Individuum a verfügbare Information enthält, die für Ea statistisch relevant ist (vgl. die Beispiele in Kap. 4.4). Diese Bedingung entspricht der Anwendungsregel nichtmonotoner Schlüsse (Ms. 2.6-4) für IS-Argumente. Deshalb sind IS-Argumente eben nur Quasi-Argumente (was insbesondere von Salmon 1989, § 3.6, betont wurde). Ein Beispiel: das obige IS-Argument, demzufolge Jones deshalb an Malaria erkrankt ist (Ea), weil er mit Plasmodium Vivax infiziert wurde (Aa), und 95 % aller so Infizierten an Malaria erkranken, ist nicht mehr akzeptierbar, wenn wir erfahren, dass Jones zusätzlich eine genetische Konstellation besitzt (Ha), die in 95 % aller Fälle gegen Malaria immun macht (p(Ex|Ax^Hx) = 5 %). Das Antecedens A*a = Aa^Ha ist bestimmter als Aa. Statt über eine Kausalerklärung zu verfügen, müssen wir nun sagen, dass Jones‘ Malariaerkrankung einem tragischen Zufall zu verdanken war – er bekam Malaria, obwohl er die Genkonstellation H besaß. Die Forderung, dass nur nomologische (gesetzesartige) Antecedensprädikate als Erklärungskandidaten in Betracht kommen, trägt der Tatsache Rechnung, dass die MB1-Bedingung durch extensional definierte Prädikate wie

6.3 Probabilistische Erklärungsmodelle

z. B. „Qx :$ Hx _ x=a“ unterlaufen werden kann. In Erweiterung des Hempelschen Vorschlages (1968, 124, 127) verstehen wir unter einem nomologischen Prädikat Hx hier ein raumzeitlich universelles Prädikat im Sinne von Kap. 6.5.1.3 Bedingung (MB2) besagt schließlich, dass man in jenen Fällen, wo in W zwei rivalisierende maximal bestimmte Antecedenzien für dasselbe Explanandum ohne Bestimmtheitspräferenz vorliegen, skeptisch reagieren sollte, d. h. weder das eine noch das andere IS-Argument als Erklärung bzw. als Voraussage akzeptieren sollte. Ein Beispiel: Angenommen Mary lebt in England (A1), was es wahrscheinlich macht, dass Mary protestantisch ist (E), und zugleich ist ihr Vater Italiener (A2), was es wahrscheinlich macht, dass Mary katholisch ist (K). Wir wissen nichts über die Wahrscheinlichkeit von E bzw. K gegeben A1^A2. In einem solchen Fall sollten wir uns einer Erklärung oder Prognose besser enthalten. Das IS-Modell der Erklärung ist ebenso wie das DN-Modell epistemisch relativ, d. h., bezogen auf ein epistemisches Hintergrundsystem W. Auch die Bedingung (3b) des Hempelschen DN-Erklärungsbegriffs, derzufolge die Explanansprämissen in W gut bestätigt sein müssen, war epistemisch relativ. Coffa (1974) bezeichnete diese W-Abhängigkeit des Hempelschen DN-Modells als epistemische Relativität im Bestätigungssinn. Er argumentierte, diese Relativität sei harmlos, weil man durch Ersetzung der Bestätigungsbedingung durch die Wahrheitsbedingung ein semantisches Modell erhält. Die epistemische Relativität der maximalen Bestimmtheitsforderung ist aber nicht mehr in diesem Sinne harmlos. Coffa (1974, 151 – 153) argumentierte, dass man selbst im Fall der starken epistemischen Relativität, die in der MBBedingung steckt und die sich auf das gesamte Hintergrundsystem W erstreckt, zum semantischen Begriff einer wahren IS-Erklärung gelangen kann, und zwar dadurch, dass man das Hintergrundsystem W durch die Klasse aller wahren Aussagen in einer ideal-vollständigen Sprache ersetzt. Diesen Vorschlag griff Salmon in seinem semantischen Erklärungsmodell von (1984) auf, wenngleich er den expliziten Bezug auf eine Sprache vermeidet und statt von Prädikaten von Klassen bzw. Attributen spricht. Ersetzen wir im verbesserten IS-Erklärungsmodell (ISE) W durch die Menge aller wahren Sätze in einer idealen Sprache, so erhalten wir eine Annäherung an Salmons ,objektives‘ Erklärungsmodell – wir können hier mit Coffa von einer wahren IS-Erklärung sprechen. Das semantische Gegenstück der Hempelschen maximalen Bestimmtheitsforderung ist Salmons Bedingung, das Antecedensattribut müsse objektiv homogen sein in Bezug auf das Explanandumattribut (s. Schurz 1996a, § 6.2). Salmon verlangt darüber hinaus, A müsse die breiteste objektiv homogene Bezugsklasse für E sein – m. a. W., A darf nicht nur irrelevante Information weglassen (wie bei Hempel), sondern A muss alle irrelevanten Informationen weglassen. Wie Coffa (1974) scharf beobachtet, gilt folgendes: wäre die Welt deterministisch, dann wären die einzigen wahren IS-Erklärungen solche, in denen die statistische Wahrscheinlichkeit 1 beträgt. Denn die kausal vollständigen Antecedensinformationen müssten in einer deterministischen Welt das Ergebnis (Ex oder :Ex) immer determinieren. Genuine wahre IS-Erklärungen, so Coffa, kann es demnach nur in einer objektiv indeterministischen Welt geben.

Nomologische Prädikate

Epistemische Relativität von IS-Erklärungen

Wahre IS-Erklärungen nach Coffa Objektiv homogene Referenzklassen nach Salmon

233

234

6. Erklärung

Die LeibnizBedingung

Scrivens SyphilisParesis-Beispiel

Salmons Beispiel des Mendelschen Kreuzungsexperiments

6.3.3 Bedingungen an den Wahrscheinlichkeitswert: konfligierende Intuitionen. Auch nachdem die positive Relevanzbedingung p(Ex|Ax) > p(Ex) dem IS-Erklärungsmodell hinzugefügt wird, fragt sich noch: wie hoch muss der Wahrscheinlichkeitswert p(Ex|Ax) sein? In (1965) fordert Hempel, dieser Wert solle nahe bei 1 liegen – aber warum? Tuomela (1981, 276) und Stegmüller (1983, 972) haben argumentiert, die Minimalanforderung an den Wahrscheinlichkeitswert von p(Ex|Ax) müsse sein, dass er größer als 1/2 ist (die sogenannte ,Leibniz-Bedingung‘). Denn nur dann ist p(Ex|Ax) größer als p(:Ex|Ax), und nur in diesem Fall kann es in einem minimalen Sinne rational sein, Ea anstatt :Ea vorherzusagen. Für probabilistische Begründungen und Voraussagen ist die Leibniz-Bedingung zweifellos zutreffend, aber gilt sie auch für Erklärungen? Dann nicht, wenn man das Wesen einer probabilistischen Erklärung darin sieht, dass sie positiv relevante Kausalfaktoren zitiert. Beispielsweise erkranken nur 10 % aller Personen mit unbehandelter Syphilis (Sx) an progressiver Paralyse (Px) (Scriven 1959b). Aber Syphilis ist der einzige bekannte positive Kausalfaktor für progressive Paralyse: es gilt p(Px) = 0,01 < 0,1 = p(Px|Sx). Wenn also jemand an progressiver Paralyse erkrankt, so werden wir seine Erkrankung damit erklären, dass er über längere Zeit durch Syphilis infiziert war. Auf diese Weise motivierte Gärdenfors (1990, 110 ff.) die zentrale Bedingung seines probabilistischen Erklärungsmodells, welche besagt: die Wahrscheinlichkeit des Explanandums, gegeben das Explanans, muss gegenüber der Ausgangswahrscheinlichkeit lediglich erhöht sein. Damit verwandt ist die Forderung von van Fraassen (1990, 72 ff.), das Antecedens müsse das Explanandumereignis probabilistisch favorisieren. Der tiefste Intuitionskonflikt entsteht, wenn wir die bisherigen Bedingungen an die Wahrscheinlichkeit, welche immer etwas mit Erwartbarkeit bzw. Erwartbarkeitserhöhung des Explanandums zu tun hatten, mit der Wahrscheinlichkeitsbedingung des späteren Salmon vergleichen. Für Salmon (1971, 63; 1984) ist ein Antecedens Ax genau dann eine Erklärung für Ex, wenn es alle kausal relevanten Faktoren für Ex auflistet, egal ob diese die Wahrscheinlichkeit des Explanandums erhöhen oder erniedrigen. Zur Plausibilisierung dieser Bedingung diskutiert Salmon (1984, 109) das Beispiel eines Kreuzungsexperimentes von weißblütigen und rotblütigen Erbsenpflanzen. Das Ursachenwissen (Ax) besagt hierfür, dass die für Rotblütigkeit verantwortlichen Gene dominant sind gegenüber den für Weißblütigkeit verantwortlichen Genen. Daher werden sich unter den gekreuzten Erbsenpflanzen gemäß den Mendelschen Erbgesetzen 75 % rotblütige Erbsenpflanzen (Genotypen RR, RW, WR) und 25 % weißblütige Erbsenpflanzen (Genotyp WW) befinden. Die Ausgangswahrscheinlichkeit für rote bzw. weiße Blütenfarbe betrage 50 %. Wie Salmon nun argumentiert, ist unser kausales Situationsverständnis im Fall einer rotblütigen Erbsenpflanze genau dasselbe wie im Fall einer weißblütigen Erbsenpflanze – es gibt nichts, was wir im Falle der rotblütigen Erbsenpflanze verstehen, aber im Fall der weißblütigen Erbsenpflanze nicht verstehen. Wenn wir daher das IS-Argument „p(Rot(x)|Ax)=0,75, Aa //0,75 Rot(a)“ als Erklärung der Rotblütigkeit von a ansehen, so müssen wir auch das IS-Argument „p(Weiß(x)|Ax)=0,25, Ab //0,25 Weiß(b)“ als Erklärung der Weißblütigkeit von b ansehen, obwohl das Ante-

6.4 Normische Erklärungen und die Erklärung menschlicher Handlungen

cedens des letzteren Argumentes die Wahrscheinlichkeit des Explanandumereignisses senkt. Aus diesem Grund verlangt Salmon in seinem statistischen Relevanzmodell (SR-Modell) vom Antecedens Ax einer Erklärung nur, dass es positiv oder negativ relevant ist für Ex. Salmons Argument, dass eine Erklärung alle relevanten Kausalfaktoren auflisten muss, ist einleuchtend; auch der späte Hempel (1977, 99 f.) wurde davon überzeugt. Es scheint aber gegenintuitiv zu sein, zu sagen, ein Explanandum sei eingetreten, weil ein Faktor anwesend war, der dieses unwahrscheinlich macht (s. Cartwright 1979). Wir schließen uns dem Vorschlag von Humphreys (1989, 117) an und bezeichnen negativ relevante Antecedensfaktoren nicht als Ursachen, sondern als Gegenursachen – wir sagen in einem solchen Fall, Ex sei eingetreten, obwohl Ax eingetreten ist (für weiteren Diskussion s. Stegmüller 1973c, 281 ff.; Schurz 1990a, § I.5 – I.6; Strevens 2000). Insgesamt verbleibt für probabilistische Einzelfallerklärungen ein nur schwer lösbarer Intuitionskonflikt.

6.4 Normische Erklärungen und die Erklärung menschlicher Handlungen Die methodendualistische Schule (s. Kap. 1.2.5.6) hatte an Hempels DN-Erklärungsmodell kritisiert, dass geisteswissenschaftliche Erklärungen nicht nomothetisch, sondern idiographisch-verstehend verfahren, und daher keine covering laws benötigen. Der Philosoph der Geschichtswissenschaften Dray versuchte, dies anhand einer Fülle von Beispielen zu demonstrieren – bekannt ist sein Beispiel von Ludwig dem XIV (1957, 33): Historiker erklären dessen Unpopularität damit, dass Ludwig der XIV sein Land oft in Kriege verwickelte und dem Volk schwere Belastungen aufbürdete. Gemäß dem deduktionslogischen Modell müssten diese Historiker hierzu folgende streng allgemeine Gesetzeshypothese annehmen:

Historische Erklärungen

(6.4-1) Alle Herrscher, die ihr Land in Kriege verwickeln und dem Volk schwere Belastungen aufbürden, werden unpopulär. Jeder Historiker weiß aber, dass ein solches Gesetz nie ausnahmslos gültig sein kann – selbst dann nicht, wenn man seine Wenn-Bedingung durch weitere Konjunktionsglieder verstärkt (z. B. „und Minderheiten diskriminieren“, etc.). Ausnahmen von (6.4-1) sind (leider) gut bekannt. Hempels covering law These kann man allgemein so begründen: wann immer Historiker in historischen Erzählungen, oder Hermeneutiker in Handlungs- oder Textinterpretationen, das Wörtchen weil verwenden (und das tun sie oft), so berufen sie sich implizit auf einen generellen Zusammenhang (s. auch Scholz 2001). Diese generellen Zusammenhänge sind nicht strikter, sondern unsicherer Natur. Kann man sie alternativ durch numerisch-statistische Gesetze wiedergeben, so wie dies Hempel in seinem IS-Erklärungsmodell vorschlug? Ein Beispiel wäre:

Das „Weil“ verweist auf nomologische Zusammenhänge

235

236

6. Erklärung

(6.4-2) Die Wahrscheinlichkeit, dass ein Herrscher unpopulär wird, der sein Land in Kriege verwickelt, ist 84 %. Auch dieser Vorschlag ist praktisch kaum durchführbar, denn numerische Wahrscheinlichkeitswerte von historischen Ereignissen sind in den Geschichtswissenschaften im Regelfall unbekannt (vgl. Dray 1957, S. 51 ff.). Was Historiker in ihren Erklärungen dagegen im Kopf haben, ist Dray zufolge (1957, 31 ff.) folgende Normalfallhypothese oder normische Hypothese: (6.4-3) Herrscher bzw. Regierungen, die die-und-die Bedingungen erfüllen, werden normalerweise (üblicherweise, zumeist) unpopulär. Normische Gesetzeshypothesen

Rationale Handlungserklärung

In den 1950er Jahren dominierte die Meinung, dass normische Generalisierungen ebenfalls empirisch gehaltlos wären, weil sie durch kein Gegenbeispiel widerlegt werden können (s. Dray 1957, 132; Scriven 1959a, 466). Doch die auf Popper zurückgehende These der Identifikation von ,empirisch gehaltvoll‘ mit ,falsifizierbar‘ trifft auf normische und statistische Gesetzeshypothesen nicht zu (s. Kap. 3.8). Normische Gesetzeshypothesen implizieren statistische Majoritätshypothesen (s. Kap. 3.5) und lassen sich gemäß den Methoden von Kap. 4.2.3 empirisch überprüfen. Wir schlagen daher vor, das IS-Erklärungsmodell von Hempel im Bereich der Wissenschaften evolutionärer Systeme und speziell der Handlungswissenschaften zu einem normischen Erklärungsmodell zu erweitern, in dem die statistischen Gesetzesprämissen durch normische Gesetzeshypothesen ersetzt werden (s. Schurz 2004). Die maximale Bestimmtheitsforderung ist entsprechend zu modifizieren. Generell beruhen rationale Handlungserklärungen auf folgendem normischen Rationalitätsprinzip (vgl. Dray 1957, 132 – 137; Haussmann 1991, Scholz 2001; Rosenberg, 2008, Kap. 2 – 4, u. a. m.): (6.4-4) Personen handeln normalerweise (zweck)rational, d. h.: wenn Akteur a Ziel Z hat und glaubt, Handlung H sei ein geeignetes Mittel für die Erreichung von Z, dann wird a normalerweise H zu realisieren versuchen.

Hermeneutische Rationalitätspräsumption

Natürlich kommen viele menschliche Denk- und Handlungsprozesse teilweise nichtrational zustande: sie entspringen z. B. aus unbewussten Ursachen, usw. Es gibt aber ein traditionelles Prinzip der Hermeneutik, das eine prima-facie-Präferenz von rationalen vor nichtrationalen Erklärungen behauptet. Es besagt, dass man bei der Interpretation von (sprachlichen oder nichtsprachlichen) Handlungen prima facie, d. h. in Ermangelung triftiger Gegengründe, die Handlung zunächst rational zu erklären versuchen soll. Nur wenn dies hartnäckig scheitert, sollten nichtrationalen Erklärungen zu Hilfe genommen werden. Man spricht auch vom Prinzip des Wohlwollens (Bühler 2003, 16) oder von einer hermeneutischen Rationalitätspräsumption (Scholz 2001, Teil II).

6.5 Weiterführende Themen

6.5 Weiterführende Themen 6.5.1 Gesetzesartigkeit 6.5.1.1 Naturgesetze versus Systemgesetze. In Schurz (2002b, § 6.1; 2005c, § 2) wird vorgeschlagen, zwischen Naturgesetzen und Systemgesetzen zu unterscheiden. Naturgesetze nehmen auf keine spezifischen physikalischen Systeme Bezug, sondern drücken das aus, was für beliebige Systeme in allen physikalisch möglichen Universen gilt. Z. B. sind die Newtonschen Kernaxiome (N1 – 3) von Kap. 5.2 solche Naturgesetze. Erst indem in (N1) (Kraft = Masse mal Beschleunigung) Systembedingungen eingesetzt werden, welche die anwesenden Kräfte explizit auflisten, erhält man eine konkrete lösbare Differentialgleichung für einen bestimmen Systemtyp. Solche speziellen Differentialgleichungen sind theoretische Systemgesetze, und ihre Lösungen sind abgeleitete phänomenologische Systemgesetze. Für komplexe bzw. lebende Systeme werden die phänomenologischen Systemgesetze zumeist auf empirisch-induktivem Wege gewonnen, ohne theoretisch ableitbar zu sein. Es gibt nur wenige fundamentale Naturgesetze, und man findet sie nur in der Physik (s. auch Schiffer 1991). Auch die meisten Gesetze der Physik sind Systemgesetze (z. B. das Fallgesetz, das Pendelgesetz, das Planetengesetz, usw.). Systemgesetze sind nicht physikalisch notwendig, sondern involvieren physikalisch kontingente Systembedingungen. Alle ,höheren‘ Disziplinen, von der Biologie aufwärts, haben ihre spezifischen Systemgesetze. Diese sind zumeist normischer Natur, weil sie evolutionäre Systeme beschreiben (s. Kap. 3.5). 6.5.1.2 Gesetzesartigkeit im weiten Sinn und kontrafaktische Konditionalsätze. In Kap. 3.6 gelangten wir zum Resultat, dass die Gesetzesartigkeit von raumzeitlich beschränkten generellen Sätzen gradueller Natur ist. Wir sprechen hier von Gesetzesartigkeit im weiten Sinn. Nicht nur Naturgesetze, sondern auch alle Systemgesetze sind in diesem weiten Sinn gesetzesartig. Ein hilfreicher Indikator dafür, dass wir einen Allsatz 8x(Fx!Gx) als gesetzesartig i.w.S. ansehen, besteht darin, dass wir dem zugehörigen irrealen Konditionalsatz zustimmen würden: wäre a ein F, so wäre a auch ein G (s. Goodman 1955/75, Kap. 1). Dieser Indikator charakterisiert nicht nur Naturgesetze, sondern auch Systemgesetze oder raumzeitlich beschränkte Gesetze. Z. B. ist „alle Raben sind schwarz“ keine physikalische Notwendigkeit (s. Armstrong 1983, 18); dennoch stimmen wir dem kontrafaktischen Konditionalsatz „wäre dieser Vogel ein Rabe, dann wäre er schwarz“ zu. Dagegen würden wir von diesem grünen Apfel nicht sagen, befände er sich in diesem Korb, dann wäre er rot. Dies indiziert, dass wir den Allsatz „Alle Äpfel in diesem Korb sind rot“ als akzidentell ansehen. Lewis (1973b) und andere Autoren haben eine logische Semantik für kontrafaktische Konditionalsätze entwickelt, die auf angenommenen Ähnlichkeitsordnungen zwischen möglichen Welten beruhen. Für die wissenschaftstheoretische Analyse gibt diese logische Semantik leider wenig her, denn für die inhaltliche Interpretation der Ähnlichkeitsordnung zwischen möglichen Welten benötigt zumindest eine schon vorausgesetzte Unter-

Naturgesetze vs. Systemgesetze

Kontrafaktische Konditionalsätze

237

238

6. Erklärung

scheidung zwischen Gesetzen vs. kontingenten Fakten (s. Rescher 1964; Stegmüller 1969, 320 – 334; vgl. auch Lange 2009). Es bleibt dabei, dass Gesetzesartigkeit im weiten Sinn ein vager und gradueller Begriff ist.

Physikalische Notwendigkeit

Raumzeitliche Universalität

Starke und schwache Hume-These

6.5.1.3 Gesetzesartigkeit im engen Sinn und physikalische Notwendigkeit. Um der Vagheit und Gradualität des weiten Gesetzesartigkeitsbegriffs zu entgehen, suchten Wissenschaftstheoretiker nach einem Begriff von Gesetzesartigkeit im engeren Sinn von naturgesetzlicher bzw. physikalischer Notwendigkeit. Viele Autoren hegten die Hoffnung, dass sämtliche raumzeitlich unbeschränkten Gesetze in diesem engeren Sinn gesetzesartig wären. Diese Hoffnung erwies sich aber als unbegründet. Dies zeigte uns bereits das Beispiel (3.6-9): der Allsatz (bzw. negierte Existenzsatz) „Kein Klumpen aus Gold hat einen Durchmesser von mehr als 1 Kilometer“ ist raumzeitlich universell, und drückt dennoch keine physikalische Notwendigkeit aus. Raumzeitliche Universalität ist also keine hinreichende Bedingung für Gesetzesartigkeit i. e. S.; sie ist jedoch eine plausible notwendige Bedingung dafür. Dabei nennen wir ein Prädikat P(x1, …, xn) raumzeitlich universell g.d.w. für keine Instanzierung P(a1, …, an) der Wahrheitswert schon aus der raumzeitlichen Lokalisation der darin enthaltenen Individuenkonstanten a1, …, an analytisch folgt. Eine strikte oder nichtstrikte Generalisierung heißt raumzeitlich universell, wenn sie aus raumzeitlich universellen Prädikaten gebildet ist (ähnlich Earman 1978, Kap. 1). Beispielsweise folgt die Wahrheit der Instanzierung „wenn a ein Apfel in diesem Korb ist, dann ist a rot“ für alle Individuen a, die nicht in diesem Korb sind, bereits aus der raumzeitlichen Lokalisation von a und von diesem Korb. Daher ist der Allsatz „Alle Äpfel in diesem Korb sind rot“ nicht universell, auch dann nicht, wenn man ihn durch seine Kontraposition „Alle nichtroten Gegenstände sind keine Äpfel in diesem Korb“ ersetzt (Hempels Problem in 1965, 341). Die starke Hume-These, derzufolge Universalität eine hinreichende Bedingung für Gesetzesartigkeit i. e. S. ist, haben wir als unrichtig erkannt. Die schwache Hume-These, derzufolge Universalität lediglich eine notwendige Bedingung für Gesetzesartigkeit i. e. S. ist, trifft jedoch zu. Einige Autoren haben auch die schwache Hume-These bezweifelt (z. B. Bhaskar 1978, 54). Irreduzibel-singuläre Notwendigkeitsbehauptungen scheinen aber empirisch komplett unüberprüfbar zu sein. Angenommen eine Lawine geht ab und ein Magier behauptet kurz danach, er hätte dies durch seine Gedankenkraft bewirkt, aber dies wäre ihm nur in diesem einzigen Fall gelungen. Eine solche Behauptung bewegt sich an der Grenze der Sinnlosigkeit. Die schwache Hume-These scheint uns ein fundamentales Rationalitätsprinzip an wissenschaftliche Gesetzes- und Kausalhypothesen zu sein. Eine weitere von Carnap (1947) und Hempel (1965, 267) vorgeschlagene notwendige Bedingung für Gesetzesartigkeit i. e. S. ist die auf den Physiker James C. Maxwell zurückgehende Maxwell-Bedingung, derzufolge Naturgesetze bzw. nomologische Prädikate keinen analytischen Bezug auf bestimmte Individuen oder Raumzeitpunkte enthalten dürfen. Die MaxwellBedingung ist wesentlich stärker als die Universalitätsbedingung: beispielsweise ist das Goodman-Gesetz von Kap. 5.8.3.3 „Alle Smaragde sind grot, d. h. grün, falls vor dem Zeitpunkt to beobachtet, und andernfalls rot“ raum-

6.5 Weiterführende Themen

zeitlich universell, aber es erfüllt nicht die Maxwell-Bedingung. In ähnlicher Weise hat Armstrong (1983) die These vertreten, dass Naturgesetze Implikationsbeziehungen zwischen Universalien sind, was ebenfalls bedeutet, dass nomologische Implikationsbeziehungen nicht von einzelnen sie exemplifizierenden Individuen bzw. Raumzeitstellen abhängen. In Wilson (1979) und Schurz (1983, Kap. VI.2.3) wird herausgearbeitet, dass die Maxwell-Bedingung, richtig verstanden, ein physikalisches Symmetrieprinzip darstellt, demzufolge Naturgesetze invariant sein müssen unter Translation ihrer Zeitkoordinaten, und Translation bzw. Rotation ihrer Raumkoordinaten. Daraus lassen sich grundlegende physikalische Erhaltungssätze für Energie und Impuls gewinnen. Physikalische Symmetrieprinzipien sind jedoch nicht apriori, sondern erfahrungsabhängig (vgl. Earman 1986, Kap. VII). Davon abgesehen ist die Maxwell-Bedingung für Gesetzesartigkeit i. e. S. gleichzeitig zu schwach: dies zeigt erneut das Beispiel (3.6-9) „Kein Klumpen aus Gold hat einen Durchmesser von mehr als 1 Kilometer“, denn auch dieser Allsatz erfüllt die Maxwell-Bedingung. Es hilft wissenschaftstheoretisch auch wenig weiter, wenn man den Begriff der Naturnotwendigkeit formal durch eine mögliche-Welten-Semantik charakterisiert (z. B. von Wright 1974, 32; Kripke 1972, u. a. m.), welche eine zweistellige Relation der Erreichbarkeit zwischen möglichen Welten annimmt. Denn um diese Erreichbarkeitsrelation inhaltlich zu interpretieren, muss man bereits wissen, welche generellen Regelmäßigkeiten unserer Welt echten physikalischen Notwendigkeiten entsprechen, und welche nicht (ebenso van Fraassen 1989, 44 f.). Ein weiterer Vorschlag ist der auf Mill, Ramsey und Lewis zurückgehende Vereinheitlichungsansatz („best system approach“), demzufolge die im engen Sinn gesetzesartigen Generalisierungen all jene generellen Sätze sind, die aus jenen Theorien folgen, welche die beste Vereinheitlichung der Menge aller wahren Sätze erzeugen (s. Earman 1986, 88; Lewis 1973b, 73). Es bleibt dabei aber unklar, warum man den akzidentellen Allsatz (3.6-9) bzgl. der Größe von Goldklumpen nicht zu den vereinheitlichenden Theorien mit hinzunehmen sollte, denn auch daraus folgen viele wahre Singulärsätze, sodass dies zur Vereinheitlichung von W beitragen würde (van Fraassen 1989, 47). Was für eine wissenschaftstheoretisch adäquate Charakterisierung von Gesetzesartigkeit i. e. S. benötigt werden würde, wäre ein adäquat explizierter Begriff der physikalischen Möglichkeit, demzufolge man sagen könnte, dass ein Allsatz wie (3.6-9) in unserem Universum nur aus kontingenten Gründen wahr ist, denn es wäre physikalisch möglich, gegen ihn zu verstoßen. Das Problem dabei besteht darin, dass im physikalischen Theoriennetz eine Grenzziehung zwischen physikalisch notwendigen Naturgesetzen und mehr-oder-minder kontingenten Systemgesetzen nicht oder nicht durchgängig vorhanden ist.

6.5.2 Kausalität 6.5.2.1 Singuläre und generelle Kausalbeziehung. Kausalität ist prima facie eine Relation zwischen einzelnen Ereignissen bzw. Tatsachen. Gemäß der schwachen Hume-These von Kap. 6.5.1.3 müssen singuläre Kausalbeziehungen jedoch durch generelle Kausalbeziehungen gestützt werden. Ge-

Maxwell-Bedingung als physikalisches Symmetrieprinzip

Der Mill-RamseyLewis-Ansatz

239

240

6. Erklärung

mäß Kap. 4.3.1 und 6.2.2 muss die Kausalbeziehung auf die gegebenen Umstände bezogen werden, welche alle möglicherweise kausal relevanten Parameter zu umfassen haben, um sicherzustellen, dass die generelle Regelmäßigkeit auch eine Kausalbeziehung darstellt: Singuläre und generelle Kausalbeziehung

Kontrafaktische Kausalanalyse

(Def. 6.5-1) F(a) ist eine Ursache von G(a) g.d.w. gilt: (a) F(a) und G(a) waren der Fall, (b) die Umstände, welche alle anderen möglicherweise kausal relevanten Parameter umfassen, waren U(a), und (iii) F(x) ist in Umständen U(x) eine generelle Ursache von G(x). Die Umstände U(a) in (Def. 6.5-1) umfassen nur solche Ereignisse bzw. Zustände, welche von Fa und Ga analytisch unabhängig sind. Sie müssen sich ferner auf das gesamte Zeitintervall zwischen dem Eintreten der Ursache und der Wirkung beziehen und für dieses Zeitintervall ausschließen, dass gewisse Störfaktoren auftreten, welche den zum Wirkereignis führenden Kausalprozess verhindern. Die generelle Kausalbeziehung in (Def. 6.5-1) kann strikter oder probabilistischer Natur sein; wir lassen beide Fälle zu. Wir sprechen von einer und nicht von der Ursache, weil neben Fa noch andere Faktoren für Ga kausal relevant gewesen sein können. Gemäß (Def. 6.5-1) ist für die singuläre Kausalbeziehung nicht nur das Eintreten der Ursache, sondern auch das Eintreten der Wirkung erforderlich. Die Forderung des Eintretens der Wirkung ist im Fall strikter Kausalität redundant, aber im Fall probabilistischer Kausalität essentiell. Denn in indeterministischen Situationen folgt aus der Tatsache, dass F in Umständen U G generell verursacht, und dass Ereignis Fa unter Umständen Ua eingetreten ist, nicht zwingend, dass auch die Wirkung Ga eingetreten ist, sondern nur mit erhöhter Wahrscheinlichkeit. Auf diesen Punkt hat Eells (1991, 8 ff.) aufmerksam gemacht (er widerlegt damit Haussmans Prinzip „G“, 1998, 102 f.). Kontrafaktische Kausalanalysen versuchen, die singuläre Kausalbeziehung durch Rekurs auf Ähnlichkeitsordnungen zwischen möglichen Welten zu explizieren, ohne dabei generelle Kausalbeziehungen vorauszusetzen. Lewis (1973a) schlug vor, die Kausalbeziehung zwischen singulären Ereignissen so zu explizieren: ein Ereignis Fa verursachte ein anderes Ereignis Ga, wenn folgendes zutrifft: wäre Fa nicht eingetreten, so wäre auch Ga nicht eingetreten. Abgesehen von den in Kap. 6.5.1.2 erläuterten grundsätzlichen Deutungsproblemen von Ähnlichkeitsordnungen über möglichen Welten ist Lewis‘ Ansatz schwerwiegenden Einwänden ausgesetzt (vgl. Haussman 1998, Kap. 6). Wir können nicht näher darauf eingehen. 6.5.2.2 Kausale Präemption und Überdetermination. Von den kausal vollständigen Umständen U hängt es ab, ob F G verursacht oder nicht. Wir erläutern dies an zwei ,berüchtigten‘ Kausalszenarien: kausale Präemption und kausale Überdetermination. In beiden Situationen treten zwei (oder mehrere) für ein Ereignis Ga nomologisch hinreichende Ereignisse F1a, F2a (…) ein. Ein Beispiel: in einem Erschießungsszenario steht Fia jeweils für

6.5 Weiterführende Themen

eine auf Person a zielgenau abgefeuerte Kugel Nr. i. Sowohl Fa1 wie Fa2 erzwingen es nomologisch, dass Person a kurz danach tot ist (Ga). Im Fall der kausalen Präemption treffen die Kugeln zu unterschiedlichen Zeitpunkten ein: die Kugel 1 traf als erstes ein, und somit war sie die Ursache für den Tod von Person a; als die andere Kugel eintraf, war Person a bereits tot. Im Fall der kausalen Überdetermination treffen die Kugeln dagegen gleichzeitig ein; beide waren Mitursachen an dem traurigen Ereignis des Todes von Person a (für weitere Beispiele s. Haussman 1998, 49 – 52). Kausale Präemption und Überdetermination zeigen folgendes: selbst dann, wenn ein Ereignis vom Typ F ohne Vorliegen weiterer relevanter Faktoren eine Ursache für G wäre, kann dies durch das Vorliegen weiterer relevanter Faktoren verhindert werden. Die Kausalbeziehung ist daher selbst im strikten Fall nichtmonotoner Natur, weshalb wir uns in den Umständen U von Def. (6.5-1) auf alle möglicherweise kausal relevanten Umstandsfaktoren beziehen müssen. Welche Umstände in der Liste möglicherweise kausal relevanter Umstände tatsächlich kausal relevant sind, ergibt die generelle Kausalanalyse, der wir uns nun zuwenden, und welche mit drei Parametern arbeitet: erstens die Liste von Umständen U, die wir im Hintergrundwissen W als möglicherweise kausal relevant einschätzen, zweitens die in W verfügbaren statistischen Informationen I, und drittens die in W akzeptierten Hintergrundtheorien K über Kausalprozesse. Das Tripel (U,I,K) entspricht dem ,maximal vollständigen‘ Kausalmodell M(A,E|W) von (Ms. 6.2-1)(ii). 6.5.2.3 Kausale Prozesse. Eine Reihe von Autoren haben Kausalprozesstheorien entwickelt und damit den Bestandteil „K“ des Tripels „(U,I,K)“ zu explizieren versucht (z. B. Salmon 1984, 1997). Üblicherweise werden dabei Kausalprozesse als Prozesse charakterisiert, in denen sich eine gewisse physikalische Quantität, z. B. Energie oder Impuls, in Raum und Zeit ausbreitet. Abgesehen von speziellen Problemen (z. B. Dowe 1992, 8, 13 ff.) liegt die hauptsächliche Schwäche von Kausalprozesstheorien in folgendem: diese Theorien liefern keine Explikation von „Ereignis F ist Ursache von Ereignis G, gegeben Umstände U“ (s. auch Haussman 1998, 14). Dafür, dass F die Ursache von G in Umständen U ist, ist mehr nötig als bloß, dass irgendein kausaler Prozess von F in U nach G führt: dieser Prozess muss einerseits durch F und U bewirkt worden sein und andererseits G bewirken. Beispielsweise ist jedes Ereignis, das am Tage auf der Erde stattfindet, mit der Sonne durch einen Kausalprozess verbunden, weil die Sonne beständig Photonen auf die Erde schickt – deshalb ist nicht die Sonne schon die Ursache aller Ereignisse auf der Erde, denn die Sonnenstrahlen bewirken diese Ereignisse nicht. Um zu explizieren, was es heißt, dass ein physikalischer Kausalprozess von einem Ereignis F bewirkt wird und ein Ereignis G bewirkt, müssten die Ereignisse F und G selbst im physikalischen Detail beschrieben werden. Für komplexere Ereignisse ist dies im Regelfall aber nicht möglich. Physikalisch grundlegend ist die Annahme, dass Kausalprozesse nur zeitlich vorwärts gerichtet sein können. Einem verbreiteten Vorschlag zufolge wird die Zeitrichtung durch den zweiten Hauptsatz der Thermodynamik bestimmt, also dem Gesetz von der Zunahme der Entropie geschlossener Sys-

Multiple Ursachen: Präemption versus Überdetermination

Kausale Prozesse

Das Problem der Zeitrichtung

241

242

6. Erklärung

teme mit der Zeit. Aber das Entropiegesetz ist kein Fundamentalgesetz, sondern ein makroskopisches Gesetz über Wahrscheinlichkeitsverteilungen für Teilchenensembles. Wie Reichenbach (1956, 136 ff.) herausarbeitet, basiert die Entropierichtung der Zeit auf der Annahme, dass das Universum in einem Zustand relativ niedriger Entropie gestartet ist – ein Universum in einem Zustand maximaler Entropie könnte durchaus gelegentlich in einen Zustand geringerer Entropie übergehen (näheres s. Savitt 1996). Verschiedene Autoren haben die Begründbarkeit einer objektiven Zeitrichtung bezweifelt (z. B. Price 1996), und bis heute ist die physikalische Begründung der Zeitrichtung ein weitgehend ungelöstes Problem geblieben (für eine Diskussion einiger Ansätze s. Schurz 2001a, Kap. 4). Davon abgesehen lässt sich das Kriterium der Zeitrichtung nicht auf die Kausalrichtung in Koexistenzgesetzen anwenden (vgl. Kap. 6.2.1). Man bezeichnet die Ursachen hier auch als unabhängige und die Wirkungen als abhängige Variablen.

Kausale Graphen

MarkovReichenbach Bedingungen für probabilistische Kausalität

6.5.2.4 Kausale Graphen und probabilistische Kausalanalyse. Die Theorie der kausalen Graphen nimmt dagegen nur abstrakte Kausalprozesse an, deren Details unbekannt sind. Diese Kausalitätstheorie ist in jüngerer Zeit beeindruckend weiterentwickelt worden (s. Glymour et al. 1991, Spirtes et al. 1993, Pearl 2000). In den Standardtheorien kausaler Graphen wird der kausale Graph nicht mithilfe von Merkmalen bzw. Ereignistypen F(x) konstruiert, sondern mithilfe von Merkmalsvariablen bzw. Ereignisvariablen XF im Sinne der Statistik (Kap. 4.1.2), welche angewandt auf Individuen unterschiedliche Werte annehmen können. Ein kausaler Graph besteht aus einer Menge K von Variablen X1, …, Xn (Knoten) und einer Menge P von Pfeilen zwischen einigen dieser Knoten. X!Y bedeutet in einem kausalen Graph, dass Variable X eine direkte Ursache für eine Variable Y ist. Eine Variable X wird eine (direkte oder indirekte) Ursache für Y genannt, wenn im Graph von X ein gerichteter Pfad nach Y führt. Ein kausaler Graph, der alle gemeinsamen Ursachen (von Variablen des Graphen) umfasst, heißt kausal hinreichend (Glymour et al. 1991, 154). Über der Variablenmenge eines kausalen Graphen wird eine empirisch bekannte Wahrscheinlichkeitsverteilung angenommen. Die Pfeile eines kausalen Graphen drücken Kausalhypothesen aus. Die fundamentale kausaltheoretische Annahme der kausalen Graphentheorie besagt, dass kausal hinreichende Graphen die folgenden auf Markov und Reichenbach zurückgehende Bedingungen erfüllen (Glymour et al. 1991, 151, 156): (Ms. 6.5-1) (MR) Markov-Reichenbach-Bedingungen für kausal hinreichende Graphen ({X1,…,Xn}, P) mit gegebener Wahrscheinlichkeitsverteilung p: Für alle Xi 2 K und Variablenmengen K*  K, die keine Wirkungen von Xi enthalten, gilt: Xi und K* sind probabilistisch unabhängig, gegeben Xi’s direkte Ursachen U(Xi)  K. D. h. p(xi | u(xi), k*) = p(xi | k*) für alle Wertinstanziierungen xi, u(xi) und k* von Xi, U(Xi) und K*, respektive.

6.5 Weiterführende Themen

Konsequenzen von (MR): (MR1) Sind Xi und Xj probabilistisch abhängig, dann ist Xi entweder Ursache für Xj, oder Xj Ursache für Xi, oder Xi und Xj sind Wirkungen einer gemeinsamen Ursache. (MR2): Ist Xi weder Ursache von Xi noch umgekehrt, dann schirmt die Menge aller den beiden gemeinsamen Ursachen Xi und Xj voneinander ab. (MR 3) Kausaleinfluss wird pfadweise transportiert, d. h.: die Menge aller direkten Ursachen von Xi schirmt sämtliche indirekte Ursachen von Xi ab. Die Bedingung (MR) ist äquivalent mit der Pearlschen Bedingung der d-Verbindung und der Markov-Kompatibilität (Pearl 2000, 16, 19). Man beachte, dass eine probabilistische Unabhängigkeit zwischen zwei Variablen X, Y, gegeben eine dritte Variable Z, impliziert, dass diese Unabghängigkeit für alle Wertausprägungen x, y, z dieser Variablen besteht. In einem kausal hinreichenden Graphen spiegelt sich jede kausale Unabhängigkeit in einer probabilistischen Unabhängigkeit wider, und jede probabilistische Abhängigkeit in einer kausalen Verbindung. Die Umkehrung gilt nicht, und der Grund dafür sind die in Kap. 4.3.1 erläuterten Scheinunabhängigkeiten: wenn z. B. von X eine direkte positive und eine indirekte negative Kausalwirkung nach Y führt, dann können sich für spezielle p-Verteilungen die probabilistischen Effekte so kompensieren, dass insgesamt p(X,Y) = p(X)Np(Y) gilt. Eine Verteilung p über einem Graphen (K,P) heißt graphentreu g.d.w. die Reichenbach-Bedingungen erfüllt sind und jede von p implizierte (konditionale) probabilistische Unabhängigkeit auch einer kausalen Unabhängigkeit entspricht (Glymour et al. 1991, 159: „faithfulness“; Pearl 2000, 48: „stability“). Für graphentreue Wahrscheinlichkeitsverteilungen p über einer Variablenmenge X1, …, Xn haben Glymour et al. (1991, 170 ff.) und Pearl (2000, 50) einen kausalen Entdeckungsalgorithmus entwickelt, der alle kausal hinreichenden Graphen konstruiert, die diese Verteilung gemäß den Reichenbach-Bedingungen erzeugen. Diese möglichen und empirisch nicht diskriminierbaren kausalen Graphen heißen auch stark statistisch ununterscheidbar (Glymour et al. 1991, 168 f.). Dies wird durch folgendes Beispiel von Pearl (2000, 15) in Abb. 6.5-1 illustriert.

Rutschigkeit X5

Stark statistisch ununterscheidbare Graphen:

Nässe X4 Sprinkler X2

Regen X3

X5

X5

X4

X4

X2 Jahreszeit X1

X3 X1

X2

X3 X1

Abb. 6.5-1: Drei statistisch stark ununterscheidbare Graphen

Probabilistische versus kausale Unabhängigkeit

Kausaler Entdeckungsalgorithmus

Statistisch ununterscheidbare kausale Graphen

243

244

6. Erklärung

Zusammengefasst informieren die Resultate von Glymour et al. und Pearl über die Möglichkeiten und Grenzen probabilistischer Kausalanalysen. Selbst unter den Annahmen, dass die Variablenmenge K kausal hinreichend ist (keine abschirmende gemeinsame Ursachen wurde übersehen) und dass die Wahrscheinlichkeitsverteilung p graphentreu ist (beobachtete statistische Unabhängigkeiten sind keine Scheinunabhängigkeiten), lassen sich die wahren Kausalbeziehungen zwischen Variablen nicht immer eindeutig, sondern nur unvollständig aus der vorliegenden Wahrscheinlichkeitsverteilung erschließen.

Haussmans Interventionstheorie

6.5.2.5 Interventionistischer Ansatz. Der interventionistische Ansatz geht zurück auf den handlungstheoretischen Kausalitätsbegriff, der von Von Wright (1974, 73) und Menzies und Price (1993) entwickelt wurde. Danach ist A Ursache von B, wenn durch Realisierung von A mithilfe einer Handlung H die Wirkung B herbeigeführt werden kann. Dieser Ansatz erscheint aber insofern zirkulär, als darin die kausale Relation des Herbeiführens eines Ereignisses durch eine Handlung unerklärt angenommen wird. Davon abgesehen gibt es eine Reihe von Variablen (z. B. ,Jahreszeit‘ oder ,Regen‘ in Abb. 6.5-1), die wir nicht durch unsere Handlungen manipulieren können. Aus diesen Gründen schlägt Haussman (1989) vor, die Handlungstheorie der Kausalität durch eine abstrakte kausale Interventionstheorie zu ersetzen, welche auf folgender Annahme beruht: wannimmer ein Ereignis F entweder eine Ursache eines anderen Ereignisses G ist, oder aber F und G Wirkungen einer gemeinsamen Ursache sind, dann gibt es eine weitere Ursache H (6¼F) für G, die nicht mit F kausal verbunden ist (im Sinne von Ms. 6.5-1, Ra1), und durch welche G vom kausalen Einfluss von F entkoppelbar ist (Haussman (1998, 64). Falls F eine Ursache für G ist, kann eine solche entkoppelnde Intervention nur für die Wirkung G und nicht für die Ursache F existieren (denn wäre H Ursache für F, dann wäre H auch indirekte Ursache für G und somit mit G kausal verbunden). Mit Haussmans Kriterium kann also herausgefunden werden, ob F eine Ursache von G ist, gegeben dass F und G kausal verbunden sind sofern Haussmans Annahme erfüllt ist. Das Problem von Haussman’s Ansatz liegt aber darin, dass seine Annahme in vielen Situationen nicht erfüllt ist (Glymour et al. 1991, 166), und dass sein Kriterium nur gilt, wenn die Wahrscheinlichkeitsverteilung graphentreu ist (Schurz 2001a, Kap. 2).

6.6 Zusammenfassung, einführende Literatur und Übungen 6.6.1 Zusammenfassung (Auswahl). Hempel hatte eine deduktiv-nomologische (DN) Erklärung eines Explanandums als die Herleitung desselben aus übergeordneten strikten Gesetzen und Antecedensbedingungen charakterisiert. Gemäß dieser Charakterisierung müssten Erklärung einerseits und Voraussage bzw. Begründung andererseits dieselbe Struktur besitzen. Die Folgediskussion zeigte aber, dass dies nicht zutrifft. Für Erklärungen ist es nötig,

6.6 Zusammenfassung, einführende Literatur und Übungen

dass das Antecedens Ursachen des Explanandums anführt; für Voraussagen bzw. Begründungen ist dies nicht zu fordern. 20 Jahre später hatte Hempel sein Modell der induktiv-statistischen (IS) Erklärung entwickelt, um der Tatsache Rechnung zu tragen, dass die Gesetzesbeziehungen in Sozial- und Humanwissenschaft im Regelfall nicht-strikter Natur sind. Der grundlegende Unterschied von IS-Erklärungen im Vergleich zu DN-Erklärungen ist ihre Nichtmonotonie, welche sich darin äußert, dass ihr Antecedens die Bedingung der maximalen Bestimmtheit erfüllen muss. In der Folgediskussion entwickelte sich eine bislang ungelöste Meinungsverschiedenheit über die Höhe der Wahrscheinlichkeit, welche das Antecedens dem Explanandum verleihen soll. Im Fall der Erklärung menschlicher Handlungen erscheint es schließlich angebracht, die statistischen Gesetze des IS-Modells durch normische Gesetze zu ersetzen, welche häufig hermeneutischen Rationalitätspräsumptionen entsprechen. Gesetzesartigkeit und Kausalität gehören bis heute zu den schwierigsten wissenschaftstheoretischen Begriffen. Durch die kausale Graphentheorie wurden einige grundlegende Erkenntnisse darüber erzielt, wie aus Wahrscheinlichkeitsverteilungen über Variablenmengen auf die möglichen dahinter liegenden Kausalstrukturen geschlossen werden kann. 6.6.2 Einführende Literatur. Einführungen in Hempels Erklärungsmodelle liefern Hempel (1977) und Stegmüller (1983). Weiterführende Arbeiten und Überblicke finden sich in Schurz (1990, Hg.) und Salmon (1989). Weiterführendes zum Gesetzesartigkeitsproblem findet sich in Armstrong (1983), Earman (1986) und van Fraassen (1989). Unter den vielen weiterführenden Büchern zur Kausalität sind Haussman (1998) und Pearl (2000) hervorzuheben.

6.6.3 Fragen und Aufgaben Zu Kap. 6.1 – 6.2: 1) Rekapitulieren Sie die Bedingungen des Hempelschen DN-Erklärungsmodells. 2) Betrachten Sie folgende Argumente (die Gesetzeshypothesen sind jeweils weggelassen): (a) Peter ist gekommen, weil er zu mir kommen wollte, (b) Peter wird kommen, weil er mir gesagt hat „ich werde kommen“, (c) Peter wird kommen, weil er zu mir kommen will, (d) Peter wollte zu mir kommen, weil er zu mir gekommen ist, (e) Peter wird kommen, weil ich es glaube. – Welche dieser Argumente liefern eine Begründung?, welche eine Voraussage, und welche eine Retrodiktion? Welche der Argumente liefern eine (aktuale) Erklärung, und welche eine potentielle Erklärung? 3) (a) Welche Arten von DN-Begründungen gibt es, die keine DN-Erklärungen sind? Geben Sie jeweils ein Beispiel. (b) Welche DN-Erklärungen können nicht als DN-Voraussagen oder -Begründungen fungieren? Geben Sie ein Beispiel. Zu Kap. 6.3 – 6.4: 1) Rekapitulieren Sie Hempels IS-Modell der Erklärung. 2) Betrachten Sie folgende Erklärung (Gesetzesprämisse weggelassen): „Herr Müller ist gestorben, weil er 82 Jahre alt war“. Wie muss mein Hintergrundwissen aussehen, damit diese Erklärung für mich die Bedingung der maximalen Bestimmtheit erfüllt?

245

246

6. Erklärung

3) Jemand prognostiziert: „Herr Müller wird bald sterben, weil er 82 Jahre alt ist.“ Daraufhin kontert jemand: „Herr Müller ist doch bei bester Gesundheit!“ Erläutern Sie an diesem Beispiel die Nichtmonotonie von IS-Argumenten. 4) Suchen Sie nach Beispielen für wahrscheinlichkeitssenkende Erklärungen im Sinne von Salmon. 5) Jemand läuft zum Brunnen und trinkt Wasser. Liefern Sie hierfür eine rationale normische Erklärung. Unter welchen Umständen ließe sich diese Handlung nicht mehr rational erklären? – lassen Sie Ihre Phantasie spielen. Zu Kap. 6.5: 1) Welcher der folgenden Allsätze ist raumzeitlich universell, und welcher erfüllt die Maxwell-Bedingung: (i) alle Äpfel sind rot, (ii) alle Äpfel in Müllers Scheune sind blau, (iii) alle Äpfel sind rot, außer den blauen Äpfeln in Müllers Scheune. 2) Was versteht man unter kausaler Präemption, und was unter kausaler Überdetermination? Geben Sie jeweils ein Beispiel. 3) Was versteht man unter einer graphentreuen Wahrscheinlichkeitsverteilung? 4) Erläutern Sie den handlungstheoretischen Kausalitätsansatz und seine Probleme.

Literaturverzeichnis Adam, M. (2002): Theoriebeladenheit und Objektivität, Dr. Hänsel-Hohenhausen, Frankfurt/M. Adams, E. W. (1975): The Logic of Conditionals, Reidel, Reidel, Dordrecht. Adorno, T. W. et al. (1950): „The Authoritarian Personality“, New York. Adorno, T. W. et al. (1969): Der Positivismusstreit in der deutschen Soziologie, Suhrkamp, Frankfurt/M. (4. Aufl. 1975). Albert, H., (1980): Traktat über kritische Vernunft, J.C.B. Mohr, Tübingen, 4. Aufl. (5. erw. u. verb. Aufl. 1991). Albert, H. und Topitsch, E. (1971): Werturteilsstreit, Wissenschaftliche Buchgesellschaft, Darmstadt. Alston, W. P. (1971): „The Place of the Explanation of Particular Facts in Science“, Philosophy of Science 38, 13 – 34. Anderson, J. R. (2001): Kognitive Psychologie, Spektrum, Heidelberg, 3. Aufl. Anderson, A. R., and Belnap, N. D. (1975): Entailment. The Logic of Relevance and Necessity, Princeton Univ. Press, Princeton. Apel, K.-O. (1976, Hg.): Charles Sanders Peirce: Schriften zum Pragmatismus und Pragmatizismus, Frankfurt/M. (2. Aufl.). Apel, K.-O. (1979): Die Erklären:Verstehen-Kontroverse in transzendentalpragmatischer Sicht, Suhrkamp, Frankfurt/M. Aqvist, L. (1984): „Deontic Logic“, in: Gabbay und Guenthner (1984, Hg.), Handbook of Philosophical Logic. Vol. II: Extensions of Classical Logic, Reidel, Dordrecht, 605 – 714. Armstrong, D. M. (1983): What Is a Law of Nature?, Cambridge Univ. Press., Cambridge. Ausubel, D. et al. (1978): Educational Psychology: A Cognitive View, Holt, New York (dt. als Psychologie des Unterrichts, Beltz, Weinheim 1974). Bacchus, F. (1990): Representing and Reasoning with Probabilistic Knowledge, MIT Press, Cambridge/ MA. Balzer, W. (1982): Empirische Theorien: Modelle, Strukturen, Beispiele, Vieweg, Braunschweig. Balzer, W. (1985): Theorie und Messung, Springer, Berlin. Balzer, W., und Moulines, W. (1980): „On Theoreticity“, Synthese 44, 467 – 494.

Balzer, W., und Mühlhölzer, F. (1982): „Klassische Stoßmechanik“, Zeitschrift für Allgemeine Wissenschaftstheorie 13, 22 – 39. Balzer, W. et al. (1987): An Architectonic for Science, Reidel, Dordrecht. Barnes, E. (1992): „Explanatory Unification and the Problem of Asymmetry“, Philosophy of Science 59, 558 – 571. Bartelborth, T. (1996): Begründungsstrategien. Ein Weg durch die analytische Erkenntnistheorie, Akademie Verlag, Berlin. Bartels, A. (1996): Grundprobleme der modernen Naturphilosophie, Schöningh, Paderborn. Bauer, H. (1978): Wahrscheinlichkeitstheorie und Grundzüge der Maßtheorie, Berlin, New York (5. neubearb. Aufl. 2002; engl. Aufl. 1996). Berlin, B., und Kay, P. (1999): Basic Colour Terms: Their Universality and Evolution, CSLI Publications, Stanford (Orig. 1969). Bhaskar, R. (1978): A Realist Theory of Science, Harvester Press, Sussex. Bird, A. (1998): Philosophy of Science, McGillQueen’s University Press, Montreal & Kingston. Böhm, Jan M. et al. (2002, Hg.): Karl Poppers kritischer Rationalismus heute, Mohr Siebeck, Tübingen. Bortz, J. (1985): Lehrbuch der Statistik, Springer, Berlin, 2. Aufl. (Neuaufl. als Statistik für Human- u. Sozialwissenschaftler, 6. überarb. Aufl. 2005). Bortz, J., und Döring, N. (2002): Forschungsmethoden und Evaluation, Springer, Berlin (3. Aufl.). Bourbaki (1961): Topologie Générale, Hermann, Paris. Brainerd, C. (1978): Piaget’s Theory of Intelligence, Prentice-Hall, Englewood Cliffs. Brendel, E. (1999): Wahrheit und Wissen, Mentis, Paderborn. Bridgeman, P. W. (1936): The Nature of Physical Theory, Princeton Univ. Press, Princeton. Bromberger, S. (1965): „An Approach to Explanation“, in: Butler, R. J. (Hg.), Analytical Philosophy (Second Series), Oxford, 72 – 105. Bromberger, S. (1966): „Why-Ouestions“, in: Colodny, R. (Hg.): Mind and Cosmos, University of Pittsburgh Press, Pittsburgh, 86 – 111. Bühler, A. (2003, Hg.): Hermeneutik, Synchron, Heidelberg.

248

Literaturverzeichnis Bühler, C., und Allen, M. (1982): Einführung in die humanistische Psychologie, Klett, Stuttgart. Bunge, M. (1961): „Kinds and Criteria of Scientific Laws“, Philosophy of Science 28, 260 – 281. Bunge, M. (1967): Scientific Research, Bd. I+II, Springer, Berlin. Bunge, M. (1974): Semantics I: Sense and Reference, Reidel, Dordrecht. Campbell, D. T. (1984): „Evolutionary Epistemology“, in: Radnitzky, G. /Bartley, III, W.W. (Hg.), Evolutionary Epistemology, Rationality, and the Sociology of Knowledge, La Salle. Carnap, R. (1928): Der logische Aufbau der Welt, Felix Meiner, Hamburg 1961. Carnap, R. (1932/33): „Über Protokollsätze“, Erkenntnis 3, 215 – 228. Carnap, R. (1936/37): „Testability and Meaning“, Philosophy of Science, Vol. 3, 419 – 471 und Vol. 4, 2 – 40 (selbstständig erschienen: New Haven 1954). Carnap, R. (1939): „The Interpretation of Physics“, in: ders., Foundations of Logic and Mathematics, Univ. of Chicago Press, 56 – 69. Carnap, R. (1947): „On the Application of Inductive Logic“, Philosophy and Phenomenological Research 8, 133 – 147. Carnap, R. (1950a): „Empiricism, Semantics and Ontology“, Revue Intern. de Phil. 4, 20 – 40; dt. Übersetzung als Anh. A von Carnap 1972 (zitiert danach). Carnap, R. (1950b): Logical Foundations of Probability, Univ. of Chicago Chicago. Dt. Kurzfassung (1959). Carnap, R. (1956): „The Methodological Character of Theoretical Concepts“, in: Feigl, H./ Scriven, M. (Hg.), Minnesota Studies in the Philosophy of Science Vol. I, Univ. of Minnesota Press, Minneapolis, 38 – 76. Carnap, R. (1961): Scheinprobleme der Philosophie, Suhrkamp, Frankfurt/M. (Orig. 1928). Carnap, R. (1963): „Carl G. Hempel on Scientific Theories“, in: Schilpp, P. A. (Hg.), The Philosophy of Rudolf Carnap, La Salle, 958 – 965. Carnap, R. (1972): Bedeutung und Notwendigkeit, Springer, Berlin (engl. Orig. 1956). Carnap, R. (1973): Grundlagen der Logik und Mathematik, Nymphenburger Verlagsbuchhandlung, München (engl. Orig. 1939). Carnap, R. (1976): Einführung in die Philosophie der Naturwissenschaft, 3. A., Nymphenburger Verlagshandlung, München (englisches Original 1966). Carnap, R., und Jeffrey, R. (1971): Studies in Inductive Logic and Probability, Univ. of California Press, Berkeley.

Carrier, M. (2003): „Experimental Success and the Revelation of Reality: The Miracle Argument for Scientific Realism“, in: P. Blanchard et al. (eds.), Science, Society and Reality, Springer, Heidelberg. Cartwright, N. (1979): „Causal Laws and Effective Strategies“, Nous 13, 419 – 437. Cartwright, N. (1983): How the Laws of Physics Lie, Clarendon Press, Oxford. Chalmers, A. F. (1994): Wege der Wissenschaft, Springer, Berlin (3. Aufl.). Coffa, J. (1974): „Hempel’s Ambiguity“, Synthese 28, 141 – 163. Curd, M., und Cover, J. A. (1998, Hg.): Philosophy of Science, Norton, New York. Czaniera, U. (2001): Gibt es moralisches Wissen?, Mentis, Paderborn. Dahms, H.-J. (1994): Positivismusstreit, Suhrkamp, Frankfurt/M. Dancy, J. (1985): An Introduction to Contemporary Epistemology, B. Blackwell, Oxford 1985. De Finetti, B. (1970): Wahrscheinlichkeitstheorie, Oldenbourg, München 1981 (zuerst 1970 italienisch; 1974 als Theory of Probability bei John Wiley, New York). Dowe, P. (1992): „Wesley Salmon’s Process Theory of Causality and the Conserved Quantity Theory“, Philosophy of Science 59, 195 – 216. Dray, W. (1957): Laws and Explanation in History, Oxford Univ. Press, Oxford. Dubs, R. (1982): Der Führungsstil des Lehrers im Unterricht, Institut für Wirtschaftspädagogik, St. Gallen. Duerr, H. P. (1978): Traumzeit, Syndikat, Frankfurt/ Main. Duhem, P. (1908): Ziel und Struktur der physikalischen Theorien, Felix Meiner, Hamburg 1978. Eagle, A. (2004): „Twenty-One Arguments Against Propensity Analyses of Probability“, Erkenntnis 60, 371 – 416. Earman, J. (1986): A Primer on Determinism, Reidel, Dordrecht. Earman, J. (1992): Bayes or Bust?, MIT Press, Cambridge/Mass. Earman, J. et al. (2002, Hg.): Ceteris Paribus Laws, Erkenntnis 57, No. 3 (Sonderband). Ebbinghaus, H.-D. (2003): Einführung in die Mengenlehre, Spektrum Verlag, Heidelberg (4. Aufl.). Eells, E. (1991): Probabilistic Causality, Cambridge Univ. Press, New York. Essler, W. et al. (1991): Grundzüge der Logik Bd. I, Vittorio Klostermann, Frankfurt/M. Essler, W. et al. (2000): Theorie und Erfahrung, Karl Alber, Freiburg.

Literaturverzeichnis Etchemendy, J. (1990): The Concept of Logical Consequence, Harvard University Press, Cambridge/ Mass. Evans, J. St. B. (1982): The Psychology of Deductive Reasoning, Routledge & Kegan Paul, London. Feyerabend, P. (1976): Wider den Methodenzwang. Skizze einer anarchistischen Erkenntnistheorie, Suhrkamp, Frankfurt/M. (engl. Original 1975). Field, H. (1980): Science without Numbers, Princeton Univ. Press, Princeton. Fisher, R. A. (1956): Statistical Methods and Scientific Inference, Hafner Press, New York (erw. Neuaufl. Oxford Univ. Press 1995). Flach, P., und Kakas, A. (2000, Hg.): Abduction and Induction, Kluwer, Dordrecht. Fodor, J. (1984): „Observation Reconsidered“, Philosophy of Science 51, 23 – 43. Fodor, J. (1987): Psychosemantics, MIT Press, Cambridge/Mass. Fodor, J. (1990): A Theory of Content, MIT Press, Cambridge/Mass. Follesdal, D. (2003): „Hermeneutik und die Hypothetisch-Deduktive Methode“, in: Bühler (2003, Hg.), 157 – 176 (engl. Orig. 1979 in Dialectica 33, No 3 – 4, 337 – 356). Forster, M., und Sober, E. (1994): „How to Tell when Simpler, More Unified, or Less Ad Hoc Theories will provide More Accurate Predictions“, British Journal for the Philosophy of Science 45, 1 – 35. Frankena, W. K. (1994): Analytische Ethik, dtv, München, 5. Aufl. (engl. Orig. 1963). Frege, G. (1892): „Über Sinn und Bedeutung“, in: ders., Funktion, Begriff, Bedeutung, hg. v. G. Patzig, 4. Aufl., Vandenhoeck & Ruprecht, Göttingen 1975, 40 – 65. Frege, G. (1918): „Der Gedanke“, in: ders., Logische Untersuchungen, hg. von G. Patzig, Vandenhoeck & Ruprecht, Göttingen 1976 (zitiert danach). French, S. (2008): „The Structure of Theories“, in: S. Psillos, M. Curd (eds.), The Routledge Companion to Philosophy of Science, Routledge, London and New York, 269 – 280. Friedman, M. (1990): „Erklärung und wissenschaftliches Verstehen“, in: Schurz (1990, Hg.), 171 – 191; engl. Original in Journal of Philosophy 71, 1974. Gabbay, D. M. et al. (1994, Hg.): Handbook of Logic in Artificial Intelligence, Vol. 3: Nonmonotonic Reasoning and Uncertain Reasoning, Clarendon Press, Oxford. Gadamer, H. (1975): Wahrheit und Methode, J.C.B. Mohr, Tübingen. Gadenne, V. (1984): Theorie und Erfahrung in der psychologischen Forschung, Mohr, Tübingen.

Gärdenfors, P. (1990): „Die Epistemologie von Erklärungen“, in: Schurz, G. (1990, Hg.), 91 – 124 (Erweit. des engl. Originals von 1980). Garnham, A., und Oakhill, J. (1994): Thinking and Reasoning, B. Blackwell, Oxford. Gemes, K. (1993): „Hypothetico-Deductivism, Content, and the Natural Axiomatization of Theories“, Philosophy of Science, 54, 477 – 487. Gerthsen, C., und Kneser, H. (1971): Physik, Springer, Berlin (Neuaufl. als Gerthsen-Physik, hg. v. D. Meschede, 22. neubearb. Aufl. 2004). Giere, R. (1988): „Laws, Theories, and Generalizations“, in: Grünbaum/Salmon (Hg., 1988), 37 – 46. Giere, R. (1999): Science without Laws, Univ. of Chicago Press, Chicago. Gillies, D. (2000): Philosophical Theories of Probability, Routledge, London. Glasersfeld, E.v. (1985): „Konstruktion der Wirklichkeit und des Begriffs der Objektivität“, in: Glasersfeld, E.v. et al., Einführung in den Konstruktivismus, Oldenbourg, München. Glymour, C. (1980): „Hypothetico-Deductivism is Hopleless“, Philosophy of Science 47, 322 – 325. Glymour, C. (1981): Theory and Evidence, Princeton Univ. Press, Princeton. Glymour, C. et al. (1991): „Causal Inference“, Erkenntnis 35, 151 – 189. Gödel, K. (1947): „What Is Cantor’s Continuum Problem?“, The American Mathematical Monthly 54; wiederabgedruckt in Benacerraf/Putnam (1964, Hg.), 258 – 273 (zitiert danach). Goldszmidt, M., und Pearl, J. (1996): „Qualitative Probabilities for Default Reasoning, Belief Revision and Causal Modeling“, Artificial Intelligence 84, 57 – 112. Good, I. J. (1983): Good Thinking. The Foundations of Probability and Its Applications, Univ. of Minnesota Press, Minneapolis. Goodman, N. (1955/75): Tatsache, Fiktion, Voraussage, Suhrkamp, Frankfurt/M. 1975 (Neuaufl. 1988; engl. Original 1955). Goodman, N. (1978): Ways of Worldmaking, Harvester Press, Hassocks/Sussex. Greeno, J. (1970): „Evaluating of Statistical Hypotheses Using Information Transmitted“, Philosophy of Science 37, 279 – 293; wiederabgedruckt in Salmon (1971), 89 – 104 (zitiert danach). Grice, H. P. (1975): „Logic and Conversation“, dt. in: Meggle, G. (1993, Hg.), Handlung, Bedeutung, Kommunikation, Suhrkamp, Frankfurt/M., 243 – 265 (zitiert danach). Grünbaum, A. (1972): Philosophical Problems of Space and Time, Reidel, Dordrecht.

249

250

Literaturverzeichnis Grünbaum, A. (1976): „Ad hoc Auxiliary Hypotheses and Falsificationism“, The British Journal for the Philosophy of Science 27, 329 – 362. Grünbaum, A., und Salmon, W. (1988, Hg.): The Limitations of Deductivism, University of California Press, Berkeley Habermas, J. (1968): „Erkenntnis und Interesse“, in: Habermas, J., Technik und Wissenschaft als ,Ideologie‘, Suhrkamp, Frankfurt/M. 1968. Hanson, N. R. (1958): Patterns of Discovery, Cambridge Univ. Press, Cambridge. Hare, R. (1952): The Language of Morals, Oxford Univ. Press, Oxford. Harman, G. (1965): „The Inference to the Best Explanation“, Philosophical Review 74, 88 – 95. Hart, W. D. (1996, Hg.): The Philosophy of Mathematics, Oxford University Press, Oxford. Haussman, D. (1998): Causal Asymmetries, Cambridge University Press, Cambridge. Haussmann, T. (1991): Erklären und Verstehen, Suhrkamp, Frankfurt/M. Hays, W., und Winkler, R. (1970): Statistics: Probability, Inference, and Decision, Holt, New York (2. Aufl. 1975). Hempel, C. G. (1942): „The Function of General Laws in History“, in: The Journal of Philosophy 39, 35 – 48 (abgedruckt in ders., 1965, 231 – 243, zit. danach). Hempel, C. G. (1951): „The Concept of Cognitive Significance: A Reconsideration“, Proc. of the Amer. Academy of Arts and Sciences 80; abgedruckt in Sinnreich, J. (1972, Hg.), 126 – 144, zitiert danach (gekürzte engl. Fassung in Hempel 1965, Kap. II.1). Hempel, C. G. (1958): „The Theorectican’s Dilemma. Studies in the Logic of Theory Construction“, in: Feigl/Scriven/Maxwell (1958), wiederabgedruckt in: Hempel 1965, 173 – 228; zitiert danach. Hempel, C. G. (1965): Aspects of Scientific Explanation and Other Essays in the Philosophy of Science, Free Press, New York-London. Hempel, C. G. (1968): „Maximal Specifity and Lawlikeness in Probabilistic Explanation“, Philosophy of Science 35, 116 – 133. Hempel, C. G. (1969): „Reduction: Ontological and Linguistic Facts“, in: S. Morgenbesser et al. (Hg.), Philosophy, Science, and Method, St. Martin’s Press, New York, 179 – 199. Hempel, C. G. (1974): Philosophie der Naturwissenschaften, dtv, München (engl. Orig. 1966). Hempel, C. G. (1977): Aspekte wissenschaftlicher Erklärung, W. de Gruyter, Berlin (Übersetzung von Hempel 1965, Kap. IV.12, inkl. neuem Nachwort).

Hempel, C. G. (1988): „Provisos: A Problem Concerning the Inferential Function of Scientific Theories“, in: Grünbaum/Salmon (eds.), 19 – 36. Hempel, C./Oppenheim, P. (1948): „Studies in the Logic of Explanation“, Philosophy of Science 15, 135 – 175; wiederabgedruckt in: Hempel (1965), 245 – 290; zitiert danach. Hilgendorf, E. (2000): „Das Problem der Wertfreiheit in der Jurisprudenz“, in: Hilgendorf und Kuhlen (2000), 1 – 32. Hilgendorf, E., und Kuhlen, L. (2000, Hg.): Die Wertfreiheit in der Jurisprudenz, C.F. Müller, Heidelberg. Hintikka, J., und Suppes, P. (1966, Hg.): Aspects of Inductive Logic, North-Holland Publ. Comp., Amsterdam. Hitchcock C. and Sober, E. (2004): „Prediction Versus Accommodation and the Risk of Overfitting“, British Journal for the Philosophy of Science 55: 1 – 34. Howson, C., und Urbach, P. (1996): Scientific Reasoning: The Bayesian Approach, Open Court, Chicago (2. Aufl.). Howson, C. (2000): Hume’s Problem: Induction and the Justification of Belief, Clarendon Press, Oxford. Hoyningen-Huene, P. (1989): Die Wissenschaftsphilosophie Thomas S. Kuhns, Friedr. Vieweg & Sohn, Braunschweig/Wiesbaden. Huber, O. (1987): Das psychologische Experiment: Eine Einführung, Huber, Bern. Huff, D. (1956): Wie lügt man mit Statistik, Sanssouci, Zürich. Hughes, G. E., und Cresswell, M. J. (1996): A New Introduction to Modal Logic, Routledge, London. Hume, D. (1748): Eine Untersuchung über den menschlichen Verstand, reclam, Hamburg. Hummell, H. J., und Ziegler, E. (1976): Korrelation und Kausalität, F. Enke, Stuttgart. Humphreys, P. (1989): The Chances of Explanation, Princeton Univ. Press, Princeton. Humphreys, P. (1993): „Greater Unification Equals Greater Understanding?“, Analysis 53/3, 183 – 188. Kamlah, W., und Lorenzen, P. (1973): Logische Propädeutik, Bibliographisches Institut, Mannheim. Kanitscheider, B. (1981): Wissenschaftstheorie der Naturwissenschaften, de Gruyter, Berlin. Kelsen, H. (1960): Reine Rechtslehre, Deuticke, Wien (2. vollst. neu bearb. u. erw. Aufl.). Ketland, J. (2004): „Empirical Adequacy and Ramsification“, British Journal for the Philosophy of Science 55, 287 – 300. Keuth, H. (1978): „Methodologische Regeln des kritischen Rationalismus – Eine Kritik“, Zeitschrift für Allgemeine Wissenschaftstheorie 9, 236 – 255.

Literaturverzeichnis Kitcher, P. (1990): Erklärung durch Vereinheitlichung, in: Schurz (1990, Hg.), 193 – 231; engl. Original in Philosophy of Science 48, 1981. Klenk, V. (1989): Understanding Symbolic Logic, Prentice Hall, Englewood Cliffs, NJ, 2. Aufl. (4. Aufl. 2001). Körner, S. (1947): „On Entailment“, Proceedings of the Aristotelean Society 21, 143 – 162. Kolmogorov, A. N. (1933): Foundations of the Theory of Probability, Chelsea Publ. Comp., New York 1950 (dt. Orig. 1933 im Zentralblatt der Mathematik, 2. Band, Springer, Berlin). Konegen, N., und Sondergeld, K. (1985): Wissenschaftstheorie für Sozialwissenschaftler, Leske Verlag, Opladen. Kornblith, H. (1994, Hg.): Naturalizing Epistemology, MIT Press, Cambridge/MA. Krantz, D. et. al. (1971): Foundations of Measurement. Vol. I, Academic Press, New York. Krebs, D. L. (1998): „The Evolution of Moral Behaviour“, in: Crawford, C. und Krebs, D.L. (Hg.): Handbook of Evolutionary Psychology, Lawrence Erlbaum Assoc., Mahwah/NJ., 337 – 368. Kripke, S. (1972): Naming and Necessity, Basil Blackwell, Oxford (2. Auflage 1980). Kriz, J. et al.: Wissenschafts- und Erkenntnistheorie. Eine Einführung für Psychologen und Humanwissenschaftler, Leske Verlag, Opladen. Kromrey, H. (2002): Empirische Sozialforschung, Leske+Budrich, Opladen (10. Aufl.). Kuhlen, L. (2000): „Wertfreiheit in der Jurisprudenz?“, in: Hilgendorf und Kuhlen (2000), 33 – 49. Kuhn, T. S. (1967): Die Struktur wissenschaftlicher Revolutionen, Suhrkamp, Frankfurt/M. (2. rev. Aufl. 2002, engl. Original 1962). Kuhn, T. S. (1977): Die Entstehung des Neuen, Suhrkamp, Frankfurt/M. (5. Aufl. 1997). Kuipers, T. A. F. (2000): From Instrumentalism to Constructive Realism, Kluwer, Dordrecht. Kutschera, F. v. (1972): Wissenschaftstheorie, Bd. I und II, Fink, München. Kutschera, F. v. (1975): Sprachphilosophie, W. Fink, München. Ladyman, J. (2002): Understanding Philosophy of Science, Routledge, London. Ladyman, J., and Ross, D. (2007): Every Thing Must Go. Metaphysics Naturalized, Oxford University Press. Oxford. (With D. Spurrett and J. Collier.) Lakatos, I. (1974): „Falsifikation und die Methodologie wissenschaftlicher Forschungsprogramme“, in: Lakatos, I., und Musgrave, A., Kritik und Erkenntnisfortschritt, Vieweg, Braunschweig (engl. Original 1970).

Lamnek, S. (1988): Qualitative Sozialforschung, Bd. 1: Methodologie, Psychologie Verlags Union, München. Lange, M., (2009): Laws and Lawmakers, Oxford University Press, Oxford. Langley, P. et al. (1987): Scientific Discovery. Computational Explorations of the Creative Process, MIT Press, Cambridge/Mass. Laudan, L. (1997): „A Confutation of Convergent Realism“, in: Papineau (Hg., 1997), 107 – 138. Lauth, B., und Sareiter, J. (2002): Wissenschaftliche Erkenntnis. Eine ideengeschichtliche Einführung in die Wissenschaftstheorie, mentis, Paderborn. Lenzen, W. (1974): Theorie der Bestätigung wissenschaftlicher Hypothesen, Holzboog, Stuttgart-Bad Cannstatt. Leplin, J. (1984, Hg.): Scientific Realism, Univ. of California Press, Berkeley. Lewis, D. (1970): „How to Define Theoretical Terms“, wiederabgedruckt in: ders., Philosophical Papers Vol. I, Oxford Univ. Press, New York 1983, Kap. 6, zitiert danach. Lewis, D. (1973a): „Causation“, Journal of Philosophy 70, 556 – 567. Lewis, D. (1973b): Counterfactuals, Basil Blackwell, Oxford. Lewis, D. (1980): „A Subjectivist’s Guide to Objective Chance“, in: Jeffrey, R. C. (Hg., 1980), Studies in Inductive Logic and Probability, Vol. 2, Berkeley, Univ. of California Press. Lipton, P. (1991): Inference to the Best Explanation, Routledge, London. Losee, J. (1977): Wissenschaftstheorie. Eine historische Einführung, C.H. Beck, München (engl. Orig. 1972). Mackie, J. L. (1975): „Causes and Conditions“, in: Sosa, E. (Hg.): Causation and Conditionals, Oxford Univ. Press, Oxford, 15 – 38. Makinson, D. (1965): „The Paradox of the Preface“, Analysis 25, 205 – 207. Maturana, H. R., und Varela, F. (1984): Der Baum der Erkenntnis, Goldmann Verlag, München. Maxwell, G. (1962): „The Ontological Status of Theoretical Entities“, in: Feigl, H., and Maxwell, G. (Hg., 1962): Minnesota Studies in the Philosophy of Science Vol. III, Univ. of Minnesota Press, Minneapolis, 3 – 27. Maxwell, N. W. (1998): The Comprehensibility of the Universe, Clarendon Press, Oxford. Mayntz, R. et al. (1974): Einführung in die Methoden der empirischen Soziologie, Westdeutscher Verlag, Opladen (4. Aufl.). Melia, J., and Saatsi, J. (2006): „Ramsification and Theoretical Content“, British Journal for the Philosophy of Science 57, 561 – 581.

251

252

Literaturverzeichnis Menzies, P., und Price, H. (1993): „Causation as a Secondary Quality“, British Journal for the Philosophy of Science 44, 187 – 203. Milgram, S. (1974): Obedience to Authority: An Experimental View, Harper and Row, New York. Mill, J. St. (1865): System of Logic, London; deutsch: als Bde 2 – 3 der Gesammelten Werke von J. St. Mill, hg. von T. Gomperz, Leipzig 1872, 8. Auflage., zitiert danach. Mortimer, C. E. (1973): Chemie, Georg Thieme Verlag, Stuttgart (8. überarb. u. erw. Aufl. 2003). Moser, P. K. (1989): Knowledge and Evidence, Cambridge Univ. Press. Mumford, S. (1998): Dispositions, Oxford Univ. Press, Oxford. Musgrave, A. (1988): „The Ultimate Argument for Scientific Realism“, in: Nola, R. (Hg., 1988), Relativism and Realism in Sciences, Reidel, Dordrecht, 229 – 252. Musgrave, A. (2002): „Karl Poppers kritischer Rationalismus“, in: J. Böhm et al. (2002), 25 – 42. Nagel, E. (1961): The Structure of Science, Routledge & Kegan Paul, London. Nagel, E. (1970): „Issues in the Logic of Reductive Explanations“, in: Munitz, M.K. (Hg., 1970), Contemporary Philosophic Thought, Bd. 2, Albany, New York, 117 – 137. Nagel, E. (1979): Teleology Revisited and Other Essays, Columbia University, New York. Neurath, O. (1914/15): „Zur Klassifikation von Hypothesensystemen“, in: Haller, R. und Heiner, R. (Hg.), Gesammelte philosophische und methodologische Schriften, Hölder-Pichler-Tempsky, Wien 1981, 85 – 102. Neurath, O. (1934): „Radikaler Physikalismus und ,Wirkliche Welt‘“, in: Neurath, O., Wissenschaftliche Weltauffassung, hrsg. v. R. Hegselmann, Suhrkamp, Frankfurt/M. 1979, 102 – 119. Niiniluoto, I. (1981): „Statistical Explanation Reconsidered“, Synthese 48, 437 – 472. Niiniluoto, I. (1999): „Defending Abduction“, Philosophy of Science 66 (Proceedings), S436 – S451. Nolting, H.-P. (1978): Lernfall Aggression, rororo, Reinbek bei Hamburg. Novak, J. (1980): „Eine Alternative zu Piagets Psychologie“, in: Jung, W. (Hg., 1980), Piaget und Physikdidaktik. Physica Didactica 7 (2. Sonderheft), 17 – 46. Otte, R. (1981): „A Critique of Suppes‘ Theory of Probabilistic Causality“, Synthese 48, 167 – 189. Pap, A. (1978): „Disposition Concepts and Extensional Logic“, in: Tuomela, R. (Hg., 1978), 27 – 54. Papineau, D. (1993): Philosophical Naturalism, Blackwell, Oxford.

Papineau, D. (1996): „Theory-dependent Terms“, Philosophy of Science 63, 1 – 20. Papineau, D. (1997, Hg.): The Philosophy of Science, Oxford Univ. Press, Oxford. Patry, J.-L. (1991): Transsituationale Konsistenz des Verhaltens und Handelns in der Erziehung, Lang, Bern. Pearl, J. (2000): Causality, Cambridge Univ. Press, Cambridge (2. Ausgabe 2009). Peirce, C. S. (1878): „Deduction, Induction, and Hypothesis“, dt. in: Apel (1976), 229 – 250. Peirce, C. S. (1903): „Lecures on Pragmatism“, dt. in: Apel (1976), 337 – 427. Pflüger, J., und Schurz, Robert (1987): Der Maschinelle Charakter, Westdeutscher Verlag, Opladen. Piaget, J. (1975): Die Entwicklung der physikalischen Mengenbegriffe beim Kinde, Ges. Werke 4, Klett, Stuttgart. Pietroski, P., and Rey, G. (1995): „When Other Things Aren‘t Equal: Saving Ceteris Paribus Laws from Vacuity“, British Journal for the Philosophy of Science 46, 81 – 110. Popper, K. (1935/76): Logik der Forschung, 6. Auflage (mit neuen Anhängen), J. C. B. Mohr, Tübingen 1976 (10. Aufl. 2004). Popper, K. (1974): Objektive Erkenntnis. Ein evolutionärer Entwurf, Hoffmann und Campe (4. verbess. Aufl. 1998; engl. Original 1972). Popper, K. (1983): Realism and the Aim of Science, London, Hutchinson. Popper, K. (1992): Die offene Gesellschaft und ihre Feinde. Band 2, 7. Auflage (mit weitgehenden Verbeserungen und neuen Anhängen), J. C. B. Mohr, Tübingen. Price, H. (1996): Time’s Arrow and Archimedes‘ Point, Oxford Univ. Press, New York. Prim, R., und Tilman, H. (1979): Grundlagen einer kritisch-rationalen Sozialwissenschaft, Quelle & Meyer, Heidelberg. Putnam, H. (1962): „What Theories are Not“, in: Nagel, E., Suppes, P., und Tarski, A. (Hg.): Logic, Methodology and Philosophy of Science, Stanford, 240 – 251. Putnam, H. (1978): Meaning and the Moral Sciences, Routledge and Kegan Paul, London. Putnam, H. (1979, hg. v. Spohn, W.): Die Bedeutung von „Bedeutung“, Klostermann, Frankfurt/M. (engl. Original 1975). Putnam, H. (1990): Vernunft, Wahrheit und Geschichte, Suhrkamp, Frankfurt/M. (engl. Orig. 1982). Putnam, H. (1995): Pragmatismus. Eine offene Frage, Campus, Frankfurt/M.

Literaturverzeichnis Pylyshyn, Z. (1999): „Is Vision Continuous with Cognition?“, Behavioral and Brain Sciences 22, 341 – 365. Quine, W. v. O. (1951): „Two Dogmas of Empiricism“, Philosophical Review 60, 20 – 43; wiederabgedruckt in Quine (1979), Kap. II (zitiert danach). Quine, W. v. O. (1960): Word and Object; dt. Wort und Gegenstand, reclam, Stuttgart 1980; zitiert danach. Quine, W. v. O. (1976): Die Wurzeln der Referenz, Suhrkamp, Frankfurt/M. (engl. Original 1974). Quine, W. v. O. (1995): Unterwegs zur Wahrheit, Schöningh, Paderborn (engl. Orig. 1992). Raiffa, H. (1973): Einführung in die Entscheidungstheorie, Oldenbourg, München (engl. Original 1968). Ramsey, F. P. (1926): „Truth and Probability“, wiederabgedruckt in: ders., Philosophical Papers, hg. von H. D. Mellor, Cambridge Univ. Press, Cambridge 1990. Ramsey, F. P. (1931): The Foundations of Mathematics, Kegan Paul, London (engl. Neuausgabe Ramsey 1978; dt. bei Frommann-Holzboog, StuttgartBad Cannstatt 1980). Rautenberg, W. (2002): Einführung in die mathematische Logik, Vieweg, Braunschweig. Rawls, J. (1979): Eine Theorie der Gerechtigkeit, Suhrkamp, Frankfurt/M. (engl. Orig. 1971). Reichenbach, H. (1938): Experience and Prediction, University of Chicago Press, Chicago. Reichenbach, H. (1949): The Theory of Probability, University of California Press, Berkeley (engl. Erweiterung der dt. Fassung von 1935). Reichenbach, H. (1956): The Direction of Time, Univ. of California Press, Berkeley. Reichenbach, H. (1968): Der Aufstieg der wissenschaftlichen Philosophie, Vieweg, Braunschweig (engl. Orig. 1951). Rescher, N. (1964): Hypothetical Reasoning, Van Gorcum, Amsterdam (Neuaufl. 1976). Rescher, N. (1977): „Die Kriterien der Wahrheit“, in: Skirbekk, G. (1977, Hg.): Wahrheitstheorien, Suhrkamp, Frankfurt/M., 337 – 390 (engl. Original 1973). Rescher, N. (1987): Induktion, Philosophia Verlag, München (engl. Orig. 1980). Rescher, N. (1998): „Pragmatism in Crisis“, in: P. Weingartner et al. (Hg.), The Role of Pragmatics in Contemporary Philosophy, Hölder-Pichler-Tempsky, Vienna 1998, 24 – 38. Rips, L. J. (1994): The Psychology of Proof, MIT Press, Cambridge. Rock, I. (1984): Perception, Scientific American Books, New York (dt. als Wahrnehmung, Spektrum Akad. Verlag, Heidelberg 1998).

Rorty, R. (1982): Consequences of Pragmatism, Harvester Press, Brighton. Rosenberg, A. (2008): Philosophy of Social Science, 3rd., Westview Press, Boulder. Rott, H. (1992): Reduktion und Revision, P. Lang, Bern. Rott, H. (1994): „Zur Wissenschaftsphilosophie von Imre Lakatos“, Philosophia Naturalis 31/1, 25 – 62. Runggaldier, E. (1990): Analytische Sprachphilosophie, Kohlhammer, Stuttgart. Salmon, W. et al. (1971): Statistical Explanation and Statistical Relevance (with Contributions by R. C. Jeffrey and J. G. Greeno), Univ. of Pittsburgh Press, London. Salmon, W. (1984): Scientific Explanation and the Causal Structure of the World, Princeton Univ. Press. Salmon, W. (1989): Four Decades of Scientific Explanation, Univ. of Minnesota Press, Minneapolis. Salmon, W. (1997): „Causality and Explanation: A Reply to Two Critiques“, Philosophy of Science 64, 461 – 477. Savigny, E. v. (1976): Grundkurs im wissenschaftlichen Definieren, 4. Aufl., dtv, München (5. Aufl. 1980). Savigny, E. v., et al. (1976): Juristische Dogmatik und Wissenschaftstheorie, C. H. Beck, München. Savitt, S. F. (1996): „The Direction of Time“, British Journal for the Philosophy of Science 47, 347 – 370. Schiefele, H. et al. (1979): „,Interesse‘ als Ziel und als Weg der Erziehung“, Zeitschrift für Pädagogik 25, Heft 1, 1 – 20. Schiffer, S. (1991): „Ceteris Paribus Laws“, Mind 100, 1 – 17. Schlick, M. (1930/31): „Die Wende der Philosophie“, in: Erkenntnis, Bd. l, 4 – 11; wiederabgedruckt in: Schleichert, M. (1975, Hg.), 12 – 19. Schmidt, P. F. (1971): „Ethische Normen in der wissenschaftlichen Methode“, in: Albert/Topitsch (1971), 353 – 364. Scholz, O. (2001): Verstehen und Rationalität, Vittorio Klostermann, Frankfurt/M. (2. Auflage). Schurz, G. (1982): „Ein Logisch-Pragmatisches Modell von Deduktiv-Nomologischer Erklärung (Systematisierung)“, Erkenntnis 17, 321 – 341. Schurz, G. (1983): Wissenschaftliche Erklärung. Ansätze zu einer logisch-pragmatischen Wissenschaftstheorie, dbv-Verlag für die TU Graz, Graz. Schurz, G. (1985): „Denken, Sprache und Erziehung: Die aktuelle Piaget-Kontroverse“, Zeitschrift für Semiotik, 7, Heft 4, 1985, 335 – 366.

253

254

Literaturverzeichnis Schurz, G. (1990, Hg.): Erklären und Verstehen in der Wissenschaft, Oldenbourg, München (hardcover 1988). Schurz, G. (1990a): „Was ist wissenschaftliches Verstehen?“, in: Schurz (1990, Hg.), 235 – 267. Schurz, G. (1990b): „Paradoxical Consequences of Balzer’s and Gähde’s Criteria of Theoreticity. Results of an Application to Ten Scientific Theories“, Erkenntnis 32, 161 – 214. Schurz, G. (1991): „Relevant Deduction“, Erkenntnis 35, 391 – 437. Schurz, G. (1994): „Relevant Deduction and Hypothetico-Deductivism: A Reply to Gemes“, Erkenntnis 41, 183 – 188. Schurz, G. (1996a): „Scientific Explanation: A Critical Survey“, Foundation of Science I/3, (1995/95), 429 – 465. Schurz, G. (1996b): „Kinds of Unpredictability in Deterministic Systems“, in: P. Weingartner/G. Schurz (eds.), Law and Prediction in the Light of Chaos Research, Springer, Berlin 1996, 123 – 141. Schurz, G. (1997a): The Is-Ought Problem. An Investigation in Philosophical Logic, Kluwer (Studia Logica Library), Dordrecht 1997. Schurz, G. (1997b): „Die Goodman-Paradoxie: Ein Invarianz- und Relevanzproblem“, in: W. Lenzen (Hg.), Das weite Spektrum der analytischen Philosophie, de Gruyter, Berlin 1997, 290 – 306. Schurz, G. (1997c): „Probabilistic Default Reasoning Based on Relevance- and Irrelevance Assumptions“, in: D. Gabbay et al. (eds.), Qualitative and Quantitative Practical Reasoning, Springer, Berlin 1997, 536 – 553. Schurz, G. (1998a): „Koexistenzweisen rivalisierender Paradigmen“, in: Schurz, G., und Weingartner, P. (Hg.), 1 – 52. Schurz, G. (1998b): „Das Problem der Induktion“, in: H. Keuth (Hg.), Karl Popper. Logik der Forschung, Akademie-Verlag, Berlin 1998, 25 – 40. Schurz, G. (1998c): „Probabilistic Semantics for Delgrande’s Conditional Logic and a Counterexample to his Default Logic“, Artificial Intelligence 102, No. 1, 81 – 95. Schurz, G. (1999): „Tarski and Carnap on Logical Truth – or: what is Genuine Logic?“, in: J. Wolenski and E. Köhler (Hg.), Alfred Tarski and the Vienna Circle, Kluwer, Dordrecht 1999, 77 – 94. Schurz, G. (2001a): „Causal Asymmetry, Independent Versus Dependent Variables, and The Direction of Time“, in: W. Spohn et al. (Hg.), Current Issues in Causation, Mentis Verlag, Paderborn, 47 – 67. Schurz, G. (2001b): „Normische Gesetzeshypothesen und die wissenschaftsphilosophische Bedeutung des nichtmonotonen Schließens“, Zeitschrift

für Allgemeine Wissenschaftstheorie 32, 2001, 65 – 107. Schurz, G. (2001c): „What Is ,Normal‘? An EvolutionTheoretic Foundation of Normic Laws and Their Relation to Statistical Normality“, Philosophy of Science 28, 2001, 476 – 497. Schurz, G. (2001d): „Pietroski and Rey on Ceteris Paribus Laws“, The British Journal for the Philosophy of Science 52, 359 – 370. Schurz, G. (2002a): „Karl Popper, Deduktion, Induktion, und Abduktion“, in: J. M. Böhm, H. Holweg und C. Hook (Hg.), Karl Poppers Kritischer Rationalismus Heute, Mohr-Siebeck, Tübingen 2002, 126 – 143. Schurz, G. (2002b): „Ceteris Paribus Laws: Classification and Deconstruction“, in: Earman et al. (Hg.), 351 – 372. Schurz, G. (2003): „Wissenschafts- und Erkenntnistheorie, Logik und Sprache: Positivismus, Neopositivismus und das Umfeld“. In: K. Acham (Hg.), Geschichte der österreichischen Humanwissenschaften. Band 6.1, Passagen Verlag, Wien 2004, 227 – 298. Schurz, G. (2004): „Erklären und Verstehen“, in: Jaeger, F., und Straub, J. (Hg.), Handbuch der Kulturwissenschaften. Band 2 (Kap. 8.5), J. B. Metzler Verlag, Stuttgart, 156 – 174. Schurz, G. (2005a): „Semantic Holism and (Non-) Compositionality of Scientific Theories“, in: M. Werning et al. (Hg.), The Compositionality of Meaning and Content. Vol. I, Ontos-Verlag, Frankfurt, 271 – 284. Schurz, G. (2005b): „Kuipers‘ Account to H-D Confirmation and Truthlikeness“, in: Festa, R. (Hg.), Logics of Scientific Discovery. Essays in Debate With Theo Kuipers, Rodopi, Amsterdam 2005, 141 – 159. Schurz, G. (2005c): „Laws of Nature versus System Laws“, in: Faye, J. et al. (ed., 2005): Nature’s Principles, Kluwer, Dordrecht, 255 – 268. Schurz, G. (2008a): „Patterns of Abduction“, Synthese 164, 201 – 234. Schurz, G. (2008b): „The Meta-Inductivist’s Winning Strategy in the Prediction Game: A New Approach to Hume’s Problem“, Philosophy of Science 75, 278 – 305. Schurz, G. (2009): „When Empirical Success Implies Theoretical Reference: A Structural Correspondence Theorem“, British Journal for the Philosophy of Science 60/1, 101 – 133. Schurz, G. (2013a): „Bayesian Pseudo-Confirmation, Use-Novelty, and Genuine Confirmation“, Studies in History and Philosophy of Science 45, 2013, 87 – 96.

Literaturverzeichnis Schurz, G. (2013b): „Criteria of Theoreticity: Bridging Statement and Non Statement View“, Erkenntnis 2014 (link.springer.com/article/10.1007/s10670013-9581-x). Schurz, G., und Lambert, K. (1994): „Outline of a Theory of Scientific Understanding“, Synthese 101/1, 65 – 120. Schurz, G., und Weingartner, P. (1987): „Verisimilitude Defined by Relevant Consequence-Elements“, in: Kuipers, T. A. (Hg.), What Is Closer-ToThe-Truth?, Rodopi, Amsterdam, 47 – 78. Schurz, G., und Weingartner, P. (1998, Hg.): Koexistenz rivalisierender Paradigmen, Westdeutscher Verlag, Opladen/Wiesbaden. Schurz, G., und Weingartner, P. (2010): „Zwart and Franssen’s Impossibility Theorem Holds for PossibleWorld-Accounts but not for Consequence-Accounts to Verisimilitude“, Synthese 172, 2010, 415 – 436. Scriven, M. (1959a): „Truisms as Grounds for Historical Explanations“, in: P. Gardiner (Hg.), Theories of History, New York, The Free Press; wiederabgedruckt in Giesen, B., und Schmidt, M. (Hg.), Theorie, Handeln und Geschichte, Hoffmann und Campe, Hamburg. Scriven, M. (1959b): „Explanation and Prediction in Evolutionary Theory“, Science 130, 477 – 482. Shapiro, S. (2000): Thinking About Mathematics. The Philosophy of Mathematics, Oxford Univ. Press, Oxford. Sher, G. (1991): The Bounds of Logic, MIT Press, Cambridge/Mass. Siegel, L. (1978): „The Relationship of Language and Thought in the Preoperational Child“, in: Siegel, L., Brainerd, C. (Hg., 1978), Alternatives to Piaget, Academic Press, New York, 43 – 67. Skyrms, B. (1989): Einführung in die induktive Logik (hg. von G. J. W. Dorn), Peter Lang, Frankfurt/M. (dt. Übersetz. und Bearbeit. des engl. Orig. von 1975). Sneed, J. D. (1971): The Logical Structure of Mathematical Physics, Reidel, Dordrecht. Sober, E. (2007): „Evidence and Value Freedom“, in: Kincaid, H. et al. (Hg.), Value-Free Science?, Oxford Univ. Press, Oxford, 109 – 119. Spirtes, P., et al. (1993): Causation, Prediction, and Search, Springer, New York (Neuauflage 2000). Stadler, F. (1997): Studien zum Wiener Kreis, Suhrkamp, Frankfurt/M. Stegmüller, W. (1969): Probleme und Resultate der Wissenschaftstheorie und Analytischen Philosophie. Band I: Wissenschaftliche Erklärung und Begründung, Springer, Berlin. Stegmüller, W. (1970): Probleme und Resultate … (wie 1969). Band II: Theorie und Erfahrung. 1. Halbband (Studienausg. Teil A – C), Springer, Berlin.

Stegmüller, W. (1971): „Das Problem der Induktion“, in: Lenk, H. (1971, Hg.): Neue Aspekte der Wissenschaftstheorie, Vieweg, Braunschweig, 13 – 74. Stegmüller, W. (1973a): Probleme und Resultate … (wie 1969). Band II. 2. Halbband: Theorienstrukturen und Theoriendynamik (Studienausg. Teil D – E), Springer, Berlin. Stegmüller, W. (1973b), Probleme und Resultate … (wie 1969). Band IV. 1. Halbband: Personelle Wahrscheinlichkeit (Studienausg. Teil A – C), Springer, Berlin. Stegmüller, W. (1973c), Probleme und Resultate … (wie 1969). Band IV. 2. Halbband: Statistisches Schließen (Studienausgabe Teil D+E), Springer, Berlin. Stegmüller, W. (1979a): „Walter von der Vogelweides Lied von der Traumliebe und Quasar 3 C 273“, in Stegmüller (1979c), 27–86 (wiederabgedruckt in Bühler 2003). Stegmüller, W. (1979b): „Wertfreiheit, Interessen, und Objektivität“, in: Stegmüller (1979c), 177 – 203. Stegmüller, W. (1979c): Rationale Rekonstruktion von Wissenschaft und ihrem Wandel, reclam, Stuttgart. Stegmüller, W. (1983): Probleme und Resultate … (wie 1969). Band I: Erklärung – Begründung – Kausalität. 2. verbesserte und erw. Auflage, Springer, Berlin. Stegmüller, W. (1986): Probleme und Resultate … (wie 1969). Band II. 3. Teilband: Die Entwicklung des neuen Strukturalismus seit 1973, Springer, Berlin. Stich, S. (1990): The Fragmentation of Reason: Preface to Pragmatic Theory of Cognitive Evaluation, MIT Press, Cambridge/Mass. Strevens, M.: (2000): „Do Large Probabilities Explain Better?“, Philosophy of Science 67, 366 – 390. Strevens, M. (2004): „Bayesian Confirmation Theory: Inductive Logic, or Mere Inductive Framework“, Synthese 141, 365 – 379. Suppes, P. (1957): Introduction to Logic, Princeton, New Jersey. Suppes, P. (1970): A Probabilistic Theory of Causality, North-Holland, Amsterdam. Swinburne, R. (1974, Hg.): The Justification of Induction, Oxford University Press, Oxford. Tarski, A. (1936a): „Der Wahrheitsbegriff in den formalisierten Sprachen“, Studia Philosophica Vol. 1, 261 – 405; in engl. Version in: Tarski (1956), Kap. VIII; zitiert danach. Tarski, A. (1936b): „Über den Begriff der logischen Folgerung“, Actes du Congrès International de Philosophie Scientifique Vol. 7, Paris, 1 – 11; engl. Version in: Tarski, (1956), Kap. XVI; zitiert danach.

255

256

Literaturverzeichnis Tarski, A. (1956): Logic, Semantics, Metamathematics, Clarendon Press, Oxford. Tarski, A. (1986): „What are Logical Notions“, in: History and Philosophy of Logic 7, 143 – 154 (Vortragsmanuskript von 1966). Thagard, P. (1999): Kognitionswissenschaft, KlettCotta, Stuttgart (engl. Original 1996). Toulmin, S. (1950): The Place of Reason in Ethics, Cambridge University Press, Cambridge (Neuauflage 1968). Tuomela, R. (1973): Theoretical Concepts, Springer, Berlin. Tuomela, R. (Hg., 1978): Dispositions, Reidel, Dordrecht. Tuomela, R. (1981): „Inductive Explanation“, Synthese 48, 257 – 294. Van Fraassen, B. (1980): The Scientific Image, Clarendon Press, Oxford (Neuaufl. 1990). Van Fraassen, B. (1989): Laws and Symmetry, Clarendon Press, Oxford. Van Fraassen, B. (1990): „Die Pragmatik des Erklärens“, in: Schurz (1990a, Hg.), 31 – 90. Vickers, John (2010): „The Problem of Induction“, The Stanford Encyclopedia of Philosophy (Spring 2010 Edition), Edward N. Zalta (ed.), Von Mises, R. (1964): Mathematical Theory of Probability and Statistics, Academic Press, New York. Von Wright, G. H. v. (1974): Erklären und Verstehen, Fischer, Frankfurt/M. (engl. Original 1971). Watkins, J. W. N. (1984): Science and Skepticism, Hutchinson, London.

Weber, M. (1968): Gesammelte Aufsätze zur Wissenschaftslehre, hg. v. J. Winckelmann, 3. erw. u. verb. Aufl., J. C. B. Mohr, Tübingen (Ersterscheinung 1922). Weingartner, P. (1978): Wissenschaftstheorie I: Einführung in die Hauptprobleme, problemata, frommann-holzboog (2. Auflage). Weingartner, P. (1996): Logisch-Philosophische Untersuchungen zu philosophie-historischen Themen, P. Lang, Frankfurt/M. Weingartner, P., und Schurz, G. (1986): „Paradoxes Solved by Simple Relevance Criteria“, Logique et Analyse 113, 3 – 40. Westermann, R., und Hager, W. (1982): „Entscheidung über statistische und wissenschaftliche Hypothesen“, Zeitschrift für Sozialpsychologie 13, 13 – 21. Whorf, B. L. (1963): Sprache, Denken, Wirklichkeit, Rowohlt, Reinbek bei Hamburg (engl. Orig. 1956). Wilson, M. (1979): „Maxwell’s Condition – Goodman’s Problem“, British Journal for the Philosophy of Science 30, 107 – 123. Wittgenstein, L. ( 1921): Tractatus logico-philosophicus, 9. Auflage Suhrkamp, Frankfurt/Main 1973. Wittgenstein, L. (1945): Philosophische Untersuchungen, in: Werkausgabe Bd 1, Suhrkamp, Frankfurt/M. 1984. Woodward, J. (2002): „There Is No such Thing as a Ceteris Paribus Law“, in: Earman et al. (2002), 303 – 328. Worrall, J. (1997): „Structural Realism: The Best of Both Worlds?“, in: Papineau, D. (Hg., 1997), 139 – 165.

Definitionen, Merksätze, und Abbildungen Definitionen: (Def. 2.1-1) Methode der rationalen Rekonstruktion 24 (Def. 2.5-1) Schema des Zweck-Mittel-Schlusses 41 (Def. 2.6-1) Monotonie 55 (Def. 3.2-1) Deskriptive vs. normative Sätze 80 (Def. 3.2-2) Inhaltliche Satzeinteilung 82 (Def. 3.3.-1) Logische Wahrheit 84 (Def. 3.3-2) Logische Gültigkeit 85 (Def. 3.4-1) Definitorische Wahrheit 86 (Def. 3.5-1) Essentielle Satztypen 93 (Def. 3.7-1) Logischer und empirischer Gehalt 97 (Def. 3.8-1) Verifikation, Falsifikation, Bestätigung und Schwächung 98 (Def. 3.9-1) Statistische und subjektive Wahrscheinlichkeit 99 (Def. 3.9-2) Bedingte Wahrscheinlichkeit 100 (Def. 3.9-3) Prinzip der engsten Referenzklasse 101 (Def. 3.10-1) Nomologisch definierte Dispositionen 103 (Def. 3.10-2) Relevanz in logischen Schlüssen 107 (Def. 3.10-3) Relevante Konsequenzelemente 109 (Def. 3.10-4) Relevanter Gehalt 109 (Def. 4.1-1) Strikte Relevanzbedingung 121 (Def. 4.1-2) Irreduzible Repräsentation 123 (Def. 4.1-3) Statistische Relevanzbedingung 126 (Def. 4.2-1) Akzeptanzintervall 136 (Def. 4.2-2) Signifikante Stichprobendifferenz 138 (Def. 4.5-1) Mittelwert und Streuung 157 (Def. 5.3-1) T-theoretischer Begriff 186 (Def. 5.3-2) Vortheorie 187 (Def. 6.5-1) Singuläre Kausalität 240

Merksätze: (Ms. 2.5-1) Wertneutralitätsforderung 45 (Ms. 2.6-1+2) Monotonie und Nichtmonotonie 55 (Ms. 2.6-3+4) Anwendungsregel monotoner und nicht-monotoner Schlüsse 56 (Ms. 3.4-1) Empirische Nichtkreativität 88 (Ms. 3.5-1) Folgerungsbeziehungen 92 (Ms. 3.9-1) Grundaxiome der Wahrscheinlichkeit 102 (Ms. 3.10-1) Theoreme der Wahrscheinlichkeit 110 (Ms. 3.10-1) Statistisches principal principle 115

(Ms. 4.1-1) Statistische Relevanz und Korrelation 125 (Ms. 4.2-1) Überprüfung von strikten Gesetzeshypothesen 131 (Ms. 4.2-2) Überprüfung von statistischen Gesetzeshypothesen 139 (Ms. 4.3-1) Reichenbach-Bedingungen für gemeinsame Ursachen 148 (Ms. 4.5-1) Mittelwert und Streuung von Stichprobenmittelwerten 158 (Ms. 4.5-2) Subjektive Rechtfertigung der LikelihoodIntuition 161 (Ms. 5.1-1) Synthetische Natur von Zuordnungsgesetzen 168 (Ms. 5.1-2+3) Methodische Indikatorregeln 175 f. (Ms. 5.4-1) Deduktionslogischer Überprüfungsholismus 190 (Ms. 5.6-1 – 5.6-3) Falsifikation, Erfolg, und Mißerfolg einer Theorieversion 200 (Ms. 5.6-4) Bewährund und Schwächung einer Theorieversion 201 (Ms. 5.6-4*) Dynamische Wahrheitsnähe einer Theorieversion 202 (Ms. 5.6-5) Intertheoretischer Vergleich und Theorienfortschritt 203 (Ms. 5.6-6) Rationale Akzeptanz und Zurückweisung einer Theorieversion 206 (Ms. 5.6-7) Empirischer Gehaltsüberschuss 207 (Ms. 5.6-8+9) Bewährung und Schwächung von Theoriekernen 207 (Ms. 5.6-10) Empirischer Erfolg und rationale Präferenz für Theoriekerne 208 (Ms. 5.6-11+12) Rationale Akzeptanz und Zurückweisung von Theoriekernen 208 (Ms. 6.1-1+2) DN-Ereignis- und Gesetzeserklärung 223+225 (Ms. 6.2-1) Gesetzesartigkeit und Kausalität für DNErklärungen 228 (Ms. 6.2-2) Prognostische Funktion von DN-Argumenten 229 (Ms. 6.2-3) Deduktive Relevanz von DN-Argumenten 230 (Ms. 6.3-1) Induktiv-statistische Erklärung 230 f. (Ms. 6.3-2) Maximale Bestimmtheit 232 (Ms. 6.5-1) Markov-Reichenbach-Bedingungen für kausal hinreichende Graphen 243 f.

258

Definitionen, Merksätze, und Abbildungen Abbildungen: Abb. 2.1-1: Methode der rationalen Rekonstruktion 25 Abb. 2.3-1: Drei Ebenen der wissenschaftlichen Methode 31 Abb. 2.3-2: Beispiel für die drei Ebenen 31 Abb. 2.4-1 Klassifikation der Realwissenschaften 38 Abb. 2.5-1: Schematische Darstellung der Wertneutralitätsforderung 46 Abb. 2.6-1: Das induktiv-deduktive Schema 49 Abb. 2.6-2: Zusammenwirken von epistemischer Induktion und Abduktion 54 Abb. 3.1-1: Klassifikation von Begriffen nach ihrem logischen Typ 67 Abb. 3.1-2: Klassifikation von Begriffen nach ihrem Inhaltstyp 71 Abb. 3.1-3: Klassifikation von Begriffen nach ihrem Skalentyp 74 Abb. 3.1-4: Rangskalierung von D 75 Abb. 3.2-1: Klassifikation von Sätzen nach ihrem Inhaltstyp 79 Abb. 3.5-1: Klassifikation von Sätzen nach ihrem Allgemeinheitsgrad 89 Abb. 3.5-2: Statistische Generalisierung 90 Abb. 3.8-1: Überprüfbarkeit von Generalisierungsarten 98 Abb. 3.9-1: Nichtmonotonie statistischer Generalisierungen 100

Abb. 3.10-1: Drei Binomialverteilungen 112 Abb. 4.1-1: Irrelevanz und Witz 124 Abb. 4.2-1: Akzeptanzintervall 135 Abb. 4.2-2: Wahrscheinlichkeitsverteilung von Stichprobendifferenzen 138 Abb. 4.3-1: Gemeinsame Ursache 146 Abb. 4.3-2: Gemeinsame Ursache vs. intervenierende Variable 148 Abb. 4.3-3: Scheinunabhängigkeit 151 Abb. 4.3-4: Kausalrichtung 152 Abb. 4.5-1: Normalverteilung 156 Abb. 4.5-2+3: Kurvenanpassung 159 Abb. 5.1-1: Theoretisches Merkmal und empirische Dispositionen 167 Abb. 5.1-2: Balken- und Federwaage auf Erde und Jupiter 170 Abb. 5.2-1: Theorienverband der klassischen Mechanik 180 Abb. 5.3-1,2,3: Untergliederungen einer Theorie 186 Abb. 5.6-1: Historische Theorienfolge 199 Abb. 5.6-2: Empirischer Erfolg von Theorien 203 Abb. 5.6-3: Empirisch komplementäre Theorieversionen 204 Abb. 5.6-4: Erfolgsbewertungen von Theorien – Realbeispiel 205 Abb. 6.5-1: Drei statistisch stark ununterscheidbare Graphen 243

Autorenregister Adam, M. 59, 64 Adams, E. 174 Adorno, T. W. 19, 40, 208 f. Albert, H. 19, 40, 208 Alston, W. P. 12, 40, 58, 64 Anderson, A. R. 108 Anderson, J. R. 19 Apel, K.-O. 19 f., 28 Aqvist, L. 41 Aristoteles 12, 49 Armstrong, D. M. 237, 239, 245 Ausubel, D. 196 Bacchus, F. 110 Balzer, W. 18, 21, 71, 78, 173, 177, 186 f., 190, 214 Barnes, E. 189 Bartelborth, T. 53, 64 Bartels, A. 29 Bauer, H. 101, 110, 112., 126, 156, 158 Belnap, N. D. 108 Berlin, B. 60 Bhaskar, R. 211, 238 Bird, A. 12, 18, 21, 52 Bortz, J. 73, 78, 101, 126, 134 f., 141 f., 158 f., 163, 174 Bourbaki 34 Brainerd, C. 192, 194 f. Brendel, E. 29 Bridgeman, P. W. 73 Bromberger, S. 225, 227 f. Bühler, A. 19, 236 Bühler, C. 43 Bunge, M. 204 f., 212 Campbell, D. T. 18 Carnap, R. 14 f., 25 f., 34, 44, 47 – 51, 56, 60 f., 70, 74, 78, 83, 88, 94 f., 101 – 105, 110, 114 – 117, 129, 156, 162, 171 f., 211, 214 – 219, 221, 238 Carrier, M. 180, 212 Cartwright, N. 91, 149, 152, 182, 235 Chalmers, A. F. 57 f., 83 Coffa, J. 233 Cover, J. A. 221

Cresswell, M. J. 69 Curd, A. 221 Czaniera, U. 62 Dahms, H.-J. 40, 64 Dancy, J. 58 De Finetti, B. 101, 111, 114 f. Döring, N. 126, 163, 174 Dowe, P. 241 Dray, W. 224, 235 f. Dubs, R. 41 Duerr, H. P. 121 Duhem, P. 189, 224 Eagle, A. 113 Earman, J. 52, 91, 101, 103, 114 f., 160, 162 f., 218, 238 f., 245 Ebbinghaus, H. D. 69, 117 Eells, E. 147, 149, 152, 240 Essler, W. 69, 103 Etchemendy, J. 104 f. Evans, J. St. B. 195 Feyerabend, P. 17 Field, H. 34 Fisher, R. A. 101, 134, 137, 142, 160 Flach, P. 53 Fodor, J. 58, 71, 106, 216 Follesdal, D. 35 Forster, M. 159, 204 Frankena, W. K. 40 Frege, G. 35, 71 Friedman, M. 189, 225 Gabbay, D. M. 55 Gadamer, H. 35 Gadenne, V. 91 Gärdenfors, P. 234 Garnham, A. 60, 76 Gemes, K. 108, 217 Gerthsen, C. 178 Giere, R. 21, 57, 59, 94, 182 Gillies, D. 101, 113, 117 Glasersfeld, E. v. 19, 56 Glymour, C. 107, 159, 201, 217, 241 f., 244 Gödel, K. 35

Goldszmidt, M. 127 Good, I. J. 116 Goodman, N. 26, 103, 218 f., 221, 237 f. Greeno, J. 112 Grice, H. P. 108 Grünbaum, A. 146, 183 f., 227 Habermas, J. 19 f., 40, 42 f., 65 Hanson, N.R. 16, 58 f. Hare, R. 71 Harman, G. 53 f. Hart, W. D. 34 Haussman, D. 240 – 242, 244 f. Haussmann, T. 19, 236 Hays, W. 112, 134, 142 f., 156, 160 f. Hempel, C. G. 14 f., 22, 60, 83, 86, 91, 93 – 95, 116, 122 f., 131, 163, 171, 177, 184, 191, 211, 213, 221, 223 – 238, 244 f. Hilgendorf, E. 36, 43 Hintikka, J. 51 Howson, C. 48, 51, 101, 113 – 115, 134, 142 f., 162 f., 217 f. Hoyningen-Huene, P. 16 f., 20 Huber, O. 145, 174 Huff, D. 144 f. Hughes, G. E. 69 Hume, D. 13, 48, 52, 150, 238 f. Hummell, H. J. 126, 146 Humphreys, P. 189, 235 Jeffrey, R. 51, 74, 101, 114, 116, 156, 162, 218 Kakas, A. 53 Kamlah, W. 28 Kanitscheider, B. 171 Kay, P. 60 Kelsen, H. 36 Ketland, J. 213 f. Keuth, H. 20, 198 Kitcher, P. 181, 189 Klenk, V. 69, 85, 117 Körner, S. 107 Kolmogorov, A. N. 51, 102, 110, 114

260

Autorenregister Konegen, N. 42 Kornblith, H. 18 Krantz, D. 78, 117 Krebs, D. L. 176 Kripke, S. A. 18, 66, 239 Kriz, J. 43, 56 Kromrey, F. 141 Kuhlen, L. 36 Kuhn, T. S. 16 f., 20 – 25, 58 f., 171, 181, 198 f., 216 Kuipers, T. A. F. 52, 54, 57, 59, 202, 211 f., 216 f. Kutschera, F. v. 60, 70, 81, 97, 101, 113, 115, 117, 162 Lambert, K. 169, 189, 198, 201, 204 Ladyman, J. 48, 221 Lakatos, I. 15, 32, 181 – 184, 194, 196 – 199, 201, 206 f., 220 f. Lamnek, S. 19, 39 Langley, P. 22, 50 Laudan, L. 212 Lauth, B. 112, 156, 181 Lazarsfeld, P. F. 147 Leibniz, G. W. 13, 130, 234 Lenzen, W. 216 f. Leplin, J. 20, 211 Lewis, D. 81, 115, 214 – 216, 237, 239 f. Lipton, P. 53 Lorenzen, P. 28 Losee, J. 12, 20, 128, 211 Mackie, J. L. 121 f. Makinson, D. 97 Maturana, H. R. 19 Maxwell, G. 211, 213 Maxwell, J. C. 238 f., 246 Maxwell, N. W. 189 Mayntz, R. 141 f., 147, 149, 175 Menzies, P. 244 Milgram, S. 208 – 210, 221 Mill, J. St. 13, 128, 163, 239 Mortimer, C. E. 166, 173 Moser, P. K. 53 Moulines, C. U. 188 Mühlhölzer, F. 187 Mumford, S. 103 f. Musgrave, A. 51 f., 212 Nagel, E. 57, 95, 172, 191, 225 Neurath, O. 14, 189 f. Newton, I. 13, 53, 171, 176 f., 179

Niiniluoto, I. 53, 230 f. Nolting, H.-P. 204 Novak, J. 196 Oakhill, J. 60, 76 Oppenheim, P. 44, 93, 223, 225, 227, 229 Otte, R. 150 Pap, A. 103 Papineau, D. 18, 214 – 216, 221 Patry, J.-L. 41 Pearl, J. 217, 150, 242 – 245 Peirce, C. S. 17, 28, 53 Pflüger, J. 153 Piaget, J. 1175, 192 – 196, 203, 221, 225 Pietroski, P. 91 Popper, K. 17, 20 f., 40, 49 – 52, 54, 65, 81, 83, 98, 113, 128, 130, 184, 197, 201 – 203, 206, 226, 236 Price, H. 242, 244 Prim, R. 21, 42, 63 Putnam, H. 17 f., 44, 59, 72, 83, 211 f. Pylyshyn, Z. 58 Quine, W. v. O. 15, 18, 61, 72, 83, 104 – 106, 117, 189 f., 212, 220 Raiffa, H. 41 Ramsey, F. P. 101, 114, 172, 213 – 216, 221, 239 Rautenberg, W. 71, 117 Rawls, J. 26 Reichenbach, H. 13, 21, 48, 56, 95, 100 f., 116, 125, 146 – 150, 232, 242 f. Reiter, M. 18 Rescher, N. 17, 29, 48, 64, 238 Rey, G. 91 Rips, L. J. 108 Rock, I. 58, 64 Rorty, R. 17 Rott, H. 191, 196 Runggaldier, E. 70, 117 Salmon, W. 116, 121 – 124, 146 f., 223, 230, 232 – 235, 241, 245 f. Sareiter, J. 112, 181 Savigny, E. v. 36, 87

Savitt, S. F. 242 Schiefele, H. 43 Schiffer, S. 237 Schlick, M. 14 Schmidt, P. F. 45 Scholz, O. 19, 235 f. Schurz, G. 14, 16 f., 20, 41 f., 48 – 55, 62, 78, 83, 90 f., 104, 107 f., 127, 155, 169, 172 f., 181 f., 188 – 192, 195, 198, 201, 203 f., 212, 214, 217 – 219, 223, 227 – 230, 233 – 239, 242, 244 f. Schurz, R. 153 Scriven, M. 90, 229, 234, 236 Shapiro, S. 34 Sher, G. 104 Siegel, L. 195 Skyrms, B. 48, 163 Sneed, J. D. 18, 177 f., 186, 188, 211, 214 Sober, E. 199, 204 Sondergeld, K. 42 Spirtes, P. 242 Stadler, F. 14, 20 Stegmüller, W. 15, 17 f., 21, 23, 35, 39, 48, 64, 73, 94, 101, 103, 112, 117, 142, 172, 188, 213, 217, 221, 223, 226, 228 f., 234, 238, 245 Stich, S. 17 Suppes, P. 18, 51, 87 f., 150 Swinburne, R. 48 Tarski, A. 26, 71, 84, 104 Thagard, P. 19 Tilman, H. 42, 63 Topitsch, E. 40, 64 Toulmin, S. 63 Tuomela, R. 81, 213 – 215, 221, 234 Urbach, P. 101, 113 – 115, 134, 142 f., 162 f., 217 f. Van Fraassen, B. 17, 54, 58, 60, 81, 95, 147, 151 f., 182, 211 f., 225, 228, 234 f., 245 Varela, F. 19 Von Mises, R. 101, 113 Von Wright, G. H. v. 239, 244 Watkins, J. W. N. 52 Weber, M. 39 – 41, 43

Autorenregister Weingartner, P. 20, 23, 29, 62, 87, 107 – 109, 191, 201, 203 Westermann, R. 142 Whorf, B. L. 60

Wilson, M. 239 Winkler, R. 112, 134, 142 f., 156, 160, 162 Wittgenstein, L. 58, 83, 224

Woodward, J. 91 Worrall, J. 180, 212 Ziegler, E. 126, 146, 163

261

Sachregister Abduktion 52 – 55 Abgrenzungsproblem 16, 20, 39, 41 – 45 Abschirmung 147 – 150 Adorno-Milgram-Theorie 208 – 210, 221 ad-hoc Hypothese 183 f., 197 f., 201 f., 205 f., 220 f. Akzeptanzintervall 134 – 139, 142, 157 – 159, 163 f. Allgemeinheit 79, 89, 91, 93, 100, 117 f. Allsatz 67, 81, 89 – 95, 120, 122 f., 127, 227, 237 f. analytisch 13 – 15, 19, 22, 38, 40 – 43, 71, 74, 76, 79 – 81, 85 f., 88 f., 97, 104 – 106, 109, 116 – 118, 166, 171 f., 189, 214 f., 220 f. , 238 f. Antecedens 89 – 91, 100, 110, 115, 120 – 126, 128 – 134, 140, 155, 163, 223 – 235, 244 Approximation 26, 182, 191 Äquivalenz (materiale) 68 Argument 47 – 57 (s. auch Schluss) Asymmetrie (Poppersche) 15, 98, 128 Axiomatisierung, natürliche 185

93 f., 97 – 99, 113, 127, 183 f., 200, 217 – 220 Bestätigung (auch: Bewährung) 31, 51 f., 98, 101, 108, 128 f., 133 f., 137, 160, 180, 184 f., 201 f., 207, 216 – 219 Binomialverteilung 111 f., 134 – 138, 156, 158

Basissatz 83 Bayesianismus 101, 110, 113, 161 – 163, 217 f. Bedeutung 15, 18, 27, 44, 61, 66, 70 – 73, 79 – 89, 104 – 106, 166 – 174, 186, 189 f., 212, 214 – 216, 220 Begriffsarten 66 – 69, 71 – 78, 80, 186 Begründung 212, 225 – 227, 229 – 231, 233 f. Begründungszusammenhang 21, 45 f., 50, 64 Beobachtung(ssatz) 14 – 17, 27 – 29, 30 f., 37, 47 – 51, 57 – 64, 72 f., 76 f., 79 – 83,

Einheitswissenschaft 44 Einzelfall 121, 143, 154 f., 232 empirische Kreativität 180, 188, 220 Empirismus 12 – 15, 20, 23 f., 27, 37, 44, 52, 64, 171, 211 Entstehungszusammenhang 21, 45 f. Erfolgsbilanz einer Theorie 200 – 208 Erkenntnisziel, oberstes 23, 29 Erklärung 25, 29 – 31, 52 – 54, 92, 116, 122, 154 f., 179 – 183, 200, 223 ff. deduktiv-nomologische (DN) – 223 – 231, 233, 235, 244 f.

Carnap-Satz (einer Theorie) 214 – 216, 221 Ceteris paribus-Klausel (-Gesetz) 89, 91, 122, 182, 197 covering law 224, 235 Default-Annahme 127, 174 Definition 14, 27, 29, 34, 68, 70 f., 85 – 89, 102 f., 105 f., 141, 168, 171, 173, 215 f., 219 Determinismus 93, 96 Differentialgleichung 178, 181 f., 237 Disjunktion 68, 83, 108, 110, 122, 190 Dispositionsbegriff 72 f., 102 – 104, 113, 117, 153, 166 – 169 Doppelblindtest 144 Duhem-Neurath-Quine-These 189 f.

induktiv-statistische (IS) – 230 – 234 rationale – 236 f. Evidenz 53 f., 93, 101, 129, 160 f., 195, 217 f., 223, 229, 231 Existenzsatz 81 – 83, 89, 92 f., 97, 185, 213 f., 237 Experiment 25, 29, 35, 37 f., 63, 78, 91, 133, 143, 145, 148, 150 f., 189, 209 – 211 experimentum crucis 189 Explanandum 223 – 226, 230 – 235 Explanans 223 f., 231, 234 Explikation 25 f., 169, 188, 201, 204, 241 Extension 102, 105 f., 109, 122, 213 – 215 Fallibilität 14, 24, 34, 65 Falsifikation 15, 25, 98, 128, 130, 133 f., 142, 183, 197, 200, 202 Falsifikationismus 15, 198 Falsifizierbarkeit 15, 65, 98, 117 Folgerung logisch-deduktive – 85, 92, 101, 190 wahrscheinlichkeitstheoretische – 110, 230 f. Formalisierung 70, 83 f. Forschungsprogramm 181, 196, 198 f. Fundamentalismus 12 f., 18 g. d. w. 55 (s. Äquivalenz) Gehalt empirischer – 44, 83, 88, 97 f., 113, 168, 171, 177 – 180, 188 f., 197 f., 203 – 208, 213 f., 236 logischer – 97, 103 probabilistischer – 97, 174, 184 relevanter – 109, 117, 184 Generalisierung 47, 49 f., 89 – 93, 95 f., 99, 123 – 127, 134, 179, 194, 196, 236, 239

264

Sachregister Gesamtdatum 56, 101, 129 Gesetz Ceteris-Paribus – 89, 91, 122, 182, 197 Koexistenz – 227, 242 Natur- 93, 95 f., 182, 224, 227, 237 – 239 normisches – 36, 89 – 91, 127, 173 f., 194, 210, 235 – 237, 245 Sukzessions – 227 System – 237 f. Zuordnungs – 166 – 174, 177 f., 181, 185 – 187, 192 f., 209, 220 f. Gesetzesartigkeit 67, 72, 93 – 96, 103 f., 121, 227 – 232, 237 – 239 – i. e. S. 238 f. – i. w. S. 237 Globalität 180, 188 f. Größe extensive – 77 f. intensive – 78 Gründe Glaubens – 225 – 229 Real- (s. Ursachen) 225 – 229 Grundgesamtheit (s. Population) Handlungerklärung 235 f. Häufigkeitsgrenzwert 90, 100, 112 f., 116 f., 162 Hermeneutik (s. Verstehen) 19, 39, 235 f. Hintergrundwissen(-system) 18, 31 f., 53, 56, 58 – 61, 72, 88, 94, 115, 153, 184, 201, 223, 228, 231 – 233, 241 Holismus 99, 106, 182 f., 189 f., 211, 220 f. Homogenität – von theoretischen Begriffen 174 f. – von Theorien 190, 207 f. objektive 233 Hume-These 238 f. hypothetisch-deduktiv 107, 217 f., 222 Idealisierung 50, 78, 182, 198 Immunisierungsthese 197 Implikation (materiale) 68, 102, 129 Indeterminismus

epistemischer – 96 objektiver – 96 Indifferenzprinzip 161 – 163 Indikator(en) 143, 161, 166, 170, 173 – 176, 193 f., 210 f., 227, 237 Induktion 15, 17, 47 – 53, 181, 219 f. methodische – 49 f., 65, 131 logische – 50 – 54, 65 epistemische – 51 – 64, 65 Goodmans Rätsel der – 219 induktiv-deduktives Schema 49 f. Inkommensurabilität 16, 18, 20, 216 Instrumentalismus 54, 211, 221 Interpretation 16, 35, 39, 58 – 61, 71, 80, 83 – 85, 104, 143, 154, 175, 213, 235 f. Intersubjektivität 27 f., 35, 62 Intertheoretische Beziehungen 190 intervenierende Variable (Mittlerursache) 148 f., 151, 164 Interventionstheorie der Kausalität 244 Invarianz 93, 192 – 194, 217 irreduzible Repräsentation 123, 185, 217, 230 Irrelevanz 37, 106 f., 123, 128 f., 131, 174, 201, 210, 217, 225 f., 229 f. deduktive – 107 f., 122, 201, 225, 229 f. statistische – 123 – 125, 127, 138 f.

singuläre – 238 f. Kohärentismus 26, 187 Konditionalisierung 115 f., 127, 162 Konfidenzintervall 133, 137, 139, 142 f., 164 Konjunktion 68, 83, 92, 97, 107 – 109, 120, 122 f., 128, 185, 188, 190, 201, 206, 213, 215 f., 228 Konsequens 89 – 91, 100, 121 – 124, 163 Konsequenzelement, relevante 108 f., 201 Konstruktivismus 19, 56, 65 Kontextabhängigkeit 73 Kontinuitätsargument 60 Kontrafaktisches Konditional 72, 103, 237, 240 Korrelation 125 f., 139 f., 146 – 154, 163, 174 f. Kreationismus 12 Kritische Theorie 19, 40 Kurvenpassung 159 f.

Jurisprudenz 36

L-äquivalent 93, 109, 122 f., 214 f., 217, 227 Leibniz-Bedingung 234 Lewis-Definition theoretischer Begriffe 215 f. Likelihood 115, 142 f., 160 f., 218 Logik 13 f., 19, 22, 27 f., 33, 39, 41, 47 f., 64, 66 – 71, 79, 83 – 85, 87, 93, 107 f., 127 f., 190, 195 logische Form 70 f., 83 – 87 L-wahr (= logisch wahr) 83 – 88, 118

Kausal -analyse 241, 243 -e Präemption 240 f., 245 -e Überdetermination 240 f., 245 -er Graph 242 – 244 -prozess 153, 226 – 228, 240 – 242 Kausalität 15, 124, 146, 163, 226 – 229, 239 – 245 generelle – 239 f. Richtung der – 148, 152 – 154, 241 f. Schein – 146 – 152 scheinnegative – 151 f.

Mathematik (Philosophie der – ) 34 Maximale Bestimmtheit 231 – 233, 236 Maxwell-Bedingung 238 f. Medienkritik 154 Mengentheorie 18, 67, 69 – 71 Merkmal funktionale – 103 f., 166 Struktur – 103 f., 166 Messung 29, 755 – 78, 83, 88, 127, 159, 170 f., 178 – 180, 182, 188 Methode(n) – der Übereinstimmung 128, 131 – 134

Sachregister – des Unterschieds 128 f., 131 – 134, 137, 139 qualitative – 19, 39 quantitative – 19, 39 Methodendualismus 19 Metrisierung 76 – 79, 117, 186 Milgram-Experiment 210 Mittel hinreichendes – 41 notwendiges – 41 optimales – 24, 41 -wert 125 f., 138, 156 – 160, 175 Modell 18, 24 – 26, 34 f, 53, 71, 109, 177 f., 181 f., 188, 198, 214 f., 223 f., 233 Mögliche Welt(en) 18, 47, 84, 114, 143, 215, 237, 239 f. Naturalismus 18 f. Negation 68, 83, 92, 97 f., 128, 140, 190, 202, 206, 208 nichtmonoton 54 f., 65, 91, 100, 127, 154, 190, 231 f., 241, 244 non statement view 18, 71 Normalbedingung(en) – der Wahrnehmung 62 -sargument 62, 64 f. Normalwissenschaft 16, 25 f. normisch 36, 89 – 91, 93, 98, 103, 127, 173, 194, 200, 210, 235 f. Notwendigkeit logische – 83 – 88 metaphysische – 18, 130, 224 physikalische – 237 – 239 Nullwahrscheinlichkeit 162 Nutzen (subjektiver) 41 f., 78, 116 Objektivität 24, 27 f., 35, 39, 57, 63 f. Ockhamsches Rasiermesser 172 Ordnungsrelation 75 f. ostensiv (Ostension) 29, 60 – 62, 66, 72, 88, 106, 219 f. Overfitting 159 f. Paradigma 16, 39, 42, 59, 181, 196, 198 Paradox Carnaps – 103 Goodman- 218 f. Irrelevanz- 107, 121, 123, 201, 217

Klebe- 206 Lotterie- 97 Bestätigungs- 217, 221 Preface- 97 Simpson- 152 Piagetsche Entwicklungspsychologie 192 – 195 Population 124, 126, 129, 133 – 138, 140 – 142, 150 f., 156 – 161 Positivismusstreit 40, 64 Prädikat 57, 66 – 71, 100, 107 f., 120, 127, 134, 155, 166, 218 f., 232 f., 238 nomologisches – 119, 232, 237 ostensiv erlernbares – 61, 219 f. pathologisches – 219 f. qualitatives – 219 Pragmatik 17, 70 erkenntnisinterne – 17 erkenntnisexterne – 17 principal principle 51, 115, 161, 231 prognostische Funktion 226, 229, 231 Propensität 113, 115 Proposition 28, 56, 71, 93, 123, 224 Quantor 68 f., 81 f., 90 – 92, 105, 108 f., 213 f. Ramsey-Satz (einer Theorie) 172, 213 – 215 Rationale Rekonstruktion 23 – 26, 64 f. Rationalismus 12 f., 15 f. klassischer – 12 f. kritischer – 15 f., 20, 23, 44, 49 Rationalitätspräsumption 236, 244 raumzeitliche Universalität 95, 233, 238 Realismus 28 f., 34. 54, 56 – 58, 211 f., 220 f. hypothetisch-konstruktiver – 28, 56 f., 65, 212 metaphysischer – 17, 28, 57, 212 minimaler – 24, 26, 28 f., 34 f., 64 Rechtspositivismus 36

Reduktion 14, 44, 103, 118, 171 f., 191 Reduktionismus 14, 20 Redundanz 108, 122, 201, 217, 225, 229 Referenz 18, 70 f., 211 – 213, 215 Referenzklasse 56, 101, 115 f., 127, 155, 232 f. Reichenbach-Bedingungen (für Kausalität) 148 – 150, 232, 242 f. Relativismus 16 f. relevante Konsequenzelemente 108 f., 201 Relevanz (s. Irrelevanz) Repräsentativität 129, 130, 133, 141, 143 f. Retrodiktion 53, 226, 245 Revolution, wissenschaftliche 16, 25 f., 199 Sachverhalt 29 f., 71, 80, 100, 200, 228 salva validitate 107 Satzoperator 67 – 69, 80, 105 intensionaler – 69 wahrheitsfunktionaler – 67 f., 104 Schluss – auf die beste Erklärung 28, 52 – 54, 64 f., 224 deduktiver – 47, 49, 55, 64, 85 induktiver – 47 – 50, 52, 54 – 56, 64, 141 f., 159, 162, 218 f. monotoner – 54 – 56, 65 nicht-monotoner – 54 – 56, 65, 232 Schwächung (s. Bestätigung) Scientific Community 16, 188, 201, 223 Selbstregulation 90 f. Semantik 70 f., 85, 105, 237 f. Signifikanz empirische – 172, 186 -koeffizient 135, 138 f. Skalen 66, 73 – 79, 117, 126, 174 – 176 -einheit 77 -niveau 78 f. -nullpunkt 77 -transformation 78

265

266

Sachregister Sprachabhängigkeit 60, 162, 219, 221 Statistik 33, 39, 74, 101, 126, 128 f., 147, 156, 158, 160 – 163, 242 Inferenz- 142, 161 Test- 142, 160 f. Stellenauswahl 112 f. Stichprobe 51, 110 – 113, 115, 127 – 131, 133 – 145, 150, 157 – 165, 217 f. Strukturalismus 34, 188, 214 Symmetrie 76, 110, 122, 135, 162, 228, 239 Syntax 70 Systembedingung 91, 178 – 180, 182 f., 185, 197 f., 221, 224, 237 Tatsache 39, 71, 223, 239 theoretisch 73, 82, 186 – 188, 213 – 216 – i. e. S. 73, 82 – i. w. S. 73, 82 T- 73, 82, 186 – 188, 199, 216 vor- 82, 176, 184 – 187, 199, 216 Theorie(n) -bewertung 52, 199, 204, 220 empirisch äquivalente – 190, 207, 212 f. empirisch komplementäre – 203 f., 221 -fortschritt 25, 199, 203, 206, 220 -kern 176, 183 – 185, 189, 195 f., 198 – 200, 207 f., 210, 220 f. -peripherie 176, 182 f., 185 f., 193, 195 – 199, 202, 210, 220 -version 16, 183, 185, 189, 197 – 204, 206 – 208, 211, 216, 221 Theorieabhängigkeit 17 f., 59, 64 – der Bedeutung 18, 72 f., 106, 169 – 172, 189 – der Beobachtung 56 – 60 t-Test 158, 175 Überlegungsgleichgewicht 26 Überprüfung 14 f., 27, 30 f., 35 f., 44, 58, 90 f., 98 f., 216 – 219 – von quantitativen Hypothesen 157 – 160

– von statistischen Gesetzen 98, 133 – 143 – von strikten Gesetzen 98, 128 – 130 – von Theorien 98, 182 – 184, 196 – 207 Unabhängigkeit probabilistische – 102, 111, 243 Schein- 151 f. , 243 f. kausale – 243 Unabhängigkeitsargument 62 – 65 Ursache direkte – 148, 242 f. Gegen – 235 gemeinsame – 92, 146 – 149, 154, 227, 242, 244 Handlungs- 151 indirekte – 148, 242 – 244 Variable 66 – 69, 74 f., 82 f., 109, 216, 135, 144 – 152, 156, 159, 164, 175 – 177, 181, 213 f., 242 – 244 Varianz 126, 157 Vektoraddition 177 Vereinheitlichung 168, 180 f., 185, 188 f., 191, 239 Verifikation 15, 98, 121, 128 Verstehen 19, 24, 35, 216, 224, 235 f. Vertauschbarkeit 162, 219 Verteilung Binomial- 111 f., 134, 136, 138, 156, 158 – der Stichprobenmittelwerte 157 f. graphentreue – 243 f., 246 nichtdogmatische – 162, 219 Normal – 112, 134, 136, 138 f., 156 – 158 Verwertungszusammenhang 45 f. Voraussage(n) 29 f., 96, 120, 127, 146, 159, 178 – 183, 185, 220, 226 f., 231, 234, 244 – im epistemischen Sinn 226 – im zeitlichen Sinn 226 qualitativ neue – 180 f., 212, 222 – und Erklärung 30 f., 160, 212, 224 – 228, 231, 233 Vp (Versuchsperson) 61, 75 f., 142 – 145, 174 – 176, 209

Wahrheit definitorische – 28 f., 85 f., 117 Korrespondenztheorie der – 26, 28 f. logische – 83 – 85, 104 f. Wahrheitsnähe 26, 51, 54, 185, 201 – 203 Wahrnehmung 19, 27, 29, 53, 56, 58 – 64, 106, 166 Wahrscheinlichkeit 23, 27, 48 – 51, 67, 90, 96 – 102, 109 – 117, 124 – 127, 133 – 143, 155 – 163, 216 – 219, 231 – 236, 242 – 246 apriori – 115 Axiome der – 97, 102, 109, 114, 174, 218 bedingte – 55, 90, 97, 100, 10, 116, 120, 124, 127, 150, 154 statistische – 90, 92, 99 – 101, 111 – 115, 117, 134, 138, 142 f., 156 – 160, 163, 231 f. subjektive – 99 – 101, 111 f., 114 – 117, 143, 155, 160 – 163, 211, 213, 231 f. Theoreme der – 110 Warum-Frage(n) 225 Werte 21 f., 37, 39 – 46, 62 f., 71, 80 f. wissenschaftsinterne – 45 f. wissenschaftsexterne – 45 f., 64 Wertneutralität 20, 39, 41 – 46, 62, 64 f., 80 Werturteilsstreit 40, 64 Wettquotient 114 Wiener Kreis 13 – 15, 40, 83 Wirkung – s. Ursache Wunderargument 212, 220, 222 Zeit 77 f., 153 f., 162, 227, 241 f. Zirkularität (Zirkelschluss) 48, 58, 113, 187, 216 Zufallsexperiment 111 – 113, 143 Zuordnungsgesetz 166 – 174, 177 f., 181, 185 – 187, 192, 209, 221 Zurückweisungsintervall 134 – 136, 138, 140, 142, 160 Zweck-Mittel-Schluss 41, 43, 44, 65

Lösungen zu ausgewählten Aufgaben (Hinweis: bei nicht angeführten Aufgaben findet sich die Lösung direkt im Text.) Kap. 2.2, 2): Ein Satz ist falsifizierbar, wenn er in einer möglichen Faktenlage falsifiziert werden kann; er ist falsifiziert, wenn er de fakto falsifiziert wurde, und er ist fallibel, wenn er nicht absolut gewiss, sondern fehlbar ist. Kap. 2.4, 2): Erziehungswissenschaft: empirisch, auch experimentell (gelegentlich spekulativ); Humanmedizin: sezierend und experimentell; Kulturwissenschaft: maximal empirisch (häufig spekulativ). Kap. 2.6, 1): (i): Induktiv-statistischer Spezialisierungsschluss, (ii): deduktiver Schluss, vom Allgemeinen auf das Besondere, (iii) induktiver Voraussageschluss, (iv) abduktiver Schluss. Kap. 2.6, 4): (ii) monoton; (i), (iii) und (iv) nichtmonoton. Kap. 2.7, 2): (a) psychologische Normalbedingung der Wahrhaftigkeit, (b) biologische Normalbedingung, (c) psychologische Normalbedingung, (d) physikalische Normalbedingung. Kap. 3.1, 1): (a) Eigenname, (b): einstellige Funktion, (c): funktionaler singulärer Term, (d) zweistellige Relation 1. Stufe, (e): intensionale Satzoperatoren, (f) wahrheitsfunktionaler Satzoperator, (g): Quantor. – 2): (a): Wertbegriff, (b): Beobachtungsbegriff, (c): theoretischer Begriff der Physik, (d): empirischer Dispositionsbegriff der Physik, (e): theoretischer Begriff der Psychologie, (f): empirischer Dispositionsbegriff der Biologie, (g): politischer Normbegriff. – 3): (a): Nominalskala, (b): Absolutskala, (c): Nominalskala, (d): Ordinalskala, (e): Intervallskala, (f): Intervallskala, (g): Verhältnisskala, (h): physikalisch Verhältnisskala, psychologisch Ordinalskala. Kap. 3.2, 1): (a): deskriptiv, (b): gemischt, (c): präskriptiv, (d): deskriptiv. – 2): (a): theoretischer Satz (Singulärsatz), (b): Beobachtungssatz, (c): empirischer Satz (zeitlich-generell), (d): Beobachtungssatz, (e): empirischer Allsatz, (f): theoretischer Satz (Allsatz). Kap. 3.3 – 3.4, 2): (a) log. wahr, (b): def. wahr, (c); synth. wahr, (d): def. wahr, (e): log. wahr, (f): synth. wahr. Kap. 3.5 – 3.6, 1): (a): generell, statistisch, (raumzeitlich) beschränkt, (b): strikt generell, unbeschränkt (negierter Existenzsatz $ Allsatz), (c): Singulärsatz, (d): statistisch generell, unbeschränkt (qualitativ-statistisch), (e): strikter Allsatz, beschränkt (lokalisiert), (f): beschränkter Existenzsatz, (g), strikter Allsatz, unbeschränkt, (h) statistisch-genereller Satz, unbeschränkt. – 2): (b), (g) und (h) sind gesetzesartig, evtl. auch (d); (a) und (e) sind akzidentell. Kap. 3.7 – 3.8, 1): (i): weder-noch, (ii): veri- und falsifizierbar, (iii) wedernoch, (iv): falsifizierbar, aber nicht verifizierbar, (v) veri- und falsifizierbar.

Lösungen zu ausgewählten Aufgaben

Kap. 3.9, 1): (a): die meisten Menschen, die so wenig wie du geschlafen haben, sind am nächsten Morgen müde, (b) die meisten Menschen, die einer anderen Person, die so infiziert war wie er, dieser Person so nahe gekommen sind wie du, werden sich von dieser anderen Person anstecken, (c) meistens sind die Berge morgens an Tagen, denen eine ähnliche Wetterentwicklung vorausging wie in den letzten drei Tagen, und in Regionen wie der unseren, nicht nebelfrei, (d) die meisten Plastikspielzeuge, die dem deines Kindes in relevanten Hinsichten ähneln, gehen nach spätestens drei Wochen kaputt. Kap. 3.9, 2): (i): 100 – (20+30+27) = 23%, (ii): 27+23 = 50%, (iii): 20+30 = 50%, (iv): 30+23 = 53%, (v): 20+27 = 47%, (vi): 20+30+23 = 73%, (vii): 20+27+23 = 70%, (viii): 100%, (ix): 0%; (x): 23/53, (xi): 27/47, (xii): 30/50, (xiii): 20/50, (xiv): F gegeben (U_M): es gilt (F^(U_M)) $ (F^U); daher = 20/73, (xv): es gilt (V_M)^M $ M, daher = 1. Kap. 3.10, 1): (i): wenn man x unter den-und-den Standardbedingungen um Hilfe bittet, dann gilt (normalerweise): x ist hilfreich $ x hilft; (ii): wenn sich x einem Reaktionstest unterzieht, dann gilt: x ist reaktionsschnell $ x reagiert (normalerweise) innerhalb einer so-und-so-kurzen Zeitspanne. – 3): (a): gültig, irrelevant (,weiblich s.v. ersetzbar), (b): gültig, relevant, (c): gültig, irrelevant (,der ein soziales Wesen ist s.v. ersetzbar), (d): ungültig, (e): gültig, irrelevant (,Mensch und ,allein sein s.v. ersetzbar). – 4): „[n:k]“ stehe für „n-über-k“. Gemäß Binomialformel ist [10:5] N 1/25 N 1/2(10 – 5) = [10:5] N (1/210) die Wahrscheinlichkeit, von 10 Würfen genau 5 Mal Kopf zu würfeln. Das Ergebnis lautet daher: (1/210) N R5k10 [10:k]. Bekanntlich gilt [n:k] = [n: (n – k)], und R0kn [n:k] = 2n. Daraus folgt R5k10 [10:k] = (210+[10:5])/2. Es folgt das Ergebnis: 1/2 + [10:5]N(1/211), was geringfügig größer als 1/2 ist. Kap. 4.1, 2): (i) H ! F, (ii) H^G ! F, (iii) H^:G ! F, (iv) H ! G, (v) H^F! G, (vi) H^:F ! G (Hinweis: kontrapositionelle Umformungen wie z. B. :F ! :H, äquivalent mit (i), sind nicht extra angeführt). (i) und (iv) sind relevant. – 3): Korr(M,V) = p(V|M) – p(V) = 46/106 – 53/106 = – 7/106; Korr(F,V) = p(V|F) – p(V) = 54/94 – 47/94 = +7/94; Korr(U,M) = p(M|U) – p(M) = 60/100 – 53/100 = +7/100; Korr(U,F) = p(F|U) – p(F) = 40/100 – 47/100 = – 7/100. Kap. 4.2, 1): (G1) wird (gemäß Übereinstimmungsmethode) durch Stichprobe 1 und 2 bestätigt, (G2) wird durch Stichprobe 1 bestätigt, und (G3) wird durch Stichprobe 3 falsifiziert. – (G2) ist im Antecedensfaktor „über 40“ durch Vergleich mit Stichprobe 2 irrelevant, und im Antecedensfaktor „männlich“ durch Vergleich mit Stichprobe 3 relevant. (G1) ist durch Vergleich mit Stichprobe 3 relevant. – 2): (a) Schwach bestätigt, wenn die Stichprobenhäufigkeit von Rauchern im Akzeptanzintervall, also zwischen 282 und 318 von 500 liegt; andernfalls stark geschwächt. (b): Das Konfidenzintervall lautet [297,333]. – 3): Sie dürfte, auf ganze Prozente aufgerundet, nicht größer sein als 15%, damit auf eine signifikante Korrelation geschlossen werden kann; und sie müsste mindestens 16% betragen, damit nicht auf eine solche geschlossen werden kann. – 5): Kuno hat (a) die Bekannten durch seine Erzählung positiv vorbeeinflusst, er hat (b) 3 von 7 Fälle aus seinem Sample weggelassen (die, die sich nicht ,

,

,

,

268

Lösungen zu ausgewählten Aufgaben

mehr gemeldet haben), und er hatte (c) nur eine experimentelle Gruppe, aber keine Kontrollgruppe (schwer erkältete Bekannte, denen er die Pflanze nicht gab). pffiffiffi korr Kap. 4.5, 1): Gemäß der Formel l(X) pffiffiffiffiffiffi l 1,96 N rsn = n beträgt das Akzeptanzintervall 15 l 1,96 N 2,5 / 25 = [14, 16] – 2): Gemäß der p(ca.) ffiffiffiffiffiffiffiffiffiffiffiffiffiffiffiffiffiffiffiffiffiffiffiffiffiffiffiffiffiffi Formel berechnen wir Dsign = 1,96 N r ð1=nÞ þ ð1=mÞ = 1,96 N 2,8 N pffiffiffiffiffiffiffiffiffiffiffiffiffiffiffiffiffiffiffiffiffiffiffiffiffiffiffi 1=30 þ 1=25 = (ca.) 1,5. Kap. 5.1, 2): (a): In kaum gefährlichen Situationen schreckt die Person leicht auf, und: die Person macht sich mehr Sorgen als nötig um ihre Kinder (usw.). (b): Legt man Spannung an, so fließt ein Strom; und: erhitzt man, so wird die Wärme hervorragend geleitet (usw.). Empirischer Gehalt: (a): eine Person, die in kaum gefährlichen Situationen leicht aufschreckt, wird sich um ihre Kinder mehr als nötig Sorgen machen. (b): Eine Festsubstanz, welche bei Spannungsanlegung Strom leitet, wird auch die Wärme hervorragend leiten. – 3): Eine versteckte Variable bei der Selbsteinschätzung von Vpn ist die wohlbekannte Tatsache, dass die Selbsteinschätzung einen Bias die Richtung besitzt, wie sich die Vp gerne sehen möchte, und wie sie die Erwartung des Interviewers beurteilt. Einseitigkeit wäre gegeben, wenn der Glückszustand konkret nur durch Gesundheitszustand und Wohlstand gemessen werden würde. Kap. 5.5 – 5.7, 3): Eine ad-hoc-Hypothese wäre z.B.: Modus Tollens involviert, im Gegensatz zu Modus Ponens, die Negationsoperation, und diese wird erst äußerst spät gelernt. Diese Hypothese wäre unplausibel, weil in gewissen Situationen die Negation schon vom Kleinkind begriffen wird. – 4): Der Experte ,im weißen Kittel repräsentiert in modernen Gesellschaften das ,Allgemeinverbindliche . Auch Politiker berufen sich auf Experten. Somit besteht ein hoher indirekter Konformitätsdruck, sich diesem ,Allgemeinverbindlichen anzupassen. Kap. 5.8, 5): „x ist wün g.d.w. x vor t0 beobachtet wurde und weiß ist, oder andernfalls grün ist“, sowie „x ist greiß g.d.w. x vor t0 beobachtet wurde und grün ist, oder andernfalls weiß ist“. Damit ergibt sich: „x ist weiß g.d.w. x vor t0 beobachtet wurde und wün ist, oder andernfalls greiß ist“, und „x ist grün g.d.w. x vor t0 beobachtet wurde und greiß ist, oder andernfalls wün ist“. Kap. 6.1 – 6.2, 2): (a): Aktuale Erklärung, (b): Voraussage, keine potentielle Erklärung, weil das Kommen-Wollen, aber nicht das Aussprechen des Satzes, die Ursache für das Kommen ist, (c): Voraussage und potentielle Erklärung (Begründung wie für b), (d): Retrodiktion, keine potentielle Erklärung (hier wird von der Wirkung auf die Ursache geschlossen), (e): Weder Begründung noch Erklärung. Kap. 6.3 – 6.4, 2): Ich weiß über Herrn Müllers körperlichen Allgemeinzustand nichts, außer, dass er 82 Jahre alt war. – 3): Die Information, dass Herr Müller trotz seiner 82 Jahre bei bester Gesundheit ist, senkt die Wahrscheinlichkeit seines baldigen Sterbens nicht nur gegenüber der in diesem Alter zu erwartenden Durchschnittswahrscheinlichkeit, sondern auch unter 50%. Daher liegt kein minimal akzeptables IS-Argument vor. Zusatzinformation macht also ein akzeptables IS-Argument inakzeptabel – ein Beispiel von Nichtmonotonie. – 5): Die rationale Handlungserklä-

269

,

,

,

270

Lösungen zu ausgewählten Aufgaben

rung lautet: weil diese Person durstig war (d. h. Wasser trinken wollte) und glaube (wusste), dass das Trinken des Wassers aus diesem Brunnen ein geeignetes Mittel dazu ist (plus die normische Gesetzesprämisse (6.44). Wenn hingegen diese Person nicht aufhört, Wasser zu trinken, obwohl sie vom vielen Wasser schon einen Blähbauch hat, muss irgendein ein irrationaler Faktor im Spiel sein. Kap. 6.5, 1): (i): universell und Maxwell, (ii): weder universell noch Maxwell, (iii): universell, aber nicht Maxwell.